Annotation of db/baza/okmar15.txt, revision 1.22

1.12      boris       1: Чемпионат:
                      2: Окский марафон - 2015 (Пущино)
1.1       rubashki    3: 
                      4: URL:
                      5: /znatoki/boris/reports/201501Pushchino.html
                      6: 
                      7: Дата:
                      8: 31-Jan-2015
                      9: 
                     10: Инфо:
1.16      rubashki   11: В качестве первых двух туров марафона использовался синхрон "Азовский
                     12: бриз", а в качестве последних полутора - синхрон "Кубок 359".
1.1       rubashki   13: 
                     14: Тур:
                     15: 3 тур. "ЛУК" (Архангельск)
                     16: 
                     17: Вопрос 1:
                     18: Внимание, в вопросе есть замены.
                     19:    В тексте песни группы "Ундервуд" (куда же без нее на марафоне!)
                     20: строчка "Новый год от ИКСА до ИГРЕКА" рифмуется со строкой "Новый год,
                     21: приготовьтесь, будет круто!". Что мы заменили на ИКС и ИГРЕК, если они
                     22: отличаются друг от друга всего одной буквой?
                     23: 
                     24: Ответ:
                     25: Салат и салют.
                     26: 
                     27: Комментарий:
                     28:    Новый год от салата до салюта.
                     29:    Новый год, дед Мороз не так уж стар.
                     30:    Новый год, приготовьтесь, будет круто.
                     31:    А пока-пока-пока, а пока-пока-пока,
                     32:    Все ждут двенадцатый удар.
                     33:    Всё как в жизни - начинаем салатом, заканчиваем салютом. Команда
                     34: "ЛУК" приветствует вас в новом 2015 году и как бы намекает:
                     35: приготовьтесь, будет действительно круто.
                     36: 
                     37: Источник:
                     38:    1. http://morepesen.ru/34810-undervud-novogodnij-car.html
                     39:    2. http://www.youtube.com/watch?v=J0ganCpggo0
                     40: 
                     41: Автор:
                     42: Иван Нефёдов
                     43: 
                     44: Вопрос 2:
                     45: В романе Александра Иличевского театральный режиссер Штейн держит дома
                     46: человеческий череп, который появился у него в доме в 1931 году. Штейн
                     47: уверяет мальчишек, пришедших в гости, что это череп известного человека,
                     48: приводя в доказательство подлинности отсутствие... Чего?
                     49: 
                     50: Ответ:
                     51: Носа.
                     52: 
                     53: Комментарий:
                     54: Штейн неохотно отвечал мальчишкам: "Череп Гоголя. Вот здесь не хватает
                     55: носа. Где нос? Нос пошел гулять". В 1931 году перезахоранивали Николая
                     56: Васильевича.
                     57: 
                     58: Источник:
                     59: Александр Иличевский. Перс. - М.: АСТ: Астрель, 2010. - С. 325.
                     60: 
                     61: Автор:
                     62: Иван Нефёдов
                     63: 
                     64: Вопрос 3:
                     65: [Ведущему: кавычки в вопросе не озвучивать.]
                     66:    Прослушайте список, в котором две буквы в одном из слов лишние:
                     67: "Динозавры", "Хищники", "Черный налим", "Грызуны", "Пингвины", "Крючки".
                     68: Такая российская классификация была предложена в одном из выпусков
                     69: журнала "Forbes" за 2012 год. Шесть видов чего были обозначены таким
                     70: образом?
                     71: 
                     72: Ответ:
                     73: Коррупция.
                     74: 
                     75: Зачет:
                     76: Взяточничество, денежные вымогательства, взятки, способы "дать на лапу".
                     77: 
                     78: Комментарий:
                     79: Не налим, а нал. Причем черный. Если вдруг интересно: "Динозавры" -
                     80: коррупционные практики, постепенно уходящие в прошлое. "Хищники" -
                     81: коррупционные схемы, основанные на вымогании денег у бизнеса органами
                     82: государственного контроля. "Черный нал" - понятно. "Грызуны" -
                     83: использование ресурсов компании для личного обогащения. "Пингвины" -
                     84: случаи использования в коррупционных целях дружбы и прочих личных
                     85: отношений. "Крючки" - использование в своих интересах рычагов
                     86: воздействия на сотрудников, партнеров и ответственных лиц через сбор
                     87: конфиденциальной и компрометирующей информации.
                     88: 
                     89: Источник:
                     90: http://www.forbes.ru/svoi-biznes/biznes-i-vlast/80739-6-vidov-vzyatok-v-rossii
                     91: 
                     92: Автор:
                     93: Иван Нефёдов
                     94: 
                     95: Вопрос 4:
                     96: В апреле 1998 года двое американцев решили своеобразным образом
                     97: поддержать налоговую реформу. Они выплыли на середину портовой гавани
                     98: одного из прибрежных городов. Достали старый сундук, положили в него
                     99: налоговый кодекс и выбросили всё это за борт. Мы не просим вас назвать
                    100: город, рядом с которым происходило это мероприятие. Ответьте, какое
                    101: короткое слово было написано на сундуке.
                    102: 
                    103: Ответ:
                    104: Чай.
                    105: 
                    106: Зачет:
                    107: Tea.
                    108: 
                    109: Комментарий:
                    110: Своего рода символ Бостонского чаепития 1773 года.
                    111: 
                    112: Источник:
                    113:    1. http://hpn.asu.edu/archives/Apr98/0204.html
                    114:    2. http://www.historynewsnetwork.org/article/94858
                    115:    3. http://en.wikipedia.org/wiki/Boston_Tea_Party
                    116: 
                    117: Автор:
                    118: Иван Нефёдов, Александр Усачёв
                    119: 
                    120: Вопрос 5:
                    121: В сентябре 2011 года сайт smithsonian.com [смитсОниан точка ком]
                    122: составил список самых главных произведений мировой литературы, в который
                    123: среди прочих вошли поэма Гомера "Маргит", роман Хемингуэя о Первой
                    124: мировой, пьеса Шекспира "Карденио", роман Джейн Остин "Сэндитон" и роман
                    125: Германа Мелвилла "Остров Креста". Напишите слово, которое мы пропустили
                    126: в предыдущем предложении.
                    127: 
                    128: Ответ:
                    129: Утерянных.
                    130: 
                    131: Зачет:
                    132: Потерянных, утраченных и т.п. по смыслу.
                    133: 
                    134: Комментарий:
                    135: Это были книги, которые мы уже никогда не сможем прочесть, - список
                    136: самых главных утерянных произведений мировой литературы.
                    137: 
                    138: Источник:
                    139: http://www.smithsonianmag.com/arts-culture/The-Top-10-Books-Lost-to-Time.html
                    140: 
                    141: Автор:
                    142: Иван Нефёдов
                    143: 
                    144: Вопрос 6:
                    145: [Ведущему: перечислять фразы с соответствующими паузами после них.]
                    146:    Статья Александра Харламова от 20 ноября 2012 года была посвящена
                    147: известному футбольному клубу и состояла из нескольких абзацев,
                    148: начинавшихся следующими фразами: "львиная доля", "уже довольно давно",
                    149: "честолюбивый тренер", "ершистость", "сумасшедшая победная серия", "как
                    150: ни крути", "убедительная игра". Назовите фамилию тренера, успешно
                    151: возглавлявшего клуб в тот момент.
                    152: 
                    153: Ответ:
                    154: Луческу.
                    155: 
                    156: Комментарий:
                    157: По первым буквам абзацев.
                    158: 
                    159: Источник:
                    160: http://www.sports.ru/football/144829723.html
                    161: 
                    162: Автор:
                    163: Иван Нефёдов
                    164: 
                    165: Вопрос 7:
                    166: У группы "Сатана печет блины" есть песня с не совсем приличным
                    167: названием. В ней поется о нерадивом хозяине, который не заботился о
                    168: своих животных, и за это его не простят ни еж, ни пес, ни енот, но вот
                    169: ОНА, может быть, простит. Назовите ЕЕ.
                    170: 
                    171: Ответ:
                    172: Утка.
                    173: 
                    174: Зачет:
                    175: Уточка.
                    176: 
                    177: Источник:
                    178: "Сатана печет блины" - "Простит утка".
                    179: 
                    180: Автор:
                    181: Денис Овчинников
                    182: 
                    183: Вопрос 8:
                    184: Прослушайте цитату из книги Терри Пратчетта "Пятый элефант": "Господин
                    185: Ваймс, это мой фамильный топор. Он принадлежит моей семье почти
                    186: девятьсот лет, понятно? Конечно, пару раз меняли лезвие. Несколько раз -
                    187: топорище. Меняли конструкцию металлических частей, подновляли узоры...
                    188: но разве от этого он перестал быть девятисотлетним фамильным топором?".
                    189: Этот топор упоминается в статье Википедии, посвященной проблеме,
                    190: появившейся еще во времена античности. Напишите название этой статьи.
                    191: 
                    192: Ответ:
                    193: Корабль Тесея.
                    194: 
                    195: Зачет:
                    196: Парадокс Тесея.
                    197: 
                    198: Комментарий:
                    199: Согласно греческому мифу, пересказанному Плутархом, корабль, на котором
                    200: Тесей вернулся с Крита в Афины, долгое время хранился афинянами. При
                    201: починке в нем постепенно заменяли доски, до тех пор пока среди философов
                    202: не возник спор, тот ли это еще корабль или уже другой, новый?
                    203: 
                    204: Источник:
                    205:    1. http://ru.wikipedia.org/wiki/Корабль_Тесея
                    206:    2. Терри Пратчетт. Пятый элефант.
                    207: http://www.e-reading.link/bookreader.php/46326/Pratchett_24_Pyatyii_elefant.html
                    208: 
                    209: Автор:
                    210: Денис Овчинников
                    211: 
                    212: Вопрос 9:
                    213: (pic: 20150134.jpg)
                    214:    Внимание, в вопросе есть замена.
                    215:    В одном из эпизодов мультсериала "Южный парк" утверждается, что
                    216: каждый раз, когда Морган Фриман что-то объясняет, он получает АЛЬФУ. У
                    217: главного героя мультфильма 1971 года, которого несправедливо обвиняли в
                    218: убийстве, тоже были АЛЬФЫ. Назовите АЛЬФУ.
                    219: 
                    220: Ответ:
                    221: Веснушка.
                    222: 
                    223: Комментарий:
                    224: Мультфильм 1971 года - "Рыжий, рыжий, конопатый".
                    225: 
                    226: Источник:
                    227:    1. Мультсериал "Южный парк", s16e14.
                    228:    2. http://ru.wikipedia.org/wiki/Рыжий,_рыжий,_конопатый
                    229: 
                    230: Автор:
                    231: Денис Овчинников
                    232: 
                    233: Вопрос 10:
                    234: (pic: 20150135.jpg)
                    235:    Перед вами предмет, принадлежащий одному из персонажей мультсериала
                    236: "Папский городок". Назовите профессию этого персонажа двумя словами.
                    237: 
                    238: Ответ:
                    239: Швейцарский гвардеец.
                    240: 
                    241: Комментарий:
                    242: Швейцарская гвардия - один из видов вооруженных сил Ватикана, используют
                    243: алебарды как парадно-церемониальное оружие. В мультфильме лезвие
                    244: заменено на швейцарский нож.
                    245: 
                    246: Источник:
                    247: http://ru.wikipedia.org/wiki/Швейцарская_гвардия
                    248: 
                    249: Автор:
                    250: Денис Овчинников
                    251: 
                    252: Вопрос 11:
                    253: (pic: 20150136.jpg)
                    254:    Художница Джессика Хэйджи с помощью такой схемы отметила то, к каким
                    255: последствиям в повседневной жизни приводят все возможные парные
                    256: сочетания ИХ. Например, AC для нее - "проституция", CF - "реклама", FD -
                    257: "социальное пособие" DB - "суббота", BG - "толстяк в микроплавках", GF -
                    258: "второе место", FE - "язвительность". Назовите ИХ тремя словами, два из
                    259: которых начинаются на одну и ту же букву.
                    260: 
                    261: Ответ:
                    262: Семь смертных грехов.
                    263: 
                    264: Комментарий:
                    265: Примеров, кстати, семь. Вот грехи: A - блуд, B - чревоугодие, C -
                    266: алчность, D - леность, E - гнев, F - зависть, G - гордыня. А вот список
                    267: сочетаний:
                    268:    AB - съедобные трусы,
                    269:    AC - проституция,
                    270:    AD - быстро перепихнуться,
                    271:    AE - домашнее насилие,
                    272:    AF - адюльтер,
                    273:    AG - брак по расчету,
                    274:    BC - последний пончик,
                    275:    BD - суббота,
                    276:    BE - булимия,
                    277:    BF - быстрый метаболизм,
                    278:    BG - толстяк в микроплавках,
                    279:    CD - аферы "разбогатей сейчас",
                    280:    CE - грабеж,
                    281:    CF - реклама,
                    282:    CG - символы статуса,
                    283:    DE - пассивная агрессия,
                    284:    DF - социальное пособие,
                    285:    DG - халтурщики,
                    286:    EF - язвительность,
                    287:    EG - бокс,
                    288:    FG - второе место.
                    289: 
                    290: Источник:
                    291: http://esquire.ru/thisisindexed
                    292: 
                    293: Автор:
                    294: Иван Нефёдов
                    295: 
                    296: Вопрос 12:
                    297:    <раздатка>
                    298:    Статья в журнале "Esquire" о том, как фотограф Мэттью Таун на
                    299: протяжении нескольких лет снимает портреты учеников летной школы на юге
                    300: Англии, называется "Как я провел этим летом".
                    301:    </раздатка>
                    302:    [Ведущему: читать "лётной".]
                    303:    Статья в журнале "Esquire" о том, как фотограф Мэттью Таун на
                    304: протяжении нескольких лет снимает портреты учеников летной школы на юге
                    305: Англии, называется "Как я провел этим летом". В предложенном вам тексте
                    306: мы дважды пропустили две буквы. Напишите пропавшие буквы в любом
                    307: порядке.
                    308: 
                    309: Ответ:
                    310: А и Б.
                    311: 
                    312: Зачет:
                    313: Б и А.
                    314: 
                    315: Комментарий:
                    316: А упало, Б пропало. Школа балетная.
                    317: 
                    318: Источник:
                    319: http://esquire.ru/photo/ballet-boys
                    320: 
                    321: Автор:
                    322: Иван Нефёдов
                    323: 
                    324: Вопрос 13:
                    325: Внимание, в вопросе есть замены.
                    326:    Действие американской компьютерной игры "Singularity" происходит в
                    327: секретном советском научном комплексе. В одной из лабораторий можно
                    328: увидеть Периодический ИКС из ИГРЕКИ. Напишите слова, которые заменены на
                    329: ИКС и ИГРЕК.
                    330: 
                    331: Ответ:
                    332: Стол, элемент.
                    333: 
                    334: Комментарий:
                    335: (pic: 20150137.jpg)
                    336:    Разработчики пытались воссоздать атмосферу советской лаборатории и
                    337: использовали русский язык. "Периодический стол из элементы" - неудачный
                    338: перевод "Periodical table of elements".
                    339: 
                    340: Источник:
                    341:    1. http://sadtranslations.livejournal.com/1151914.html
                    342:    2. http://ru.wikipedia.org/wiki/Singularity_(игра)
                    343: 
                    344: Автор:
                    345: Денис Овчинников
                    346: 
                    347: Вопрос 14:
1.22    ! rubashki  348: В романе ХалЕда ХоссейнИ пожилая мать в разговоре со своим сыном
1.1       rubashki  349: упоминает Джеймса, Джорджа, Роберта, Джона. Саму себя она связывает с Лу
                    350: и задает логичный вопрос: "Как мужчинам удалось монополизировать ИХ?".
                    351: Догадавшись о чем идет речь, назовите любую из фамилий упомянутых
                    352: мужчин.
                    353: 
                    354: Ответ:
                    355: Гериг.
                    356: 
                    357: Зачет:
                    358: Паркинсон, Хангтингтон, Грейвз, Даун.
                    359: 
                    360: Комментарий:
                    361: "Джеймс Паркинсон, Джордж Хантингтон, Роберт Грейвз, Джон Даун. А теперь
                    362: еще и этот мой Лу Гериг. Как мужчинам удалось монополизировать и
                    363: названия болезней?".
                    364: 
                    365: Источник:
                    366: Халед Хоссейни. И эхо летит по горам. - М.: Фантом Пресс, 2013. - С.
                    367: 370.
                    368: 
                    369: Автор:
                    370: Иван Нефёдов
                    371: 
                    372: Вопрос 15:
                    373: Название статьи в русскоязычном журнале "Esquire" [эсквайр] с портретами
                    374: молодых американок, одетых подобно своим куклам, на слух совпадает с
                    375: названием государства, которое по площади территории занимает двадцатое
                    376: место в мире и на гербе которого присутствуют волны. Воспроизведите
                    377: название этой статьи из двух слов.
                    378: 
                    379: Ответ:
                    380: "Республика чад".
                    381: 
                    382: Комментарий:
                    383: США по форме правления, так же как и Чад, является президентской
                    384: республикой. Волны - символ озера Чад.
                    385: 
                    386: Источник:
                    387:    1. http://esquire.ru/photo/american-girls
                    388:    2. http://ru.wikipedia.org/wiki/Чад
                    389: 
                    390: Автор:
                    391: Иван Нефёдов
                    392: 
                    393: Вопрос 16:
1.9       rubashki  394: В компьютерной игре "The binding of Isaac" [зэ бАйндинг оф Айзэк]
1.1       rubashki  395: ("Жертвоприношение Исаака") некоторые артефакты являются реально
                    396: существующими предметами, например, Библия, карты Таро или Спиритическая
                    397: доска. На одном из таких артефактов нарисована буква "А", и при его
                    398: использовании появляется несколько бомб. Назовите этот артефакт тремя
                    399: словами.
                    400: 
                    401: Ответ:
                    402: Поваренная книга анархиста.
                    403: 
                    404: Комментарий:
                    405: Отсылка к "Поваренной книге анархиста" - пособию по использованию
                    406: обычных исходных материалов, веществ и предметов для изготовления в
                    407: домашних условиях наркотических веществ, оружия, взрывных устройств,
                    408: ядов; буква "А" в круге - символ анархии.
                    409: 
                    410: Источник:
                    411:    1. http://ru.bindingofisaac.wikia.com/wiki/Предметы
                    412:    2. http://ru.wikipedia.org/wiki/Поваренная_книга_анархиста
                    413: 
                    414: Автор:
                    415: Денис Овчинников
                    416: 
                    417: Вопрос 17:
                    418: Бернард Шоу сравнил ЕЕ со спасательным кругом, который "бросают пловцу,
                    419: когда тот уже благополучно достиг берега". Совпадение или нет, но,
                    420: начиная с 2010 года, Малую ЕЕ можно "наблюдать" в декабре в одной из
                    421: северных республик РФ. Назовите ЕЕ двумя словами.
                    422: 
                    423: Ответ:
                    424: Нобелевская премия.
                    425: 
                    426: Комментарий:
                    427: Компания "Нобель Ойл" учредила в республике Коми "Малую Нобелевскую
                    428: премию" для поддержки учащихся и учителей. Процедура награждения
                    429: Нобелевской премией происходит ежегодно, 10 декабря, в столицах двух
                    430: стран - Швеции и Норвегии. Ну и как всем известно, Бернард Шоу получил
                    431: свою Нобелевку по литературе в 1925 году.
                    432: 
                    433: Источник:
                    434:    1. http://ru.wikipedia.org/wiki/Нобелевская_премия
                    435:    2. http://www.komi-nobel.ru
                    436: 
                    437: Автор:
                    438: Иван Нефёдов
                    439: 
                    440: Вопрос 18:
                    441: В романе Александра Иличевского подросток мечтательно описывает тюльпан
                    442: Эйхлера - персидский экземпляр, с ослепительно черным зеркалом, за
                    443: луковичку которого в Голландии перед восстанием гёзов могли расплатиться
                    444: каретой с лошадьми. Назовите цвет тюльпана абсолютно точно.
                    445: 
                    446: Ответ:
                    447: Аленький.
                    448: 
                    449: Зачет:
                    450: Алый. Незачет: Другие оттенки красного и сам красный.
                    451: 
                    452: Комментарий:
                    453: Вот такая аллюзия на известную сказку, богатый купец в которой
                    454: отправлялся "по торговым делам за море". А младшая дочь, видимо, была не
                    455: только самая любимая, но и самая прозорливая. В сказке "Красавица и
                    456: чудовище" в качестве "аленького цветка" была роза.
                    457: 
                    458: Источник:
                    459: Александр Иличевский. Перс. - М.: АСТ: Астрель, 2010. - С. 324.
                    460: 
                    461: Автор:
                    462: Иван Нефёдов
                    463: 
                    464: Вопрос 19:
1.9       rubashki  465: В компьютерной игре "The binding of Isaac" [зэ бАйндинг оф Айзэк]
1.1       rubashki  466: ("Жертвоприношение Исаака") некоторые артефакты являются реально
                    467: существующими предметами, например, Библия, карты Таро или Спиритическая
                    468: доска. Один из артефактов представляет собой ключ, похожий на ИКС, и
                    469: дает возможность открыть почти все двери. Назовите ИКС.
                    470: 
                    471: Ответ:
                    472: Скелет.
                    473: 
                    474: Комментарий:
                    475: Словосочетание "Skeleton Key" (он же "passingkey") обозначает
                    476: специальный ключ, сделанный таким образом, чтобы открывать все замки в
                    477: определенном месте, например, в гостиницах. Данное словосочетание можно
                    478: перевести дословно как "ключ-скелет".
                    479: 
                    480: Источник:
                    481:    1. http://ru.bindingofisaac.wikia.com/wiki/Предметы
                    482:    2. http://en.wikipedia.org/wiki/Skeleton_key
                    483: 
                    484: Автор:
                    485: Денис Овчинников
                    486: 
                    487: Вопрос 20:
                    488: [Ведущему: кавычки в вопросе не озвучивать.]
                    489:    Отмечая новаторский подход режиссера фильма "Добро пожаловать в
                    490: Зомбиленд", Кирилл Андреев парадоксально замечает, что кинолента придает
                    491: АЛЬФУ "ИКСАМ". "АЛЬФА" - произведение средневекового автора, а "ИКСЫ"
                    492: появились на следующий год после выхода упомянутого фильма. Какие четыре
                    493: слова мы заменили на АЛЬФУ ИКСОВ?
                    494: 
                    495: Ответ:
                    496: Новая жизнь ходячих мертвецов.
                    497: 
                    498: Комментарий:
                    499: Новая жизнь (итал. La Vita Nuova) - сборник произведений, написанных
                    500: Данте Алигьери в 1283-1293 гг. Фильм "Добро пожаловать в зомбиленд"
                    501: вышел на экраны в октябре 2009 года и первоначально планировался как
                    502: пилотный эпизод сериала. Через год в октябре 2010 года состоялась
                    503: премьера сериала "Ходячие мертвецы". "Новаторский" - подсказка.
                    504: 
                    505: Источник:
                    506:    1. http://www.film.ru/articles/pod-druzhnoe-rychanie
                    507:    2. http://www.kinopoisk.ru/film/427122/
                    508:    3. http://en.wikipedia.org/wiki/The_Walking_Dead_(TV_series)
                    509: 
                    510: Автор:
                    511: Александр Усачёв
                    512: 
                    513: Вопрос 21:
                    514: Комментируя успешный пуск ракеты с атомной субмарины Б-138 "Обнинск"
                    515: проекта 671РТМК, посетитель сайта topwar.ru [топвар точка ру] упомянул
                    516: ПЕРВУЮ и ВТОРОГО. Догадавшись, кто такая ПЕРВАЯ, ответьте, кем был
                    517: ВТОРОЙ в произведении 1884 года.
                    518: 
                    519: Ответ:
                    520: Идеалистом.
                    521: 
                    522: Комментарий:
                    523: Кодовое название подлодок проекта 671РТМК - "Щука". Выражение - "щука в
                    524: воде, чтобы карась не дремал". В 1884 году был опубликован рассказ
                    525: Салтыкова-Щедрина "Карась-идеалист".
                    526: 
                    527: Источник:
                    528:    1. http://topwar.ru/64259-s-apl-obninsk-proizveden-uspeshnyy-pusk-krylatoy-rakety-iz-podvodnogo-polozheniya.html
                    529:    2. http://dic.academic.ru/dic.nsf/michelson_old/65/
                    530:    3. http://az.lib.ru/s/saltykow_m_e/text_0480.shtml
                    531: 
                    532: Автор:
                    533: Александр Усачёв
                    534: 
                    535: Вопрос 22:
                    536: Тайфун "Хагупит", обрушившийся на Филиппины в начале декабря 2014 года,
                    537: принес множество разрушений и стал серьезным испытанием для всех жителей
                    538: островного государства. Особенно страшными выдались первые удары стихии,
                    539: пришедшиеся на города в восточной части страны. Многие филиппинцы
                    540: получили тяжелые ранения, были и погибшие. Одно из значений слова
                    541: "хагупит" на тагальском языке - ОН. Назовите ЕГО.
                    542: 
                    543: Ответ:
                    544: Кнут.
                    545: 
                    546: Зачет:
                    547: Хлыст. Незачет: Плеть (слово женского рода).
                    548: 
                    549: Комментарий:
                    550: "Обрушившийся", "удары", "испытание" - прямые отсылки к наказанию или
                    551: пытке кнутом. Тяжелые ранения и последующая смерть были обычным делом
                    552: при этом. Также некоторые считают, что первые удары были наиболее
                    553: страшными и тяжелыми, так как кнут был еще твердым и не размокшим.
                    554: 
                    555: Источник:
                    556:    1. http://ru.euronews.com/2014/12/07/more-than-a-million-take-shelter-as-typhoon-hagupit-hits-the-philippines
                    557:    2. http://www.rg.ru/2014/12/08/taifun-site.html
                    558: 
                    559: Автор:
                    560: Александр Усачёв
                    561: 
                    562: Вопрос 23:
                    563: Автор статьи на сайте "РБК", комментируя большое количество косвенных
                    564: аргументов в пользу возвращения на пост главного тренера "Челси" летом
                    565: 2013 года Жозе Моуринью, имеющего действующий контракт с мадридским
                    566: "Реалом", называет эту ситуацию "защитой Чубакки". В одном из разделов
                    567: Википедии "защита Чубакки" стоит между "Святой Русью" и "серым
                    568: кардиналом". Что мы заменили словами "защита Чубакки"?
                    569: 
                    570: Ответ:
                    571: Секрет Полишинеля.
                    572: 
                    573: Комментарий:
                    574: Это всё фразеологизмы, а ситуация с Моуринью превратилась к концу мая в
                    575: "секрет Полишинеля".
                    576: 
                    577: Источник:
                    578:    1. http://sport.rbc.ru/football/newsline/17/05/2013/391651.shtml
                    579:    2. http://ru.wikipedia.org/wiki/Секрет_Полишинеля
                    580: 
                    581: Автор:
                    582: Иван Нефёдов
                    583: 
                    584: Вопрос 24:
                    585: Наблюдая, как часть солнца начинает пропадать во время затмения, а рядом
1.22    ! rubashki  586: сидящая девушка переписывается на онлайн-форуме, герой романа ХалЕда
        !           587: ХоссейнИ сравнивает солнце с НИМ. Назовите ЕГО двумя словами.
1.1       rubashki  588: 
                    589: Ответ:
                    590: Логотип "Apple".
                    591: 
                    592: Комментарий:
                    593: "От солнца с северного бока уже откушен ломтик, и теперь оно похоже на
                    594: логотип "Эппл" на ноутбуке Талии, - она время от времени рапортует на
                    595: какой-то онлайн-форум".
                    596: 
                    597: Источник:
                    598: Халед Хоссейни. И эхо летит по горам. - М.: Фантом Пресс, 2013. - С.
                    599: 370.
                    600: 
                    601: Автор:
                    602: Иван Нефёдов
                    603: 
                    604: Тур:
                    605: 4 тур. "IDDQD-UNR.I.P" (Москва - Раменское)
                    606: 
                    607: Вопрос 1:
                    608: Говоря об одной из созданных им телепередач, Сергей Супонев упомянул
                    609: "самый трудный возраст", а также продолжительный хронометраж каждого из
                    610: ее выпусков. Напишите название этой передачи, которое, по одной из
                    611: версий, появилось благодаря популярной головоломке.
                    612: 
                    613: Ответ:
                    614: "Марафон-15".
                    615: 
                    616: Комментарий:
                    617: По словам Супонева, "15 - самый трудный возраст", а 15 сюжетов в каждой
                    618: из программ - "это марафон". "Марафон-15" - телепередача для подростков,
                    619: выходившая в эфир с 1989 по 1998 год, каждый выпуск которой состоял из
                    620: 15 отдельных сюжетов. Пятнашки, или "Игра в 15", - головоломка,
                    621: связанная с выстраиванием последовательности из пятнадцати костяшек.
                    622: 
                    623: Источник:
                    624:    1. http://ru.wikipedia.org/wiki/Марафон-15
                    625:    2. http://ru.wikipedia.org/wiki/Пятнашки
                    626: 
                    627: Автор:
                    628: Виктор Бойков (Раменское)
                    629: 
                    630: Вопрос 2:
                    631: В пародийном фильме один герой обещает раздавить другого, а затем
                    632: обвести мелком. Какие два слова мы пропустили в предыдущем предложении?
                    633: 
                    634: Ответ:
                    635: Против тараканов.
                    636: 
                    637: Зачет:
                    638: От тараканов; для тараканов; как таракана.
                    639: 
                    640: Комментарий:
                    641: В ходе расследований криминалисты для большей наглядности обводят трупы
                    642: мелом. "Мелок от тараканов" - популярное средство борьбы с данным видом
                    643: насекомых, на которых, кстати, намекает прозвучавшее в вопросе слово
                    644: "раздавить". Авторская группа надеется, что по итогам тура ее не
                    645: постигнет участь одного из героев данного вопроса.
                    646: 
                    647: Источник:
                    648:    1. http://www.abur.tv/news/prigodnick/508204-prosto-gavrilo-i-kubok-mogila-2013.html
                    649: (2:07:25)
                    650:    2. http://www.sanobrabotka.com/blog/2013/01/16/preparaty-protiv-tarakanov/
                    651: 
                    652: Автор:
                    653: Виктор Бойков (Раменское)
                    654: 
                    655: Вопрос 3:
                    656: Юрий Денисов, говоря об известных исследователях, приходит к выводу, что
                    657: их впечатления от объекта соперничества в сравнении оказались ТАКИМИ.
                    658: Автор этого вопроса считает, что ТАКИМИ можно назвать впечатления
                    659: каждого из этих соперников по отдельности. Какими ТАКИМИ?
                    660: 
                    661: Ответ:
                    662: Полярными.
                    663: 
                    664: Комментарий:
                    665: Речь идет о полярных исследователях Руале Амундсене и Роберте Скотте,
                    666: соперничавших за звание первооткрывателя Южного полюса. Амундсен
                    667: описывает Антарктиду восторженно, Скотт же называет ее "ужасным местом".
                    668: 
                    669: Источник:
                    670:    1. http://db.chgk.info/question/spbvuz07.9/11
                    671:    2. http://slovari.yandex.ru/полярный/правописание/
                    672: 
                    673: Автор:
                    674: Евгений Кравченко (Москва)
                    675: 
                    676: Вопрос 4:
                    677: Внимание, в вопросе есть замены.
                    678:    Рекламный слоган российского туристического агентства гласит:
                    679:    Скуку, сырость, холод не КОРИ -
                    680:    На солнечном КОРИНФЕ отдыхай!
                    681:    Какое слово мы заменили словом "КОРИНФ"?
                    682: 
                    683: Ответ:
                    684: ХайнАнь.
                    685: 
                    686: Комментарий:
                    687: В оригинале стихотворный слоган звучит следующим образом:
                    688:    Скуку, сырость, холод не хай -
                    689:    На солнечном ХайнАне отдыхай!
                    690:    ХайнАнь - популярный курортный остров у побережья КНР. Согласно
                    691: "Толковому словарю живого великорусского языка" В.И. Даля, одним из
                    692: синонимов глагола "хаять" является глагол "корить".
                    693: 
                    694: Источник:
                    695:    1. ЛОАВ.
                    696:    2. http://dic.academic.ru/dic.nsf/enc2p/372394/
                    697: 
                    698: Автор:
                    699: Виктор Бойков (Раменское)
                    700: 
                    701: Вопрос 5:
                    702: (pic: 20150138.jpg)
                    703:    На картине Александра Бейдемана Эскулап негодующе машет рукой на
                    704: профессоров медицины, суетящихся вокруг пациента. Назовите фамилию
                    705: человека, которого Бейдеман изобразил на картине справа, в образе
                    706: печального старца.
                    707: 
                    708: Ответ:
                    709: Ганеман.
                    710: 
                    711: Комментарий:
                    712: Полотно Бейдемана, написанное в 1857 году, носит название "Гомеопатия,
                    713: взирающая на ужасы аллопатии". Немецкий ученый Христиан Фридрих Самуэль
                    714: Ганеман - основоположник гомеопатии (вида альтернативной медицины,
                    715: предполагающей использование сильно разведенных препаратов). Аллопатия -
                    716: термин, предложенный сторонниками гомеопатии и других альтернативных
                    717: направлений медицины для обозначения фармакотерапии и других методов
                    718: общепринятой медицины.
                    719: 
                    720: Источник:
                    721:    1. http://omeopathy.livejournal.com/19743.html
                    722:    2. http://ru.wikipedia.org/wiki/Гомеопатия
                    723:    3. http://ru.wikipedia.org/wiki/Аллопатия
                    724: 
                    725: Автор:
                    726: Евгений Кравченко (Москва)
                    727: 
                    728: Вопрос 6:
                    729: [Ведущему: максимально четко прочитать словосочетание "ФЕДЕРАЛЬНЫЙ КУБОК
                    730: ЯРМАРОК".]
                    731:    В мае 2013 года на сайте Lenta.ru [лента точка ру] вышла статья под
                    732: заголовком "ФЕДЕРАЛЬНЫЙ КУБОК ЯРМАРОК". Ответьте двумя словами, что мы
                    733: заменили на "ФЕДЕРАЛЬНЫЙ КУБОК ЯРМАРОК".
                    734: 
                    735: Ответ:
                    736: Бундеслига чемпионов.
                    737: 
                    738: Комментарий:
                    739: Статья была посвящена финалу футбольной Лиги чемпионов, в котором
                    740: встречались мюнхенская "Бавария" и дортмундская "Боруссия". Оба этих
                    741: клуба представляют высшую лигу Чемпионата Германии по футболу, которая
                    742: официально именуется Бундеслигой. Корень "бундес-" переводится с
                    743: немецкого языка как "федеральный". Кубок ярмарок - название футбольного
                    744: турнира - предшественника Кубка УЕФА и Лиги Европы.
                    745: 
                    746: Источник:
                    747:    1. http://lenta.ru/articles/2013/05/02/league/
                    748:    2. http://ru.wikipedia.org/wiki/Кубок_ярмарок
                    749: 
                    750: Автор:
                    751: Виктор Бойков (Раменское)
                    752: 
                    753: Вопрос 7:
                    754: Внимание, в вопросе есть замена.
                    755:    Статья новостного агентства "РИА Новости" о событиях на Корейском
                    756: полуострове была озаглавлена "КНДР ОТЖАЛА на себя санкции". Герой
                    757: произведения 1965 года рассказывает, как ОТЖАЛ, и упоминает два
                    758: числительных. Какие?
                    759: 
                    760: Ответ:
                    761: 10000, 500.
                    762: 
                    763: Зачет:
                    764: Десять тысяч, пятьсот; десять тыщ, пятьсот.
                    765: 
                    766: Комментарий:
                    767: Заголовок новостного материала о взрыве в рамках северокорейских ядерных
                    768: испытаний звучал так: "КНДР РВАНУЛА на себя санкции". Герой написанной в
                    769: 1965 году Владимиром Высоцким "Песни про конькобежца на короткие
                    770: дистанции..." признавался: "... Я на десять тыщ РВАНУЛ, как на пятьсот,
                    771: - и спекся". Жим, как и рывок, - одно из упражнений тяжелой атлетики.
                    772: 
                    773: Источник:
                    774:    1. http://ria.ru/world/20130212/922452250.html
                    775:    2. http://www.kulichki.com/vv/pesni/desyat-tysyach-i-vsego.html
                    776:    3. http://ru.wikipedia.org/wiki/Жим_над_головой
                    777: 
                    778: Автор:
                    779: Виктор Бойков (Раменское)
                    780: 
                    781: Вопрос 8:
                    782: В англоязычных странах так изначально называли изображения на
                    783: геральдических щитах и церковные хоругви. В современной жизни мы чаще
                    784: всего сталкиваемся с НИМИ в Интернете или на спортивных мероприятиях, да
                    785: и просто на улицах. Назовите ИХ.
                    786: 
                    787: Ответ:
                    788: Баннеры.
                    789: 
                    790: Источник:
                    791:    1. http://en.wikipedia.org/wiki/Banner
                    792:    2. http://ru.wikipedia.org/wiki/Баннер
                    793: 
                    794: Автор:
                    795: Евгений Кравченко (Москва)
                    796: 
                    797: Вопрос 9:
                    798: Журнал "Time" [тайм] отмечает, что некая шиитская традиция сыграла
                    799: ключевую роль в событиях 1978 года в Иране, когда волны протестов,
                    800: захлестнувшие страну, переросли в исламскую революцию. Какое число
                    801: "Time" [тайм] упоминает в связи с этой традицией?
                    802: 
                    803: Ответ:
                    804: 40.
                    805: 
                    806: Комментарий:
                    807: По шиитской традиции, поминальные службы о погибшем тоже идут 40 дней.
                    808: Соответственно, волнения в Иране проходили с явным интервалом в 40 дней,
                    809: ширясь и ширясь за счет новых смертей. И вопрос девятый, что говорит еще
                    810: об одной традиционной поминальной дате.
                    811: 
                    812: Источник:
                    813:    1. http://www.time.com/time/world/article/0,8599,1906049,00.html
                    814:    2. http://ru.wikipedia.org/wiki/Исламская_революция_в_Иране
                    815: 
                    816: Автор:
                    817: Евгений Кравченко (Москва)
                    818: 
                    819: Вопрос 10:
                    820: Эмблема болгарской партии, представляющей интересы граждан - сторонников
                    821: европейского развития страны, представляет собой ряд звезд, опоясывающих
                    822: слово "ЩИТ". Что мы заменили словом "ЩИТ"?
                    823: 
                    824: Ответ:
                    825: ГЕРБ.
                    826: 
                    827: Комментарий:
                    828: На эмблеме партии "Граждане за европейское развитие Болгарии" звезды,
                    829: символизирующие Евросоюз, опоясывают аббревиатуру названия партии,
                    830: составляющую слово "ГЕРБ". В геральдике гербы часто имеют форму щита.
                    831: 
                    832: Источник:
                    833: http://www.gerb.bg/
                    834: 
                    835: Автор:
                    836: Сергей Филин (Красноармейск)
                    837: 
                    838: Вопрос 11:
                    839: В сатирической песне, посвященной коррупции в системе МВД, автор сетует,
                    840: что с "оборотнями в погонах" нельзя СДЕЛАТЬ ЭТО, т.к. взяточники ДЕЛАЮТ
                    841: ТО. Какие три слова мы заменили словами "СДЕЛАТЬ ЭТО", а какие три -
                    842: словами "ДЕЛАТЬ ТО"?
                    843: 
                    844: Ответ:
                    845: Поймать за руку, брать на лапу.
                    846: 
                    847: Зачет:
                    848: Ловить за руку, взять на лапу.
                    849: 
                    850: Комментарий:
                    851: "Поймать за руку" - устойчивое выражение, означающее "уличить кого-либо
                    852: или поймать с поличным". "Брать на лапу" - также устойчивое выражение,
                    853: означающее "получить взятку". Действительно, сложно поймать за руку
                    854: того, кто берет на лапу. Упоминание в вопросе "оборотней" намекает на
                    855: периодическое превращение рук в лапы и наоборот.
                    856: 
                    857: Источник:
                    858:    1. http://music.yandex.ru/#!/track/4230086/album/479578
                    859:    2. http://ru.wikipedia.org/wiki/Оборотень
                    860:    3. http://phraseology.academic.ru/6218/
                    861:    4. http://phrase_dictionary.academic.ru/65/
                    862: 
                    863: Автор:
                    864: Виктор Бойков (Раменское)
                    865: 
                    866: Вопрос 12:
                    867: Рассуждая о роли Твиттера и Фейсбука в волнениях на территории
                    868: североафриканских государств, Евгений Козловский переосмысливает
                    869: известные пушкинские строки. Напишите название стихотворения, из
                    870: которого они взяты.
                    871: 
                    872: Ответ:
                    873: "Утопленник".
                    874: 
                    875: Комментарий:
                    876: Twitter и Facebook - современные социальные сети. Говоря о погибших в
                    877: народных волнениях на территории Магриба, автор вспоминает фразу "Тятя,
                    878: тятя, наши сети притащили мертвеца" из стихотворения А.С. Пушкина
                    879: "Утопленник" (1825).
                    880: 
                    881: Источник:
                    882:    1. http://ria.ru/authors/20110301/340574046.html
                    883:    2. http://ru.wikisource.org/wiki/Утопленник_(Пушкин)
                    884: 
                    885: Автор:
                    886: Евгений Кравченко (Москва)
                    887: 
                    888: Вопрос 13:
                    889: Согласно современной шутке, "настоящая женщина должна родить сына,
                    890: воспитать мужа и построить МОНСТРА". Какое слово в предыдущем
                    891: предложении мы заменили другим?
                    892: 
                    893: Ответ:
                    894: Свекровь.
                    895: 
                    896: Комментарий:
                    897: Замена является отсылкой к русскоязычному варианту названия известной
                    898: голливудской комедии "Если свекровь - монстр".
                    899: 
                    900: Источник:
                    901:    1. http://www.inpearls.ru/comments/329985
                    902:    2. http://ru.wikipedia.org/wiki/Если_свекровь_%E2%80%94_монстр
                    903: 
                    904: Автор:
                    905: Виктор Бойков (Раменское)
                    906: 
                    907: Вопрос 14:
                    908: (pic: 20150139.jpg)
                    909:    Блиц.
                    910:    Перед вами шуточные валентинки американского художника Бена Клинга.
                    911:    1. Какие четыре буквы мы закрыли от вас на первой картинке?
                    912:    2. Какое слово мы закрыли от вас на второй картинке?
                    913:    3. Какое слово испанского происхождения мы закрыли от вас на третьей
                    914: картинке?
                    915: 
                    916: Ответ:
                    917:    1. Майн.
                    918:    2. Председатель.
                    919:    3. Эмбарго.
                    920: 
                    921: Зачет:
                    922:    1. Mein.
                    923:    2. Chairman.
                    924:    3. Embargo.
                    925: 
                    926: Комментарий:
                    927: Первая валентинка обыгрывает название известной книги Адольфа Гитлера
                    928: "Mein Kampf" ("Моя борьба"). Вторая посвящена первому председателю КНР и
                    929: ЦК Коммунистической партии Китая Мао Цзэдуну, также часто именуемому
                    930: "председателем Мао". Третья посвящена экс-лидеру Кубы Фиделю Кастро, чья
                    931: родина с 1961 года находится под торговым эмбарго со стороны США.
                    932: 
                    933: Источник:
                    934:    1. http://www.benkling.com/valentines
                    935:    2. http://ru.wikipedia.org/wiki/Моя_борьба
                    936:    3. http://ru.wikipedia.org/wiki/Мао_Цзэдун
                    937:    4. http://ru.wikipedia.org/wiki/Эмбарго
                    938: 
                    939: Автор:
                    940: Виктор Бойков (Раменское)
                    941: 
                    942: Вопрос 15:
                    943: В известном романе ОНИ "распустились в тумане, как кристаллики соли в
                    944: воде". А по мнению Линды Маккартни, если бы ИМИ оборудовали некоторые
                    945: сооружения, то весь мир отказался бы от мяса. Назовите ИХ двумя словами,
                    946: начинающимися на одну и ту же букву.
                    947: 
                    948: Ответ:
                    949: Стеклянные стены.
                    950: 
                    951: Комментарий:
                    952: В романе-антиутопии Евгения Замятина "Мы" элементом постоянного контроля
                    953: жизни граждан со стороны тоталитарного государства являются стеклянные
                    954: стены жилых домов. Линда Маккартни предлагала оборудовать стеклянными
                    955: стенами бойни.
                    956: 
                    957: Источник:
                    958:    1. http://az.lib.ru/z/zamjatin_e_i/text_0050.shtml
                    959:    2. http://ru.wikipedia.org/wiki/Маккартни,_Линда
                    960: 
                    961: Автор:
                    962: Евгений Кравченко (Москва)
                    963: 
                    964: Вопрос 16:
                    965: В материале на сайте sports.ru [спортс точка ру] журналист пожелал нашим
                    966: прыгунам-синхронистам чисто выполнить свою программу. Пожелания были
                    967: снабжены ремаркой: "Чтобы ПЕРВЫЙ ПРОПУСК зал, важно не ВТОРОЙ ПРОПУСК
                    968: бассейн". Заполните оба пропуска однокоренными словами.
                    969: 
                    970: Ответ:
                    971: Зарукоплескал, расплескать.
                    972: 
                    973: Зачет:
                    974: Любые подходящие по смыслу формы данных слов.
                    975: 
                    976: Комментарий:
                    977: Качество входа в воду в спортивных прыжках оценивается, в частности, по
                    978: отсутствую брызг.
                    979: 
                    980: Источник:
                    981: http://www.sports.ru/others/london2012/142333658.html
                    982: 
                    983: Автор:
                    984: Евгений Кравченко (Москва)
                    985: 
                    986: Вопрос 17:
                    987: Недалеко от бывшего здания фабрики по производству шляп в
                    988: Сан-Жуан-да-Мадейра расположен памятник "Черные ногти", поставленный в
                    989: честь работниц фабрики. Традиционный местный головной убор "карапУца"
                    990: имеет особенную форму, напоминающую об одном литературном персонаже.
                    991: Назовите этого персонажа.
                    992: 
                    993: Ответ:
                    994: Железный дровосек.
                    995: 
                    996: Комментарий:
                    997: У работниц фабрики из-за особенностей производства ногти с годами
                    998: приобретали черную окраску. Постепенно все жители Сан-Жуана получили в
                    999: Португалии прозвище "черные ногти". КарапУца внешне напоминает воронку -
                   1000: считается, что ее удлиненный хвост позволяет стягивать головной убор, не
                   1001: запачкав его грязными руками.
                   1002: 
                   1003: Источник:
                   1004:    1. http://pt.wikipedia.org/wiki/S%C3%A3o_Jo%C3%A3o_da_Madeira
                   1005:    2. http://www.moda-dic.ru/html/k/karapuca.html
                   1006:    3. ЛОАВ, материалы экскурсионных рассказов.
                   1007:    4. http://ru.wikipedia.org/wiki/Железный_Дровосек
                   1008: 
                   1009: Автор:
                   1010: Евгений Кравченко (Москва)
                   1011: 
                   1012: Вопрос 18:
                   1013: МоасИр БарбОза - выдающийся голкипер футбольного клуба "Васко да Гама" и
                   1014: сборной Бразилии середины XX века. Однако именно его обвиняют в
                   1015: поражении от уругвайцев на домашнем чемпионате мира. Статья о его судьбе
                   1016: называется так же, как музыкальная композиция 1991 года. Воспроизведите
                   1017: это название.
                   1018: 
                   1019: Ответ:
                   1020: "[The] Unforgiven".
                   1021: 
                   1022: Зачет:
                   1023: "Непрощенный".
                   1024: 
                   1025: Комментарий:
                   1026: В Бразилии такое не прощают. Всю последующую жизнь Барбоза ощущал на
                   1027: себе последствия злополучного второго гола в финале ЧМ-1950. "The
                   1028: Unforgiven" - песня американской музыкальной группы "Metallica" с
                   1029: вышедшего в 1991 году альбома "The Black Album".
                   1030: 
                   1031: Источник:
                   1032:    1. http://en.wikipedia.org/wiki/The_Last_Save_of_Moacyr_Barbosa
                   1033:    2. http://ru.wikipedia.org/wiki/Барбоза,_Моасир
                   1034:    3. http://ru.wikipedia.org/wiki/The_Unforgiven
                   1035: 
                   1036: Автор:
                   1037: Евгений Кравченко (Москва)
                   1038: 
                   1039: Вопрос 19:
                   1040: (pic: 20150140.jpg)
                   1041:    Перед вами иллюстрация к статье доктора физико-математических наук
                   1042: Сергея Сухинина, демонстрирующая понятие волнового фронта. Иллюстрация
                   1043: опровергает тезис, выдвинутый в известном анекдоте. На ней мы скрыли от
                   1044: вас некий предмет. Какой?
                   1045: 
                   1046: Ответ:
                   1047: Кирпич.
                   1048: 
                   1049: Комментарий:
                   1050: Анекдот, соответственно, такой:
                   1051:    Создали чукчи научный институт. Просят привезти им железнодорожный
                   1052: состав кирпича. Прислали. Через некоторое время просят привезти еще.
                   1053: Опять прислали. Потом еще. Заинтересовались в Москве: "Что же они там
                   1054: такое строят?". Выслали комиссию проверить строительство. Приезжают.
                   1055: Видят - стоят на берегу моря несколько человек и бросают в воду кирпичи.
                   1056: "Что это вы такое делаете?" - спрашивают их. А чукчи и отвечают: "А вы
                   1057: посмотрите, кирпичи-то квадратные, а круги от них по воде круглые!".
                   1058:    Из рисунка видно, что волны, конечно же, не круглые.
                   1059: 
                   1060: Источник:
                   1061: http://elementy.ru/lib/430289?context=1532471
                   1062: 
                   1063: Автор:
                   1064: Евгений Кравченко (Москва)
                   1065: 
                   1066: Вопрос 20:
                   1067: Статья газеты "Московский комсомолец" о компании "Де Бирс",
                   1068: присматривающейся к некоторым российским месторождениям, носит название
                   1069: "У Де Бирс [ПРОПУСК]. На чужие деньги". Заполните пропуск устойчивым
                   1070: словосочетанием.
                   1071: 
                   1072: Ответ:
                   1073: Глаз - алмаз.
                   1074: 
                   1075: Комментарий:
                   1076: "Де Бирс" - мировой монополист алмазной промышленности.
                   1077: 
                   1078: Источник:
                   1079: http://socarchive.narod.ru/digest/pr/archive/0313/20.html
                   1080: 
                   1081: Автор:
                   1082: Виктор Бойков (Раменское)
                   1083: 
                   1084: Вопрос 21:
                   1085:    <раздатка>
                   1086:    Белые овечки, белые овечки на голубом холме,
                   1087:    Когда ветер утихнет, вы смирно стоите,
                   1088:    Когда ветер подует, уходите прочь.
                   1089:    Белые овечки, белые овечки, куда же уходите вы?
                   1090:    </раздатка>
                   1091:    [Ведущему: сделать ярко выраженную паузу между словами "загаданное" и
                   1092: "в песне".]
                   1093:    Мы не спрашиваем, что загадано в розданном вам стихотворении
                   1094: английской поэтессы Кристины Россетти. Ответьте, с чем сравнивалось
                   1095: загаданное в песне на слова советского поэта и переводчика Леонида
                   1096: ЯхнинА.
                   1097: 
                   1098: Ответ:
                   1099: С [белыми] корабликами. Незачет: С [белогривыми] лошадками.
                   1100: 
                   1101: Комментарий:
                   1102: Совпадение или нет, но песня "Белые кораблики" содержит следующие слова:
                   1103: "Белые кораблики, белые кораблики // По небу плывут. // Белые кораблики,
                   1104: белые кораблики // Дождики везут". Последние слова песни: "Белые
                   1105: кораблики, белые кораблики - // Это облака". Слова "овца" и "корабль"
                   1106: по-английски звучат почти одинаково.
                   1107: 
                   1108: Источник:
                   1109:    1. http://www.poets.org/poetsorg/poem/clouds
                   1110:    2. http://www.a-pesni.org/baby/belkorab.php
                   1111: 
                   1112: Автор:
                   1113: Антон Тилипман (Москва)
                   1114: 
                   1115: Вопрос 22:
                   1116: Добавив в лермонтовскую строчку притяжательное и указательное
                   1117: местоимения, этот бард превратил ямб в амфибрахий. В результате
                   1118: получилась песня, ставшая сквозной темой классической советской комедии.
                   1119: Назовите фамилию барда.
                   1120: 
                   1121: Ответ:
                   1122: Ким.
                   1123: 
                   1124: Комментарий:
                   1125: При написании сквозной музыкальной темы фильма Марка Захарова
                   1126: "Двенадцать стульев" (1975) бард Юлий Ким вставил в строку "Белеет парус
                   1127: одинокий" из стихотворения М.Ю. Лермонтова местоимения "мой" и "такой".
                   1128: 
                   1129: Источник:
                   1130: http://ru.wikipedia.org/wiki/12_стульев_(фильм,_1976)
                   1131: 
                   1132: Автор:
                   1133: Евгений Кравченко (Москва)
                   1134: 
                   1135: Вопрос 23:
                   1136: В этот день 2002 года в США было объявлено о самом крупном за всю
                   1137: историю страны банкротстве. В тот же день восемь лет спустя в Аргентине
                   1138: вступил в силу закон об однополых браках. В точности через год произошли
                   1139: кровавые теракты в Норвегии. День приближения чего отмечается 22 июля?
                   1140: 
                   1141: Ответ:
                   1142: [Число] Пи.
                   1143: 
                   1144: Зачет:
                   1145: [Число] &pi;.
                   1146: 
                   1147: Комментарий:
                   1148: Одной из дат, связанных с числом Пи, является 22 июля, называемое "Днем
                   1149: приближения числа Пи" (англ. Pi Approximation Day). Дело в том, что в
                   1150: европейском формате дат этот день записывается как 22/7, а значение
                   1151: данной дроби является приближенным значением числа Пи с точностью до
                   1152: двух знаков после запятой. А наш тур приближается к логическому
                   1153: завершению.
                   1154: 
                   1155: Источник:
                   1156:    1. http://ru.wikipedia.org/wiki/22_июля
                   1157:    2. http://ru.wikipedia.org/wiki/Пи_(число)
                   1158: 
                   1159: Автор:
                   1160: Антон Тилипман (Москва)
                   1161: 
                   1162: Вопрос 24:
                   1163: Согласно шутке, представители некоторой профессии могут устроить КРАСНУЮ
                   1164: ЖАРУ, если их не поздравить с этим праздником. Поэтому с одним из
                   1165: последних таких праздников этих людей поздравляли особенно тепло. Что мы
                   1166: заменили словами "КРАСНАЯ ЖАРА"?
                   1167: 
                   1168: Ответ:
                   1169: Конец света.
                   1170: 
                   1171: Комментарий:
                   1172: Широко известный благодаря масс-медиа потенциальный "конец света",
                   1173: ожидавшийся 21 декабря 2012 года, по случайности совпал с днем
                   1174: энергетика. "Красная жара" (1988) и "Конец света" (1999) - фильмы с
                   1175: участием известного американского актера Арнольда Шварценеггера. На этом
                   1176: наш тур завершается.
                   1177: 
                   1178: Источник:
                   1179:    1. http://ru.wikipedia.org/wiki/День_энергетика
                   1180:    2. http://ru.wikipedia.org/wiki/Конец_света_(2012)
                   1181:    3. http://ru.wikipedia.org/wiki/Шварценеггер,_Арнольд
                   1182: 
                   1183: Автор:
                   1184: Евгений Кравченко (Москва)
                   1185: 
                   1186: Тур:
                   1187: 5 тур. "Канатчикова дача" (Москва)
                   1188: 
                   1189: Вопрос 1:
                   1190: Внимание, в вопросе есть замены.
                   1191:    а где тут руль промолвил кедр
                   1192:    деревня хмыкнула заря
                   1193:    еще спроси а где тут вожжи
                   1194:    еще ПРОПУСК скажи
                   1195:    Догадавшись, какие слова мы заменили, заполните пропуск.
                   1196: 
                   1197: Ответ:
                   1198: Поехали.
                   1199: 
                   1200: Комментарий:
                   1201: Во время первого полета в космос позывным Гагарина был Кедр, Королёва -
                   1202: Заря-1.
                   1203: 
                   1204: Источник:
                   1205:    1. http://www.perashki.ru/piro/29313/
                   1206:    2. http://www.cosmoworld.ru/spaceencyclopedia/gagarin/index.shtml?doc10.html
                   1207: 
                   1208: Вопрос 2:
                   1209: В конце XIX века ОН совершил дерзкое убийство, всколыхнувшее Англию и
                   1210: вызвавшее небывалый переполох в определенных кругах. Все хорошо знали
                   1211: виновника, но его невозможно было наказать. Существует легенда, что сама
                   1212: королева Виктория была настолько возмущена его поступком, что вынудила
                   1213: вышеупомянутого джентльмена добровольно искупить свою вину. От вас же
                   1214: требуется всего лишь назвать его имя - ведь это так просто!
                   1215: 
                   1216: Ответ:
                   1217: Артур.
                   1218: 
                   1219: Комментарий:
                   1220: Речь, конечно же, идет об Артуре Конан Дойле и об убийстве Шерлока
                   1221: Холмса в схватке с профессором Мориарти.
                   1222: 
                   1223: Источник:
                   1224: http://ru.wikipedia.org/wiki/Шерлок_Холмс
                   1225: 
                   1226: Вопрос 3:
                   1227: В середине 40-х годов прошлого века в Америке были запатентованы
                   1228: необычные шахматы с фигурами в виде танков, самолетов и прочего. В этих
                   1229: шахматах пешка, доходя до последней линии, становилась АЛЬФОЙ. Назовите
                   1230: АЛЬФУ двумя словами.
                   1231: 
                   1232: Ответ:
                   1233: Атомная бомба.
                   1234: 
                   1235: Источник:
                   1236: Е. Гижицкий. С шахматами через века и страны. - Warszawa: Sport I
                   1237: Turystyka, 1970. - С. 75.
                   1238: 
                   1239: Вопрос 4:
                   1240: В средневековой Японии гейша не имела права вступать в брак. Поэтому
                   1241: вместо церемонии бракосочетания была принята другая процедура, во время
                   1242: которой влюбленные обменивались ИКСАМИ вместо колец. Назовите комедию
                   1243: 1995 года, в которой ИКС появился в пентхаусе.
                   1244: 
                   1245: Ответ:
                   1246: "Четыре комнаты".
                   1247: 
                   1248: Комментарий:
                   1249: ИКС - отрубленный палец.
                   1250: 
                   1251: Источник:
                   1252:    1. А.Н. Мещеряков. Книга японских символов. Книга японских
                   1253: обыкновений. - М.: Наталис, 2004. - С. 510.
                   1254:    2. http://ru.wikipedia.org/wiki/Четыре_комнаты
                   1255: 
                   1256: Вопрос 5:
                   1257: Согласно одной кхмерской легенде, раньше ОНИ были огромного роста, даже
                   1258: больше грифов, у них были огромные пасти, отличались они необыкновенной
                   1259: прожорливостью и питались исключительно людьми. Чтобы не дрожать всю
                   1260: жизнь в ожидании неминуемой смерти, люди бросали жребий, кому быть
                   1261: съеденным ИМИ в следующую ночь. Назовите ИХ.
                   1262: 
                   1263: Ответ:
                   1264: Комары.
                   1265: 
                   1266: Источник:
                   1267: http://www.bibliotekar.ru/azia/71.htm
                   1268: 
                   1269: Вопрос 6:
                   1270: "На берегу гладкого озера выросли кривые вербы", - говорят калмыки про
                   1271: АЛЬФЫ. А одна группа "родственников" всерьез уверена, что с помощью АЛЬФ
                   1272: можно летать. Назовите этих "родственников".
                   1273: 
                   1274: Ответ:
                   1275: "Братья Гримм".
                   1276: 
                   1277: Комментарий:
                   1278: АЛЬФЫ - ресницы.
                   1279: 
                   1280: Источник:
                   1281:    1. http://ru.wikiquote.org/wiki/Калмыцкие_загадки
                   1282:    2. http://megalyrics.ru/lyric/bratia-grimm/riesnitsy.htm
                   1283: 
                   1284: Вопрос 7:
                   1285: Этот топоним любят заимствовать для названия чего-нибудь грандиозного.
                   1286: Например, так называется один из популярных сервисов для автоматического
                   1287: перевода. Чаще всего это название встречается в единственном числе,
                   1288: однако интернет-словарь редких и забытых слов приводит определение для
                   1289: множественного числа данного слова: ОНИ - это "извилины, кривые или
                   1290: ломаные линии, вычурный узор". Почти наверняка вы хорошо ИХ помните
                   1291: благодаря советской кинокомедии 1964 года. А где ОНИ были в этом фильме?
                   1292: 
                   1293: Ответ:
                   1294: На голове.
                   1295: 
                   1296: Комментарий:
                   1297: ОНИ - это вавилоны, а вы наверняка помните фразу из кинофильма "Добро
                   1298: пожаловать, или Посторонним вход воспрещен!": "А Митрофанова вавилоны на
                   1299: голове крутит!". Ну а программа, как вы наверняка знаете, носит название
                   1300: "Babylon".
                   1301: 
                   1302: Источник:
                   1303:    1. http://www.babylon.com/
                   1304:    2. http://www.zabytye-slova.ru/vavilony/
                   1305:    3. http://frazochka.ru/authors/6703.html
                   1306: 
                   1307: Вопрос 8:
                   1308: (pic: 20150141.jpg)
                   1309:    Поисково-прицельная система "Новелла-П-38", устанавливаемая при
                   1310: модернизации самолетов Ил-38, получила среди авиаторов прозвище "АЛЬФА".
                   1311: В статье Википедии, посвященной АЛЬФЕ, упоминаются республики Поволжья и
                   1312: Крым. Назовите АЛЬФУ.
                   1313: 
                   1314: Ответ:
                   1315: Тюбетейка.
                   1316: 
                   1317: Источник:
                   1318:    1. http://www.sdelanounas.ru/blogs/54229/
                   1319:    2. http://ru.wikipedia.org/wiki/Тюбетейка
                   1320: 
                   1321: Вопрос 9:
                   1322: Во время президентства Джорджа Буша-старшего его советником по
                   1323: национальной безопасности был генерал-лейтенант Брент Скоукрофт, который
                   1324: отличался тем, что часто ДЕЛАЛ ЭТО. Буш даже учредил специальную премию
                   1325: в его честь и сам лично выбирал победителей. В России, если верить
                   1326: известному выражению, в среде "младших коллег" Скоукрофта как раз в
                   1327: обычае ДЕЛАТЬ ЭТО, так как их работе это никак не мешает. Что мы
                   1328: заменили на "ДЕЛАТЬ ЭТО"?
                   1329: 
                   1330: Ответ:
                   1331: Спать на совещаниях.
                   1332: 
                   1333: Зачет:
                   1334: Спать на работе.
                   1335: 
                   1336: Комментарий:
                   1337: Премией Скоукрофта награждались те, кто уснул во время встречи с
                   1338: президентом, причем тот оценивал кандидатов по трем показателям -
                   1339: продолжительности и глубине сна, а также храпу. Известная русская
                   1340: поговорка - "Солдат спит - служба идет".
                   1341: 
                   1342: Источник:
                   1343: http://ru.wikipedia.org/wiki/Скоукрофт,_Брент
                   1344: 
                   1345: Вопрос 10:
                   1346:    <раздатка>
                   1347:    В 2009 году 67-летний Стивен Кинг был удостоен одной из двух высших
                   1348: наград США для гражданских лиц - президентской медали свободы, которой
                   1349: награждаются люди, "внесшие существенный вклад в безопасность и защиту
                   1350: национальных интересов США, в поддержание мира во всём мире, а также в
                   1351: общественную и культурную жизнь США и мира.
                   1352:    </раздатка>
                   1353:    Какие две буквы мы опустили в раздаче?
                   1354: 
                   1355: Ответ:
                   1356: Хо.
                   1357: 
                   1358: Комментарий:
                   1359: Стивену Кингу на данный момент всё еще 67 лет, а медаль получил Стивен
                   1360: Хокинг.
                   1361: 
                   1362: Источник:
                   1363:    1. http://ru.wikipedia.org/wiki/Хокинг,_Стивен_Уильям
                   1364:    2. http://ru.wikipedia.org/wiki/Кинг,_Стивен
                   1365: 
                   1366: Вопрос 11:
                   1367: Иван Дмитриевич Папанин в своем дневнике "Жизнь на льдине" писал: "От
                   1368: лагеря до лебедки, на расстоянии одного километра, мы протянули веревку,
                   1369: чтобы, в случае сильной пурги, можно было двигаться, держась за нее и не
                   1370: рискуя заблудиться". Ответьте двумя словами, как назвали папанинцы "это
                   1371: новое веревочное сооружение".
                   1372: 
                   1373: Ответ:
                   1374: Троллейбусная линия.
                   1375: 
                   1376: Комментарий:
                   1377: Номер вопроса напоминает троллейбусные штанги.
                   1378: 
                   1379: Источник:
                   1380: И.Д. Папанин. Жизнь на льдине. Дневник. - 2-е издание. - М.:
                   1381: Художественная литература, 1940. - С. 230.
                   1382: 
                   1383: Вопрос 12:
                   1384: Бернард Шоу как-то сказал: "Я бы променял все картины с изображением
                   1385: Христа на одну АЛЬФУ". По мнению физика Джона Джексона, такой АЛЬФОЙ
                   1386: является ОНА. Назовите ЕЕ абсолютно точно.
                   1387: 
                   1388: Ответ:
                   1389: Туринская плащаница.
                   1390: 
                   1391: Комментарий:
                   1392: АЛЬФА - это фотография. А Джон Джексон возглавляет центр по изучению
                   1393: Туринской плащаницы.
                   1394: 
                   1395: Источник:
                   1396:    1. http://www.photoquotations.com/a/614/George+Bernard+Shaw
                   1397:    2. http://www.youtube.com/watch?v=1c3rJKt8wjA
                   1398: 
                   1399: Вопрос 13:
                   1400: В 1855 году в газетах писали об одном из экспонатов Парижской выставки:
                   1401: ОН был впервые "показан публично, и при этом стало ясно, что бОльшую
                   1402: часть того, что о нем говорилось, следует приписать фантазии и
                   1403: легковерности широкой публики. Вместо ожидаемых гор ЕГО там оказалось
                   1404: всего 12 маленьких брусочков, общим весом около килограмма, что,
                   1405: конечно, немного для открытия, которое казалось способным перевернуть
                   1406: мир". Назовите ЕГО.
                   1407: 
                   1408: Ответ:
                   1409: Алюминий.
                   1410: 
                   1411: Комментарий:
                   1412: Алюминий - 13-й элемент периодической системы, на что, надеемся,
                   1413: намекает номер вопроса. И хотя алюминий впервые был получен в 1825 году,
                   1414: только в 1854 году Анри Сент-Клер Девиль изобрел первый способ его
                   1415: промышленного производства.
                   1416: 
                   1417: Источник:
                   1418:    1. http://www.ng.ru/koncep/2008-02-28/9_element.html
                   1419:    2. http://ru.wikipedia.org/wiki/Алюминий
                   1420: 
                   1421: Вопрос 14:
1.7       rubashki 1422: Фанаты общественного транспорта называют ИМ автобус ЛиАЗ-677. В 1993
1.1       rubashki 1423: году на аукционе Sotheby's ОН был продан за 68.500 долларов Ричарду
                   1424: Гэрриотту. Назовите ЕГО сложносоставным словом.
                   1425: 
                   1426: Ответ:
                   1427: Луноход.
                   1428: 
                   1429: Комментарий:
1.7       rubashki 1430: В 1993 году на аукционе Sotheby's, посвященном космической тематике,
1.1       rubashki 1431: "Луноход-2" был продан за 68.500 долларов Ричарду Гэрриотту,
                   1432: американскому коллекционеру космических артефактов и сыну астронавта
                   1433: NASA.
                   1434: 
                   1435: Источник:
                   1436:    1. http://lurkmore.to/ЛиАЗ-677
1.7       rubashki 1437:    2. http://ru.wikipedia.org/wiki/Луноход-2
1.1       rubashki 1438: 
                   1439: Вопрос 15:
                   1440: Существует распространенное мнение, что спортсмены не отличаются высоким
                   1441: интеллектом. Например, есть такой анекдот про известного итальянского
                   1442: футболиста Франческо Тотти. В результате пожара в доме Тотти сгорела вся
                   1443: его библиотека. Сам футболист так прокомментировал данный инцидент:
                   1444: "Конечно же, мне очень жаль обе книжки! Тем более, одну из них я даже не
                   1445: успел полностью прочитать...". В комментарии Тотти мы заменили одно
                   1446: слово. Напишите его в первоначальном варианте.
                   1447: 
                   1448: Ответ:
                   1449: Раскрасить.
                   1450: 
                   1451: Комментарий:
                   1452: Это были даже не книжки, а детские раскраски.
                   1453: 
                   1454: Источник:
                   1455: http://francescototti.narod.ru/jokebook.htm
                   1456: 
                   1457: Вопрос 16:
                   1458: На соревнованиях "Бегущий город. Москва. 2011" участникам выдавалась
                   1459: аудиозагадка - по "минусовке" требовалось угадать "Песенку об Арбате"
                   1460: Булата Окуджавы. У некоторых участников это вызывало затруднения, и
                   1461: судьи давали им совет из двух слов, совпадающий со слоганом социальной
                   1462: рекламы. Воспроизведите этот слоган.
                   1463: 
                   1464: Ответ:
                   1465: "Позвоните родителям".
                   1466: 
                   1467: Источник:
                   1468:    1. Личный опыт автора вопроса.
                   1469:    2. http://www.youtube.com/watch?v=SBhuqZnzcwA
                   1470: 
                   1471: Вопрос 17:
                   1472: Когда-то X являлось для американцев символом надежды и защиты, но
                   1473: некоторое время назад X стало вызывать и прямо противоположные
                   1474: ассоциации. Чему же равно X?
                   1475: 
                   1476: Ответ:
                   1477: 911.
                   1478: 
                   1479: Источник:
                   1480:    1. http://ru.wikipedia.org/wiki/Номера_телефонов_экстренных_служб
                   1481:    2. http://ru.wikipedia.org/wiki/Террористические_акты_11_сентября_2001_года
                   1482: 
                   1483: Вопрос 18:
                   1484: Согласно британскому ученому Ричарду Докинзу, ИКСЫ во многом ведут себя
                   1485: подобно генам, хотя и не являются материальными объектами. Несмотря на
                   1486: довольно долгий срок существования этого термина, наибольшее
                   1487: распространение он получил лишь в последнее десятилетие, в чем явная
                   1488: заслуга Интернета. Так что сегодня можно с уверенностью заявить, что
                   1489: число ИКСОВ превышает 9000. Назовите ИКС коротко и ясно.
                   1490: 
                   1491: Ответ:
                   1492: Мем.
                   1493: 
                   1494: Источник:
                   1495:    1. http://ru.wikipedia.org/wiki/Эгоистичный_ген
                   1496:    2. http://lurkmore.to/Более_9000
                   1497: 
                   1498: Вопрос 19:
                   1499: Внимание, в вопросе есть замены.
                   1500:    Рок-группа "Фактор страха" появилась в 1987 году и за свою историю
                   1501: выпустила 13 альбомов. В январе 2009 года появилась шутка, что все
                   1502: последние новости делятся на две темы: фактор страха и фактор без
                   1503: страха. Какие два слова мы заменили в этих предложениях?
                   1504: 
                   1505: Ответ:
                   1506: Сектор, Газа.
                   1507: 
                   1508: Зачет:
                   1509: Сектор, газа.
                   1510: 
                   1511: Комментарий:
                   1512: В январе 2009 года имели место газовый конфликт между Россией и Украиной
                   1513: и операция Вооруженных Сил Израиля в Секторе Газа.
                   1514: 
                   1515: Источник:
                   1516:    1. http://ru.wikipedia.org/wiki/Сектор_Газа_(группа)
                   1517:    2. http://ru.wikipedia.org/wiki/Фактор_Страха_(группа)
                   1518:    3. http://ru.wikipedia.org/wiki/Январь_2009_года
                   1519:    4. http://www.anekdot.ru/id/379690/
                   1520: 
                   1521: Вопрос 20:
                   1522: По словам Нины Ватолиной, "ОН - первый насмешник, первый агитатор,
                   1523: первый сказочник, первый свидетель жизни народной. ОН - демократ по
                   1524: рождению. ОН - для всех". К его помощи прибегали Малевич, Маяковский,
                   1525: Бурлюк. Назовите ЕГО одним словом.
                   1526: 
                   1527: Ответ:
                   1528: Лубок.
                   1529: 
                   1530: Комментарий:
                   1531: В августе 1914 года авангардисты К. Малевич, А. Лентулов, В.В.
                   1532: Маяковский, Д.Д. Бурлюк создали группу "Сегодняшний лубок", возродившую
                   1533: старинные традиции батального лубка XIX века. Эта группа выпустила,
                   1534: используя традицию лубочного примитива, серию из 22 листов на военные
                   1535: сюжеты.
                   1536: 
                   1537: Источник:
                   1538:    1. Н.Н. Ватолина. Пейзажи Москвы. - М.: Советский художник, 1983. -
                   1539: С. 37.
                   1540:    2. http://blog.trud.ru/users/rodich2007/post88950294/
                   1541: 
                   1542: Вопрос 21:
                   1543: Однажды, гуляя поздним вечером по пустынным улицам города Туниса, автор
                   1544: вопроса услышал звуки, доносящиеся из закрытого магазина. Как оказалось,
                   1545: эти звуки были призваны отпугнуть воров. Ответьте абсолютно точно, на
                   1546: кого же надеялся владелец магазина.
                   1547: 
                   1548: Ответ:
                   1549: На Аллаха.
                   1550: 
                   1551: Комментарий:
                   1552: Как известно, большинство населения Туниса исповедуют ислам. Владелец
                   1553: магазина оставил включенным магнитофон с записью чтения Корана, надеясь,
                   1554: что вор, если таковой будет, окажется человеком богобоязненным и,
                   1555: услышав слова священной книги, задумается над своим поступком. А слова
                   1556: "на кого же надеялся" - наводка на пословицу "На Бога надейся, а сам не
                   1557: плошай".
                   1558: 
                   1559: Источник:
                   1560: Личный опыт автора вопроса.
                   1561: 
                   1562: Вопрос 22:
                   1563: В начале 80-х годов XX века автор вопроса занимался фехтованием на
                   1564: саблях. Оружие, защитные маски и куртки предоставлялись в секции, а
                   1565: перчатки приходилось покупать самим. Лучше всего для защиты руки
                   1566: подходили мотоциклетные краги, которые стоили недешево и были дефицитом.
                   1567: В итоге все ими обзавелись, но те, кому меньше повезло, пользовались
                   1568: ТАКИМИ крагами. Некоторые машинки компании "VooV", выпускающиеся по
                   1569: известной франшизе тоже могут быть ТАКИМИ. Какими ТАКИМИ?
                   1570: 
                   1571: Ответ:
                   1572: Вывернутыми наизнанку.
                   1573: 
                   1574: Зачет:
                   1575: По смыслу.
                   1576: 
                   1577: Комментарий:
                   1578: Перчатка нужна одна, а продаются они парами. Покупали одну пару на
                   1579: двоих, и одному из двух правшей приходилось выворачивать левую перчатку
                   1580: наизнанку, чтобы можно было надеть ее на правую руку. Компания "VooV"
                   1581: выпускает машинки-трансформеры, которые при трансформации выворачиваются
                   1582: шиворот-навыворот.
                   1583: 
                   1584: Источник:
                   1585:    1. ЛОАВ.
                   1586:    2. http://milamama.ru/category/voov_2_in_1
                   1587: 
                   1588: Вопрос 23:
                   1589: Первое упоминание об этой организации встречается в именном указе Петра
                   1590: I от 1724 года, где к знакомому нам словосочетанию было добавлено "и
                   1591: курьезных художеств" - ведь в то время помимо прочих премудростей ОНА
                   1592: отвечала за организацию массовых празднеств и фейерверков. Назовите ЕЕ
                   1593: двумя словами.
                   1594: 
                   1595: Ответ:
                   1596: Академия наук.
                   1597: 
                   1598: Источник:
                   1599:    1. И. Словцова. История петербургских районов: Энциклопедия. - М.:
                   1600: Астрель; СПб.: Астрель-СПб, 2012. - С. 69.
                   1601:    2. http://ru.wikipedia.org/wiki/Санкт-Петербургская_академия_наук
                   1602: 
                   1603: Вопрос 24:
                   1604: У некоторых из вас АЛЬФА может ассоциироваться с ударным музыкальным
                   1605: инструментом импортного производства, у других - с однолетним растением
                   1606: семейства тыквенных, обогащенным хлоридом натрия, а у третьих - с полым
                   1607: кондитерским изделием на основе масла какао. Примечательно, что в
                   1608: зависимости от вашей ассоциации можно сделать некоторые выводы о том,
                   1609: где проходило ваше детство. А какие два слова, написанные через дефис,
                   1610: мы заменили на АЛЬФУ?
                   1611: 
                   1612: Ответ:
                   1613: Жадина-говядина.
                   1614: 
                   1615: Зачет:
                   1616: Ябеда-корябеда.
                   1617: 
                   1618: Комментарий:
                   1619: В разных городах дети по-разному продолжают эту дразнилку. В Москве,
                   1620: например, это был "турецкий барабан", в Питере - "пустая шоколадина", в
                   1621: некоторых других городах - "соленый огурец".
                   1622: 
                   1623: Источник:
                   1624: http://forum.lingvo.ru/actualthread.aspx?tid=117253
                   1625: 
                   1626: Тур:
                   1627: 6 тур. "По знакомству" (Москва)
                   1628: 
                   1629: Вопрос 1:
                   1630: (pic: 20150142.jpg)
                   1631:    Первый вопрос команды "По знакомству"!
                   1632:    В этом вопросе словом "ИКС" заменено три других слова.
                   1633:    Перед вами первый ИКС. На одном коллаже нынешний ИКС выложен
                   1634: кокаином. Каким слоганом из трех слов подписан этот коллаж?
                   1635: 
                   1636: Ответ:
                   1637: "Оставайтесь на первом".
                   1638: 
                   1639: Комментарий:
                   1640: Кокаин на жаргоне называют первым, так же как амфетамин называют феном.
                   1641: ИКС - логотип Первого канала.
                   1642: 
                   1643: Источник:
                   1644:    1. http://ru.wikipedia.org/wiki/Первый_канал_(Россия)#.D0.9B.D0.BE.D0.B3.D0.BE.D1.82.D0.B8.D0.BF.D1.8B
                   1645:    2. http://demotivators.to/p/393071/ostavajtes-na-pervom.htm
                   1646: 
                   1647: Автор:
                   1648: Амаль Имангулов (Самара - Москва)
                   1649: 
                   1650: Вопрос 2:
                   1651: Жительница города Химки пострадала в результате пожара, вызванного тем,
                   1652: что она пыталась сжечь фотографии и вещи своего бывшего молодого
                   1653: человека. По выражению сообщества Лентач, девушку убила АЛЬФА.
                   1654: Распространение АЛЬФЫ заметно увеличилось с тех пор, как мексиканских
                   1655: пчел и колибри научились заменять трудом человека. Что мы заменили на
                   1656: АЛЬФУ?
                   1657: 
                   1658: Ответ:
                   1659: Ваниль.
                   1660: 
                   1661: Источник:
                   1662:    1. https://vk.com/oldlentach?w=wall-29534144_1501839
                   1663:    2. http://www.vokrugsveta.ru/vs/article/7546/
                   1664: 
                   1665: Автор:
                   1666: Вера Звягинцева (Москва)
                   1667: 
                   1668: Вопрос 3:
                   1669: Вопреки достаточно распространенному мнению, ОН не является
                   1670: изобретателем МАНСАРДЫ - он лишь способствовал ее распространению,
                   1671: руководствуясь идеями гуманизма и равенства. Согласно афоризму
                   1672: Станислава Ежи Леца, автор весь день не мог вспомнить слово "МАНСАРДА" -
                   1673: голова сопротивлялась. Догадайтесь, что мы заменили на слово "МАНСАРДА",
                   1674: и назовите человека, которого мы заменили на слово "ОН".
                   1675: 
                   1676: Ответ:
                   1677: Гильотен.
                   1678: 
                   1679: Комментарий:
                   1680: Мансарда тоже названа в честь конкретного человека.
                   1681: 
                   1682: Источник:
                   1683:    1. http://www.vokrugsveta.ru/chronograph/2601/
                   1684:    2. http://lib.ru/ANEKDOTY/lec.txt_with-big-pictures.html
                   1685: 
                   1686: Автор:
                   1687: Вера Звягинцева (Москва)
                   1688: 
                   1689: Вопрос 4:
                   1690: Согласно одной из распространенных версий, когда канцлер британского
                   1691: казначейства Уильям Гладстон спросил у Майкла Фарадея, какой толк в
                   1692: открытии электромагнитной индукции, ученый ответил, что когда-нибудь с
                   1693: его открытием смогут СДЕЛАТЬ ЭТО. ЭТО СДЕЛАЛИ с солнцем на Балеарских
                   1694: островах и с гражданским браком в Китае. Что мы заменили на "СДЕЛАТЬ
                   1695: ЭТО"?
                   1696: 
                   1697: Ответ:
                   1698: Обложить налогом.
                   1699: 
                   1700: Зачет:
                   1701: По смыслу.
                   1702: 
                   1703: Источник:
                   1704:    1. У. Гратцер. Эврики и эйфории. Об ученых и их открытиях. - М.:
                   1705: КоЛибри, Азбука-Аттикус, 2011.
                   1706:    2. http://m.forbes.ru/article.php?id=234249
                   1707: 
                   1708: Автор:
                   1709: Вера Звягинцева (Москва)
                   1710: 
                   1711: Вопрос 5:
                   1712: Ученый Джон Рид назвал "невоспетым героем" стереохимии и автором многих
                   1713: открытий ТАКОГО ЕГО - ему ТАКОЙ ОН нечаянно помог изменить ход
                   1714: эксперимента и совершить прорыв в его исследовании. Маловероятно, что
                   1715: Майкл Фарадей был ТАКИМ, однако ИМ он был с 1812 по 1815 год. Какие два
                   1716: слова, начинающиеся на одну и ту же букву, мы заменили на "ТАКОЙ ОН"?
                   1717: 
                   1718: Ответ:
                   1719: Ленивый лаборант.
                   1720: 
                   1721: Источник:
                   1722:    1. У. Гратцер. Эврики и эйфории. Об ученых и их открытиях. - М.:
                   1723: КоЛибри, Азбука-Аттикус, 2011.
                   1724:    2. http://ru.wikipedia.org/wiki/Фарадей,_Майкл
                   1725: 
                   1726: Автор:
                   1727: Вера Звягинцева (Москва)
                   1728: 
                   1729: Вопрос 6:
                   1730: В вопросе словом "ИКС" заменено выражение из двух слов.
                   1731:    "ИКС" - ироничный заголовок статьи, рассказывающей о том, что
                   1732: Владимир Путин принял решение повысить зарплаты чиновникам и
                   1733: представителям силовых структур. Согласно уставу, регулирующему
                   1734: отношения между военными, перед тем как произнести фразу "ИКС",
                   1735: военнослужащий должен обратиться к начальнику "товарищ" и далее назвать
                   1736: его полное воинское звание. Что мы заменили на ИКС?
                   1737: 
                   1738: Ответ:
                   1739: Разрешите доложить.
                   1740: 
                   1741: Источник:
                   1742:    1. http://www.gazeta.ru/politics/2013/10/01_a_5677533.shtml
                   1743:    2. http://www.consultant.ru/document/cons_doc_LAW_165097/?frame=1
                   1744: 
                   1745: Автор:
                   1746: Вера Звягинцева (Москва)
                   1747: 
                   1748: Вопрос 7:
                   1749:    <раздатка>
                   1750:    Нельзя отвращаться. Врубать заднего. Мол, этот хороший, я ему помогу,
                   1751: а этот нет - пусть подыхает. Если ты помогаешь тем, кто тебе нравится,
                   1752: ты просто расчесываешь его. Когда твой компас начинает крутиться как
                   1753: сумасшедший, ты не обращаешь на него внимания. Потому что ты делаешь это
                   1754: не для себя. В этом всё дело. Понимаешь?
                   1755:    </раздатка>
                   1756:    Перед вами отрывок из романа Виктора Пелевина "Бэтман Аполло". Какую
                   1757: букву мы заменили в приведенной цитате?
                   1758: 
                   1759: Ответ:
                   1760: Э.
                   1761: 
                   1762: Комментарий:
                   1763: Расчесываешь эго.
                   1764: 
                   1765: Источник:
                   1766: http://www.zhlob.info/?p=736
                   1767: 
                   1768: Автор:
                   1769: Вера Звягинцева (Москва)
                   1770: 
                   1771: Вопрос 8:
                   1772: Дэнни Де Вито утверждает, что ИКС - как атомная бомба: он есть у всех,
                   1773: значит, есть и у него. Согласно одному афоризму, хороший ИКС - всегда
                   1774: стОящий. Какое слово мы заменили на ИКС?
                   1775: 
                   1776: Ответ:
                   1777: Адвокат.
                   1778: 
                   1779: Источник:
                   1780:    1. http://esquire.ru/wil/danny-devito
                   1781:    2. http://www.aphorism.ru/comments/jth838dsai.html
                   1782: 
                   1783: Автор:
                   1784: Вера Звягинцева (Москва)
                   1785: 
                   1786: Вопрос 9:
                   1787: Героиня Алексея Иванова при встрече с бывшим любовником снова начинает
                   1788: выяснять отношения. Автор шутит, что этим она как бы хочет СДЕЛАТЬ ЭТО.
                   1789: ЭТО нужно СДЕЛАТЬ, чтобы звуки читались раздельно. Ответьте точно, какие
                   1790: пять слов мы заменили словами "СДЕЛАТЬ ЭТО".
                   1791: 
                   1792: Ответ:
                   1793: Поставить вторую точку над i.
                   1794: 
                   1795: Комментарий:
                   1796: Например, во французском словосочетании "na&iuml;ve remark" &iuml; [и с
                   1797: двумя точками] ставится, чтобы показать, что "a" и "i" читаются
                   1798: раздельно.
                   1799: 
                   1800: Источник:
                   1801:    1. А. Иванов. Блуда и МУДО.
                   1802:    2. http://ru.wikipedia.org/wiki/%C3%8F_(латиница)
                   1803: 
                   1804: Автор:
                   1805: Амаль Имангулов (Самара - Москва)
                   1806: 
                   1807: Вопрос 10:
                   1808: В одной шутке известный уроженец Эр-Рияда просит оценить свою внешность.
                   1809: Ответьте как можно точнее, какую оценку он получает.
                   1810: 
                   1811: Ответ:
                   1812: 9/11.
                   1813: 
                   1814: Зачет:
                   1815: 9 из 11.
                   1816: 
                   1817: Комментарий:
                   1818: Часто оценки маркируют дробью: 10/10 [десять слэш десять] и т.д. Город,
                   1819: в котором на свет появился Усама бен Ладен, мог натолкнуть на необычную
                   1820: шкалу измерения.
                   1821: 
                   1822: Источник:
                   1823: Тредшот автора вопроса с сайта 4chan.
                   1824: 
                   1825: Автор:
                   1826: Амаль Имангулов (Самара - Москва)
                   1827: 
                   1828: Вопрос 11:
                   1829: Говоря об очевидных проблемах в расчете одного из численных методов,
                   1830: Александр Злотник заметил, что ОНИ лежат на поверхности. Назовите ИХ.
                   1831: 
                   1832: Ответ:
                   1833: Подводные камни.
                   1834: 
                   1835: Источник:
                   1836: ЛОАВ.
                   1837: 
                   1838: Автор:
                   1839: Амаль Имангулов (Самара - Москва)
                   1840: 
                   1841: Вопрос 12:
                   1842: Герои называющейся двукоренным словом песни группы "Птицу Емъ" приводят
                   1843: загипнотизированных жертв в свою секту, ДЕЛАЯ ЭТО. "ДЕЛО ВОЛШЕБНОГО
                   1844: ЭТОГО" - книга мастера сексуальной коммуникации Екатерины Федоровой.
                   1845: Какие три слова мы заменили словами "ДЕЛАТЬ ЭТО"?
                   1846: 
                   1847: Ответ:
                   1848: Играть на флейте.
                   1849: 
                   1850: Комментарий:
                   1851: Песня называется "Крысоловы". "Игра на волшебной флейте" - это, кхм-кхм,
                   1852: частый синоним всяких там пошлостей.
                   1853: 
                   1854: Источник:
                   1855:    1. Птицу Емъ - Крысоловы.
                   1856:    2. http://www.fedorova-e.ru/about/my_book
                   1857: 
                   1858: Автор:
                   1859: Амаль Имангулов (Самара - Москва)
                   1860: 
                   1861: Вопрос 13:
                   1862: Дуплет.
                   1863:    1. Эдуард I вынужден был запретить ИКС. ИКС имеет схожее
                   1864: происхождение с английским глаголом to hurt [ту хёрт]. Назовите ИКС.
                   1865:    2. Кристофер Бакли в книге "Флоренс Аравийская" описывает
                   1866: французско-американский конфликт. В том числе, он пишет, как в США
                   1867: поджигались Пежо, а ОНИ разрывались напополам. Назовите ИХ одним словом.
                   1868: 
                   1869: Ответ:
                   1870:    1. Бугурт.
                   1871:    2. Багет.
                   1872: 
                   1873: Комментарий:
                   1874:    1. Рыцарское сражение.
                   1875:    2. Антифранцузские настроения.
                   1876:    Оба ответа на дуплет являются в Интернете синонимами
                   1877: распространенного понятия "butthurt".
                   1878: 
                   1879: Источник:
                   1880:    1. http://ru.wikipedia.org/wiki/Бугурт
                   1881:    2. К. Бакли. Флоренс Аравийская.
                   1882: 
                   1883: Автор:
                   1884: Амаль Имангулов (Самара - Москва)
                   1885: 
                   1886: Вопрос 14:
                   1887:    <раздатка>
                   1888:    - А я вс-сё думал, кто же это так жаждет со мной пообщатьс-ся, -
                   1889: ПРОПУСК1 ему прямо в ухо знакомый холодный голос. Зельевар вздрагивает.
                   1890: Он запоздало вспоминает, что недавно, в связи с ремонтом западного крыла
                   1891: Малфой-мэнора (затеянного после одного не слишком удачного эксперимента
                   1892: Волдеморта), Лорд переселился на второй этаж восточного крыла
                   1893:    Автор рассказа, отрывок из которого перед вами, ПРОПУСК2 Волдеморта и
                   1894: Северуса Снэйпа.
                   1895:    </раздатка>
                   1896:    Восстановите пропуски, содержащие по одному слову.
                   1897: 
                   1898: Ответ:
                   1899: Шипит, шиппит.
                   1900: 
                   1901: Зачет:
                   1902: Шипит, пэйрит.
                   1903: 
                   1904: Комментарий:
                   1905: Шиппинг, или пэйринг, - настойчивое соединение персонажей вселенной
                   1906: вместе. Ну а антагонисты мира Гарри Поттера только и делали, что
                   1907: общались со змеями и всячески шипели.
                   1908: 
                   1909: Источник:
                   1910: http://www.ficbook.net/readfic/160147
                   1911: 
                   1912: Автор:
                   1913: Амаль Имангулов (Самара - Москва)
                   1914: 
                   1915: Вопрос 15:
                   1916: Согласно Википедии, в эпоху Возрождения АЛЬФА ассоциировалась с
                   1917: Сатурном. АЛЬФА ПЕРВОГО, как ни странно, не показана в АЛЬФЕ ВТОРОГО. Мы
                   1918: не спрашиваем, какое слово греческого происхождения мы заменили на
                   1919: АЛЬФУ. Назовите ПЕРВОГО и ВТОРОГО.
                   1920: 
                   1921: Ответ:
                   1922: Дюрер, Триер.
                   1923: 
                   1924: Комментарий:
                   1925: АЛЬФА - меланхолия.
                   1926: 
                   1927: Источник:
                   1928:    1. http://ru.wikipedia.org/wiki/Меланхолия
                   1929:    2. Общие знания.
                   1930: 
                   1931: Автор:
                   1932: Анна Кузнецова (Липецк - Москва)
                   1933: 
                   1934: Вопрос 16:
                   1935: Героя фильма "Lego Movie" перебивают на середине мотивационной речи.
                   1936: Ответьте точно, какое английское ругательство ему кричат в ответ на его
                   1937: следующую реплику.
                   1938: 
                   1939: Ответ:
                   1940: Butt.
                   1941: 
                   1942: Комментарий:
                   1943: На русский язык эту шутку перевели как - "Нет, тут должно было быть
                   1944: "но"" - "Сам ты дно!". Первая половина мотивационной речи часто
                   1945: посвящена тому, что всё плохо, но - не надо отчаиваться, нужно
                   1946: продолжать бороться.
                   1947: 
                   1948: Источник:
                   1949: Фильм "Lego Movie", английский оригинал и русский перевод.
                   1950: 
                   1951: Автор:
                   1952: Амаль Имангулов (Самара - Москва)
                   1953: 
                   1954: Вопрос 17:
                   1955: Одним из главных научных достижений этого английского ученого стали его
                   1956: открытия в области атомной теории строения вещества. Однако его имя
                   1957: стало известно во многом благодаря его неудачным занятиям ботаникой: он
                   1958: не мог разобраться в классификации цветущих растений, долгое время
                   1959: объясняя свою неспособность запутанностью названий. Назовите термин,
                   1960: который его прославил.
                   1961: 
                   1962: Ответ:
                   1963: Дальтонизм.
                   1964: 
                   1965: Источник:
                   1966: У. Гратцер. Эврики и эйфории. Об ученых и их открытиях. - М.: КоЛибри,
                   1967: Азбука-Аттикус, 2011. - С. 134.
                   1968: 
                   1969: Автор:
                   1970: Вера Звягинцева (Москва)
                   1971: 
                   1972: Вопрос 18:
                   1973: Герой песни группы "Птицу Емъ", находясь ПРОПУСК, каждой клеточкой тела
                   1974: почувствовал музыку и проболтался милиционерам о преступлении своих
                   1975: друзей. Героиня Джеффри Евгенидиса, находясь ПРОПУСК, решила написать
                   1976: гневное и бессмысленное письмо в газету. В каком заведении она
                   1977: находилась?
                   1978: 
                   1979: Ответ:
                   1980: Парикмахерская.
                   1981: 
                   1982: Комментарий:
                   1983: В этом вопросе обыгрываются жаргонные названия наркотиков: как кокаин
                   1984: называют первым, так и амфетамин называют феном. Пропущено "под колпаком
                   1985: фена" (под феном). В первой части вопроса речь идет об амфетамине,
                   1986: пробивающем на болтливость, танцы и бессмысленного содержания письма.
                   1987: 
                   1988: Источник:
                   1989:    1. Птицу Емъ - Нумеро Уно.
                   1990:    2. Джеффри Евгенидис. Девственницы-самоубийцы.
                   1991: 
                   1992: Автор:
                   1993: Амаль Имангулов (Самара - Москва)
                   1994: 
                   1995: Вопрос 19:
                   1996: (pic: 20150143.jpg)
                   1997:    Дуплет.
                   1998:    1. Назовите фильм, который зашифрован на первом постере из серии
                   1999: "Классика кино".
                   2000:    2. Назовите фильм, который зашифрован на втором постере из серии
                   2001: "Классика кино".
                   2002: 
                   2003: Ответ:
                   2004:    1. "В джазе только девушки".
                   2005:    2. "Последнее танго в Париже".
                   2006: 
                   2007: Источник:
                   2008:    1. http://www.poster.pl/poster/homework_pol_zartem/pl
                   2009:    2. http://www.poster.pl/poster/homework_ostatnie_tango/pl
                   2010: 
                   2011: Автор:
                   2012: Ольга Старчикова (Тамбов - Москва)
                   2013: 
                   2014: Вопрос 20:
                   2015: Писательница Мариам Петросян дает неопрятному герою такое описание: Он
                   2016: был равнодушен к еде и неряшлив в одежде, носил траур и подолгу не менял
                   2017: носков. Какие два слова мы пропустили в предыдущем предложении?
                   2018: 
                   2019: Ответ:
                   2020: Под ногтями.
                   2021: 
                   2022: Источник:
                   2023: Мариам Петросян. Дом, в котором... - М.: Live Book, 2014. - С. 106.
                   2024: 
                   2025: Автор:
                   2026: Наталия Павлова (Москва)
                   2027: 
                   2028: Вопрос 21:
                   2029:    <раздатка>
                   2030:    P&oacute;jd&#378;&#380;e, ki&#324; t&#281; chmurno&#347;&#263; w
                   2031: g&#322;&#261;b flaszy.
                   2032:    </раздатка>
                   2033:    Фразу, розданную вам, можно перевести как "Давай, брось свою тоску на
                   2034: дно бутылки". Назовите то, чем эта фраза является, двумя словами на одну
                   2035: и ту же букву.
                   2036: 
                   2037: Ответ:
                   2038: Польская панграмма.
                   2039: 
                   2040: Источник:
                   2041: http://pl.wikipedia.org/wiki/Pangram
                   2042: 
                   2043: Автор:
                   2044: ???
                   2045: 
                   2046: Вопрос 22:
                   2047: Прослушайте стихотворение Александра Матюшкина-Герке:
                   2048:    Вспыхает небо, разбужая ветер,
                   2049:    Проснувший гомон птичьих голосов;
                   2050:    Проклинывая всё на белом свете,
                   2051:    Я вновь бежу в нетоптанность лесов.
                   2052:    Шуршат зверушки, выбегнув навстречу,
                   2053:    Приветливыми лапками маша,
                   2054:    Я среди тут пробуду целый вечер,
                   2055:    Бессмертные творения пиша.
                   2056:    Но, выползя на миг из тины зыбкой,
                   2057:    Болотная зелёновая тварь
                   2058:    Совает мне с заботливой улыбкой
                   2059:    Большой...
                   2060:    Закончите двумя словами.
                   2061: 
                   2062: Ответ:
                   2063: "... орфографический словарь".
                   2064: 
                   2065: Источник:
                   2066: http://pikabu.ru/story/kosmatyiy_oblak_667250
                   2067: 
                   2068: Автор:
                   2069: Наталия Павлова (Москва)
                   2070: 
                   2071: Вопрос 23:
                   2072: Серия постеров художника Тан Яу Хунга посвящена известным цитатам. На
                   2073: одном из постеров изображены два яблочных огрызка, которые обрисовывают
                   2074: портрет автора цитаты. Напишите эту цитату.
                   2075: 
                   2076: Ответ:
                   2077: Stay hungry, stay foolish.
                   2078: 
                   2079: Зачет:
                   2080: В русском переводе.
                   2081: 
                   2082: Источник:
                   2083:    1. http://www.tangyauhoong.com/portfolio/quote-illustration/
                   2084:    2. http://ok-english.ru/stay-hungry-stay-foolish-9-vyiskazyivaniy-stiva-dzhobsa/
                   2085: 
                   2086: Автор:
                   2087: Наталия Павлова (Москва)
                   2088: 
                   2089: Вопрос 24:
                   2090: В ответе на этот вопрос могут упоминаться: просторечное упоминание
                   2091: интуиции, карликовая лошадь, слово, к которому восходит слово "ассасин".
                   2092: Назовите этот вопрос.
                   2093: 
                   2094: Ответ:
                   2095: Что курил автор вопроса?
                   2096: 
                   2097: Комментарий:
                   2098: Интуиция - это "чуйка", сленговое название сорта "чуйская долина",
                   2099: карликовая лошадь - "пони", сленговое название взрощенной на водном
                   2100: парнике гидропоники, ну а ассасин, или хашишин, произошло от слова
                   2101: "гашиш".
                   2102: 
                   2103: Источник:
                   2104: ЛОАВ.
                   2105: 
                   2106: Автор:
                   2107: Амаль Имангулов (Самара - Москва)
                   2108: 
                   2109: Тур:
                   2110: 7 тур. "Сэр Пухов" (Серпухов)
                   2111: 
                   2112: Вопрос 1:
                   2113: Жан Эффель курил так много, что, по воспоминаниям друзей, "в день ему
                   2114: требовалась всего одна ОНА...". Назовите двумя словами на одну и ту же
                   2115: букву место, где ОНА была в цене.
                   2116: 
                   2117: Ответ:
                   2118: Планета Плюк.
                   2119: 
                   2120: Зачет:
                   2121: Планета Пацак.
                   2122: 
                   2123: Комментарий:
                   2124: ОНА - спичка. После того как от спички Эффель прикуривал первую
                   2125: сигарету, остальные он прикуривал от предыдущей сигареты.
                   2126: 
                   2127: Источник:
                   2128:    1. О. Сердобольский. Автографы в антракте. - СПб.: Нотабене, 2001. -
                   2129: С. 55.
                   2130:    2. http://ru.wikipedia.org/wiki/Кин-дза-дза!
                   2131: 
                   2132: Автор:
                   2133: Олег Холодов, Евгений Кононенко
                   2134: 
                   2135: Вопрос 2:
                   2136: Сотрудница секретной лаборатории на вопрос Шерлока Холмса, чем она здесь
                   2137: занимается, отвечает: "Смешиваю гены и скрещиваю ИХ". При помощи одного
1.8       rubashki 2138: из НИХ легко определить, находитесь ли вы в зоне поражения во время
1.1       rubashki 2139: ядерного взрыва. Назовите его абсолютно точно.
                   2140: 
                   2141: Ответ:
                   2142: Большой палец.
                   2143: 
                   2144: Комментарий:
                   2145: Есть простой способ определить, находится ли человек в зоне поражения во
                   2146: время ядерного взрыва. Для этого нужно вытянуть руку с оттопыренным
                   2147: большим пальцем в сторону "грибка". Если он выше пальца - плохо.
                   2148: 
                   2149: Источник:
                   2150:    1. "Шерлок", s01e02.
                   2151:    2. http://copypast.ru/2014/08/28/interesnye_fakty_o_bolshom_palce.html
                   2152: 
                   2153: Автор:
                   2154: Олег Холодов
                   2155: 
                   2156: Вопрос 3:
                   2157: Дуплет.
                   2158:    1. По версии писателя Алексея Вязовского, ОН появился в конце XII
                   2159: века, во время противостояния кланов Минамото и Тайра, как бы
                   2160: символизируя победу клана Минамото. Назовите ЕГО.
                   2161:    2. Артемий Лебедев в своем твите от 10 августа 2014 года сообщил, что
                   2162: в Японии после первой брачной ночи принято выставлять ЕГО на всеобщее
                   2163: обозрение. Назовите ЕГО.
                   2164: 
                   2165: Ответ:
                   2166:    1. Японский флаг.
                   2167:    2. Японский флаг.
                   2168: 
                   2169: Комментарий:
                   2170:    1. Минамото выступали под белыми знаменами, а Тайра - под красными.
                   2171: 
                   2172: Источник:
                   2173:    1. http://samlib.ru/i/isaew_a_w/mikado.shtml
                   2174:    2. http://ru.wikipedia.org/wiki/Флаг_Японии
                   2175:    3. https://plus.google.com/+temalebedev/posts/EahhiFS3vbt
                   2176: 
                   2177: Автор:
                   2178: Олег Холодов
                   2179: 
                   2180: Вопрос 4:
                   2181: В рассказе Брайана Олдисса "Слюнное дерево" герои запасаются мукой и
                   2182: идут сражаться с пришельцем, обладающим тем же свойством, какое приобрел
                   2183: герой произведения 1897 года. Мы не просим назвать этого героя. Назовите
                   2184: его врожденную особенность.
                   2185: 
                   2186: Ответ:
                   2187: Альбинизм.
                   2188: 
                   2189: Зачет:
                   2190: Альбинос.
                   2191: 
                   2192: Комментарий:
                   2193: Пришелец - невидимый, мука облепит его, и это позволит эффективнее с ним
                   2194: сражаться. Произведение - "Человек-невидимка". Кстати, рассказ "Слюнное
                   2195: дерево" написан к столетию со дня рождения Герберта Уэллса. Для полной
                   2196: невидимости человек должен был быть альбиносом, каковым Гриффин и
                   2197: являлся.
                   2198: 
                   2199: Источник:
                   2200:    1. http://ru.wikipedia.org/wiki/Человек-невидимка
                   2201:    2. Брайан Олдисс. Слюнное дерево.
                   2202: http://www.flibusta.net/b/309248/read
                   2203: 
                   2204: Автор:
                   2205: Ася Самойлова, Надежда Богданова
                   2206: 
                   2207: Вопрос 5:
                   2208: Недалеко от антарктической станции "Амундсен - Скотт" учеными установлен
                   2209: столб, к которому участники экспедиций прибивают указатели, на которых
                   2210: написаны расстояния и направления до различных географических точек
                   2211: Земли. Есть таблички с указателями направления на Вашингтон, Нью-Йорк,
                   2212: Лондон, Москву и т.д. Все они выполнены в оригинальной манере. Назовите
                   2213: словом голландского происхождения то, в виде чего сделан указатель на
                   2214: Северный полюс.
                   2215: 
                   2216: Ответ:
                   2217: Флюгер.
                   2218: 
                   2219: Комментарий:
                   2220: Столб находится точно на Южном полюсе. К нему прикреплен флюгер, на нем
                   2221: написано "Nord" (север), и он всегда указывает на север, так как ветер
                   2222: всегда указывает на север, куда бы он ни дул. Только вот каждый год
                   2223: из-за дрейфа ледников приходится столб переносить.
                   2224: 
                   2225: Источник:
                   2226: "Дискавери". Антарктическая станция "Амундсен - Скотт".
                   2227: 
                   2228: Автор:
                   2229: Святослав Холодов
                   2230: 
                   2231: Вопрос 6:
                   2232: Французский юморист Ги Бедо накануне своего юбилея пошутил:
                   2233: "Единственное утешение в том, что скоро я задую шестидесятую свечку, -
                   2234: это то, что в шестидесятилетнем возрасте есть ОН". Назовите ЕГО
                   2235: односложным словом.
                   2236: 
                   2237: Ответ:
                   2238: Sex.
                   2239: 
                   2240: Комментарий:
                   2241: Во французском слове "sexag&eacute;naire" - шестидесятилетний.
                   2242: 
                   2243: Источник:
                   2244: Мирей Гильяно. Француженки подтяжек не делают.
                   2245: 
                   2246: Автор:
                   2247: Виктория Холодова
                   2248: 
                   2249: Вопрос 7:
                   2250: Интернет наводнили разные советы для поклонниц одного популярного
                   2251: сериала. Среди советов есть и такой: "Дождитесь полнолуния и ровно в
                   2252: полночь съешьте селедочный хвост, запив стаканом молока. Для закрепления
                   2253: результата примите горячую ванну". Какого эффекта мечтают достигнуть
                   2254: поклонницы, используя эти советы?
                   2255: 
                   2256: Ответ:
                   2257: Стать русалкой.
                   2258: 
                   2259: Зачет:
                   2260: По смыслу.
                   2261: 
                   2262: Комментарий:
                   2263: Сериал - "H2O: Просто добавь воды" про девочек-подростков, которые
                   2264: однажды попадают в древнюю пещеру и превращаются в русалок. В
                   2265: поисковиках на сегодня очень популярен запрос "как стать русалкой".
                   2266: 
                   2267: Источник:
                   2268:    1. http://www.sovetprost.ru/kak-legko-i-bystro-stat-nastoyashhej-rusalkoj.html
                   2269:    2. http://www.ymni4ka.ru/kak-stat-rusalkoj-s-siloj/
                   2270: 
                   2271: Автор:
                   2272: Надежда Богданова
                   2273: 
                   2274: Вопрос 8:
                   2275:    <раздатка>
                   2276:    Я был так взволнован, что едва мог усидеть на месте от мыслей в
                   2277: голове. Такое приятное чувство возбуждения испытывает только ПРОПУСК.
                   2278: ПРОПУСК в начале долгого пути.
                   2279:    </раздатка>
                   2280:    Перед вами цитата из фильма "Побег из Шоушенка" - естественно, в
                   2281: русском переводе. Заполните пропуск двумя словами.
                   2282: 
                   2283: Ответ:
                   2284: Свободный человек.
                   2285: 
                   2286: Комментарий:
                   2287: Это говорил герой Моргана Фримена. Фримен = свободный человек.
                   2288: 
                   2289: Источник:
                   2290: Фильм "Побег из Шоушенка" (1994).
                   2291: 
                   2292: Автор:
                   2293: Надежда Богданова
                   2294: 
                   2295: Вопрос 9:
                   2296: Во времена Второй мировой войны А. Бабкиным и В. Сосновским был
                   2297: предложен препарат из НЕЕ для восполнения кровопотери раненых. В 2010
1.8       rubashki 2298: году шотландская компания представила мировую новинку - ЕЕ в бутылке.
1.1       rubashki 2299: Среди главных покупателей своего товара компания видит рестораны. ОНА
                   2300: станет незаменимым продуктом для тех поваров, кто уделяет особое
                   2301: внимание вкусовым качествам блюд. Назовите ЕЕ.
                   2302: 
                   2303: Ответ:
                   2304: Морская вода.
                   2305: 
                   2306: Источник:
                   2307:    1. http://www.trinity-metody.com.ua/read/id=107/
                   2308:    2. http://www.seryogina.ru/sea-water-aquamara/
                   2309: 
                   2310: Автор:
                   2311: Надежда Богданова, Олег Холодов
                   2312: 
                   2313: Вопрос 10:
                   2314: По одной из легенд, пять ЕЕ лепестков напоминают о пяти чувствах,
                   2315: дарованных человеку, и о пяти ранах Христа. В Англии есть грустное
                   2316: поверье: якобы ОНИ появляются в больших количествах на полях сражений,
                   2317: где погибло много солдат. Назовите ЕЕ.
                   2318: 
                   2319: Ответ:
                   2320: Незабудка.
                   2321: 
                   2322: Комментарий:
                   2323: Синие цветы - словно письма, посланные погибшими с того света: "Не
                   2324: забывайте нас!".
                   2325: 
                   2326: Источник:
                   2327:    1. http://c-amazonka.ru/post111681395/
                   2328:    2. http://www.liveinternet.ru/users/3090603/post124901208/
                   2329: 
                   2330: Автор:
                   2331: Олег Холодов
                   2332: 
                   2333: Вопрос 11:
                   2334: Автор вопроса Татьяна Соломенникова хоть и не является моделью, но
                   2335: считает, что чем-то похожа на Лесли. К тому же ее прозвище похоже на
1.8       rubashki 2336: прозвище Лесли, которое та получила почти полвека назад и под которым
                   2337: стала известна всему миру. Мы не спрашиваем у вас прозвище автора
                   2338: вопроса. Назовите прозвище Лесли.
1.1       rubashki 2339: 
                   2340: Ответ:
                   2341: Твигги.
                   2342: 
                   2343: Зачет:
                   2344: Тростинка, Соломинка.
                   2345: 
                   2346: Комментарий:
                   2347: Была тонкой, как тростинка. Прозвище автора вопроса пошло от ее фамилии.
                   2348: 
                   2349: Источник:
                   2350: http://ru.wikipedia.org/wiki/Твигги
                   2351: 
                   2352: Автор:
                   2353: Татьяна Соломенникова
                   2354: 
                   2355: Вопрос 12:
                   2356: Внимание, в вопросе есть замены.
                   2357:    В романе Олега Бубелы "Герой" полководец жалеет, что "единственный ОН
                   2358: быстро перешел в разряд ТАКИХ". ТАКИЕ ОНИ - один из предметов, который
                   2359: изучают в "Школе Монстров". Какой термин мы заменили на "ТАКОЙ ОН"?
                   2360: 
                   2361: Ответ:
                   2362: Мертвый язык.
                   2363: 
                   2364: Комментарий:
                   2365: У героя в результате пыток умер язык (пленный). В "Школе Монстров", так
                   2366: как они все вымирающие, один из основных предметов - "мертвые языки",
                   2367: чтобы монстры могли общаться друг с другом.
                   2368: 
                   2369: Источник:
                   2370:    1. Олег Бубела. Герой. http://www.flibusta.net/b/231586/read
                   2371:    2. http://www.mhfans.ru/interestingly/23-chto-takoe-mertvye-yazyki.html
                   2372: 
                   2373: Автор:
                   2374: Олег Холодов
                   2375: 
                   2376: Вопрос 13:
                   2377: Эбису - один из семи богов удачи в синтоизме. Обычно изображается в
                   2378: высокой шляпе с удочкой в руках и большой рыбой. Назовите предельно
                   2379: точно общественные места, в которых часто встречаются изображения Эбису.
                   2380: 
                   2381: Ответ:
                   2382: Рестораны, где подают рыбу фугу.
                   2383: 
                   2384: Зачет:
                   2385: По смыслу.
                   2386: 
                   2387: Комментарий:
                   2388: Да сопутствует вам удача при поедании рыбы фугу!
                   2389: 
                   2390: Источник:
                   2391: http://ru.wikipedia.org/wiki/Эбису
                   2392: 
                   2393: Автор:
                   2394: Олег Холодов
                   2395: 
                   2396: Вопрос 14:
                   2397: Хозяин одного мини-отеля использует понятную систему информирования с
                   2398: помощью цветных флагов. Например, поднятый красный флаг сигнализирует о
                   2399: запрете купания, зеленый - о штиле на море, желтый - "волны, будьте
                   2400: осторожны". В каких случаях хозяин отеля поднимет белый флаг?
                   2401: 
                   2402: Ответ:
                   2403: При наличии в отеле свободных номеров.
                   2404: 
                   2405: Зачет:
                   2406: По смыслу.
                   2407: 
                   2408: Комментарий:
                   2409: Номера в отеле сдаются.
                   2410: 
                   2411: Источник:
                   2412: ЛОАВ.
                   2413: 
                   2414: Автор:
                   2415: Олег Холодов
                   2416: 
                   2417: Вопрос 15:
                   2418: Википедия утверждает, что прототип ЭТОГО появился в 1733 году. Среди
                   2419: различных видов ЭТОГО встречаются: рюкзак, книжка, чемодан. Согласно
                   2420: афоризму, будущее нации зависит от количества ЭТОГО. Назовите ЭТО.
                   2421: 
                   2422: Ответ:
                   2423: Детская коляска.
                   2424: 
                   2425: Источник:
                   2426:    1. http://ru.wikipedia.org/wiki/Детская_коляска
                   2427:    2. http://www.pervenez.ru/kolyaska.html
                   2428:    3. http://www.novate.ru/blogs/110613/23190/
                   2429:    4. http://citaty.info/quote/man/211744
                   2430: 
                   2431: Автор:
1.2       rubashki 2432: Олег Холодов, Ася Самойлова
1.1       rubashki 2433: 
                   2434: Вопрос 16:
                   2435: "Это даже не рисунок мальчишки, а узор не окончившего ПТУ резчика". Так
                   2436: о "НЕЙ" отозвался Борис Гребенщиков. В 2010 году о правах на НЕЕ заявила
                   2437: некая Анхелес Дуран. Впрочем, как ехидно замечают журналисты, вступить в
                   2438: права ей будет, мягко сказать, затруднительно. Назовите ЕЕ четырьмя
                   2439: словами.
                   2440: 
                   2441: Ответ:
                   2442: Звезда по имени Солнце.
                   2443: 
                   2444: Комментарий:
                   2445: Так Гребенщиков высказался о песни Цоя "Звезда по имени Солнце". Анхелес
                   2446: Дуран провозгласила себя хозяйкой Солнца.
                   2447: 
                   2448: Источник:
                   2449:    1. http://volna.afisha.ru/archive/onesong_zvezda/
                   2450:    2. http://izvestia.ru/news/481810
                   2451: 
                   2452: Автор:
                   2453: Татьяна Соломенникова
                   2454: 
                   2455: Вопрос 17:
                   2456: Внимание, в вопросе есть замена.
                   2457:    При правильном подборе коромысла всегда учитывается вес и рост
                   2458: владельца. На самом деле, если верить производителям, существует много
                   2459: разновидностей коромысел. В частности, коромысло для кормления грудью,
1.5       rubashki 2460: коромысло для новорождённых. Недавно американский архитектор Пол Кветон
                   2461: совместил коромысло с домиком для кошки. Его изобретение стало хитом
                   2462: среди зажиточных жителей США и Европы. Какое сложное слово, пишущееся
                   2463: через дефис, мы заменили на "коромысло"?
1.1       rubashki 2464: 
                   2465: Ответ:
                   2466: Кресло-качалка.
                   2467: 
                   2468: Комментарий:
                   2469: Обоснование замены: по-английски кресло-качалку часто называют "rocker",
                   2470: на русский это слово также переводится как "коромысло".
                   2471: 
                   2472: Источник:
                   2473:    1. http://www.kreslo-kachalka.ua/blog/?p=31
                   2474:    2. http://www.drunov.ru/news/kreslo_kachalka_dlya_dvoikh/
                   2475:    3. http://www.malutka.net/kreslo-kachalka-dlya-novorozhdennogo
                   2476:    4. http://www.mebelniykrug.ru/kak-sdelat-kreslo-kachalku-svoimi-rukami.html
                   2477:    5. http://translate.academic.ru/кресло-качалка/ru/en/1
                   2478: 
                   2479: Автор:
                   2480: Алексей Паклин
                   2481: 
                   2482: Вопрос 18:
                   2483: Дуплет.
                   2484:    1. В 2011 году во время хорошо вам известного мероприятия, по данным
                   2485: агентства "РИА Новости", один человек ответил на 90 вопросов за 4 часа
                   2486: 33 минуты. Кто этот человек?
                   2487:    2. Как ни удивительно, но провести почти 19 дней без сна и установить
                   2488: рекорд Роберту Макдональду помогло ОНО. Назовите ЕГО.
                   2489: 
                   2490: Ответ:
                   2491:    1. [Владимир] Путин.
                   2492:    2. Кресло-качалка.
                   2493: 
                   2494: Комментарий:
                   2495:    1. Речь идет о ежегодной передаче "Прямая линия" с Владимиром
                   2496: Путиным.
                   2497:    2. С 14 марта по 2 апреля добровольно провел без сна 453 часа 40
                   2498: минут, борясь с ним в кресле-качалке.
                   2499: 
                   2500: Источник:
                   2501:    1. http://ria.ru/infografika/20130424/934270082.html
                   2502:    2. http://www.para-psychology.ru/perception7.htm
                   2503: 
                   2504: Автор:
                   2505: Олег Холодов, Надежда Богданова
                   2506: 
                   2507: Вопрос 19:
                   2508: Этот эпоним имеет много значений, большинство из которых на данный
                   2509: момент устарело. Например, Готье де Куанси в одном из стихотворений
                   2510: использовал это слово как метафору для характеристики абсолютно
                   2511: никчёмного человека. В средние века этим словом называли сочинения по
                   2512: искусству счета. В XVIII веке в одном из германских словарей это слово
                   2513: объяснялось как понятие о четырех арифметических операциях. Назовите это
                   2514: слово.
                   2515: 
                   2516: Ответ:
                   2517: Алгоритм.
                   2518: 
                   2519: Комментарий:
                   2520: Слово "алгоритм" происходит от имени хорезмского ученого Абу Абдуллах
                   2521: Мухаммед ибн Муса аль-Хорезми (алгоритм - аль-Хорезми). В первой
                   2522: половине XII века книга аль-Хорезми в латинском переводе проникла в
                   2523: Европу. Переводчик, имя которого до нас не дошло, дал ей название
                   2524: "Algoritmi de numero Indorum" ("Алгоритмы о счете индийском"). Таким
                   2525: образом, мы видим, что латинизированное имя среднеазиатского ученого
                   2526: было вынесено в заглавие книги, и сегодня считается, что слово
                   2527: "алгоритм" попало в европейские языки именно благодаря этому сочинению.
                   2528: 
                   2529: Источник:
                   2530: http://ru.wikipedia.org/wiki/Алгоритм
                   2531: 
                   2532: Автор:
                   2533: Святослав Холодов
                   2534: 
                   2535: Вопрос 20:
                   2536: По одной версии, дружеское прозвище композитора повлияло на выбор имени
                   2537: героини комедии - мол, именно так композитор закрепил в массовом
                   2538: сознании свое авторство знаменитой песни. Позже прозвище перешло по
                   2539: наследству. Назовите этого композитора.
                   2540: 
                   2541: Ответ:
                   2542: Исаак Дунаевский.
                   2543: 
                   2544: Комментарий:
                   2545: Прозвище Дунаевского, "переданное" и его сыну Максиму, также
                   2546: композитору, - "Дуня". По сюжету фильма "Волга-Волга", музыку к которому
                   2547: написал Исаак Дунаевский, разыскивается автор песни, о которой известно
                   2548: лишь то, что "ее написала Дуня".
                   2549: 
                   2550: Источник:
                   2551:    1. Документальный фильм "Максим Дунаевский. Жизнь по завещанию".
                   2552:    2. http://ru.wikipedia.org/wiki/Волга-Волга
                   2553: 
                   2554: Автор:
                   2555: Евгений Кононенко
                   2556: 
                   2557: Вопрос 21:
                   2558: В сериале "Две легенды" бизнесмен сравнивает ИХ с мужской бижутерией. ИХ
                   2559: качество и количество указывает на статус владельца. Алан Пинкертон
                   2560: пошутил, что для защиты от экономического кризиса нужно приобретать ИХ.
                   2561: Назовите ИХ сложносоставным словом, которое является названием "самого
                   2562: простого фильма Акиры Куросавы".
                   2563: 
                   2564: Ответ:
                   2565: Телохранители.
                   2566: 
                   2567: Комментарий:
                   2568: Алан Пинкертон в 1850 году в Чикаго организовал Национальное детективное
                   2569: агентство Пинкертона. На сегодняшний день Pinkerton Government Services
                   2570: является одним из крупнейших охранных агентств в мире. "Телохранитель" -
                   2571: кинофильм режиссера Акиры Куросавы, снятый в жанре дзидайгэки. Остальные
                   2572: работы японца - философские притчи.
                   2573: 
                   2574: Источник:
                   2575:    1. Сериал "Две легенды", s01e01.
                   2576:    2. "Алан Пинкертон - история успеха".
                   2577:    3. http://kino-leone.narod.ru/tel.htm
                   2578: 
                   2579: Автор:
                   2580: Олег Холодов
                   2581: 
                   2582: Вопрос 22:
                   2583: Во времена работы в издательстве "Алконост" Александр Блок за привычку
                   2584: одевать строгие черные костюмы или смокинги для фотографирования получил
                   2585: прозвище ШУМАН. Это прозвище настолько хорошо передавало сущность поэта,
                   2586: что Владимир Орлов свою биографическую книгу назвал "ШУМАН. Жизнь
                   2587: Александра Блока". Какое слово мы заменили в этом вопросе?
                   2588: 
                   2589: Ответ:
                   2590: Гамаюн.
                   2591: 
                   2592: Комментарий:
                   2593: У птицы Гамаюн черная окраска. Александру Блоку шел девятнадцатый год,
                   2594: когда он написал стихотворение "Гамаюн, птица вещая". По одной из
                   2595: версий, слово "Гамаюн" произошло от славянского слова "гам" - шум.
                   2596: 
                   2597: Источник:
                   2598:    1. http://blok.ouc.ru/andrei-turkov.html
                   2599:    2. http://az.lib.ru/b/blok_a_a/text_0430.shtml
                   2600: 
                   2601: Автор:
                   2602: Олег Холодов
                   2603: 
                   2604: Вопрос 23:
                   2605: По одной из версий, ЕЕ завезли в Россию немцы, которые таким способом
                   2606: пытались прикрыть свои попытки соблазнять русских барышень. Есть даже
                   2607: версия, что ЕЕ название возникло от традиционного немецкого приглашения
                   2608: к интимной близости. Назовите ЕЕ и не перепутайте.
                   2609: 
                   2610: Ответ:
                   2611: Фига.
                   2612: 
                   2613: Комментарий:
                   2614: Fick-fick machen - так звучало традиционное немецкое приглашение к
                   2615: интимной близости. Фиговый листок - переносное лицемерное прикрытие
                   2616: заведомо бесстыдных действий, нечестных поступков; существует много
                   2617: различных версий, в которых утверждается что первая одежда была сделана
                   2618: не из фиговых листьев, а листьев каких-то других растений. Чаще всего
                   2619: упоминается яблоко.
                   2620: 
                   2621: Источник:
                   2622: http://copypast.ru/2014/08/28/interesnye_fakty_o_bolshom_palce.html
                   2623: 
                   2624: Автор:
                   2625: Олег Холодов
                   2626: 
                   2627: Вопрос 24:
                   2628: Внимание, в вопросе есть замена.
                   2629:    АВТОГОЛ Тотти стал причиной недавнего раздора между президентами
                   2630: "Ромы" и "Лацио". В России в 2014 году надзирающий орган рекомендовал
                   2631: обходиться без АВТОГОЛОВ. Какое слово мы заменили на "АВТОГОЛ"?
                   2632: 
                   2633: Ответ:
                   2634: Селфи.
                   2635: 
                   2636: Комментарий:
                   2637: Русский вариант слова "селфи" - себяшка, самострел. Франческо Тотти
                   2638: забрал телефон у тренера вратарей "Ромы" после своего второго гола в
                   2639: ворота "Лацио" и сфотографировал себя на фоне беснующейся от восторга
                   2640: фанатской трибуны, что стало причиной перебранки президентов клубов.
                   2641: Селфи является главной причиной распространения вшей, заявило курское
                   2642: управление Роспотребнадзора в октябре 2014 года.
                   2643: 
                   2644: Источник:
                   2645:    1. http://football.sport-express.ru/reviews/52631/
                   2646:    2. http://ru.wikipedia.org/wiki/Селфи
                   2647: 
                   2648: Автор:
                   2649: Олег Холодов
                   2650: 
                   2651: Тур:
                   2652: 8 тур. "Водопой кротов" (Тверь)
                   2653: 
                   2654: Вопрос 1:
                   2655: Героиня Маркеса в молодости втайне ДЕЛАЛА ЭТО вместе со своей двоюродной
                   2656: сестрой, причем они ДЕЛАЛИ ЭТО шиворот-навыворот, как солдаты во время
                   2657: войны по ночам. Какой глагол мы заменили на "ДЕЛАТЬ ЭТО"?
                   2658: 
                   2659: Ответ:
                   2660: Курить.
                   2661: 
                   2662: Комментарий:
                   2663: Солдаты курили сигареты огоньком внутрь, чтобы не выдать свое
                   2664: местоположение ночью. В дальнейшем уважаемые знатоки убедятся, что и
                   2665: авторы пакета курили что попало и тоже шиворот-навыворот.
                   2666: 
                   2667: Источник:
                   2668: Г. Гарсиа Маркес. Любовь во время чумы.
                   2669: http://www.flibusta.net/b/96270/read
                   2670: 
                   2671: Автор:
                   2672: Станислав Попов (Тверь)
                   2673: 
                   2674: Вопрос 2:
                   2675: В магазине жена автора вопроса повертела в руках блузку и посетовала,
                   2676: что производители экономят на декоративных элементах, а в результате
                   2677: получается ОНА. Назовите ЕЕ двумя словами, начинающимися на одну и ту же
                   2678: букву.
                   2679: 
                   2680: Ответ:
                   2681: Перевязь Портоса.
                   2682: 
                   2683: Комментарий:
                   2684: Блестки, пайетки и прочий декор располагался только на лицевой стороне.
                   2685: 
                   2686: Источник:
                   2687: Личный опыт жены автора вопроса.
                   2688: 
                   2689: Автор:
                   2690: Станислав Попов (Тверь)
                   2691: 
                   2692: Вопрос 3:
                   2693: Одна американская компания выпустила разновидность ЭТОГО под названием
                   2694: "Impossible", отличающуюся неровными краями и наличием пяти
                   2695: дополнительных деталей. Назовите ЭТО одним заимствованным словом.
                   2696: 
                   2697: Ответ:
                   2698: Паззл.
                   2699: 
                   2700: Комментарий:
                   2701: Паззл стал намного сложнее, так как его нельзя начать собирать с более
                   2702: простой "рамки".
                   2703: 
                   2704: Источник:
                   2705: http://www.ugames.com
                   2706: 
                   2707: Автор:
                   2708: Станислав Попов (Тверь)
                   2709: 
                   2710: Вопрос 4:
                   2711: (pic: 20150144.jpg)
                   2712:    Перед вами фотографии Ангелы Меркель, использованные в качестве
                   2713: рекламы одного товара. Лучшие образцы этого товара выдерживают более 200
                   2714: килограммов на квадратный сантиметр. Назовите этот товар.
                   2715: 
                   2716: Ответ:
                   2717: Суперклей.
                   2718: 
                   2719: Зачет:
                   2720: Клей.
                   2721: 
                   2722: Источник:
1.14      rubashki 2723:    1. http://9gag.com/gag/26728
1.1       rubashki 2724:    2. http://ru.wikipedia.org/wiki/Цианоакрилат
                   2725: 
                   2726: Автор:
                   2727: Станислав Попов (Тверь)
                   2728: 
                   2729: Вопрос 5:
                   2730: В Риме после официального принятия христианства были уничтожены все
                   2731: статуи языческих императоров, кроме памятника Марку Аврелию, потому что
                   2732: его ПРОПУСК. Дмитрий Дибров рассказывал, как однажды в Киеве его ПРОПУСК
                   2733: Хабенского. Заполните пропуск тремя словами.
                   2734: 
                   2735: Ответ:
                   2736: Приняли за Константина.
                   2737: 
                   2738: Источник:
                   2739:    1. http://en.wikipedia.org/wiki/Equestrian_Statue_of_Marcus_Aurelius
                   2740:    2. http://www.ddibrov.com/press/dmitriy-dibrov-kogda-ya-v-efire-u-menya-v-glazah-smertelnaya-skuka/
                   2741: 
                   2742: Автор:
                   2743: Станислав Попов (Тверь)
                   2744: 
                   2745: Вопрос 6:
                   2746: Врач-ортодонт, оперирующий в крупном городе, сделал логотип своей
                   2747: клиники в виде силуэта известного здания, расположенного там же. В каком
                   2748: городе это произошло?
                   2749: 
                   2750: Ответ:
                   2751: Сидней.
                   2752: 
                   2753: Комментарий:
                   2754: (pic: 20150145.jpg)
                   2755:    Ортодонты, в основном, занимаются выправлением прикуса. Оперный театр
                   2756: Сиднея в профиль выглядит как ОЧЕНЬ кривой ряд зубов. Не зря в вопросе
                   2757: стоит слово "оперирующий".
                   2758: 
                   2759: Источник:
                   2760: http://www.findingnemo2.co.uk/p-sherman-42-wallaby-way-sydney-apparently-real-orthodontic-clinic/
                   2761: 
                   2762: Автор:
                   2763: Станислав Попов (Тверь)
                   2764: 
                   2765: Вопрос 7:
                   2766: Роман Тонино Бенаквисты "Малавита" написан, в основном, на французском,
                   2767: но содержит и английские реплики. К примеру, в разговоре об одной опере
                   2768: один из героев замечает: "Если она достаточно хороша для тебя, она и для
                   2769: меня достаточно хороша". Назовите заглавного героя этой оперы.
                   2770: 
                   2771: Ответ:
                   2772: Борис Годунов.
                   2773: 
                   2774: Комментарий:
                   2775: Игра слов "Godunov = Good Enough".
                   2776: 
                   2777: Источник:
                   2778: Тонино Бенаквиста. Малавита. http://www.flibusta.net/b/316583/read
                   2779: 
                   2780: Автор:
                   2781: Станислав Попов (Тверь)
                   2782: 
                   2783: Вопрос 8:
                   2784: Есть легенда, что поэт Ли Бо утонул в реке, когда ночью в лодке спьяну
                   2785: попытался СДЕЛАТЬ ЭТО. На некоторых алхимических гравюрах ЭТО ДЕЛАЕТ
                   2786: Солнце. Какие два слова мы заменили на "ДЕЛАТЬ ЭТО"?
                   2787: 
                   2788: Ответ:
                   2789: Обнять Луну.
                   2790: 
                   2791: Зачет:
                   2792: Поймать Луну, поцеловать Луну и т.п.
                   2793: 
                   2794: Источник:
                   2795: http://en.wikipedia.org/wiki/Li_Bai
                   2796: 
                   2797: Автор:
                   2798: Станислав Попов (Тверь)
                   2799: 
                   2800: Вопрос 9:
                   2801: Ученый и философ Дуглас Хофштадтер сформулировал такой шуточный закон:
                   2802: "Что бы вы ни делали, это всегда займет больше времени, чем
                   2803: планировалось, даже если учесть ЕГО". Назовите ЕГО двумя словами.
                   2804: 
                   2805: Ответ:
                   2806: Закон Хофштадтера.
                   2807: 
                   2808: Источник:
                   2809: http://en.wikipedia.org/wiki/Hofstadter's_law
                   2810: 
                   2811: Автор:
                   2812: Станислав Попов (Тверь)
                   2813: 
                   2814: Вопрос 10:
                   2815: После мятежа на "Баунти" на поиски предателей был послан корабль. Экипаж
                   2816: напал на след мятежников, задержал их и поместил в крохотную камеру на
                   2817: судне. Однако на пути назад корабль разбился о риф и начал тонуть,
                   2818: поэтому арестантов пришлось выпустить, но ничего катастрофического в
                   2819: результате не случилось. А какое имя носил этот корабль?
                   2820: 
                   2821: Ответ:
                   2822: "Пандора".
                   2823: 
                   2824: Комментарий:
                   2825: А камеру на корабле так и называли - "ящик Пандоры".
                   2826: 
                   2827: Источник:
                   2828: http://en.wikipedia.org/wiki/HMS_Pandora_(1779)
                   2829: 
                   2830: Автор:
                   2831: Станислав Попов (Тверь)
                   2832: 
                   2833: Вопрос 11:
                   2834: На сайте kinonews.ru приводится следующая справка: "Алехандро Аменабар
                   2835: родился 31 марта 1973 года, был режиссером фильмов: "Море внутри"
                   2836: (2004), "ПРОПУСК" (2001), "Дипломная работа" (1996) и ПРОПУСК".
                   2837: Восстановите любой из пропусков.
                   2838: 
                   2839: Ответ:
                   2840: Другие.
                   2841: 
                   2842: Источник:
                   2843: http://www.kinonews.ru/person_3736/alejandro-ameni
                   2844: 
                   2845: Автор:
                   2846: Станислав Попов (Тверь)
                   2847: 
                   2848: Вопрос 12:
                   2849: (pic: 20150146.jpg)
                   2850:    Автор вопроса составил себе такую таблицу, чтобы не запутаться. Какая
                   2851: английская фамилия скрыта от вас в последней строчке?
                   2852: 
                   2853: Ответ:
                   2854: Windsor.
                   2855: 
                   2856: Зачет:
                   2857: Виндзор.
                   2858: 
                   2859: Комментарий:
                   2860: Это памятка, как вязать различные галстучные узлы.
                   2861: 
                   2862: Источник:
                   2863:    1. ЛОАВ.
                   2864:    2. http://www.agreeordie.com
                   2865: 
                   2866: Автор:
                   2867: Станислав Попов (Тверь)
                   2868: 
                   2869: Вопрос 13:
                   2870: Объясняя один из способов завязывания галстучного узла, видеоблогер
                   2871: Алекс Красны советует с самого начала оставить один конец галстука очень
                   2872: длинным, так как узел очень ТАКОЙ. А что было ТАКИМ в названии фильма
                   2873: 2012 года?
                   2874: 
                   2875: Ответ:
                   2876: Игры.
                   2877: 
                   2878: Источник:
                   2879:    1. http://www.agreeordie.com/
                   2880:    2. http://ru.wikipedia.org/wiki/Голодные_игры_(фильм)
                   2881: 
                   2882: Автор:
                   2883: Станислав Попов (Тверь)
                   2884: 
                   2885: Вопрос 14:
                   2886: Герой романа Фаулза "Коллекционер" похищает девушку и втайне ото всех
                   2887: держит ее в подвале. Он даже устанавливает специальную печь, назначение
                   2888: которой он описывает фразой, практически совпадающей с русской
                   2889: поговоркой. Воспроизведите эту поговорку.
                   2890: 
                   2891: Ответ:
                   2892: Сор из избы не выносить.
                   2893: 
                   2894: Источник:
                   2895: Дж. Фаулз. Коллекционер. http://www.flibusta.net/b/338811/read
                   2896: 
                   2897: Автор:
                   2898: Станислав Попов (Тверь)
                   2899: 
                   2900: Вопрос 15:
                   2901:    <раздатка>
                   2902:    "***** Vina Musica"
                   2903:    </раздатка>
                   2904:    Группа "Corvus Corax", исполняющая музыку в стиле "средневековый
                   2905: рок", не чужда идеалам XX века. Один из их альбомов называется "ПРОПУСК
                   2906: Vina Musica". Имя какого божества мы пропустили?
                   2907: 
                   2908: Ответ:
                   2909: Venus.
                   2910: 
                   2911: Зачет:
                   2912: Венера.
                   2913: 
                   2914: Комментарий:
                   2915: Средневековый аналог "Sex, Drugs, Rock'n'Roll".
                   2916: 
                   2917: Источник:
                   2918: http://www.corvuscorax.de
                   2919: 
                   2920: Автор:
                   2921: Станислав Попов (Тверь)
                   2922: 
                   2923: Вопрос 16:
                   2924: Французская певица Брижит Лен, также увлекающаяся средневековой музыкой,
                   2925: выпустила альбом песен XII-XIII веков, воспевающих женщин. Название
                   2926: этого альбома представляет собой короткий палиндром из двух слов.
                   2927: Воспроизведите это название.
                   2928: 
                   2929: Ответ:
                   2930: "Ave Eva".
                   2931: 
                   2932: Источник:
                   2933: http://www.amazon.com/Ave-Eva-Brigitte-Lesne/dp/B000005W48/
                   2934: 
                   2935: Автор:
                   2936: Станислав Попов (Тверь)
                   2937: 
                   2938: Вопрос 17:
                   2939: В конце 1930-х годов влиятельная американская организация "Союз Дочерей
                   2940: Конфедерации" заявила, что лучше пусть ЕЮ станет англичанка, чем янки.
                   2941: Назовите имя и фамилию этой англичанки.
                   2942: 
                   2943: Ответ:
                   2944: Вивьен Ли.
                   2945: 
                   2946: Комментарий:
                   2947: Речь шла о кастинге на роль Скарлетт О'Хары - типичной южанки.
                   2948: 
                   2949: Источник:
                   2950: http://diletant.ru/state_exam/?id=23033858
                   2951: 
                   2952: Автор:
                   2953: Станислав Попов (Тверь)
                   2954: 
                   2955: Вопрос 18:
                   2956: Известно, что Жерар Депардье сбежал от французских налогов сначала в
                   2957: Бельгию, а потом в Россию. Годом раньше в Лондон эмигрировал и другой
                   2958: французский актер - Кристиан Клавье. В посвященной этому заметке на
                   2959: сайте lenta.ru говорится, что ОНИ-то за свою родину сражались. Назовите
                   2960: ИХ имена.
                   2961: 
                   2962: Ответ:
                   2963: Астерикс и Обеликс.
                   2964: 
                   2965: Источник:
                   2966: В вопросе.
                   2967: 
                   2968: Автор:
                   2969: Станислав Попов (Тверь)
                   2970: 
                   2971: Вопрос 19:
                   2972: Как рассказывает статья в журнале "Трансаэро", венский мастер Михаэль
                   2973: Тонет в середине XIX века предложил новую технику изготовления мебели, и
                   2974: хотя поначалу результат многим не понравился, мастер продолжал
                   2975: НАСТАИВАТЬ НА УБЕЖДЕНИЯХ. Песня "НАСТАИВАЙ НА УБЕЖДЕНИЯХ" стала основной
                   2976: темой отечественного фильма. Какого?
                   2977: 
                   2978: Ответ:
                   2979: "Брат-2".
                   2980: 
                   2981: Комментарий:
                   2982: Мастер продолжал гнуть свою линию.
                   2983: 
                   2984: Источник:
                   2985: Журнал "Трансаэро", 2013, N 8. - С. 232-234.
                   2986: 
                   2987: Автор:
                   2988: Станислав Попов (Тверь)
                   2989: 
                   2990: Вопрос 20:
                   2991: На афише, анонсирующей выход фильма Ларса фон Триера "Нимфоманка", были
                   2992: запечатлены лица сыгравших актеров, изображающих момент оргазма. Подпись
                   2993: внизу афиши состояла из двух английских слов. Воспроизведите эти слова.
                   2994: 
                   2995: Ответ:
                   2996: Coming soon.
                   2997: 
                   2998: Комментарий:
                   2999: Двойной смысл словосочетаний "скоро выходит" (в вопросе указано, что это
                   3000: анонс) и "скоро кончит".
                   3001: 
                   3002: Источник:
                   3003: http://culturmag.de/litmag/lars-von-trier-nymphmaniac-1-2/80397
                   3004: 
                   3005: Автор:
                   3006: Станислав Попов (Тверь)
                   3007: 
                   3008: Вопрос 21:
                   3009: (pic: 20150147.gif)
                   3010:    Внимание, в вопросе есть замены.
                   3011:    Город Нэшвилл в штате Теннесси, где расположена штаб-квартира
                   3012: Национальной Конвенции Баптистов, иногда называют "ВЕРШИНОЙ БЕРМУДСКОГО
                   3013: ТРЕУГОЛЬНИКА". На розданном вам рисунке - одна из моделей ВЕРШИНЫ
                   3014: ТРЕУГОЛЬНИКА в натуральную величину в разрезе. Какие слова мы заменили
                   3015: на "ВЕРШИНА БЕРМУДСКОГО ТРЕУГОЛЬНИКА"?
                   3016: 
                   3017: Ответ:
                   3018: Пряжка Библейского пояса.
                   3019: 
                   3020: Комментарий:
                   3021: "Библейский пояс" - собирательное название группы южных штатов США, где
                   3022: сильны христианские традиции. На звание "Пряжки Библейского пояса"
                   3023: претендуют и несколько других городов.
                   3024: 
                   3025: Источник:
                   3026:    1. http://www.findpatent.ru/img_show/230060.html
                   3027:    2. http://en.wikipedia.org/wiki/Bible_Belt
                   3028: 
                   3029: Автор:
                   3030: Станислав Попов (Тверь)
                   3031: 
                   3032: Вопрос 22:
                   3033: Про эту птичку в Англии есть легенда, что она вынимала шипы из ран
                   3034: Христа, испачкавшись в Его крови. Согласно другой легенде, эта птичка
                   3035: вовсе не благочестива, а наоборот, склонна к одному из смертных грехов,
                   3036: поэтому литературный персонаж и получил свое имя. Назовите этого
                   3037: персонажа, использовав дефис.
                   3038: 
                   3039: Ответ:
                   3040: Робин-Бобин.
                   3041: 
                   3042: Комментарий:
                   3043: Речь идет о малиновке, которая по-английски, как известно, - robin.
                   3044: 
                   3045: Источник:
                   3046: Е. Коути, Н. Харса. Суеверия викторианской Англии.
                   3047: http://www.flibusta.net/b/269090/read
                   3048: 
                   3049: Автор:
                   3050: Станислав Попов (Тверь)
                   3051: 
                   3052: Вопрос 23:
                   3053: В 1795 году из-за угрозы голода премьер-министр Великобритании Уильям
                   3054: Питт ввел высокий налог на использование ЭТОГО вещества в
                   3055: непродовольственных целях. С другой стороны, елизаветинские времена
                   3056: давно уже прошли, и мода начала меняться сама собой. О каком веществе
                   3057: идет речь?
                   3058: 
                   3059: Ответ:
                   3060: Крахмал.
                   3061: 
                   3062: Комментарий:
                   3063: Крахмал использовался для огромных воротничков и манжет, а также для
                   3064: производства пудры. К началу XIX века эти аксессуары и сами вышли из
                   3065: моды.
                   3066: 
                   3067: Источник:
                   3068: С.Дж. Винсент. Анатомия моды. Манеры одеваться от эпохи Возрождения до
                   3069: наших дней. - М.: Новое литературное обозрение, 2015.
                   3070: 
                   3071: Автор:
                   3072: Станислав Попов (Тверь)
                   3073: 
                   3074: Вопрос 24:
                   3075: Ироничный ирландец Дилан Моран задается вопросом: "Как изображение
                   3076: совершенно обычной ирландской улыбки должно заставить меня СДЕЛАТЬ
                   3077: ЭТО?". Какие два слова мы заменили на "СДЕЛАТЬ ЭТО"?
                   3078: 
                   3079: Ответ:
                   3080: Бросить курить.
                   3081: 
                   3082: Зачет:
                   3083: Не курить, близкие по смыслу ответы из двух слов.
                   3084: 
                   3085: Комментарий:
                   3086: Моран комментирует изображения всяких ужасов на пачках с сигаретами, а
                   3087: именно - желтые зубы с пародонтозом. Авторы пакета на этом тоже бросают
                   3088: курить что попало и шиворот-навыворот и прощаются с вами.
                   3089: 
                   3090: Источник:
                   3091: Dylan Moran. What It Is (2009).
                   3092: 
                   3093: Автор:
                   3094: Станислав Попов (Тверь)
                   3095: 
                   3096: Тур:
                   3097: 9 тур. "Питер Пингвинс" (Санкт-Петербург)
                   3098: 
                   3099: Вопрос 1:
                   3100: (pic: 20150148.jpg)
                   3101:    Подписи под полученными вами картинками почти совпадают.
                   3102: Воспроизведите подпись под нижней.
                   3103: 
                   3104: Ответ:
                   3105: Ни пуха ни пера.
                   3106: 
                   3107: Комментарий:
                   3108: На первой картинке изображены персонажи "Винни-Пуха" без Винни-Пуха и
                   3109: персонажи "Питера Пэна" без Питера Пэна ("pen" - перо по-английски). Со
                   3110: второй картинкой всё ясно. Команда "Питер Пингвинс" желает всем удачи!
                   3111:    z-checkdb: Peter Pan не имеет отношения к слову "pen".
                   3112: 
                   3113: Автор:
                   3114: Мария Топаж
                   3115: 
                   3116: Вопрос 2:
                   3117: С 1990 года ИХ освободили от "школы", но это не значит, что о параграфах
                   3118: и скобках можно было забыть. Назовите фильм 1993 года, не так давно
                   3119: вдохновивший ту из НИХ, которая, тем не менее, еще в школе.
                   3120: 
                   3121: Ответ:
                   3122: "Список Шиндлера".
                   3123: 
                   3124: Комментарий:
                   3125: Параграфы и скобки входят в программу обязательных фигур в фигурном
                   3126: катании, которую называют "школой", - она была отменена в 1990 году. На
                   3127: знаменитую произвольную программу прошлого года Юлию Липницкую и ее
                   3128: команду вдохновила музыка из "Списка Шиндлера".
                   3129: 
                   3130: Источник:
                   3131:    1. http://ru.wikipedia.org/wiki/Обязательные_фигуры
                   3132:    2. http://ru.wikipedia.org/wiki/Липницкая,_Юлия_Вячеславовна
                   3133: 
                   3134: Автор:
                   3135: Мария Топаж
                   3136: 
                   3137: Вопрос 3:
                   3138: В книге Евгения Лукина "Алая аура протопарторга" описывается
                   3139: православно-социалистический город Лыцк. Протопарторг - третий чин
                   3140: лыцкой иерархии. Первый чин имеет глава города - партиарх. Назовите
                   3141: второй чин лыцкой иерархии, который имеют, например, замы партиарха.
                   3142: 
                   3143: Ответ:
                   3144: Митрозамполит.
                   3145: 
                   3146: Источник:
                   3147: Евгений Лукин. Алая аура протопарторга.
                   3148: http://www.flibusta.net/b/132417/read
                   3149: 
                   3150: Автор:
                   3151: Михаил Якубсон
                   3152: 
                   3153: Вопрос 4:
                   3154: Комментатор матча Аргентина - Нигерия назвал атаку сборной Аргентины с
                   3155: участием ЭзекьЕля ЛавЕсси ТАКОЙ. Если ИКС не ТАКОЙ - он белый. Мы не
                   3156: спрашиваем вас, что мы заменили на "ИКС". Назовите слово, которое
                   3157: заменили на "ТАКОЙ".
                   3158: 
                   3159: Ответ:
                   3160: Рифмованный.
                   3161: 
                   3162: Комментарий:
                   3163: В атаке принимали участие Месси и Лавесси.
                   3164: 
                   3165: Источник:
                   3166: Трансляция матча Аргентина - Нигерия.
                   3167: 
                   3168: Автор:
                   3169: Татьяна Шарапа
                   3170: 
                   3171: Вопрос 5:
                   3172: Вступив в Киев со стороны ПечЕрска, полковник петлюровской армии
                   3173: БолботУн принял решение ДЕЛАТЬ ЭТО. Вальсируя, кавалеру следует обратить
                   3174: особое внимание на то, чтобы не ДЕЛАТЬ ЭТО. Что мы заменили словами
                   3175: "ДЕЛАТЬ ЭТО"?
                   3176: 
                   3177: Ответ:
                   3178: Наступать на Подол.
                   3179: 
                   3180: Зачет:
                   3181: Наступать на подол.
                   3182: 
                   3183: Комментарий:
                   3184: Подол - район Киева.
                   3185: 
                   3186: Источник:
                   3187: М. Булгаков. Белая гвардия. http://www.flibusta.net/b/261423/read
                   3188: 
                   3189: Автор:
                   3190: Александр Топаж
                   3191: 
                   3192: Вопрос 6:
                   3193: Недавно под автостоянкой в Лестере английскими археологами было
                   3194: обнаружено захоронение легендарного короля Ричарда III. Анализ останков
                   3195: показал, что традиционное представление о степени физического уродства
                   3196: этого монарха было сильно преувеличенным. Воспроизведите заглавие
                   3197: заметки об этом на сайте "Русская планета", состоящее из трех слов.
                   3198: 
                   3199: Ответ:
                   3200: Могила исправила горбатого.
                   3201: 
                   3202: Зачет:
                   3203: С точностью до порядка слов.
                   3204: 
                   3205: Комментарий:
                   3206: Оказалось, что горбатость Ричарда III - исключительно художественный
                   3207: вымысел авторов, например, Шекспира.
                   3208: 
                   3209: Источник:
                   3210: http://www.rusplt.ru/world/mogila-ispravila-gorbatogo-10192.html
                   3211: 
                   3212: Автор:
                   3213: Александр Топаж
                   3214: 
                   3215: Вопрос 7:
                   3216: Главный герой компьютерной игры вооружен разнообразными бластерами и по
                   3217: долгу своей нелегкой и опасной службы защищает корабль. Название этой
                   3218: игры можно с некоторой вольностью перевести с английского как "ИКС".
                   3219: Посетитель сайта geektimes.ru [гиктаймс точка ру] с ником gwer [гвер]
                   3220: считает, что наличие ИКСА поможет нам защититься от инопланетного
                   3221: вторжения. Назовите ИКС двумя словами.
                   3222: 
                   3223: Ответ:
                   3224: Космический мусор.
                   3225: 
                   3226: Комментарий:
                   3227: "Space cop" действительно можно с некоторой вольностью перевести как
                   3228: "Космический мусор".
                   3229: 
                   3230: Источник:
                   3231:    1. http://www.igrystrelyalki.ru/dlya-malchikov/1045-kosmicheskiy-policeyskiy.html
                   3232:    2. http://geektimes.ru/post/230199/
                   3233: 
                   3234: Автор:
                   3235: Алексей Палецких
                   3236: 
                   3237: Вопрос 8:
                   3238: Согласно академическому источнику, ИХ всего 11: а, б, в, ж, и, к, о, с,
                   3239: у, э, я. Назовите ИХ.
                   3240: 
                   3241: Ответ:
                   3242: Однобуквенные слова русского языка.
                   3243: 
                   3244: Зачет:
                   3245: По смыслу.
                   3246: 
                   3247: Комментарий:
                   3248: Это междометия, местоимения, предлоги, союзы и частицы.
                   3249: 
                   3250: Источник:
                   3251: Словарь Ожегова.
                   3252: 
                   3253: Автор:
                   3254: Михаил Дворкин
                   3255: 
                   3256: Вопрос 9:
                   3257: Прослушайте перевод англоязычной шутки: "Я решил себя заморозить до ИКС
                   3258: градусов по Цельсию. Жена говорит, что я умру, а я думаю, что всё будет
                   3259: ИГРЕК". Запишите точно, что мы заменили на ИКС и ИГРЕК.
                   3260: 
                   3261: Ответ:
                   3262: -273, 0K.
                   3263: 
                   3264: Источник:
                   3265: http://memegenerator.net/instance/11179723
                   3266: 
                   3267: Автор:
                   3268: Михаил Дворкин
                   3269: 
                   3270: Вопрос 10:
                   3271: "Милостивый манифест" Емельяна Пугачёва начинался словами: "Жалую вас и
                   3272: крестом, и АЛЬФОЮ, и волею, и землею, и угодьями, и лесами, и лугами, и
                   3273: рыбными ловлями, и всем беспошлинно и безданно...". АЛЬФА явилась
                   3274: важнейшим атрибутом австрийского триумфа. Что мы заменили на АЛЬФУ?
                   3275: 
                   3276: Ответ:
                   3277: Борода.
                   3278: 
                   3279: Комментарий:
                   3280: Примечательно, что борода в этом списке является даже большей ценностью,
                   3281: чем воля, хотя со времен Петровских указов прошло достаточно времени.
                   3282: Австрийский триумф - победа Кончиты Вурст на Евровидении-2014.
                   3283: 
                   3284: Источник:
                   3285:    1. А.Н. Крылов. Мои воспоминания.
                   3286: http://www.flibusta.net/b/179782/read
                   3287:    2. http://ru.wikipedia.org/wiki/Кончита_Вурст
                   3288: 
                   3289: Автор:
                   3290: Александр Топаж
                   3291: 
                   3292: Вопрос 11:
                   3293: Прослушайте строфу из поэмы Давида Самойлова "Юлий КлОмпус":
                   3294:    Прозрачнее, чем патер ЧЕстертона,
                   3295:    Среди белья крахмально выстиранного,
                   3296:    Лежал он, отрешась от женственного,
                   3297:    В печальном постиженьи истинного.
                   3298:    В зачитанном вам отрывке мы заменили подростка на взрослого, а
                   3299: русского - на англичанина. Восстановите исходный текст и назовите имя
                   3300: подростка, которого мы заменили.
                   3301: 
                   3302: Ответ:
                   3303: Варфоломей.
                   3304: 
                   3305: Комментарий:
                   3306: Замененные слова - "отрок Нестерова". Давид Самойлов образно сравнил
                   3307: лирического героя с юным героем картины Нестерова "Видение отроку
                   3308: Варфоломею".
                   3309: 
                   3310: Источник:
                   3311: Д. Самойлов. Избранное. http://www.flibusta.net/b/354708/read
                   3312: 
                   3313: Автор:
                   3314: Александр Топаж
                   3315: 
                   3316: Вопрос 12:
                   3317: (pic: 20150149.jpg)
                   3318:    На раздаточном материале вам представлен фрагмент фэнтезийного
                   3319: интернет-комикса. Главный герой на следующей панели говорит, что сделал
                   3320: ЭТО. "ЭТО" - авторский неологизм. Напишите этот неологизм.
                   3321: 
                   3322: Ответ:
                   3323: Elfie.
                   3324: 
                   3325: Зачет:
                   3326: Элфи, Эльфи, Сэльфи, а также любая подобная комбинация слов "селфи" и
                   3327: "эльф". Незачет: Сэлфи.
                   3328: 
                   3329: Комментарий:
                   3330: Как можно заметить, главный герой - эльф, который рисует себя.
                   3331: 
                   3332: Источник:
                   3333: http://www.lfg.co/page/774/
                   3334: 
                   3335: Автор:
                   3336: Алексей Палецких
                   3337: 
                   3338: Вопрос 13:
                   3339: По мнению некоторых историков, Венеция была обязана процветанием своей
                   3340: лагуне, которая слишком глубокая для ПЕРВОЙ и слишком мелкая для
                   3341: ВТОРОГО. А кем, по мнению ТРЕТЬЕГО, являются ПЕРВАЯ и ВТОРОЙ для нашей
                   3342: страны?
                   3343: 
                   3344: Ответ:
                   3345: [Единственными] союзниками.
                   3346: 
                   3347: Комментарий:
                   3348: Речь идет об армии и флоте. Согласно мнению историков, лагуна являлась
                   3349: прекрасной естественной крепостью, защищавшей Венецию от морского или
                   3350: сухопутного вторжения. Согласно известному выражению Александра III,
                   3351: армия и флот - единственные союзники России.
                   3352: 
                   3353: Источник:
                   3354:    1. А. Генис. Космополит. http://www.flibusta.net/b/358899/read
                   3355:    2. http://www.bibliotekar.ru/encSlov/19/19.htm
                   3356: 
                   3357: Автор:
                   3358: Александр Топаж
                   3359: 
                   3360: Вопрос 14:
                   3361: В стихотворении Александра Городницкого 2003 года есть строки:
                   3362:    Всё будет вновь: Ливонская война
                   3363:    И [ПРОПУСК] и Кавказа.
                   3364:    Заполните пропуск двумя словами.
                   3365: 
                   3366: Ответ:
                   3367: Покорение Крыма.
                   3368: 
                   3369: Зачет:
                   3370: Покоренье Крыма.
                   3371: 
                   3372: Источник:
                   3373: http://magazines.russ.ru/neva/2003/3/gorod.html
                   3374: 
                   3375: Автор:
                   3376: Михаил Якубсон
                   3377: 
                   3378: Вопрос 15:
                   3379: Заметка на новостной ленте Google, посвященная переговорам между Италией
                   3380: и Россией о смягчении торговых санкций, называлась: "Рим просит помощи у
                   3381: первого...". Какое слово мы пропустили в этом названии?
                   3382: 
                   3383: Ответ:
                   3384: Третий.
                   3385: 
                   3386: Комментарий:
                   3387: Москва - третий Рим.
                   3388: 
                   3389: Источник:
                   3390: Новостная лента Google.
                   3391: 
                   3392: Автор:
                   3393: Александр Топаж
                   3394: 
                   3395: Вопрос 16:
                   3396: В тексте этого вопроса заменена одна буква.
                   3397:    Выдающийся итальянский певец Франко Бонисолли называл одного своего
                   3398: коллегу "фламинго", намекая тем самым на отсутствие у того... Ответьте
                   3399: достаточно точно: чего?
                   3400: 
                   3401: Ответ:
                   3402: Нот "си" и "до" в диапазоне.
                   3403: 
                   3404: Зачет:
                   3405: По упоминанию данных нот.
                   3406: 
                   3407: Комментарий:
                   3408: Речь идет о Пласидо Доминго, имя которого Бонисолли по упомянутой
                   3409: причине сокращает до "Пламинго".
                   3410: 
                   3411: Источник:
                   3412: http://izvestia.ru/news/311888
                   3413: 
                   3414: Автор:
                   3415: Максим Мельцин
                   3416: 
                   3417: Вопрос 17:
                   3418: Согласно шутке, ОНА хранится в палате мер и весов как эталон рекурсии.
                   3419: Назовите ЕЕ четырьмя словами.
                   3420: 
                   3421: Ответ:
                   3422: Палата мер и весов.
                   3423: 
                   3424: Источник:
                   3425: https://twitter.com/FiztehRadio/status/451645816990892032/
                   3426: 
                   3427: Автор:
                   3428: Михаил Дворкин
                   3429: 
                   3430: Вопрос 18:
                   3431: Статья на портале autonews.ru [автоньюс точка ру] рассказывает о
                   3432: компаниях "Kor_Ecologic", "StreetScooter", "Maasaica" [кор эколОджик,
                   3433: стритскУтер, маасАика] и других фирмах, использующих современные
                   3434: инновационные технологии в автомобилестроении. Эти компании названы в
                   3435: упомянутой статье тем же словом, каким принято называть, например,
                   3436: Петра, Ивана или Франциска. Воспроизведите это слово.
                   3437: 
                   3438: Ответ:
                   3439: Первопечатники.
                   3440: 
                   3441: Комментарий:
                   3442: Эти компании первыми стали печатать детали автомобилей на 3D-принтере.
                   3443: 
                   3444: Источник:
                   3445: http://www.autonews.ru/autobusiness/news/1796156/
                   3446: 
                   3447: Автор:
                   3448: Александр Топаж
                   3449: 
                   3450: Вопрос 19:
                   3451: Как сообщает сайт vesti.ru [вести точка ру], в 2014 году эта организация
                   3452: произвела ребрендинг и теперь принимает в свои ряды также
                   3453: афроамериканцев, евреев и гомосексуалов. Неизменным останется лишь
                   3454: дресс-код. Назовите эту организацию.
                   3455: 
                   3456: Ответ:
                   3457: Ку-клукс-клан.
                   3458: 
                   3459: Источник:
                   3460: http://www.vesti.ru/doc.html?id=2115293
                   3461: 
                   3462: Автор:
1.2       rubashki 3463: Даниил Черкашин
1.1       rubashki 3464: 
                   3465: Вопрос 20:
                   3466: В слове по крайней мере три буквы "м" и две буквы "и". Какое слово мы
                   3467: заменили?
                   3468: 
                   3469: Ответ:
                   3470: Минимум.
                   3471: 
                   3472: Комментарий:
                   3473: Слово "минимум" заменено тремя словами "по крайней мере".
                   3474: 
                   3475: Автор:
                   3476: Татьяна Шарапа
                   3477: 
                   3478: Вопрос 21:
                   3479: На иллюстрации к июльскому событию ОНА делает фейспалм. Назовите ЕЕ.
                   3480: 
                   3481: Ответ:
                   3482: Статуя Христа-Искупителя.
                   3483: 
                   3484: Зачет:
                   3485: Статуя Христа в Рио-де-Жанейро и т.п. по смыслу.
                   3486: 
                   3487: Комментарий:
                   3488: Бразильцы на чемпионате мира по футболу 2014 года действительно дали
                   3489: повод.
                   3490: 
                   3491: Источник:
                   3492: http://www.kaztur.ru/newst/news/?id=6192
                   3493: 
                   3494: Автор:
1.2       rubashki 3495: Даниил Черкашин
1.1       rubashki 3496: 
                   3497: Вопрос 22:
                   3498: Прослушайте цитату из истории с сайта zadolba.li [задолба точка ли]:
                   3499: "Неудивительно, что встретив - внезапно - марку сливок, которая выходит
                   3500: в упаковке 200 граммов и 500 граммов, а не 470 или 180, я уже
                   3501: обрадовалась. Не тут-то было. Ставишь пачку на кухонные весы - и да,
                   3502: конечно, 200 граммов ровно. С упаковкой. Самих сливок на 15 граммов
                   3503: меньше". Запишите абсолютно точно тремя словами, как была озаглавлена
                   3504: эта история.
                   3505: 
                   3506: Ответ:
                   3507: "И ты, брутто".
                   3508: 
                   3509: Источник:
                   3510: http://zadolba.li/story/15760
                   3511: 
                   3512: Автор:
                   3513: Михаил Дворкин
                   3514: 
                   3515: Вопрос 23:
                   3516: История Европы, можно сказать, началась с АЛЬФА. АЛЬФА находится на
                   3517: расстоянии 280 километров от Европы. Назовите АЛЬФА или АЛЬФУ.
                   3518: 
                   3519: Ответ:
                   3520: Галилей.
                   3521: 
                   3522: Зачет:
                   3523: Галилея.
                   3524: 
                   3525: Комментарий:
                   3526: Галилей открыл спутник Юпитера - Европу. Галилея находится на расстоянии
                   3527: примерно 280 километров от Кипра.
                   3528: 
                   3529: Автор:
                   3530: Александр Топаж
                   3531: 
                   3532: Вопрос 24:
                   3533:    <раздатка>
                   3534:    Если бы ________ была у ________,
                   3535:    Она бы с ________ по жизни ________,
                   3536:    Но ________ была от ________ ________,
                   3537:    Поскольку она проживала с ________.
                   3538:    </раздатка>
                   3539:    В стихотворении Татьяны Щёголевой мы пропустили восемь слов: шесть
                   3540: имен собственных, глагол и наречие. Запишите пропущенный глагол и
                   3541: наречие.
                   3542: 
                   3543: Ответ:
                   3544: Шагала, вдали.
                   3545: 
                   3546: Комментарий:
                   3547:    Если бы Гала была у Шагала,
                   3548:    Она бы с Шагалом по жизни шагала,
                   3549:    Но Гала была от Шагала вдали,
                   3550:    Поскольку она проживала с Дали.
                   3551: 
                   3552: Источник:
                   3553: http://www.orlita.org/вечер-веселой-поэзии/
                   3554: 
                   3555: Автор:
                   3556: Михаил Дворкин
                   3557: 
                   3558: Тур:
                   3559: 10 тур. "Брейн" (Москва)
                   3560: 
                   3561: Вопрос 1:
                   3562: Джалли помогала своей хозяйке зарабатывать деньги, принимая участие в
                   3563: выступлениях в "чудесной" части Парижа XV века. Полли и Молли внесли
                   3564: вклад в науку на рубеже XX-XXI веков. Шрек прославился после того, как
                   3565: наконец-то был подстрижен в апреле 2004 года и стал талисманом Новой
                   3566: Зеландии. Догадавшись, кто такие Джалли, Полли, Молли и Шрек, назовите
                   3567: число, которое на сегодняшний день их объединяет.
                   3568: 
                   3569: Ответ:
                   3570: 2015.
                   3571: 
                   3572: Комментарий:
                   3573: Джалли - козочка Эсмеральды из романа "Собор Парижской Богоматери"
                   3574: Виктора Гюго. Полли и Молли - клонированные овечки, которым был введен
                   3575: человеческий ген для возможного применения в медицине. Шрек -
                   3576: мериносовый баран, который прятался от людей шесть лет. 2015 - год козы,
                   3577: овцы, барана.
                   3578: 
                   3579: Источник:
                   3580:    1. В. Гюго. Собор Парижской Богоматери.
                   3581: http://www.flibusta.net/b/151369/read
                   3582:    2. http://ru.wikipedia.org/wiki/Долли_(овца)
                   3583:    3. http://ru.wikipedia.org/wiki/Шрек_(овца)
                   3584:    4. http://ru.wikipedia.org/wiki/Китайский_гороскоп
                   3585: 
                   3586: Автор:
                   3587: Дарья Коротина (Саранск - Москва)
                   3588: 
                   3589: Вопрос 2:
                   3590: Внимание, в вопросе есть замены.
                   3591:    Один школьник, начитавшийся мифов Древней Греции, был абсолютно
                   3592: уверен, что в слове "ДОЛИНА" нужно писать букву "А" вместо "О". Он
                   3593: объяснял это тем, что слово "ДОЛИНА" произошло от слова "ДАЛЬ". Назовите
                   3594: слова, которые мы заменили словами "ДОЛИНА" и "ДАЛЬ", если первое из них
                   3595: - глагол, а второе - имя собственное.
                   3596: 
                   3597: Ответ:
                   3598: Хоронить, Харон.
                   3599: 
                   3600: Комментарий:
                   3601: Школьник убежден, что глагол "ХОРОНИТЬ" произошел от имени перевозчика
                   3602: душ умерших ХАРОНА, поэтому и пишет слово с ошибкой.
                   3603: 
                   3604: Источник:
                   3605: Личный опыт автора вопроса.
                   3606: 
                   3607: Автор:
                   3608: Дарья Коротина (Саранск - Москва)
                   3609: 
                   3610: Вопрос 3:
                   3611: В одной из статей Reuters за 2012 год сообщается, что в немецком языке
                   3612: появился неологизм "вульфить", образованный от фамилии Кристиана Вульфа,
                   3613: являвшегося на тот момент президентом ФРГ. Глагол имеет несколько
                   3614: значений. Например, уклончиво рассказывать о чем-либо, недоговаривать,
                   3615: но так, чтобы говорящего нельзя было уличить в откровенной лжи. Это
                   3616: значение связано с реакцией президента на обвинения в коррупции и
                   3617: давлении на СМИ. Также в статье упоминается глагол "меркелить", т.е.
                   3618: сомневаться, быть нерешительным. А какой неологизм используется в
                   3619: значении "украсть интеллектуальную собственность"?
                   3620: 
                   3621: Ответ:
                   3622: Сгуттенбергить.
                   3623: 
                   3624: Комментарий:
                   3625: Неологизм появился после отставки министра обороны ФРГ Карла-Теодора
                   3626: Гуттенберга из-за обвинений в плагиате при написании диссертации.
                   3627: 
                   3628: Источник:
                   3629: http://lenta.ru/news/2012/01/13/verbum/
                   3630: 
                   3631: Автор:
                   3632: Дарья Коротина (Саранск - Москва)
                   3633: 
                   3634: Вопрос 4:
                   3635: Некоторые цветы обладают ЭТИМ свойством. Например, цветок, который
                   3636: является шуточным ответом на многие вопросы. Другой цветок именно из-за
                   3637: ЭТОГО свойства был любимым цветком главной героини произведения XIX
                   3638: века. Назовите это произведение.
                   3639: 
                   3640: Ответ:
                   3641: "Дама с камелиями".
                   3642: 
                   3643: Комментарий:
                   3644: Это свойство - отсутствие запаха. Первый цветок, упомянутый в вопросе, -
                   3645: гладиолус. "Потому что гладиолус" - популярный шуточный ответ на многие
                   3646: вопросы. Главная героиня романа Александра Дюма-сына куртизанка
                   3647: Маргарита Готье была больна чахоткой и любила камелии за то, что они не
                   3648: пахнут, так как от аромата других цветов из-за болезни у нее кружилась
                   3649: голова.
                   3650: 
                   3651: Источник:
                   3652:    1. http://lurkmore.to/Потому_что_гладиолус
                   3653:    2. А. Дюма-сын. Дама с камелиями.
                   3654: http://www.flibusta.net/b/112277/read
                   3655: 
                   3656: Автор:
                   3657: Дарья Коротина (Саранск - Москва)
                   3658: 
                   3659: Вопрос 5:
                   3660: Представители знати и обладатели значительных денежных сумм традиционно
                   3661: издревле старались перещеголять соседей демонстрацией своего богатства.
                   3662: Кто-то строил вычурные дворцы, кто-то разбивал роскошные сады, а
                   3663: британские аристократы XVIII века нанимали себе в услужение особых
                   3664: людей. Основные требования к соискателям должности были таковы: носить
                   3665: только выданную одежду, ни при каких обстоятельствах не покидать
                   3666: пределов поместья, не общаться ни с кем, кроме хозяина, не стричь волос
                   3667: и ногтей; пожилой возраст и знание Библии крайне приветствовались.
                   3668: Заключались такие контракты, как правило, на срок от семи лет, но и
                   3669: жалование выплачивалось по тем временам более чем достойное. В качестве
                   3670: кого же нанимались эти люди?
                   3671: 
                   3672: Ответ:
                   3673: Отшельники.
                   3674: 
                   3675: Комментарий:
                   3676: Желая показать соседям и друзьям свою набожность или просто поразить
                   3677: всех эксцентричностью, аристократы нанимали себе декоративных
                   3678: отшельников. Эти люди жили на прилегавших к поместью землях в палатке
                   3679: или пещере и... Собственно, это всё, что они делали.
                   3680: 
                   3681: Источник:
                   3682: http://www.hermitary.com/lore/ornamental_hermits.html
                   3683: 
                   3684: Автор:
                   3685: Максим Фомин (Москва)
                   3686: 
                   3687: Вопрос 6:
                   3688: Известны случаи, когда животные используют в своих целях различные
                   3689: предметы: выдра использует камни, чтобы разбивать ракушки, птица
                   3690: шалашник использует ракушки и цветы, чтобы привлечь самку. Для чего
                   3691: речной дельфин использует обезглавленную тушку рыбы?
                   3692: 
                   3693: Ответ:
                   3694: Мастурбация.
                   3695: 
                   3696: Комментарий:
                   3697: Зафиксированы случаи, когда самцы речного дельфина использовали
                   3698: обезглавленные тушки рыб для мастурбации.
                   3699: 
                   3700: Источник:
1.18      rubashki 3701:    1. http://mixstuff.ru/archives/26868
1.1       rubashki 3702:    2. http://www.zoopage.ru/stat.php?idstat=68
                   3703:    3. http://www.youtube.com/watch?v=ownANj8dKcQ
                   3704: 
                   3705: Автор:
                   3706: Дарья Беличенко (Москва)
                   3707: 
                   3708: Вопрос 7:
                   3709: Говорят, что природа отдыхает на детях гениев. Однако прапрапраправнук
                   3710: одного классика американской литературы также смог добиться всемирной
                   3711: известности, правда, уже в качестве популярного электронного музыканта.
                   3712: А чтобы почтить именитого родственника, в качестве сценического
                   3713: псевдонима музыкант взял имя антагониста самой известного произведения
                   3714: писателя. Хотя, в отличие от этого кровожадного персонажа, музыкант -
                   3715: известный вегетарианец. Назовите книгу, о которой идет речь.
                   3716: 
                   3717: Ответ:
                   3718: "Моби Дик".
                   3719: 
                   3720: Комментарий:
                   3721: Родители Ричарда Мелвилла Холла очень гордились своим именитым предком:
                   3722: американским классиком Германом Мелвиллом, автором знаменитого романа
                   3723: "Моби Дик". Поэтому мальчику от прапрапрапрадедушки досталось не только
                   3724: второе имя, но и кличка "Моби", с которой тот не расстается и по сей
                   3725: день.
                   3726: 
                   3727: Источник:
1.13      rubashki 3728: http://en.wikipedia.org/wiki/Moby
1.1       rubashki 3729: 
                   3730: Автор:
                   3731: Антон Лазарев (Москва)
                   3732: 
                   3733: Вопрос 8:
                   3734: Оводов разделяют на три семейства в соответствии с тем, где на теле
                   3735: животного развиваются их личинки: подкожные, желудочные и ТАКИЕ. Если
                   3736: ТАКОЙ овод отложит слишком много личинок на одном животном, то оно
                   3737: умрет. Причина смерти совпадает с названием романа 2001 года. Назовите
                   3738: автора этого романа.
                   3739: 
                   3740: Ответ:
                   3741: [Чак] Паланик.
                   3742: 
                   3743: Комментарий:
                   3744: Личинки носоглоточого овода развиваются в носоглотке; когда их слишком
                   3745: много, животное не может дышать и задыхается.
                   3746: 
                   3747: Источник:
                   3748:    1. http://www.zooeco.com/int/int-nasek23-111.html
                   3749:    2. http://dic.academic.ru/dic.nsf/dic_biology/3788/
                   3750:    3. http://ru.wikipedia.org/wiki/Удушье_(роман)
                   3751: 
                   3752: Автор:
                   3753: Дарья Беличенко (Москва)
                   3754: 
                   3755: Вопрос 9:
                   3756: Фактопедия утверждает, что эти животные бывают шарообразными и
                   3757: нитевидными. Одно время на англоязычном сайте отечественной организации
                   3758: им был посвящен целый научный институт. Назовите писателя, который
                   3759: поведал об особенной способности одного из представителей этих животных
                   3760: к обработке драгоценных материалов.
                   3761: 
                   3762: Ответ:
                   3763: Пушкин.
                   3764: 
                   3765: Комментарий:
                   3766: Как переводчики РАН, так и составители Фактопедии перепутали бЕлок и
                   3767: белОк. В произведении А.С. Пушкина "Сказка о царе Салтане" Белка грызла
                   3768: золотые орехи с чистыми изумрудами.
                   3769: 
                   3770: Источник:
                   3771:    1. http://factopedia.ru/animal/belki
                   3772:    2. http://ru.wikipedia.org/wiki/Институт_белка_РАН
                   3773:    3. А.С. Пушкин. Сказка о царе Салтане.
                   3774: 
                   3775: Автор:
                   3776: Антон Рогожин (Саранск)
                   3777: 
                   3778: Вопрос 10:
                   3779: Однажды автор наткнулся на видеоролик, посвященный фитнесу. В
                   3780: комментарии к нему указывается, что, несмотря на то что ролик
                   3781: англоязычный, всё ясно без перевода, потому что объясняют... Как?
                   3782: 
                   3783: Ответ:
                   3784: На пальцах.
                   3785: 
                   3786: Комментарий:
                   3787: Речь идет о т.н. фингер-фитнесе, т.е. о тренировке пальцев.
                   3788: 
                   3789: Источник:
                   3790: https://vk.com/topic-41352307_28420063
                   3791: 
                   3792: Автор:
                   3793: Антон Рогожин (Саранск)
                   3794: 
                   3795: Вопрос 11:
                   3796: Герой ЭТОГО отечественного постмодернистского романа получил свое имя в
                   3797: честь русского писателя XX века и российского революционера XX века. Его
                   3798: имя лишь одной буквой отличается от названия города, руины которого
                   3799: находятся на территории современного Ирака. Напишите название романа,
                   3800: если известно, что фамилия героя образована от названия тюркоязычного
                   3801: народа, составляющего более половины населения региона 16.
                   3802: 
                   3803: Ответ:
                   3804: "Generation П".
                   3805: 
                   3806: Комментарий:
                   3807: Герой романа Виктора Пелевина "Generation П" был назван в честь Василия
                   3808: Аксёнова и Владимира Ленина. Его имя одной буквой отличается от названия
                   3809: города "Вавилон". Фамилия героя "Татарский" образована от названия
                   3810: народа "татары", а регион 16 - это Татарстан.
                   3811: 
                   3812: Источник:
                   3813:    1. В. Пелевин. Generation П.
                   3814:    2. http://ru.wikipedia.org/wiki/Вавилон
                   3815:    3. http://ru.wikipedia.org/wiki/Татары
                   3816: 
                   3817: Автор:
                   3818: Дарья Коротина (Саранск - Москва)
                   3819: 
                   3820: Вопрос 12:
                   3821: Прослушайте шуточное стихотворение Джеймса Линдона, в котором пропущено
                   3822: одно слово:
                   3823:    Плотная пища жен ПРОПУСК
                   3824:    Только на пользу им шла, не иначе.
                   3825:    Весили жены, согласно молве,
                   3826:    Каждая - как предыдущие две.
                   3827:    Догадавшись, о чьих женах писал Линдон, назовите роман американского
                   3828: писателя, в котором герои, для того чтобы открыть сейф в банке,
                   3829: использовали то, что изобрел вышеупомянутый муж.
                   3830: 
                   3831: Ответ:
                   3832: Код да Винчи.
                   3833: 
                   3834: Комментарий:
                   3835: В стихотворении пропущено слово "Фибоначчи". В романе Дэна Брауна в
                   3836: качестве кода доступа к сейфу герои использовали последовательность
                   3837: Фибоначчи, зашифрованную в анаграмме.
                   3838: 
                   3839: Источник:
                   3840:    1. http://www.tutoronline.ru/blog/chisla-fibonachchi-ishhem-sekret-mirozdanija
                   3841:    2. Д. Браун. Код да Винчи. - Глава 42.
                   3842: 
                   3843: Автор:
                   3844: Дарья Коротина (Саранск - Москва)
                   3845: 
                   3846: Вопрос 13:
                   3847: В книге психиатра Оливера Сакса описывается случай, когда пациент,
                   3848: страдавший от постоянных тиков и непроизвольных движений, нашел свое
                   3849: признание, став ИМ. Один из НИХ сетовал, что его судьба - видеть перед
                   3850: собой три задницы. Назовите ИХ.
                   3851: 
                   3852: Ответ:
                   3853: Барабанщики.
                   3854: 
                   3855: Комментарий:
                   3856: Пациент, страдавший от нервных тиков, стал превосходным джазовым
                   3857: барабанщиком. К слову сказать, по излечении его импровизационные
                   3858: способности практически исчезли. А слова о незавидной судьбе принадлежат
                   3859: барабанщику "Битлз" Ринго Старру.
                   3860: 
                   3861: Источник:
                   3862:    1. О. Сакс. Человек, который принял жену за шляпу.
                   3863: http://www.flibusta.net/b/95975/read
                   3864:    2. http://www.beatl.ru/beatles/citati/ringo-starr.html
                   3865: 
                   3866: Автор:
                   3867: Антон Рогожин (Саранск)
                   3868: 
                   3869: Вопрос 14:
                   3870: Казобон, герой романа Умберто Эко "Маятник Фуко", говорил, что каждое
                   3871: утро за бритьем мы глядим в глаза двойника, который обречен пожизненно
                   3872: быть ТАКИМ. В бейсболе ТАКОГО питчера называют "southpaw", что значит
                   3873: "южная лапа". Назовите ТАКОГО одним словом.
                   3874: 
                   3875: Ответ:
                   3876: Левша.
                   3877: 
                   3878: Комментарий:
                   3879: Бейсбольные поля обычно проектируются таким образом, что отбивающий
                   3880: стоит лицом на восток, чтобы полуденное или вечернее солнце не светило
                   3881: ему в глаза. Это означает, что питчеры-левши бросают своей "южной"
                   3882: рукой. Казобон говорил об отражении в зеркале.
                   3883: 
                   3884: Источник:
                   3885:    1. У. Эко. Маятник Фуко.
                   3886:    2. http://ru.wikipedia.org/wiki/Левша#.D0.9B.D0.B5.D0.B2.D1.88.D0.B0_.C2.ABSouthpaw.C2.BB
                   3887: 
                   3888: Автор:
                   3889: Дарья Коротина (Саранск - Москва)
                   3890: 
                   3891: Вопрос 15:
                   3892: Героиня американского сериала "Как я встретил вашу маму" Робин Щербатски
                   3893: и ее подруга пели, что "два ИХ лучше, чем один". Это была милая детская
                   3894: песенка, которая, однако, друзьям Робин показалась пошлой. Назовите ИХ,
                   3895: если известно, что ИХ существует два вида: обыкновенный (или речной) и
                   3896: канадский.
                   3897: 
                   3898: Ответ:
                   3899: Бобер.
                   3900: 
                   3901: Комментарий:
                   3902: Девушки вели программу для детей. У каждой из них был игрушечный бобер.
                   3903: Они пели песню о дружбе их бобров. Однако в американской табуированной
                   3904: лексике слово "beaver" (бобер) имеет совсем иное значение - женские
                   3905: половые органы. Отсюда и соответствующая реакция ее друзей.
                   3906: 
                   3907: Источник:
                   3908:    1. "How I met your mother", s06e09.
                   3909:    2. http://www.multitran.ru/c/m.exe?CL=1&s=beaver&l1=1
                   3910:    3. http://ru.wikipedia.org/wiki/Бобры
                   3911: 
                   3912: Автор:
                   3913: Дарья Коротина (Саранск - Москва)
                   3914: 
                   3915: Вопрос 16:
                   3916: В ПЕРВОМ рассказывается о мире, где государство постановило, что 2+2 =
                   3917: 5. Во ВТОРОМ - о двух мирах, в одном из которых на небе висят две луны.
                   3918: Название ВТОРОГО отличается от названия ПЕРВОГО одним символом. Этот
                   3919: символ стал кодовым именем вымышленного персонажа известной серии книг и
                   3920: еще более известной серии фильмов по этим книгам. Назовите этот символ.
                   3921: 
                   3922: Ответ:
                   3923: Q.
                   3924: 
                   3925: Комментарий:
                   3926: ПЕРВЫЙ - это роман Оруэлла "1984", ВТОРОЙ - роман Мураками "1Q84" (в
                   3927: русском переводе - "Тысяча невестьсот восемьдесят четыре"). Q - кодовое
                   3928: имя Джеффри Бутройта, вымышленного персонажа романов про Джеймса Бонда
                   3929: британского писателя Яна Флеминга.
                   3930: 
                   3931: Источник:
                   3932:    1. Дж. Оруэлл. 1984.
                   3933:    2. Х. Мураками. 1Q84.
                   3934:    3. http://en.wikipedia.org/wiki/Q_(James_Bond)
                   3935: 
                   3936: Автор:
                   3937: Дарья Коротина (Саранск - Москва)
                   3938: 
                   3939: Вопрос 17:
                   3940: ТАКОЙ ОН был куплен для могилы Железной Лошади. "ТАКОЙ ОН" - название
                   3941: отечественной музыкальной группы и известного романа. Напишите это
                   3942: название.
                   3943: 
                   3944: Ответ:
                   3945: "Черный обелиск".
                   3946: 
                   3947: Комментарий:
                   3948: Железная Лошадь - прозвище проститутки из романа Э.М. Ремарка "Черный
                   3949: обелиск", для могилы которой был куплен черный обелиск. Рок-группа взяла
                   3950: свое название в честь вышеупомянутого романа Ремарка.
                   3951: 
                   3952: Источник:
                   3953:    1. Э.М. Ремарк. Черный обелиск. http://www.flibusta.net/b/326124/read
                   3954:    2. http://ru.wikipedia.org/wiki/Чёрный_Обелиск_(группа)
                   3955: 
                   3956: Автор:
                   3957: Дарья Коротина (Саранск - Москва)
                   3958: 
                   3959: Вопрос 18:
                   3960: В песне Битлз "ТАКОЙ ОН" поется о небогатой девушке, у которой стены в
                   3961: комнате были обшиты ТАКИМ ИМ, и не было ни одного стула, чтобы присесть
                   3962: гостю. В 1987 году был написан одноименный зарубежный роман, героине
                   3963: которого очень нравилась эта песня. Назовите автора романа.
                   3964: 
                   3965: Ответ:
                   3966: Харуки Мураками.
                   3967: 
                   3968: Комментарий:
                   3969: Песня Битлз "Норвежский лес" является любимой композицией Наоко,
                   3970: возлюбленной Ватанабэ, в одноименном романе Х. Мураками.
                   3971: 
                   3972: Источник:
                   3973:    1. The Beatles, "Norwegian Wood".
                   3974:    2. Х. Мураками. Норвежский лес.
                   3975: 
                   3976: Автор:
                   3977: Дарья Коротина (Саранск - Москва)
                   3978: 
                   3979: Вопрос 19:
                   3980: В 2000 году молодой банкир с Уолл-Стрит надел на встречу со своим
                   3981: коллегой прозрачное пластиковое пальто, чтобы защитить свою одежду от
                   3982: ЭТОГО. В 2013 году судебный психиатр из Мериленда надел похожее пальто
                   3983: на встречу со своим коллегой с той же целью. От чего они хотели защитить
                   3984: одежду?
                   3985: 
                   3986: Ответ:
                   3987: От крови.
                   3988: 
                   3989: Комментарий:
                   3990: Банкир - Патрик Бэйтмен, главный герой фильма "Американский психопат"
                   3991: (2000), психиатр - Ганнибал Лектер из сериала "Ганнибал" (2013).
                   3992: 
                   3993: Источник:
                   3994:    1. Фильм "Американский психопат".
                   3995:    2. Сериал "Ганнибал".
                   3996: 
                   3997: Автор:
                   3998: Дарья Беличенко (Москва)
                   3999: 
                   4000: Вопрос 20:
                   4001: По словам известной феминистки Сюзен Энтони, ЭТО сделало больше для
                   4002: эмансипации женщин, чем всё остальное вместе взятое. В связи с
                   4003: распространением ЭТОГО в 1890-х годах женщины получили свободу, которой
                   4004: у них не было раньше. Кроме того, это помогло женщинам избавиться от
                   4005: корсетов и другой сковывающей одежды. В одном из рассказов Чехова герой
                   4006: был так поражен, увидев свою возлюбленную с ЭТИМ, что отказался от
                   4007: намерения жениться на ней. Назовите ЭТО.
                   4008: 
                   4009: Ответ:
                   4010: Велосипед.
                   4011: 
                   4012: Комментарий:
                   4013: Велосипеды дали женщинам беспрецедентную свободу передвижения; с
                   4014: распространением велосипедов связана мода на женские шаровары и одежду,
                   4015: не сковывающую движения.
                   4016: 
                   4017: Источник:
                   4018:    1. http://ru.wikipedia.org/wiki/Велосипед#.D0.A1.D0.BE.D1.86.D0.B8.D0.B0.D0.BB.D1.8C.D0.BD.D0.B0.D1.8F_.D1.80.D0.BE.D0.BB.D1.8C
                   4019:    2. А.П. Чехов. Человек в футляре.
                   4020: http://www.flibusta.net/b/82117/read
                   4021: 
                   4022: Автор:
                   4023: Дарья Беличенко (Москва)
                   4024: 
                   4025: Вопрос 21:
                   4026: Находящаяся в Ватиканском дворце фреска "Афинская школа", одна из самых
                   4027: известных работ Рафаэля, содержит в себе немало забавных секретов. Так,
                   4028: некоторым из изображенных античных философов мастер подарил лица своих
                   4029: знаменитых современников Леонардо да Винчи и Микеланджело, а в качестве
                   4030: здания школы Рафаэль при помощи архитектора Донато Браманте весьма точно
                   4031: изобразил первый этаж знаменитого собора Святого Петра. Однако это
                   4032: сходство посетители Ватикана заметили лишь спустя несколько десятилетий.
                   4033: Почему?
                   4034: 
                   4035: Ответ:
                   4036: Собор был построен позже, чем была создана фреска.
                   4037: 
                   4038: Зачет:
                   4039: По смыслу.
                   4040: 
                   4041: Комментарий:
                   4042: На момент написания фрески строительство собора еще не началось -
                   4043: Браманте, которому поручили вести строительство, поделился с Рафаэлем
                   4044: утвержденными планами строительства. Собор был открыт для посещения
                   4045: спустя столетие.
                   4046: 
                   4047: Источник:
                   4048: https://books.google.ru/books?id=MMYHuvhWBH4C&pg=PT476#v=onepage&q&f=false
                   4049: 
                   4050: Автор:
                   4051: Антон Лазарев (Москва)
                   4052: 
                   4053: Вопрос 22:
                   4054: В Берлине, современной столице одной из самых развитых европейских
                   4055: стран, немало интересных зданий, конструкций и памятников. Однако в
                   4056: первую очередь удивленные взгляды туристов притягивают, как правило,
                   4057: розовые причудливо изгибающиеся трубы, которые можно увидеть практически
                   4058: в любой точке города (длина всей системы составляет порядка 60 км!). Вот
                   4059: такое необычное решение проблемы, неплохо знакомой, в частности, жителям
                   4060: Восточного Подмосковья, нашли власти города. В чем же предназначение
                   4061: этих труб?
                   4062: 
                   4063: Ответ:
                   4064: Это водопроводные трубы.
                   4065: 
                   4066: Зачет:
                   4067: По смыслу.
                   4068: 
                   4069: Комментарий:
                   4070: Берлин, по сути, стоит на болоте (вода уже на глубине двух метров), и
                   4071: нужно эту воду отводить во избежание всяких неприятностей. Но так как
                   4072: копать там затруднительно, трубы провели по воздуху.
                   4073: 
                   4074: Источник:
                   4075: http://www.berlinglobal.org/index.php?the-berlin-mystery-of-the-pink-pipes
                   4076: 
                   4077: Автор:
                   4078: Антон Лазарев (Москва)
                   4079: 
                   4080: Вопрос 23:
                   4081: В романе Алана Фостера "Призыв к оружию" инопланетяне выбирают в
                   4082: качестве первого публичного контакта редакцию "Вашингтон Пост". Главному
                   4083: редактору издания они заявляют, что он не единственный, с кем пришельцы
                   4084: собираются контактировать. Тот сначала пугается, но вскоре
                   4085: успокаивается, говоря, что на продажах его издания это не скажется. Не
                   4086: бойтесь выбрать и ответьте, что, согласно старому анекдоту, отсутствует
                   4087: во втором издании, с которым собрались контактировать пришельцы.
                   4088: 
                   4089: Ответ:
                   4090: Правда.
                   4091: 
                   4092: Зачет:
                   4093: Известия.
                   4094: 
                   4095: Комментарий:
                   4096: Второе издание - "Известия". Советская газета вряд ли могла повлиять на
                   4097: продажи экземпляров "Вашингтон Пост". Согласно анекдоту, в "Известиях"
                   4098: нет правды, а в "Правде" - известий. Согласно другому варианту, в
                   4099: "Известиях" нет известий, а в "Правде" - правды. Так что ошибиться в
                   4100: выборе невозможно.
                   4101: 
                   4102: Источник:
                   4103:    1. А. Фостер. Призыв к оружию. http://www.flibusta.net/b/81406/read
                   4104:    2. Анекдот.
                   4105: 
                   4106: Автор:
                   4107: Антон Рогожин (Саранск)
                   4108: 
                   4109: Вопрос 24:
                   4110: В толковых словарях у этого слова есть несколько значений. Чаще всего
                   4111: оно употребляется в разговорной речи для обозначения затяжного (иногда
                   4112: бессмысленного) занятия, связанного с задержками и хлопотами. А вот для
                   4113: музыкантов, играющих на струнных инструментах, это слово обычно означает
                   4114: нечто более материальное - то, с чем им приходится иметь дело каждый
                   4115: день. Назовите это слово.
                   4116: 
                   4117: Ответ:
                   4118: Канитель.
                   4119: 
                   4120: Комментарий:
                   4121: Тонкую металлическую нить, предназначенную для вышивки или, в частности,
                   4122: для намотки на басовые струны инструментов, называют красивым испанским
                   4123: словом "канитель"; те, кто пользовались в рукоделии подобными нитями,
                   4124: наверняка поймут, почему выражение "тянуть канитель" приняло в русском
                   4125: языке такое негативное значение.
                   4126: 
                   4127: Источник:
                   4128: http://ru.wikipedia.org/wiki/Канитель
                   4129: 
                   4130: Автор:
                   4131: Максим Фомин (Москва)
                   4132: 
                   4133: Тур:
                   4134: 11 тур. "Андреевские заразы" (Москва)
                   4135: 
                   4136: Инфо:
1.3       rubashki 4137: Команда благодарит за тестирование и ценные замечания Тимура Бокова,
                   4138: Кирилла Железнова, Артема Митрофанова, Николая Некрылова, Евгения
                   4139: Перпера, Александра Попкова, Анастасию Стебалину, Алексея Тишкина и Анну
                   4140: Павлову, Олега Фею, Сергея Шорина, Вадима Штанникова, а также Игоря
                   4141: Тюнькина за самое деятельное участие в работе над пакетом.
1.1       rubashki 4142: 
                   4143: Вопрос 1:
1.3       rubashki 4144: Персонаж рассказа Чехова, неудачливый писатель, говорит о себе, что
                   4145: полсотни марок он посеял на ПЕРВОЙ и еще сотню утопил во ВТОРОЙ.
                   4146: Назовите ПЕРВУЮ и ВТОРУЮ, которые различаются одной буквой.
1.1       rubashki 4147: 
                   4148: Ответ:
1.3       rubashki 4149: Нива, Нева.
                   4150: 
                   4151: Комментарий:
                   4152: Здесь "Нива" и "Нева" - названия литературных журналов. Писатель
                   4153: отправлял рукописи со своими творениями в различные журналы, но они
                   4154: нигде не были приняты. На почтовые марки пришлось потратить заметную
                   4155: сумму.
1.1       rubashki 4156: 
                   4157: Источник:
1.3       rubashki 4158: А.П. Чехов. Мой юбилей. http://www.flibusta.net/b/10613/read
1.1       rubashki 4159: 
                   4160: Автор:
1.3       rubashki 4161: Андрей Кудрявцев
1.1       rubashki 4162: 
                   4163: Вопрос 2:
1.3       rubashki 4164: По словам Наполеона Бонапарта, высшее искусство управления состоит в
                   4165: том, чтобы не дать подчиненным совершить некое действие в своей
                   4166: должности. Главный герой произведений 2009 и 1922 годов совершает
                   4167: противоположное действие. Назовите этого героя.
1.1       rubashki 4168: 
                   4169: Ответ:
1.3       rubashki 4170: [Бенджамин] Баттон.
1.1       rubashki 4171: 
                   4172: Комментарий:
1.3       rubashki 4173: Совершить некое действие - состариться. Возможно, Наполеон имел в виду,
                   4174: что подчиненных следует своевременно повышать, но, по мнению Тарле,
                   4175: получалось иначе - у него на службе долго не жили. Обратный порядок дат
                   4176: в вопросе призван служить подсказкой - Баттон тоже молодеет.
1.1       rubashki 4177: 
                   4178: Источник:
1.3       rubashki 4179:    1. Е.В. Тарле. Наполеон
                   4180: http://www.museum.ru/1812/library/tarle/napoleon_win.txt
                   4181:    2. http://ru.wikiquote.org/wiki/Наполеон_I_Бонапарт
                   4182:    3. http://ru.wikipedia.org/wiki/Загадочная_история_Бенджамина_Баттона_(фильм)
                   4183:    4. http://ru.wikipedia.org/wiki/Загадочная_история_Бенджамина_Баттона_(рассказ)
1.1       rubashki 4184: 
                   4185: Автор:
1.3       rubashki 4186: Андрей Кудрявцев, Екатерина Лобкова
1.1       rubashki 4187: 
                   4188: Вопрос 3:
1.3       rubashki 4189: Заметка на сайте zadolba.li рассказывает о том, что социопатия - уже не
                   4190: только удел отдельных людей, но и повод для общения. Неудивительно, что
                   4191: в заголовке из социопатии пропала ОНА. Напишите ЕЕ.
1.1       rubashki 4192: 
                   4193: Ответ:
1.3       rubashki 4194: Я.
1.1       rubashki 4195: 
                   4196: Комментарий:
1.3       rubashki 4197: Заметка называется "У нас тут социопати".
1.1       rubashki 4198: 
                   4199: Источник:
1.3       rubashki 4200: http://zadolba.li/story/16196
1.1       rubashki 4201: 
                   4202: Автор:
1.3       rubashki 4203: Андрей Волыхов, Игорь Тюнькин
1.1       rubashki 4204: 
                   4205: Вопрос 4:
1.3       rubashki 4206: Учитель автора вопроса почти весь урок ДЕРЖАЛ ФОРМУ, так ничего и не
                   4207: заподозрив. Какие три слова мы заменили в вопросе?
1.1       rubashki 4208: 
                   4209: Ответ:
1.3       rubashki 4210: Сидел на кнопке.
1.1       rubashki 4211: 
                   4212: Источник:
1.3       rubashki 4213: ЛОАВ.
1.1       rubashki 4214: 
                   4215: Автор:
1.3       rubashki 4216: Андрей Кудрявцев
1.1       rubashki 4217: 
                   4218: Вопрос 5:
1.3       rubashki 4219: В меню столовой МГУ автор вопроса обнаружил дарницкий хлеб, пшеничный
                   4220: хлеб и ТАКОЙ хлеб. Чаще можно услышать, что ТАКИМ бывает другой продукт.
                   4221: Какое слово мы заменили словом "ТАКОЙ"?
1.1       rubashki 4222: 
                   4223: Ответ:
1.3       rubashki 4224: Бесплатный.
1.1       rubashki 4225: 
                   4226: Комментарий:
1.3       rubashki 4227: А вот бесплатного сыра в столовой не оказалось.
1.1       rubashki 4228: 
                   4229: Источник:
1.3       rubashki 4230: ЛОАВ.
1.1       rubashki 4231: 
                   4232: Автор:
                   4233: Андрей Волыхов
                   4234: 
                   4235: Вопрос 6:
1.3       rubashki 4236: Автор вопроса предположил, что в столовой МГУ для продуктов применяется
                   4237: ОНА. Назовите ЕЕ двумя словами.
1.1       rubashki 4238: 
                   4239: Ответ:
1.3       rubashki 4240: Китайская классификация.
1.1       rubashki 4241: 
                   4242: Зачет:
1.3       rubashki 4243: Классификация Борхеса, борхесовская классификация.
1.1       rubashki 4244: 
                   4245: Комментарий:
1.3       rubashki 4246: Там же были обнаружены, например, "разовый чай" и "зеленый чай". В одном
                   4247: эссе Борхеса приведена анекдотическая классификация животных, взятая
                   4248: якобы из китайской энциклопедии.
1.1       rubashki 4249: 
                   4250: Источник:
1.3       rubashki 4251:    1. ЛОАВ.
                   4252:    2. http://ru.wikipedia.org/wiki/Классификация_животных_(Борхес)
1.1       rubashki 4253: 
                   4254: Автор:
                   4255: Андрей Волыхов
                   4256: 
                   4257: Вопрос 7:
1.3       rubashki 4258: Хозяйка из рассказа Чехова не успевает приготовить праздничный обед и
                   4259: потому ДЕЛАЕТ ЭТО. Многим из нас необходимость регулярно ДЕЛАТЬ ЭТО
                   4260: приносила удовольствие. Что именно делать?
1.1       rubashki 4261: 
                   4262: Ответ:
1.3       rubashki 4263: Переводить часы назад.
                   4264: 
                   4265: Зачет:
                   4266: По смыслу с обязательным указанием на перевод назад.
                   4267: 
                   4268: Комментарий:
                   4269: Различные персонажи, каждый по своей причине, хотят приблизить момент
                   4270: наступления Нового года. Хозяйка же не успевает приготовить праздничный
                   4271: обед к полуночи и переводит назад минутную стрелку. Когда часы
                   4272: переводятся назад, можно спать дольше.
1.1       rubashki 4273: 
                   4274: Источник:
1.3       rubashki 4275: А.П. Чехов. Мошенники поневоле. http://www.flibusta.net/b/10615/read
1.1       rubashki 4276: 
                   4277: Автор:
                   4278: Андрей Кудрявцев
                   4279: 
                   4280: Вопрос 8:
1.3       rubashki 4281: Зал славы ТРЕТЬЕГО создан в 1983 году, но первые члены в нем появились
                   4282: лишь в 1986 году. Автор вопроса предположил, что в первые три года
                   4283: оргкомитет был слишком увлечен ПЕРВЫМ и ВТОРЫМИ. Назовите ТРЕТИЙ.
1.1       rubashki 4284: 
                   4285: Ответ:
1.3       rubashki 4286: Рок-н-ролл.
1.1       rubashki 4287: 
                   4288: Комментарий:
1.3       rubashki 4289: И слово "члены" в вопросе не случайно. Видимо, в первые три года фонд
                   4290: уделял больше внимания сексу и наркотикам, чем поиску прибежища для
                   4291: музея.
1.1       rubashki 4292: 
                   4293: Источник:
1.3       rubashki 4294:    1. http://en.wikipedia.org/wiki/Rock_and_Roll_Hall_of_Fame
                   4295:    2. Фантазии автора вопроса.
1.1       rubashki 4296: 
                   4297: Автор:
1.3       rubashki 4298: Павел Казначеев
1.1       rubashki 4299: 
                   4300: Вопрос 9:
1.3       rubashki 4301: Государство Силенд - "морская земля" - расположено недалеко от побережья
                   4302: Великобритании на платформе, с которой планировали транслировать
                   4303: независимое радио. Представители какого ресурса в 2007 году пытались
                   4304: купить Силенд?
1.1       rubashki 4305: 
                   4306: Ответ:
1.3       rubashki 4307: [The] Pirate Bay.
1.1       rubashki 4308: 
                   4309: Комментарий:
1.3       rubashki 4310: Ресурс "The Pirate Bay", видимо, захотел создать собственную пиратскую
                   4311: бухту в независимом государстве, чтобы разместить серверы.
1.1       rubashki 4312: 
                   4313: Источник:
1.3       rubashki 4314: http://en.wikipedia.org/wiki/Principality_of_Sealand
1.1       rubashki 4315: 
                   4316: Автор:
1.3       rubashki 4317: Андрей Кудрявцев, Игорь Тюнькин
1.1       rubashki 4318: 
                   4319: Вопрос 10:
1.3       rubashki 4320: Недоброжелатели положили коньки одного фигуриста на НЕЕ, чтобы тот
                   4321: провалился. В 1837 году был заложен памятник на месте другой ЕЕ, которую
                   4322: возглавлял... Кто?
1.1       rubashki 4323: 
                   4324: Ответ:
1.3       rubashki 4325: [Николай Николаевич] Раевский.
1.1       rubashki 4326: 
                   4327: Комментарий:
1.3       rubashki 4328: Разогретые на батарее коньки глубоко ушли в лед, хотя совсем под лед
                   4329: фигурист не провалился. По случаю двадцатипятилетия победы над
                   4330: Наполеоном на Бородинском поле провели торжества, а на Курганной высоте
                   4331: установили первый памятник.
1.1       rubashki 4332: 
                   4333: Источник:
1.3       rubashki 4334:    1. И. Роднина. Слеза чемпионки. http://www.flibusta.net/b/351532/read
                   4335:    2. http://www.borodinoru.ru/Pamyatniki/pamyat1812/pam52.htm
1.1       rubashki 4336: 
                   4337: Автор:
1.3       rubashki 4338: Екатерина Лобкова
1.1       rubashki 4339: 
                   4340: Вопрос 11:
1.3       rubashki 4341: Форму чего имеют точки над "ё" в логотипе фирмы "Лимузёнок"?
1.1       rubashki 4342: 
                   4343: Ответ:
1.3       rubashki 4344: Обручальные кольца.
                   4345: 
                   4346: Комментарий:
                   4347: Фирма предоставляет лимузины для торжественных событий.
1.1       rubashki 4348: 
                   4349: Источник:
1.3       rubashki 4350: Субботние наблюдения автора вопроса.
1.1       rubashki 4351: 
                   4352: Автор:
1.3       rubashki 4353: Екатерина Лобкова
1.1       rubashki 4354: 
                   4355: Вопрос 12:
1.3       rubashki 4356: Прослушайте рекламный слоган книги: "Больше, чем [два слова пропущено]
                   4357: страсти". Заполните пропуск.
1.1       rubashki 4358: 
                   4359: Ответ:
1.3       rubashki 4360: "... пятьдесят оттенков...".
1.1       rubashki 4361: 
                   4362: Источник:
1.3       rubashki 4363: Виденная автором вопроса в вагоне метро реклама какой-то книги.
1.1       rubashki 4364: 
                   4365: Автор:
1.3       rubashki 4366: Андрей Волыхов
1.1       rubashki 4367: 
                   4368: Вопрос 13:
1.3       rubashki 4369: Прослушайте цитату из песни Михаила Щербакова:
                   4370:    Вчера я видел сеть узкоколейных линий,
                   4371:    по каким ту глину поезда везли...
                   4372:    Каким словом заканчивается следующая строка песни?
1.1       rubashki 4373: 
                   4374: Ответ:
                   4375: Алюминий.
                   4376: 
1.3       rubashki 4377: Комментарий:
                   4378:    ... туда, где из нее, по слухам, алюминий
                   4379:    извлекают нынче. Веришь ли?
                   4380:    Вот миномёт. Не веришь - застрели. :-)
                   4381:    Основной источник алюминия - боксит - является глиноподобным
                   4382: минералом. Да и в любой глине основа - алюмосиликаты.
                   4383: 
1.1       rubashki 4384: Источник:
                   4385: М. Щербаков. Инициалы.
                   4386: http://www.blackalpinist.com/scherbakov/htmtexts/1996/inicialy.html
                   4387: 
                   4388: Автор:
                   4389: Андрей Волыхов
                   4390: 
                   4391: Вопрос 14:
1.3       rubashki 4392:    <раздатка>
                   4393:    Википедия утверждает, что слухи о монархизме Гитлера могли быть
                   4394: вызваны британской пропагандистской песней, написанной в 1939 году.
                   4395:    </раздатка>
                   4396:    В одном из слов в раздаточном материале мы заменили одну букву.
                   4397: Напишите исходное слово.
1.1       rubashki 4398: 
                   4399: Ответ:
1.3       rubashki 4400: Монорхизме.
1.1       rubashki 4401: 
                   4402: Комментарий:
1.3       rubashki 4403: Монорхизм - отсутствие одного яичка. Песня называлась "Hitler Has Only
                   4404: Got One Ball".
1.1       rubashki 4405: 
                   4406: Источник:
1.3       rubashki 4407:    1. http://en.wikipedia.org/wiki/Adolf_Hitler's_possible_monorchism
                   4408:    2. http://en.wikipedia.org/wiki/Hitler_Has_Only_Got_One_Ball
1.1       rubashki 4409: 
                   4410: Автор:
1.3       rubashki 4411: Андрей Волыхов, Екатерина Лобкова
1.1       rubashki 4412: 
1.10      rubashki 4413: Вопрос 15:
                   4414: [Ведущему: указать, что перед пропуском и после него есть запятые.]
1.3       rubashki 4415:    Заполните двумя словами пропуск в цитате из песни Виталия
                   4416: Калашникова:
                   4417:    Потерпим немного в своем ожиданье итога,
                   4418:    Листва облетает, и смысл по углам растасован,
                   4419:    И воздух подернут тончайшей, [ПРОПУСК],
                   4420:    И люди, как в бомбоубежище, прячутся в слово.
1.1       rubashki 4421: 
                   4422: Ответ:
1.3       rubashki 4423: "... воздушной тревогой...".
1.1       rubashki 4424: 
                   4425: Комментарий:
1.3       rubashki 4426: По-видимому, здесь "воздушная тревога" - не устойчивое словосочетание.
1.1       rubashki 4427: 
                   4428: Источник:
1.3       rubashki 4429: В. Калашников. Геннадию. http://www.bards.ru/archives/part.php?id=40298
1.1       rubashki 4430: 
                   4431: Автор:
1.3       rubashki 4432: Андрей Волыхов
1.1       rubashki 4433: 
                   4434: Вопрос 16:
1.3       rubashki 4435: (pic: 20150150.jpg)
                   4436:    Какое прилагательное дизайнер студии Артемия Лебедева использовал при
                   4437: описании этого головного убора?
1.1       rubashki 4438: 
                   4439: Ответ:
1.3       rubashki 4440: Каракулевая.
1.1       rubashki 4441: 
                   4442: Комментарий:
1.3       rubashki 4443: Из каракулей.
1.1       rubashki 4444: 
                   4445: Источник:
1.3       rubashki 4446: http://www.artlebedev.ru/everything/brain/2014/09/18/
1.1       rubashki 4447: 
                   4448: Автор:
                   4449: Павел Казначеев
                   4450: 
                   4451: Вопрос 17:
1.3       rubashki 4452: Героиня произведения Фаулза называет своего разговорившегося мужа
                   4453: кресельным социалистом и говорит, что он мог бы переманить всех в
                   4454: радиусе достижения. В одном из слов предыдущего предложения мы заменили
                   4455: одну букву. Напишите этот неологизм в изначальном виде.
1.1       rubashki 4456: 
                   4457: Ответ:
1.3       rubashki 4458: Перемаоить.
1.1       rubashki 4459: 
                   4460: Комментарий:
1.3       rubashki 4461: Мао, известный в англоязычных источниках как "chairman Mao", тоже в
                   4462: каком-то смысле кресельный социалист.
1.1       rubashki 4463: 
                   4464: Источник:
1.3       rubashki 4465: Дж. Фаулз. Туча. http://www.flibusta.net/b/349571/read
1.1       rubashki 4466: 
                   4467: Автор:
1.3       rubashki 4468: Павел Казначеев
1.1       rubashki 4469: 
                   4470: Вопрос 18:
1.3       rubashki 4471: В своей автобиографии Ирина Роднина вспоминает, что во время первого
                   4472: заграничного дебюта вместе с коллегами по сборной уже после первой
                   4473: тренировки побежала в магазин за НИМ. Роднина называет ЕГО одним словом,
                   4474: а вы назовите одним или, так и быть, двумя.
1.1       rubashki 4475: 
                   4476: Ответ:
1.3       rubashki 4477: [Нижнее] белье.
1.1       rubashki 4478: 
                   4479: Комментарий:
1.3       rubashki 4480: Девушкам было стыдно демонстрировать в общей раздевалке советское белье.
                   4481: "Своя автобиография" и "первый дебют" - плеоназмы, как и "нижнее белье".
1.1       rubashki 4482: 
                   4483: Источник:
1.3       rubashki 4484:    1. И. Роднина. Слеза чемпионки. http://www.flibusta.net/b/351532/read
                   4485:    2. http://poetique.academic.ru/407/
1.1       rubashki 4486: 
                   4487: Автор:
1.3       rubashki 4488: Екатерина Лобкова, Игорь Тюнькин
1.1       rubashki 4489: 
                   4490: Вопрос 19:
1.3       rubashki 4491: Авторы заметки на сайте housefan.ru [хаусфан точка ру] пишут, что
                   4492: некоторые врачи ДЕЛАЮТ ЭТО, когда в сериале "Доктор Хаус" пользуются
                   4493: дефибриллятором. Делают что?
1.1       rubashki 4494: 
                   4495: Ответ:
1.3       rubashki 4496: Хватаются за сердце.
1.1       rubashki 4497: 
                   4498: Комментарий:
1.3       rubashki 4499: В реальности дефибриллятор не может "запустить" остановившееся сердце.
                   4500: Его задача - остановить фибрилляцию, т.е. нескоординированное сокращение
                   4501: отдельных групп мышечных волокон сердечной мышцы, и дать возможность
                   4502: естественному водителю ритма вернуть себе контроль над сокращениями.
1.1       rubashki 4503: 
                   4504: Источник:
1.3       rubashki 4505:    1. http://www.housefan.ru/news-7mistakes.html
                   4506:    2. http://ru.wikipedia.org/wiki/Фибрилляция_сердца
                   4507:    3. http://ru.wikipedia.org/wiki/Дефибриллятор
                   4508:    4. Консультация Андрея Азова.
1.1       rubashki 4509: 
                   4510: Автор:
1.3       rubashki 4511: Андрей Кудрявцев
1.1       rubashki 4512: 
                   4513: Вопрос 20:
1.3       rubashki 4514: Узнав, что по ее фразе был написан вопрос ЧГК, Мария Логачёва упомянула
                   4515: двузначное число. Какое?
1.1       rubashki 4516: 
                   4517: Ответ:
1.3       rubashki 4518: 34.
1.1       rubashki 4519: 
                   4520: Комментарий:
1.3       rubashki 4521: Аналог "правила 34" - и про это есть вопрос! Кстати, удивительно, что
                   4522: вопросов про "правило 34" в Базе не нашлось.
1.1       rubashki 4523: 
                   4524: Источник:
1.3       rubashki 4525: Переписка в Скайпе Марии Логачёвой и Виктории Штратниковой,
                   4526: подсмотренная автором вопроса.
1.1       rubashki 4527: 
                   4528: Автор:
1.3       rubashki 4529: Андрей Волыхов
1.1       rubashki 4530: 
                   4531: Вопрос 21:
1.3       rubashki 4532: Размышляя о гендерных различиях в сходных профессиях, автор вопроса
                   4533: уподобил ИХ верхним ножкам X-хромосомы. Назовите ИХ двумя словами.
1.1       rubashki 4534: 
                   4535: Ответ:
1.3       rubashki 4536: Троллейбусные штанги.
                   4537: 
                   4538: Зачет:
                   4539: Штанги троллейбуса. Зачет с негодованием: Рога троллейбуса и прочие
                   4540: подобные ответы.
1.1       rubashki 4541: 
                   4542: Комментарий:
1.3       rubashki 4543: Женщины - водители троллейбусов встречаются намного чаще, чем женщины -
                   4544: водители автобусов.
1.1       rubashki 4545: 
                   4546: Источник:
1.3       rubashki 4547:    1. Палец автора вопроса.
                   4548:    2. http://www.the-village.ru/village/city/asking-question/131167-avtobus-trolleybus-zhenschiny
1.1       rubashki 4549: 
                   4550: Автор:
                   4551: Андрей Волыхов
                   4552: 
                   4553: Вопрос 22:
                   4554: В одном романе цитируется надпись на плакате: "Приветствуем участников
                   4555: DCLXV [шестьсот шестьдесят пятого] традиционного съезда феминисток,
                   4556: работающих в сферах геронтологии, косметологии, ботаники и межличностных
1.3       rubashki 4557: отношений". Слово, которым в романе назван этот конгресс, вероятно,
                   4558: образовано от названия дня недели. Напишите это слово.
1.1       rubashki 4559: 
                   4560: Ответ:
                   4561: ШАбаш.
                   4562: 
                   4563: Комментарий:
1.3       rubashki 4564: На съезд слетаются ведьмы. Согласно этимологическому словарю Фасмера,
                   4565: слова "шАбаш" и "шабАш" связаны с шаббатом через немецкий и польский
                   4566: языки.
1.1       rubashki 4567: 
                   4568: Источник:
                   4569:    1. С. Лукьяненко. Шестой Дозор.
1.3       rubashki 4570:    2. http://dic.academic.ru/dic.nsf/vasmer/51196/
1.1       rubashki 4571: 
                   4572: Автор:
                   4573: Екатерина Лобкова
                   4574: 
                   4575: Вопрос 23:
1.3       rubashki 4576: (pic: 20150151.jpg)
                   4577:    Перед вами вариант покроя юбки, названный в ЕЕ честь. В порту какого
                   4578: города в 1913 году установили памятник ЕЙ?
1.1       rubashki 4579: 
                   4580: Ответ:
1.3       rubashki 4581: Копенгаген.
1.1       rubashki 4582: 
                   4583: Комментарий:
1.3       rubashki 4584: Покрой "mermaid" получил свое название из-за сходства с фигурой Ариэль.
                   4585: А в Копенгагене стоит памятник андерсеновской Русалочке.
1.1       rubashki 4586: 
                   4587: Источник:
1.3       rubashki 4588:    1. https://www.google.ru/search?q=mermaid+skirt&tbm=isch
                   4589:    2. http://ru.wikipedia.org/wiki/Русалочка_(статуя)
1.1       rubashki 4590: 
                   4591: Автор:
1.3       rubashki 4592: Екатерина Лобкова
1.1       rubashki 4593: 
                   4594: Вопрос 24:
                   4595:    <раздатка>
                   4596:    Т'Пау
                   4597:    Т'Пол
                   4598:    Т'Принг
                   4599:    </раздатка>
                   4600:    Как известно, раса вулканцев из вселенной "Звездного пути" высоко
                   4601: ценит логику, интеллект и контроль над эмоциями. Перед вами несколько
                   4602: вулканских имен. Какое слово мы пропустили в тексте этого вопроса?
                   4603: 
                   4604: Ответ:
                   4605: Женских.
                   4606: 
                   4607: Зачет:
1.3       rubashki 4608: Женщин, героинь.
1.1       rubashki 4609: 
                   4610: Комментарий:
                   4611: Для пользователя Рунета забавно, что имена вулканок нередко начинаются с
                   4612: буквосочетания "тп". Но, конечно, это случайное совпадение, поскольку из
                   4613: экранного канона известны еще как минимум вулканки СаАвик и ВалЕрис.
                   4614: Живите долго и процветайте!
                   4615: 
                   4616: Источник:
                   4617: Вселенная Звездного Пути (сериалы "Star Trek: The Original Series" и
                   4618: "Enterprise", полнометражные фильмы "Звездный путь: Гнев Хана",
                   4619: "Звездный путь: В поисках Спока" и "Звездный путь: Неоткрытая страна").
                   4620: 
                   4621: Автор:
                   4622: Екатерина Лобкова
                   4623: 
                   4624: Тур:
                   4625: 12 тур. "Любовь" (Калуга)
                   4626: 
                   4627: Вопрос 1:
                   4628: Средневековая легенда рассказывает о Франческе и Паоло, которые были
                   4629: застигнуты и убиты мужем Франчески в самом начале любовных отношений.
                   4630: Произведение, по одной из версий навеянное этим сюжетом, было впервые
                   4631: показано в 1889 году. Назовите автора этого произведения.
                   4632: 
                   4633: Ответ:
                   4634: [Огюст] Роден.
                   4635: 
                   4636: Комментарий:
                   4637: Существует мнение, что на скульптуре Родена "Поцелуй" пара не касается
                   4638: друг друга губами. И это намекает, что они были убиты, не совершив
                   4639: греха.
                   4640: 
                   4641: Источник:
                   4642: http://ru.wikipedia.org/wiki/Поцелуй_(Роден)
                   4643: 
                   4644: Автор:
                   4645: ???
                   4646: 
                   4647: Вопрос 2:
                   4648: В одном из эпизодов сериала "Симпсоны" у Мардж в результате проклятия
                   4649: цыганки появилась КРАСНАЯ ШАПОЧКА. Какие слова мы заменили на "КРАСНАЯ
                   4650: ШАПОЧКА"?
                   4651: 
                   4652: Ответ:
                   4653: Синяя борода.
                   4654: 
                   4655: Комментарий:
                   4656: Мардж Симпсон, как известно, - обладательница огромной копны синих
                   4657: волос. "Синяя Борода", как и "Красная Шапочка", - название сказки Шарля
                   4658: Перро.
                   4659: 
                   4660: Автор:
                   4661: Даниил Марченко (Калуга)
                   4662: 
                   4663: Вопрос 3:
                   4664: В вопросе есть замены.
                   4665:    По шутке автора вопроса, румынская народная мудрость звучит так:
                   4666: "Если ты не нашел в своем кармане ЕГО АЛЬФУ - значит, ты живешь не в
                   4667: Румынии". На портале psifactum.ru [псифактум точка ру] утверждается, что
                   4668: в своей деятельности ОНИ стараются дотронуться до АЛЬФЫ, чтобы
                   4669: воздействовать на тактильный канал восприятия. Какие слова мы заменили
                   4670: на "ОНИ" и "АЛЬФА"?
                   4671: 
                   4672: Ответ:
                   4673: Цыгане, рука.
                   4674: 
                   4675: Источник:
                   4676:    1. http://irrichidubili.livejournal.com/14914.html
                   4677:    2. http://psifactum.ru/manipulation/521-manipulation.html
                   4678: 
                   4679: Автор:
                   4680: ???
                   4681: 
                   4682: Вопрос 4:
                   4683: В сериале "Сыны анархии" крутой сюжет перемежается с тягучей семейной
                   4684: драмой. В обзоре на этот сериал отмечено, что от городка Чарминг, где
                   4685: происходит действие, совсем недалеко до другого населенного пункта.
                   4686: Назовите его.
                   4687: 
                   4688: Ответ:
                   4689: Санта-Барбара.
                   4690: 
                   4691: Источник:
                   4692: Эфир канала "2x2", 02.11.2014 г., 23:35.
                   4693: 
                   4694: Автор:
                   4695: ???
                   4696: 
                   4697: Вопрос 5:
                   4698: Как ни странно, ИКС не упоминается в диете для набора мышечной массы. В
                   4699: 1912 году начал издаваться журнал, посвященный силовым видам спорта, под
                   4700: названием "ИКС". Какое слово мы заменили на ИКС?
                   4701: 
                   4702: Ответ:
                   4703: Геркулес.
                   4704: 
                   4705: Источник:
                   4706: http://sportswiki.ru/
                   4707: 
                   4708: Автор:
                   4709: ???
                   4710: 
                   4711: Вопрос 6:
                   4712: На Урале, чтобы облагородить цвет дымчатого кварца, крупные кристаллы
                   4713: ЖАРИЛИ НА МАСЛЕ. Рассказывают, что Державина в младенчестве ЖАРИЛИ НА
                   4714: МАСЛЕ, чтобы излечить от тяжелой болезни. Какие три слова мы заменили на
                   4715: слова "ЖАРИЛИ НА МАСЛЕ"?
                   4716: 
                   4717: Ответ:
                   4718: Запекали в тесте.
                   4719: 
                   4720: Зачет:
                   4721: Запекали в печи.
                   4722: 
                   4723: Комментарий:
                   4724: Чтобы не повредить, кристаллы обволакивали тестом и томили в печи. В
                   4725: старину младенцев тоже оборачивали тестом и сажали в печь для лечения.
                   4726: 
                   4727: Источник:
                   4728:    1. http://www.imperis.ru/art/katalog-kamnej-samocvetov/340/
                   4729:    2. http://www.vorojea.info/vega/novikov/novikovartikles/vediratmir.htm
                   4730: 
                   4731: Автор:
                   4732: Денис Карабанов (Калуга)
                   4733: 
                   4734: Вопрос 7:
                   4735: Остроумный Шеридан Саймов выпустил необычную книгу под названием "О чем
                   4736: думают все мужчины, помимо секса". В статье Википедии, рассказывающей об
                   4737: этой книге, упоминаются также уроженец Киева и уроженец Лос-Анджелеса.
                   4738: Назовите их фамилии.
                   4739: 
                   4740: Ответ:
                   4741: Малевич, Кейдж.
                   4742: 
                   4743: Комментарий:
                   4744: Книга состояла из пустых страниц, что, по мнению автора, должно отражать
                   4745: его "шокирующий вывод" о том, что, помимо секса, мужчины больше не
                   4746: думают ни о чем. Книга с пустыми страницами так же концептуальна, как
                   4747: "Черный квадрат" Малевича и музыкальная пьеса Кейджа "4 минуты 33
                   4748: секунды".
                   4749: 
                   4750: Источник:
                   4751: http://ru.wikipedia.org/wiki/Книга_Ничто
                   4752: 
                   4753: Автор:
                   4754: ???
                   4755: 
                   4756: Вопрос 8:
                   4757: В романе Стивена Кинга "11/22/63" упоминаются такие события, как
                   4758: покушение на Кеннеди, убийство Мартина Лютера Кинга в Чикаго и концерт
                   4759: мира воссоединившихся "Битлз". В одном из эпизодов главный герой слышит,
                   4760: как за окном ПРОИЗОШЛО ЭТО. Ответьте, что произошло, двумя словами,
                   4761: начинающимися на одну и ту же букву.
                   4762: 
                   4763: Ответ:
                   4764: Грянул гром.
                   4765: 
                   4766: Комментарий:
                   4767: "И грянул гром" - знаменитый рассказ Рэя Брэдбери, в котором
                   4768: иллюстрируется так называемый эффект бабочки. В романе "11/22/63"
                   4769: вернувшийся в прошлое главный герой предотвратил убийство Джона Кеннеди,
                   4770: и это повлекло за собой глобальные изменения истории.
                   4771: 
                   4772: Автор:
                   4773: Денис Карабанов (Калуга)
                   4774: 
                   4775: Вопрос 9:
                   4776: В России приобретают популярность экологически чистые источники энергии.
                   4777: По словам Веры Писаревой, с электричеством в России "ПРОИСХОДИТ ЭТО". А
                   4778: с чем ПРОИСХОДИТ ЭТО в начале стихотворения 1858 года?
                   4779: 
                   4780: Ответ:
                   4781: С травкой.
                   4782: 
                   4783: Комментарий:
                   4784: Электричество "зеленеет".
                   4785: 
                   4786: Источник:
                   4787:    1. http://www.ogoniok.com/4898/43/
                   4788:    2. http://www.a-pesni.org/popular20/last-ples.htm
                   4789: 
                   4790: Автор:
                   4791: Алексей Пономарев (Калуга)
                   4792: 
                   4793: Вопрос 10:
                   4794:    Он был монтером Ваней, но... в духе парижан,
                   4795:    Себе присвоил званье: "электротехник Жан...".
                   4796:    Этим эпиграфом Максим Кравчинский предваряет свою книгу, посвященную
                   4797: истории жанра. Назовите этот жанр, используя заимствованное слово.
                   4798: 
                   4799: Ответ:
                   4800: [Русский] шансон.
                   4801: 
                   4802: Комментарий:
                   4803: Термин "русский шансон" в начале 1990-х стал своего рода эвфемизмом
                   4804: "блатной песни". Текст Маяковского вполне подходит для песни в стиле
                   4805: "русский шансон".
                   4806: 
                   4807: Источник:
                   4808: М. Кравчинский. История русского шансона.
                   4809: http://www.flibusta.net/b/367967/read
                   4810: 
                   4811: Автор:
                   4812: ???
                   4813: 
                   4814: Вопрос 11:
                   4815: Кандидат исторических наук Артем Ковалев участвовал в конференции,
                   4816: посвященной Калужской губернии. Свое сообщение об этом в Живом Журнале
                   4817: он самоиронично озаглавил "Йа [я] ПРОПУСК". Восстановите пропуск,
                   4818: добавив одну букву к интернет-мему.
                   4819: 
                   4820: Ответ:
                   4821: Краеведко.
                   4822: 
                   4823: Комментарий:
                   4824: Историки к краеведению относятся снисходительно.
                   4825: 
                   4826: Источник:
                   4827: http://a-v-kovalev.livejournal.com/20293.html
                   4828: 
                   4829: Автор:
                   4830: Алексей Пономарев (Калуга), по идее Павла Столярова (Калуга)
                   4831: 
                   4832: Вопрос 12:
                   4833: На предприятии, где работает автор вопроса, в период 90-х годов время от
                   4834: времени заказывали рулон красной ткани, а в деревообрабатывающем цехе
                   4835: изготавливали предмет, не имеющий прямого отношения к продукции завода.
                   4836: Назовите этот предмет.
                   4837: 
                   4838: Ответ:
                   4839: Гроб.
                   4840: 
                   4841: Источник:
                   4842: ЛОАВ.
                   4843: 
                   4844: Автор:
                   4845: ???
                   4846: 
                   4847: Вопрос 13:
                   4848: Друг автора вопроса, получив на день рождения современное устройство,
                   4849: пошутил, что слишком гетеросексуален для того, чтобы пользоваться ИМ.
                   4850: Существует гипотеза, что слово "ОН" произошло от имени Анри-Луи
                   4851: Жаке-Дроза. Назовите ЕГО.
                   4852: 
                   4853: Ответ:
                   4854: Андроид.
                   4855: 
                   4856: Источник:
                   4857:    1. ЛОАВ.
                   4858:    2. http://ru.wikipedia.org/wiki/Андроид
                   4859: 
                   4860: Автор:
                   4861: ???
                   4862: 
                   4863: Вопрос 14:
                   4864: На одном из этапов создания "ИХ" звукооператор Ари Пулкиннен принес ящик
                   4865: пива, и все нахрюкались. Назовите ИХ двумя словами, начинающимися на
                   4866: соседние буквы алфавита.
                   4867: 
                   4868: Ответ:
                   4869: "Angry Birds".
                   4870: 
                   4871: Источник:
                   4872: http://www.rbcdaily.ru/autonews/562949980422921
                   4873: 
                   4874: Автор:
                   4875: ???
                   4876: 
                   4877: Вопрос 15:
                   4878: В Колумбии были найдены древние статуи с раздутыми щеками, которые
                   4879: подтверждают, что еще инки знали свойства АЛЬФЫ. В каком виде спорта
                   4880: АЛЬФА была отменена в 2009 году?
                   4881: 
                   4882: Ответ:
                   4883: Дзюдо.
                   4884: 
                   4885: Комментарий:
                   4886: АЛЬФА - кока. В первом случае - растение, листья которого жуют. Во
                   4887: втором случае - оценка в дзюдо.
                   4888: 
                   4889: Источник:
                   4890:    1. http://incamusic.narod.ru/latinorama/coca/coca.htm
                   4891:    2. http://ru.wikipedia.org/wiki/Дзюдо
                   4892: 
                   4893: Автор:
                   4894: Алексей Пономарев (Калуга)
                   4895: 
                   4896: Вопрос 16:
                   4897: Автор вопроса назвал битвы коллег по работе из-за очереди к
                   4898: микроволновке ИМИ. "ОНИ" - название произведения 2012 года. Назовите ИХ
                   4899: двумя словами.
                   4900: 
                   4901: Ответ:
                   4902: Голодные игры.
                   4903: 
                   4904: Источник:
                   4905:    1. ЛОАВ.
                   4906:    2. http://ru.wikipedia.org/wiki/Голодные_игры_(фильм)
                   4907: 
                   4908: Автор:
                   4909: ???
                   4910: 
                   4911: Вопрос 17:
                   4912: [Ведущему: после оглашения ответа обязательно прочитать комментарий!]
                   4913:    Для выполнения удара топ-спин справа в настольном теннисе следует
                   4914: отвести правую руку с ракеткой назад, повернуть плечи, а ноги согнуть в
                   4915: коленях. Рассказывая об этом, один тренер упомянул уроженца города
                   4916: ЭлевфЕры. Назовите ЕГО.
                   4917: 
                   4918: Ответ:
                   4919: МИрон.
                   4920: 
                   4921: Комментарий:
                   4922: Личный опыт автора вопроса показывает, что поза при замахе ударом
                   4923: топ-спин напоминает позу "Дискобола" в скульптуре МИрона.
                   4924: 
                   4925: Источник:
                   4926:    1. http://ru.wikipedia.org/wiki/Топ-спин
                   4927:    2. http://ru.wikipedia.org/wiki/Мирон_из_Елевфер
                   4928: 
                   4929: Автор:
                   4930: Денис Карабанов (Калуга)
                   4931: 
                   4932: Вопрос 18:
                   4933: В настольном теннисе перед ответственной игрой резиновую накладку
                   4934: ракетки отделяют от деревянной основы и заново наклеивают специальным
                   4935: клеем, что увеличивает скорость полета мяча, но сокращает срок службы
                   4936: накладки. Автор вопроса сравнил такую накладку с НЕЙ. "ОНА" - название
                   4937: литературного произведения. Назовите ЕЕ.
                   4938: 
                   4939: Ответ:
                   4940: Шагреневая кожа.
                   4941: 
                   4942: Комментарий:
                   4943: В романе исполнение желаний приводило к сжиманию шагреневой кожи и
                   4944: сокращению жизни ее обладателя.
                   4945: 
                   4946: Источник:
                   4947:    1. ЛОАВ.
                   4948:    2. http://ru.wikipedia.org/wiki/Шагреневая_кожа
                   4949: 
                   4950: Автор:
                   4951: Денис Карабанов (Калуга)
                   4952: 
                   4953: Вопрос 19:
                   4954: "Осенью 1847 года в Риме Герцен участвует в народных шествиях,
                   4955: манифестациях, посещает революционные клубы...". Прослушанный вами
                   4956: отрывок из биографии написавшего "Кто виноват?" можно назвать четырьмя
                   4957: словами. Воспроизведите эти четыре слова, которые вы сегодня уже
                   4958: слышали.
                   4959: 
                   4960: Ответ:
                   4961: Личный опыт автора вопроса.
                   4962: 
                   4963: Источник:
                   4964: http://www.c-cafe.ru/days/bio/6/036.php
                   4965: 
                   4966: Автор:
                   4967: Денис Карабанов (Калуга)
                   4968: 
                   4969: Вопрос 20:
                   4970: Пациент психоаналитика Шернера видел сон, где два ряда белокурых
                   4971: мальчиков с нежным цветом лица готовились к борьбе. Далее во сне
                   4972: произошло действие, которое обычно осуществляют в кабинете... Какого
                   4973: врача?
                   4974: 
                   4975: Ответ:
                   4976: Стоматолога.
                   4977: 
                   4978: Зачет:
                   4979: Дантиста; зубного врача.
                   4980: 
                   4981: Комментарий:
                   4982: Действие - вытаскивание зуба.
                   4983: 
                   4984: Источник:
                   4985: http://www.magister.msk.ru/library/philos/freud/freud206.htm
                   4986: 
                   4987: Автор:
                   4988: Денис Карабанов (Калуга)
                   4989: 
                   4990: Вопрос 21:
                   4991: Когда в 1968 году среди прочих подразделений в Чехословакию вошла 7-я
                   4992: воздушно-десантная дивизия, местные жители подумали, что мировая
                   4993: общественность пришла им на помощь. Причиной ошибки были ОНИ. Кстати,
                   4994: "ОНИ" - название музыкального коллектива. Назовите ИХ.
                   4995: 
                   4996: Ответ:
                   4997: Голубые береты.
                   4998: 
                   4999: Комментарий:
                   5000: Тогда советские десантники только перешли с малиновых на голубые береты,
                   5001: и местные жители спутали их с войсками ООН.
                   5002: 
                   5003: Источник:
                   5004: http://ru.wikipedia.org/wiki/Голубой_берет
                   5005: 
                   5006: Автор:
                   5007: Алексей Пономарев (Калуга)
                   5008: 
                   5009: Вопрос 22:
                   5010: В историческом словаре галлицизмов Епишкина НЕКИМ СЛОВОМ определяют
                   5011: человека, применяющего кулачную расправу. Еще одно значение -
                   5012: исследователь творчества поэта. Назовите этого поэта.
                   5013: 
                   5014: Ответ:
                   5015: Данте [Алигьери].
                   5016: 
                   5017: Комментарий:
                   5018: Это слово - "дантист", т.е. дающий в зубы.
                   5019: 
                   5020: Источник:
                   5021: http://gallicismes.academic.ru/12285/
                   5022: 
                   5023: Автор:
                   5024: ???
                   5025: 
                   5026: Вопрос 23:
                   5027: Согласно шутке команды КВН "Саботаж", среди НИХ 1% имеют юридическое
                   5028: образование, 27% - нетрадиционной сексуальной ориентации. Мы не просим
                   5029: назвать ИХ. Назовите двумя словами продукт, который упоминается в этой
                   5030: шутке в связи с остальной частью.
                   5031: 
                   5032: Ответ:
                   5033: Хозяйственное мыло.
                   5034: 
                   5035: Комментарий:
                   5036: ОНИ - футбольные судьи. Согласно шутке: остальные 72% составляют кусок
                   5037: хозяйственного мыла. 72% - это тот процент содержания жирных кислот,
                   5038: который не должен быть превышен по ГОСТу при изготовлении хозяйственного
                   5039: мыла.
                   5040: 
                   5041: Источник:
                   5042:    1. http://www.ukrgame.net/forum/topic/5126-shutki-o-futbole-iz-kvn/
                   5043:    2. http://ru.wikipedia.org/wiki/Хозяйственное_мыло
                   5044: 
                   5045: Автор:
                   5046: ???
                   5047: 
                   5048: Вопрос 24:
                   5049: Причинами, по которым в России XIX века развивались ТАКИЕ промыслы, были
                   5050: неурожаи, а также недостаточная обеспеченность крестьян землей. По
                   5051: запросу "ТАКОЕ место" Википедия перенаправляет на статью, озаглавленную
                   5052: словом французского происхождения. Что это за слово?
                   5053: 
                   5054: Ответ:
                   5055: Туалет.
                   5056: 
                   5057: Зачет:
                   5058: Сортир.
                   5059: 
                   5060: Комментарий:
                   5061: ТАКОЙ - отхожий. Крестьяне уходили на промыслы в другие земли.
                   5062: 
                   5063: Источник:
                   5064:    1. http://ru.wikipedia.org/wiki/Отхожий_промысел
                   5065:    2. http://ru.wikipedia.org/wiki/Отхожее_место
                   5066: 
                   5067: Автор:
                   5068: ???
                   5069: 
                   5070: Тур:
                   5071: 13 тур. "Львята отца Симбы" (Северодвинск - Санкт-Петербург - Коломна)
                   5072: 
                   5073: Вопрос 1:
                   5074: (pic: 20150152.jpg)
                   5075:    Уважаемые игроки, у вас есть минута, чтобы раскрутить этот вопрос!
                   5076:    Перед вами четыре не совсем обычных ИХ. Мы не просим перечислить ИХ
                   5077: все. Назовите абсолютно точно то, что чаще всего обозначается этими
                   5078: четырьмя.
                   5079: 
                   5080: Ответ:
                   5081: [Игра] "Что? Где? Когда?".
                   5082: 
                   5083: Комментарий:
                   5084: (pic: 20150153.jpg)
                   5085:    ОНИ - латинские буквы. Изображены буквы C, H, G, K. Авторы рисунка
                   5086: раскрутили буквы относительно вертикальной оси пересечения с первым
                   5087: пикселом.
                   5088: 
                   5089: Источник:
                   5090: http://melotar.blogspot.ru/2011/05/3d.html
                   5091: 
                   5092: Автор:
                   5093: Елена Ганькова, Анна Дуплищева (Северодвинск)
                   5094: 
                   5095: Вопрос 2:
                   5096: Внимание, в вопросе ИКС является заменой.
                   5097:    Вам наверняка известен ресторан быстрого питания, в названии которого
                   5098: ИКС - это сокращение от фамилии владельца, Гершеля Пинкуса Йорахама. А
                   5099: кто слышал про другой - лучший в городе ресторан, в названии которого
                   5100: ИКС соседствует с фамилией владельца?! Несмотря на стихийные различия,
                   5101: оба ресторатора не без изъяна: они частенько бывают неприветливы и
                   5102: раздражительны по отношению к окружающим. Какое слово мы заменили на
                   5103: ИКС?
                   5104: 
                   5105: Ответ:
                   5106: Красти.
                   5107: 
                   5108: Комментарий:
                   5109: Гершель Пинкус Йорахам Крастовски - клоун Красти из мультсериала
                   5110: "Симпсоны" - является владельцем "Красти Бургера". В мультсериале "Губка
                   5111: Боб Квадратные Штаны" ресторан мистера Юджина Крабса носит название
                   5112: "Красти Крабс". Слово "Красти" (Krusty) созвучно английскому "crusty" -
                   5113: резкий, неприветливый, раздражительный (по отношению к человеку). "А кто
                   5114: слышал" и "не без изъяна" - отсылки к песенке в заставке мультсериала
                   5115: "Губка Боб квадратные штаны".
                   5116: 
                   5117: Источник:
                   5118:    1. http://ru.wikipedia.org/wiki/Клоун_Красти
                   5119:    2. http://ru.wikipedia.org/wiki/Мир_мультсериала_%C2%ABГубка_Боб_Квадратные_Штаны%C2%BB#.D0.9A.D1.80.D0.B0.D1.81.D1.82.D0.B8_.D0.9A.D1.80.D0.B0.D0.B1.D1.81
                   5120: 
                   5121: Автор:
                   5122: Елена Ганькова, Анна Дуплищева (Северодвинск)
                   5123: 
                   5124: Вопрос 3:
                   5125: В одном из сюжетов "Даешь, молодежь!" Ржавый стал главой местной банды
                   5126: хулиганов и активно подражает оскароносному персонажу. Один из хулиганов
                   5127: вспоминает, как новый предводитель когда-то сходил в туалет под дверь
                   5128: ненавистному участковому. В ответ на это Ржавый цитирует известное
                   5129: выражение, заменяя одно слово на "теплым". Воспроизведите первое слово
                   5130: этой фразы.
                   5131: 
                   5132: Ответ:
                   5133: Месть.
                   5134: 
                   5135: Комментарий:
                   5136: Стал строить из себя крестного отца. Диалог звучал примерно так:
                   5137:    - Я сделаю ему предложение, от которого он не сможет отказаться!
                   5138:    - Что, опять под дверь наделаешь?
                   5139:    - Месть - это блюдо, которое подается теплым!
                   5140:    Марлон Брандо и Роберт Де Ниро, сыгравшие старшего дона Корлеоне в
                   5141: первом и втором фильмах кинотрилогии о семействе Корлеоне, -
                   5142: единственная пара актеров, удостоенных премии "Оскар" за исполнение роли
                   5143: одного и того же персонажа.
                   5144: 
                   5145: Источник:
                   5146:    1. "Даешь, молодежь!".
                   5147:    2. http://ru.wikipedia.org/wiki/Вито_Корлеоне
                   5148: 
                   5149: Автор:
                   5150: Елена Ганькова (Северодвинск)
                   5151: 
                   5152: Вопрос 4:
                   5153: [Ведущему: обязательно прочитать "КАРИТАКУРА" и "ИЗОРБАЖЕНО" именно с
                   5154: таким порядком букв!]
                   5155:    Вопрос задает Льюис Кэрролл.
                   5156:    На одной англоязычной КАРИТАКУРЕ изображен рай для людей, страдающих
                   5157: неким расстройством. Какое животное ИЗОРБАЖЕНО на этой карикатуре?
                   5158: 
                   5159: Ответ:
                   5160: Собака.
                   5161: 
                   5162: Зачет:
                   5163: Dog.
                   5164: 
                   5165: Комментарий:
                   5166: Одним из проявлений дислексии является перестановка букв при чтении.
                   5167: Люди, страдающие от этого недуга, обычную для рая надпись GOD могут
                   5168: прочитать как DOG, т.е. собака, поэтому на карикатуре вместо Бога
                   5169: изображена собака. Слова "КАРИТАКУРА" и "ИЗОРБАЖЕНО" произнесены с
                   5170: характерной перестановкой букв. Одним из известных людей, страдавших
                   5171: дислексией, был Льюис Кэрролл.
                   5172: 
                   5173: Источник:
                   5174:    1. http://jasonkatzenstein.tumblr.com/post/105812203060/artistsontumblr-cartoons-katzentoons
                   5175:    2. http://www.dyslexialearning.com/perspective.htm
                   5176: 
                   5177: Автор:
                   5178: Владимир Киц (Северодвинск - Санкт-Петербург), Константин Осичев
                   5179: (Северодвинск)
                   5180: 
                   5181: Вопрос 5:
                   5182: В вопросе есть замены.
                   5183:    Заглавный герой советского мультфильма, убеждая хозяина в своей
                   5184: полезности, пел: "Конечно, я не ПЕРВАЯ, но я и не ВТОРОЙ". С этим не
                   5185: поспоришь, ведь он действительно не ВТОРОЙ. Назовите ПЕРВУЮ и ВТОРОГО.
                   5186: 
                   5187: Ответ:
                   5188: Мельница, осел.
                   5189: 
                   5190: Комментарий:
                   5191: В сказке "Кот в сапогах" первый (старший) сын получил мельницу, второй -
                   5192: осла. А Кот в сапогах, доставшийся младшему сыну, будущему маркизу де
                   5193: Карабасу, - не осел во всех значениях этого слова.
                   5194: 
                   5195: Источник:
                   5196: Мультфильм "Кот в сапогах" (1968).
                   5197: 
                   5198: Автор:
                   5199: Елена Ганькова (Северодвинск)
                   5200: 
                   5201: Вопрос 6:
                   5202: В 2003 году на сайте www.kostyor.ru проходил конкурс, на который
                   5203: школьники присылали свои стихотворные "вредные советы". Лучшие из них
                   5204: были мастерски выполнены по всем требованиям жанра - в стилистике
                   5205: основоположника Григория Остера. Автор вопроса назвал эту подборку,
                   5206: немного изменив известное словосочетание, которое не утратило значимости
                   5207: и привлекательности и в наши дни. Напишите получившееся в итоге
                   5208: название.
                   5209: 
                   5210: Ответ:
                   5211: Сделано по Г. Остеру.
                   5212: 
                   5213: Зачет:
                   5214: С незначительными отклонениями.
                   5215: 
                   5216: Комментарий:
                   5217: Например:
                   5218:    Если ты не скушал кашу
                   5219:    И не выпил свой компот,
                   5220:    Ну а мама почему-то
                   5221:    Говорит, что нужно есть,
                   5222:    Объясни ей, что ты хочешь
                   5223:    В Книгу Гиннесса попасть,
                   5224:    Под портретом будет подпись:
                   5225:    "Самый тощий человек".
                   5226:    (Слава Левитин, Екатеринбург)
                   5227:    Словосочетание "Сделано по ГОСТу" привлекает покупателей и в наше
                   5228: время, поэтому часто используется в рекламных целях.
                   5229: 
                   5230: Источник:
                   5231:    1. http://www.kostyor.ru/4-03/oster4-03.php
                   5232:    2. http://ru.wikipedia.org/wiki/Государственный_стандарт
                   5233: 
                   5234: Автор:
                   5235: Анна Дуплищева, Елена Ганькова (Северодвинск)
                   5236: 
                   5237: Вопрос 7:
                   5238: Начинающий литератор Григорий Остер предпочитал ЕЕ. Уже пожилого
                   5239: художника Илью Репина ошибочная она, наоборот, раздражала. У того, кому
                   5240: она обязана своим появлением, ЕЕ не было. У некоторых из вас ОНА есть.
                   5241: Назовите ЕЕ абсолютно точно.
                   5242: 
                   5243: Ответ:
                   5244: [Буква] Ё в фамилии.
                   5245: 
                   5246: Зачет:
                   5247: [Буква] Ё в псевдониме.
                   5248: 
                   5249: Комментарий:
                   5250: Ранним литературным псевдонимом Остера был "Остёр"; после реформы
                   5251: орфографии 1918 года Репина, которому было за семьдесят, бесило, что ему
                   5252: иногда писали письма с его фамилией через Ё; ну а в фамилии "Карамзин" и
                   5253: его литературных псевдонимах Ё нет и не было.
                   5254: 
                   5255: Источник:
                   5256:    1. http://ru.wikipedia.org/wiki/Остер,_Григорий_Бенционович
                   5257:    2. http://www.people.su/92873_3
                   5258:    3. http://nesusvet.narod.ru/txt/yo
                   5259: 
                   5260: Автор:
                   5261: Елена Ганькова (Северодвинск)
                   5262: 
                   5263: Вопрос 8:
                   5264: Комментируя известную беседу между двумя историческими личностями,
                   5265: Валерия Новодворская сравнивала ее с более поздними реалиями: "А
                   5266: представьте, что Константин Симонов говорит даже не Сталину, а Брежневу,
                   5267: что он мог бы вступить в РОА, в армию Власова!". О разговоре кого с кем
                   5268: изначально шла речь?
                   5269: 
                   5270: Ответ:
                   5271: А.С. Пушкина и Николая I.
                   5272: 
                   5273: Зачет:
                   5274: В любом порядке.
                   5275: 
                   5276: Комментарий:
                   5277: "... Сочувствие к идеалам заставили бы его и впрямь выйти на Сенатскую,
                   5278: будь он в Петербурге 14 декабря. Здесь он Николаю I сказал правду. И
                   5279: Николай съел этот прикол и не наложил взыскания. ... К счастью, его в
                   5280: Петербурге не было (заяц по дороге помешал: спасибо ему, косому!)".
                   5281: 
                   5282: Источник:
                   5283:    1. В.И. Новодворская. Поэты и цари. - М.: АСТ, 2010. - С. 6-10.
                   5284:    2. http://www.vikent.ru/enc/4211/
                   5285: 
                   5286: Автор:
                   5287: Елена Ганькова (Северодвинск)
                   5288: 
                   5289: Вопрос 9:
                   5290: (pic: 20150154.jpg)
                   5291:    Согласно шиитской традиции, первым имамом был зять ИКСА ИГРЕК. С 23
                   5292: июля 2007 года бывший игрок NFL Кёртис Конуэй - зять ИКСА ИГРЕКА.
                   5293: Назовите ИКСА и ИГРЕКА в правильном порядке.
                   5294: 
                   5295: Ответ:
                   5296: Мухаммед, Али.
                   5297: 
                   5298: Комментарий:
                   5299: Первым имамом был Али - зять пророка Мухаммеда. 23 июля 2007 года зятем
                   5300: Мухаммеда Али стал Кёртис Конуэй, за которого Лейла Али, дочь боксера,
                   5301: вышла замуж. Ранее, 27 августа 2000 года, Али вышла замуж за Джонни
                   5302: Макклейна, который стал ее менеджером. В конце 2005 года они развелись.
                   5303: Форма вопроса напоминает бабочку, что содержит намек на знаменитое
                   5304: выражение Мухаммеда Али - "Порхай как бабочка, жаль как пчела".
                   5305: 
                   5306: Источник:
                   5307:    1. http://ru.wikipedia.org/wiki/Имам
                   5308:    2. http://ru.wikipedia.org/wiki/Али,_Лейла
                   5309:    3. http://ru.wikipedia.org/wiki/Али,_Мохаммед
                   5310: 
                   5311: Автор:
                   5312: Елена Ганькова (Северодвинск)
                   5313: 
                   5314: Вопрос 10:
                   5315: Внимание, в вопросе есть замена.
                   5316:    Миниатюра Михаила Жванецкого: "Молодая врачиха встречалась с одним
                   5317: врачом довольно долго, потом заявила: "Надоело мне это КОЧЕВОЕ жилье.
                   5318: Перехожу на ОСЕДЛОЕ"". Какие два слова латинского происхождения мы
                   5319: заменили на "кочевое" и "оседлое"?
                   5320: 
                   5321: Ответ:
                   5322: Амбулаторное, стационар.
                   5323: 
                   5324: Зачет:
                   5325: Амбулаторное, стационарное.
                   5326: 
                   5327: Источник:
                   5328: М.М. Жванецкий. Любовь (Коротко). - М.: Эксмо, 2012. - С. 74.
                   5329: 
                   5330: Автор:
                   5331: Дмитрий Гевель (Северодвинск)
                   5332: 
                   5333: Вопрос 11:
                   5334: На одной карикатуре изображены спортивные судьи на стадионе. Надпись
                   5335: внизу картинки гласит: "Даже высокие технологии иногда бесполезны".
                   5336: Действительно, даже фотофиниш не позволяет озадаченным судьям найти
                   5337: ответ на вопрос... Назовите соревнующихся "спортсменов" в любом порядке.
                   5338: 
                   5339: Ответ:
                   5340: Курица, яйцо.
                   5341: 
                   5342: Зачет:
                   5343: В любом порядке.
                   5344: 
                   5345: Комментарий:
                   5346: Судьи ищут ответ на вопрос: что было раньше - курица или яйцо?
                   5347: 
                   5348: Источник:
                   5349: http://comicsia.ru/collections/ru_comicstrip/i28175.html
                   5350: 
                   5351: Автор:
                   5352: Анна Дуплищева, Елена Ганькова (Северодвинск)
                   5353: 
                   5354: Вопрос 12:
                   5355: Фотограф Марк Никсон опубликовал снимки плюшевых медведей, которые
                   5356: служат своим хозяевам десятилетиями (и со временем стали представлять
                   5357: собой жалкое зрелище). Заголовок подборки в русском издании журнала
                   5358: "Esquire" [эсквайр] представляет собой усеченное на одну букву
                   5359: устойчивое выражение, примененное Белинским к творчеству Гоголя.
                   5360: Воспроизведите получившийся заголовок.
                   5361: 
                   5362: Ответ:
                   5363: "Мех сквозь слезы".
                   5364: 
                   5365: Комментарий:
                   5366: "Смех сквозь слезы" - Белинский о Гоголе.
                   5367: 
                   5368: Источник:
                   5369:    1. http://esquire.ru/photo/muchloved
                   5370:    2. http://www.lingvomania.info/krylatye-slova/99-2
                   5371: 
                   5372: Автор:
                   5373: Дмитрий Гевель (Северодвинск)
                   5374: 
                   5375: Вопрос 13:
                   5376: Одна из статей журнала "Вокруг света" посвящалась заимствованиям в
                   5377: русском языке из других языков, и некоторые примеры таких заимствований
                   5378: были проиллюстрированы. Рисунок, сопровождающий слово из турецкого языка
                   5379: со значением "он не знает", изображал известного человека. Назовите имя
                   5380: и фамилию этого человека.
                   5381: 
                   5382: Ответ:
                   5383: Юрий Никулин.
                   5384: 
                   5385: Комментарий:
                   5386: Слово "балбес". Никулин был нарисован в образе своего знаменитого
                   5387: персонажа - Балбеса.
                   5388: 
                   5389: Источник:
                   5390: "Вокруг света", 2012, N 1. - С. 116.
                   5391: 
                   5392: Автор:
                   5393: Елена Ганькова (Северодвинск)
                   5394: 
                   5395: Вопрос 14:
                   5396: [Ведущему: кавычки не озвучивать, четко прочитать "икс".]
                   5397:    Уважаемые знатоки! При ответе не будьте многословны.
                   5398:    Пару месяцев назад автор вопроса нашел в Базе Вопросов "Что? Где?
                   5399: Когда?" вопрос 2009 года и, используя его на тренировке, убрал из
                   5400: комментария "ИКС-". Что мы заменили в этом вопросе?
                   5401: 
                   5402: Ответ:
                   5403: Э.
                   5404: 
                   5405: Комментарий:
                   5406: Вопрос 2015 года. В.В. Путин в 2009 году был ЭКС-президентом, автор
                   5407: вопроса убрал "ЭКС-", а не "ИКС-". С 7 мая 2012 года Путин - президент
                   5408: РФ. Пожелание "не быть многословными" также намекает на него: "Буду
                   5409: краток".
                   5410: 
                   5411: Источник:
                   5412: http://db.chgk.info/question/sport09.2/10
                   5413: 
                   5414: Автор:
                   5415: Елена Ганькова (Северодвинск)
                   5416: 
                   5417: Вопрос 15:
                   5418: На одной из праздничных листовок Либерально-демократической партии
                   5419: России изображено, как ее лидер - Владимир Жириновский - рисует некий
                   5420: символ. На этой же листовке аббревиатура партии расшифрована как
                   5421: побуждающая к действию строчка из песни 1994 года. Назовите автора этой
                   5422: строчки.
                   5423: 
                   5424: Ответ:
                   5425: [Валерий] Сюткин.
                   5426: 
                   5427: Комментарий:
                   5428: На листовке ко Дню всех влюбленных Жириновский пальцем изображает
                   5429: сердечко, аббревиатура ЛДПР расшифрована как "Любите, девушки, простых
                   5430: романтиков!" - строчкой из песни группы "Браво", вошедшей в альбом
                   5431: "Дорога в облака" (1994).
                   5432: 
                   5433: Источник:
                   5434:    1. http://ibigdan.livejournal.com/14602475.html
                   5435:    2. http://www.pesnigitara.com/statpokaz.php?i=bravo__lyubite_devus
                   5436: 
                   5437: Автор:
                   5438: Елена Ганькова (Северодвинск)
                   5439: 
                   5440: Вопрос 16:
                   5441: По словам героя Сергея Лукьяненко, ОНИ - странное оружие, "единственное,
                   5442: которое применялось не для защиты, а только для убийства. Пусть даже и
                   5443: узаконенного...". Толстой воспользовался ИМИ как минимум 11 раз.
                   5444: Назовите ИХ.
                   5445: 
                   5446: Ответ:
                   5447: Дуэльные пистолеты.
                   5448: 
                   5449: Зачет:
                   5450: Пистолеты для дуэли и т.п.
                   5451: 
                   5452: Комментарий:
                   5453: Имеется в виду знаменитый дуэлянт Федор Толстой "Американец".
                   5454: Неизвестно, сколько раз в своей жизни Толстой дрался на дуэлях, однако,
                   5455: по некоторым данным, он убил на них в общей сложности одиннадцать
                   5456: человек.
                   5457: 
                   5458: Источник:
                   5459:    1. С. Лукьяненко. Способность спустить курок. / С. Лукьяненко. "Л" -
                   5460: значит люди. - М.: Изд-во "Москва", 2006. - С. 209.
                   5461:    2. http://ru.wikipedia.org/wiki/Толстой,_Фёдор_Иванович
                   5462: 
                   5463: Автор:
                   5464: Елена Ганькова (Северодвинск)
                   5465: 
                   5466: Вопрос 17:
                   5467: (pic: 20150155.jpg)
                   5468:    Перед вами изображение болида "Формулы-1" и приблизительная
                   5469: информация об одном из параметров. Слева от чисел, изображенных на
                   5470: картинке, мы убрали один и тот же знак, а справа - букву. Мы не
                   5471: спрашиваем, что это за буква. Скажите, какое слово она обозначает.
                   5472: 
                   5473: Ответ:
                   5474: Миллион.
                   5475: 
                   5476: Зачет:
                   5477: Миллионы, million, millions.
                   5478: 
                   5479: Комментарий:
                   5480: (pic: 20150156.jpg)
                   5481:    Это стоимость годового размещения рекламы на том или ином месте
                   5482: болида "Формулы-1" в миллионах долларов. С изображения мы убрали знак
                   5483: доллара и букву "m".
                   5484: 
                   5485: Источник:
                   5486: http://ibigdan.livejournal.com/15837893.html
                   5487: 
                   5488: Автор:
                   5489: Анна Дуплищева, Елена Ганькова (Северодвинск)
                   5490: 
                   5491: Вопрос 18:
                   5492: Блиц.
                   5493:    Дизайнер Александр Уткин предложил необычное оформление собрания
                   5494: сочинений Н.В. Гоголя.
                   5495:    1. На обложке первого тома собрания сочинений звезды на ночном небе
                   5496: образуют профиль автора, а рядом изображены ОНИ. Назовите ИХ обоих.
                   5497:    2. На обложке второго тома собрания сочинений на дощатом полу
                   5498: нарисован профиль автора, а рядом изображены горящие свечи и ОН.
                   5499: Назовите ЕГО.
                   5500:    3. На обложке третьего тома собрания сочинений схематически нарисован
                   5501: профиль автора, и особо обозначен ОН Гоголя. Назовите ЕГО.
                   5502: 
                   5503: Ответ:
                   5504:    1. Кузнец Вакула, черт.
                   5505:    2. Хома Брут.
                   5506:    3. Нос.
                   5507: 
                   5508: Зачет:
                   5509:    1. Вакула, черт; кузнец, черт.
                   5510: 
                   5511: Комментарий:
                   5512: (pic: 20150157.jpg)
                   5513: 
                   5514: Источник:
                   5515: https://vk.com/novlit?w=wall-26488304_5832
                   5516: 
                   5517: Автор:
                   5518: Елена Ганькова, Анна Дуплищева (Северодвинск)
                   5519: 
                   5520: Вопрос 19:
                   5521: Внимание, в вопросе есть замены.
                   5522:    Когда пожилая ПУШКИНА очень пренебрежительно высказалась о
                   5523: ПУШКАРЁВОЙ, поэт Вольпин посмеялся и заметил: "Вы просто ревнуете,
                   5524: потому что ПУШК-, да молодая, да хорошенькая!". По словам ПУШКИНОЙ,
                   5525: стихи ПУШКАРЁВОЙ "пахли хорошим кофе". Какие фамилии мы заменили на
                   5526: ПУШКИНУ и ПУШКАРЁВУ?
                   5527: 
                   5528: Ответ:
                   5529: Ахматова, Ахмадуллина.
                   5530: 
                   5531: Комментарий:
                   5532: "Вы просто ревнуете, потому что Ахма-, да молодая, да хорошенькая!".
                   5533: 
                   5534: Источник:
                   5535: Т. Катаева. Анти-Ахматова. - М.: ЕвроИНФО, 2007. - С. 396-397.
                   5536: 
                   5537: Автор:
                   5538: Елена Ганькова (Северодвинск)
                   5539: 
                   5540: Вопрос 20:
                   5541: Сергей Лукьяненко в "Ночном дозоре" пишет: "Наверное, лучшие в мире
                   5542: дипломаты - восточные. ДЕЛАЮТ ЭТО, но это вовсе не значит, что они
                   5543: воздерживаются...". Согласно одному правилу, ДЕЛАТЬ ЭТО должны участники
                   5544: известной игры. А что предписывает другое правило этой игры?
                   5545: 
                   5546: Ответ:
                   5547: Черное и белое не брать.
                   5548: 
                   5549: Зачет:
                   5550: Черное и белое не покупать/выбирать/называть и т.п.
                   5551: 
                   5552: Комментарий:
                   5553: "Наверное, лучшие в мире дипломаты - восточные. "Да" и "нет" не говорят,
                   5554: но это вовсе не значит, что они воздерживаются...". В детской игре нужно
                   5555: "да" и "нет" не говорить, черное и белое не брать.
                   5556: 
                   5557: Источник:
                   5558:    1. С. Лукьяненко. Ночной дозор. - М.: АСТ, 2006. - С. 235.
                   5559:    2. http://www.malchishki-i-devchonki.ru/chernoe%20i%20beloe.html
                   5560: 
                   5561: Автор:
                   5562: Елена Ганькова (Северодвинск)
                   5563: 
                   5564: Вопрос 21:
                   5565: Вулкан Михара - одна из главных достопримечательностей японского острова
                   5566: Идзуосима. За последние 80 лет здесь совершили самоубийство тысячи
                   5567: людей, в основном - из-за неразделенной любви. Одна из компаний даже
                   5568: умудрилась заработать на самоубийцах и жаждущих необычного зрелища
                   5569: зеваках. Мы не спрашиваем вас, что это за компания. Ответьте, что
                   5570: запретили власти Японии для снижения эпидемии суицида.
                   5571: 
                   5572: Ответ:
                   5573: Продавать билеты в один конец.
                   5574: 
                   5575: Зачет:
                   5576: По смыслу.
                   5577: 
                   5578: Комментарий:
                   5579: На остров можно попасть, в основном, по воде. Пароходная компания делала
                   5580: деньги на самоубийцах, желающих попасть на него. Совсем запретить
                   5581: продавать туда билеты власти не могли, так как там постоянно живут люди.
                   5582: 
                   5583: Источник:
                   5584:    1. http://www.factroom.ru/facts/43230
                   5585:    2. http://ru.wikipedia.org/wiki/Идзуосима
                   5586: 
                   5587: Автор:
                   5588: Владимир Киц (Северодвинск - Санкт-Петербург)
                   5589: 
                   5590: Вопрос 22:
                   5591: В 70-х годах XX века один из экспонатов зоологического музея МГУ покинул
                   5592: его почти на месяц, а всё для того, чтобы принять участие в съемках
                   5593: кинокомедии. Почти всю картину чучело парнокопытного буквально бок о бок
                   5594: "проработало" рядом с персонажем в исполнении... Кого?
                   5595: 
                   5596: Ответ:
                   5597: [Эльдара] Рязанова.
                   5598: 
                   5599: Комментарий:
                   5600: Бегемот, на котором спал Рязанов в фильме "Гараж".
                   5601: 
                   5602: Источник:
                   5603: "Москва студенческая пешком", телеканал "Культура", 13.05.2013 г.
                   5604: 
                   5605: Автор:
                   5606: Елена Ганькова (Северодвинск)
                   5607: 
                   5608: Вопрос 23:
                   5609: Внимание, вопрос с заменой.
                   5610:    "Человеческая комедия" поэта-сатирика Олега Молоткова - это
                   5611: произведение АЛЬФА. Вам наверняка известно другое стихотворение,
                   5612: которое, как и данный вопрос, тоже АЛЬФА. АЛЬФА - это два слова. Какие
                   5613: именно?
                   5614: 
                   5615: Ответ:
                   5616: Без глаголов.
                   5617: 
                   5618: Комментарий:
                   5619: Стихотворение Молоткова начинается со строк:
                   5620:    Мама, сказка, каша, кошка,
                   5621:    книжка, яркая обложка,
                   5622:    Буратино, Карабас,
                   5623:    ранец, школа, первый класс...
                   5624:    и заканчивается:
                   5625:    ....юбилей, банкет, награда,
                   5626:    речи, памятник, ограда.
                   5627:    Вот такая "Человеческая комедия" без единого глагола. В тексте
                   5628: данного вопроса нет ни одного глагола. А вам известно стихотворение
                   5629: "Шепот, робкое дыханье..." А. Фета.
                   5630: 
                   5631: Источник:
                   5632:    1. http://www.proza.ru/2010/10/22/922/
                   5633:    2. http://ru.wikisource.org/wiki/Шёпот,_робкое_дыханье_(Фет)
                   5634: 
                   5635: Автор:
                   5636: Анна Дуплищева, Елена Ганькова (Северодвинск)
                   5637: 
                   5638: Вопрос 24:
                   5639: После этого вопроса мы отдохнем.
                   5640:    Цитата из произведения В.А. Гиляровского: "... Я завернулся в облака
                   5641: и море, сунул под голову крышку гроба... и уснул сном счастливого
                   5642: человека...". А где именно устроился на ночлег повествователь?
                   5643: 
                   5644: Ответ:
                   5645: В театре.
                   5646: 
                   5647: Зачет:
                   5648: В уборной театра; по смыслу.
                   5649: 
                   5650: Комментарий:
                   5651: Будучи в Рязани, ночевал в театре и завернулся в декорации. "Мы
                   5652: отдохнем" - намек на пьесу А.П. Чехова "Дядя Ваня", т.е. на постановку в
                   5653: театре, а не только на то, что вы сможете пойти поспать, как и герой
                   5654: вопроса.
                   5655: 
                   5656: Источник:
                   5657:    1. В.А. Гиляровский. Люди театра. / В.А. Гиляровский. Трущобные люди.
                   5658: Москва и москвичи. - Кишинев, 1988. - С. 261.
                   5659:    2. http://www.bibliotekar.ru/encSlov/13/167.htm
                   5660: 
                   5661: Автор:
                   5662: Елена Ганькова (Северодвинск)
                   5663: 
                   5664: Тур:
                   5665: 14 тур. "Благоест" (Коломна)
                   5666: 
                   5667: Вопрос 1:
                   5668: Блиц.
                   5669:    1. Телепрограмма Никиты Белоголовцева, посвященная баскетболу,
                   5670: называется ОНО. Назовите ЕГО двумя словами.
                   5671:    2. На карикатуре, посвященной событию начала XXI века, изображены
                   5672: ОНИ. Назовите ИХ двумя словами на разные буквы.
                   5673:    3. Георг II правил Грецией в 1922-1924 годах. Затем в стране была
                   5674: установлена республика. В 1935 году произошло ОНО, и Георг II вновь стал
                   5675: править Грецией. Назовите ЕГО двумя словами.
                   5676: 
                   5677: Ответ:
                   5678:    1. Братство кольца.
                   5679:    2. Две башни.
                   5680:    3. Возвращение короля.
                   5681: 
                   5682: Автор:
                   5683: Кирилл Третьяков (Коломна)
                   5684: 
                   5685: Вопрос 2:
                   5686: Тетраблиц. Четыре вопроса по пятнадцать секунд.
                   5687:    1. Прослушайте стихотворение.
                   5688:    На тихих берегах ПРОПУСК
                   5689:    Церквей, венчанные крестами,
                   5690:    Сияют ветхие главЫ
                   5691:    Над монастырскими стенами.
                   5692:    Заполните пропуск.
                   5693:    2. Прослушайте стихотворение.
                   5694:    Август выдался прохладным...
                   5695:    Что же жарко мне?
                   5696:    Стало пусто в ПРОПУСК
                   5697:    Утром. На заре.
                   5698:    Заполните пропуск.
                   5699:    3. Прослушайте стихотворение.
                   5700:    Я в восторге от ПРОПУСК города.
                   5701:    Но кепчонку не сдерну с виска.
                   5702:    У советских собственная гордость:
                   5703:    На буржуев смотрим свысока.
                   5704:    Заполните пропуск.
                   5705:    4. Прослушайте стихотворение.
                   5706:    Путешественник по всей Вселенной,
                   5707:    Наш профессор говорил не раз,
                   5708:    Как, похож на уголек нетленный,
                   5709:    Умирал и возрождался ПРОПУСК.
                   5710:    Заполните пропуск.
                   5711: 
                   5712: Ответ:
                   5713:    1. Москва.
                   5714:    2. Нагасаки.
                   5715:    3. Нью-Йорка.
                   5716:    4. Марс.
                   5717: 
                   5718: Комментарий:
                   5719: Ответы отсылают к стихотворению Игоря Северянина.
                   5720: 
                   5721: Автор:
                   5722: Кирилл Третьяков (Коломна)
                   5723: 
                   5724: Вопрос 3:
                   5725: Внимание, в вопросе слово "АЛЬФА" заменяет другое слово.
                   5726:    Американский антрополог Карлтон Стивенс Кун в своем труде "Расы
                   5727: Европы" пишет: "В конце бронзового века на период два или три столетия
                   5728: завеса АЛЬФЫ падает над расовой историей Европы". Какое слово мы
                   5729: заменили на АЛЬФУ?
                   5730: 
                   5731: Ответ:
                   5732: Кремация.
                   5733: 
                   5734: Комментарий:
                   5735: Кремация трупов привела к отсутствию материалов для исследователей. Она,
                   5736: подобно занавесу, скрыла длительный период развития рас в Европе.
                   5737: 
                   5738: Источник:
                   5739: В вопросе.
                   5740: 
                   5741: Автор:
                   5742: Иван Морозов (Коломна)
                   5743: 
                   5744: Вопрос 4:
                   5745: Внимание, в вопросе слово "ОН" является заменой.
                   5746:    Польский писатель Анджей Сапковский в статье "ВаренИк, или Нет золота
                   5747: в Серых Горах" пишет: "Увы, повторяю, в случае фэнтези отсутствует
                   5748: знание канона. И метода. Нет МЕТОДА. Остался ОН...". Имя какого
                   5749: христианского святого мы заменили местоимением "ОН"?
                   5750: 
                   5751: Ответ:
                   5752: Кирилл.
                   5753: 
                   5754: Комментарий:
                   5755: Кирилл и Мефодий (по-польски - Cyryl i Metody [цырыл и методы]).
                   5756: Своеобразная игра слов. Замена звука "Ф" на "Т" вполне известна
                   5757: (например, Фома и Том). А если нет Мефодия, то остается Кирилл!
                   5758: 
                   5759: Источник:
                   5760: В вопросе.
                   5761: 
                   5762: Автор:
                   5763: Иван Морозов (Коломна)
                   5764: 
                   5765: Вопрос 5:
                   5766: Как-то раз весной автор вопроса, просматривая интернет-странички, с
                   5767: удивлением обнаружил, что некоторые особо заядлые пользователи
                   5768: Интернета, увидев ИХ, первым делом бросаются просматривать таблицы
                   5769: смайликов. Будете в следующий раз проходить мимо - обратите на это
                   5770: внимание. Мы не просим вас назвать ИХ. Напишите наречие, которое в тот
                   5771: день автор вопроса отправил в наибольшем количестве писем.
                   5772: 
                   5773: Ответ:
                   5774: Воистину.
                   5775: 
                   5776: Комментарий:
                   5777: Некоторые заядлые интернет-пользователи буквы "ХВ" воспринимают как один
                   5778: из смайликов, таких как XD, т.е. смех с зажмуренными глазами, или B-),
                   5779: т.е. надевший очки в роговой оправе. На сайте
                   5780: http://www.diary.ru/~jafnvel/p156394694.htm так прямо и сказано: "ХВ - и
                   5781: это не смайлик". :-) Кстати, Пасха дословно означает "прохождение мимо".
                   5782: Тоже намек на правильный ответ.
                   5783: 
                   5784: Автор:
                   5785: Иван Морозов (Коломна)
                   5786: 
                   5787: Вопрос 6:
                   5788: В одной из статей "Новой газеты" рассказывалось, что в преддверии
                   5789: выборов многие неугодные действующей власти статьи в ЖЖ практически
                   5790: мгновенно получали огромный отрицательный рейтинг, и наоборот -
                   5791: хвалебные статьи в адрес правящей партии за считанные минуты вырывались
                   5792: в лидеры. Высказывалось предположение, что это делается искусственно.
                   5793: Статья называлась: "На полях "Живого журнала" - высокая урожайность ЕЕ".
                   5794: Чего?
                   5795: 
                   5796: Ответ:
                   5797: БОТвы.
                   5798: 
                   5799: Зачет:
                   5800: Ботвы.
                   5801: 
                   5802: Источник:
                   5803: http://www.novayagazeta.ru/society/6070.html
                   5804: 
                   5805: Автор:
                   5806: Иван Морозов (Коломна)
                   5807: 
                   5808: Вопрос 7:
                   5809: Дуплет.
                   5810:    Внимание, словосочетания "ПЕРВЫЙ ПРОПУСК" и "ПОСЛЕДНИЙ ПРОПУСК"
                   5811: являются заменами.
                   5812:    1. В математике взаимно-однозначные отображения на торе,
                   5813: заключающиеся в растяжении по параллели и сжатии по меридиану, получили
                   5814: название "Кот ПЕРВОГО ПРОПУСКА". Напишите фамилию, которую мы заменили
                   5815: словами "ПЕРВЫЙ ПРОПУСК".
                   5816:    2. В первом фильме, где снимался ПОСЛЕДНИЙ ПРОПУСК, все реплики его
                   5817: героя пришлось дублировать другому актеру, а во втором фильме ему и
                   5818: вовсе досталась роль глухонемого киллера. Напишите фамилию, которую мы
                   5819: заменили словами "ПОСЛЕДНИЙ ПРОПУСК".
                   5820: 
                   5821: Ответ:
                   5822:    1. Арнольд.
                   5823:    2. Шварценеггер.
                   5824: 
                   5825: Комментарий:
                   5826: Владимир Игоревич Арнольд - известнейший советский и российский
                   5827: математик. Замены "первый пропуск" и "последний пропуск" - намек на
                   5828: английские "first name" и "last name", т.е. имя и фамилия. Таким
                   5829: образом, фамилия первого должна была оказаться именем второго.
                   5830: 
                   5831: Автор:
                   5832: Иван Морозов (Коломна)
                   5833: 
                   5834: Вопрос 8:
                   5835: В 2013 году в России вышла автобиографическая книга Арнольда
1.17      rubashki 5836: Шварцнеггера под названием "[Два слова пропущено]: моя необыкновенно
1.1       rubashki 5837: правдивая история". Пропущенные слова являются российским вариантом
                   5838: названия известного произведения по книге американского
                   5839: писателя-фантаста. Назовите этого писателя.
                   5840: 
                   5841: Ответ:
                   5842: ФИлип [КИндред] Дик.
                   5843: 
                   5844: Комментарий:
                   5845: Название автобиографии Шварценеггера в русском издании: "Вспомнить всё:
                   5846: моя необыкновенно правдивая история". "Вспомнить всё" - это русское
                   5847: название фильма по мотивам рассказа Дика. А сам рассказ назывался "We
                   5848: Can Remember It for You Wholesale". Это переводили как "Мы вам всё
                   5849: припомним", "В глубине памяти" или "Из глубин памяти", но никак не
                   5850: "Вспомнить всё".
                   5851: 
                   5852: Автор:
                   5853: Иван Морозов (Коломна)
                   5854: 
                   5855: Вопрос 9:
                   5856: В повести "Республика ШКИД" для предотвращения распространения ЭТОГО
                   5857: герои вынужденно меняют свою внешность, из-за чего им становится сложно
                   5858: общаться с некоторыми из своих друзей. В наши дни Роспотребнадзор
                   5859: предупреждает о возможности распространения ЭТОГО среди подростков,
                   5860: которые любят делать групповые селфи. Назовите ЭТО.
                   5861: 
                   5862: Ответ:
                   5863: Педикулез.
                   5864: 
                   5865: Зачет:
                   5866: Вшивость.
                   5867: 
                   5868: Комментарий:
                   5869: В повести всех парней побрили налысо, и им стало стыдно ходить к
                   5870: девушкам на свидания.
                   5871: 
                   5872: Источник:
                   5873:    1. Г. Белых, Л. Пантелеев. Республика ШКИД.
                   5874:    2. http://ru.wikipedia.org/wiki/Селфи
                   5875: 
                   5876: Автор:
                   5877: Изета Насырова (Коломна)
                   5878: 
                   5879: Вопрос 10:
                   5880: Сэр Генри Мерривейл - герой детективных романов Джона Диксона Карра -
                   5881: обладает тонким живым умом и склонностью к решению криминальных
                   5882: головоломок. К его помощи прибегают исключительно в безнадежных случаях.
                   5883: Коллеги по Уайтхоллу за глаза называют сэра Генри именем одного
                   5884: литературного героя, говоря, что он очень похож на этого персонажа -
                   5885: также обладает большими "дедуктивными способностями... только слишком
                   5886: ленив, чтобы использовать их на практике", что он "хранит в памяти
                   5887: информацию, как в картотеке", а его главным достоинством является
                   5888: недюжинный ум. Напишите имя, которым называют сэра Генри.
                   5889: 
                   5890: Ответ:
                   5891: Майкрофт.
                   5892: 
                   5893: Зачет:
                   5894: Майкрофт Холмс.
                   5895: 
                   5896: Комментарий:
                   5897: Шерлок не был столь ленив, а напротив - с удовольствием брался за любое
                   5898: "дело".
                   5899: 
                   5900: Источник:
                   5901: Джон Диксон Карр. Убийства в Плейг-Корте.
                   5902: http://www.flibusta.net/b/116430/read
                   5903: 
                   5904: Автор:
                   5905: Валерия Комаровская (Коломна)
                   5906: 
                   5907: Вопрос 11:
                   5908: Главный герой одного фильма стал ИКСОМ в буквальном смысле этого слова.
                   5909: В России людей, увлекающихся специфичным, по мнению автора вопроса,
                   5910: хобби, называют ИКСАМИ. В Финляндии - "выдрами" и "нерпами", в Северной
                   5911: Америке - "белыми медведями". Назовите ИКСА.
                   5912: 
                   5913: Ответ:
                   5914: Морж.
                   5915: 
                   5916: Комментарий:
                   5917: Главный герой фильма "Бивень" Уоллес Бритон попал в "лапы" маньяка, и
                   5918: тот путем нескольких операций "сшил" из него моржа. Людей, занимающихся
                   5919: зимним плаванием, называют моржами.
                   5920: 
                   5921: Источник:
                   5922:    1. Фильм "Бивень".
                   5923:    2. http://ru.wikipedia.org/wiki/Зимнее_плавание
                   5924: 
                   5925: Автор:
                   5926: Кристина Шадрина (Коломна)
                   5927: 
                   5928: Вопрос 12:
                   5929: Вопрос для тех, кто "в теме".
                   5930:    Как ни парадоксально, но публикация сайта wikiHow подробно дает
                   5931: рекомендации как стать ИМ. В частности, к таким рекомендациям относятся:
                   5932:    - заведите огород и станьте вегетарианцем;
                   5933:    - используйте слова, значения которых никто не знает, например,
                   5934: "фантасмагория", "экспроприация", "ревербератор", "экзальтированность";
                   5935:    - когда вас о чем-то спрашивают, не отвечайте напрямую, вместо этого
                   5936: ответьте расплывчато, задайте встречный вопрос или просто проявите
                   5937: сарказм и другое.
                   5938:    Назовите мультипликационного героя, который, согласно шутке, является
                   5939: первым советским ИМ.
                   5940: 
                   5941: Ответ:
                   5942: Шарик.
                   5943: 
                   5944: Зачет:
                   5945: Шарик из Простоквашино.
                   5946: 
                   5947: Комментарий:
                   5948: Шарик вместо полезных в хозяйстве вещей купил себе кеды и фоторужье.
                   5949: Хипстеры - первоначально представители особой субкультуры,
                   5950: сформировавшейся в среде поклонников джазовой музыки, понятие образовано
                   5951: от жаргонного "to be hip" ("быть в теме"). Главная задача современного
                   5952: хипстера - достижение полной независимости взглядов и действий.
                   5953: 
                   5954: Источник:
                   5955:    1. http://ru.wikihow.com/быть-хипстером
                   5956:    2. http://www.anekdot.ru/id/502287/
                   5957:    3. http://insurgent.ru/hipster
                   5958: 
                   5959: Автор:
                   5960: Александр Романкин (Коломна)
                   5961: 
                   5962: Вопрос 13:
                   5963: Модель походного вентилятора фирмы "Экспедиция" своим названием как бы
                   5964: говорит покупателю, что этот вентилятор просто делает свою работу. Это
                   5965: название созвучно слогану компании, основанной в 1964 году. Назовите эту
                   5966: компанию.
                   5967: 
                   5968: Ответ:
                   5969: "Nike".
                   5970: 
                   5971: Комментарий:
                   5972: Слоган - "Just Do It" ("Просто сделай это").
                   5973: 
                   5974: Источник:
                   5975:    1. http://www.e-xpedition.ru/products/ventiljatory-just-duet/
                   5976:    2. http://en.wikipedia.org/wiki/Just_Do_It
                   5977: 
                   5978: Автор:
                   5979: Кирилл Третьяков (Коломна)
                   5980: 
                   5981: Вопрос 14:
                   5982: Статья на сайте politonline.ru [полит онлайн точка ру], посвященная
                   5983: событиям весны 2014 года на Украине, называется "Обескровленная и
                   5984: расколотая". Какие две буквы мы заменили в предыдущем предложении?
                   5985: 
                   5986: Ответ:
                   5987: ЫМ.
                   5988: 
                   5989: Комментарий:
                   5990: "ОбескрЫМленная и расколотая".
                   5991: 
                   5992: Источник:
                   5993: http://www.politonline.ru/comments/16482.html
                   5994: 
                   5995: Автор:
                   5996: Кирилл Третьяков (Коломна)
                   5997: 
                   5998: Вопрос 15:
                   5999: Город Урюпинск в фольклоре служит символом глубинки. В прессе и
                   6000: литературе Урюпинск зачастую парадоксально называют ЕЮ. Назовите ЕЕ
                   6001: тремя словами, начинающимися на соседние буквы.
                   6002: 
                   6003: Ответ:
                   6004: Столица российской провинции.
                   6005: 
                   6006: Источник:
                   6007:    1. http://www.5-tv.ru/news/62882/
                   6008:    2. http://www.rg.ru/2014/11/11/reg-ufo/urypinsk.html
                   6009: 
                   6010: Автор:
                   6011: Кирилл Третьяков (Коломна)
                   6012: 
                   6013: Вопрос 16:
                   6014: Внимание, слово "АЛЬФА" заменяет несколько других слов.
                   6015:    Ричард Касл - герой детективного сериала - писатель, помогающий вести
                   6016: следствие полицейским убойного отдела. Он всегда полон различных
                   6017: фантастических идей и о причинах убийств, и о самих жертвах, и о
                   6018: способах умерщвления. Так, при расследовании смерти финансиста с
                   6019: Уолл-Стрит Касл предполагает, что его убила АЛЬФА, ведь на теле нет
                   6020: оборонительных ран. Назовите того, кто в XVIII веке впервые ввел понятие
                   6021: АЛЬФЫ.
                   6022: 
                   6023: Ответ:
                   6024: Адам Смит.
                   6025: 
                   6026: Комментарий:
                   6027: АЛЬФА - невидимая рука рынка.
                   6028: 
                   6029: Источник:
                   6030: Сериал "Касл", s06e19, 04:54.
                   6031: 
                   6032: Автор:
                   6033: Валерия Комаровская (Коломна)
                   6034: 
                   6035: Вопрос 17:
                   6036: (pic: 20150158.jpg)
                   6037:    На розданной вам картинке изображен Монумент Славы в виде рабочего с
                   6038: крыльями. Жители Самары в шутку именуют его "Непьющий человек", а на
                   6039: вопрос "Почему непьющий?" отвечают именно так. Как именно?
                   6040: 
                   6041: Ответ:
                   6042: [У него] руки заняты.
                   6043: 
                   6044: Источник:
                   6045: http://www.9999-777.ru/pamyatniki-samary.html
                   6046: 
                   6047: Автор:
                   6048: Иван Морозов (Коломна)
                   6049: 
                   6050: Вопрос 18:
                   6051: (pic: 20150159.jpg)
                   6052:    На одном из сайтов автор вопроса увидел данное фото. Остроумным
                   6053: комментарием к этой фотографии явилась цитата заглавного героя
                   6054: произведения 1969 года. Напишите эту цитату.
                   6055: 
                   6056: Ответ:
                   6057: Это неправильные пчелы!
                   6058: 
                   6059: Источник:
                   6060:    1. http://ibigdan.livejournal.com/10706110.html
                   6061:    2. http://ru.wikipedia.org/wiki/Винни-Пух_(мультфильм)
                   6062: 
                   6063: Автор:
                   6064: Иван Морозов (Коломна)
                   6065: 
                   6066: Вопрос 19:
                   6067: Блинов и Жарков - участники ЧГК-движения, которые должны быть знакомы
                   6068: всем присутствующим. Блинов прославился благодаря Саше Барону Коэну в
1.10      rubashki 6069: качестве человека с диктаторскими замашками, не привыкшего ограничивать
1.1       rubashki 6070: свои желания. Жарков был известен своим стремлением докапываться до
                   6071: истины, даже если это представляло опасность для его жизни. Автор
                   6072: вопроса знаком с Блиновым лично, а с Жарковым заочно и, пользуясь
                   6073: случаем, передает им привет. Какие фамилии в вопросе заменены на
                   6074: "Блинов" и "Жарков"?
                   6075: 
                   6076: Ответ:
                   6077: Аладин и Холодов.
                   6078: 
                   6079: Источник:
                   6080:    1. http://ru.wikipedia.org/wiki/Диктатор_(фильм)
                   6081:    2. http://ru.wikipedia.org/wiki/Холодов,_Дмитрий_Юрьевич
                   6082: 
                   6083: Автор:
                   6084: Александр Берелехис (Уфа)
                   6085: 
                   6086: Вопрос 20:
                   6087: (pic: 20150160.jpg)
                   6088:    Перед вами ИКСЫ. ИКС является символом ОМЕГИ. "ОМЕГА" - название
                   6089: русского романа второй половины XIX века. Назовите подмосковный город,
                   6090: название которого происходит от ОМЕГИ и на гербе которого изображен ИКС.
                   6091: 
                   6092: Ответ:
                   6093: Воскресенск.
                   6094: 
                   6095: Комментарий:
                   6096: ИКС - Феникс: слева - актер Хоакин Феникс, а справа - созвездие Феникс;
                   6097: ОМЕГА - возрождение, воскрешение; у Льва Толстого есть роман
                   6098: "Воскресение".
                   6099: 
                   6100: Источник:
                   6101:    1. http://www.filmz.ru/photos/181107/
                   6102:    2. http://ru.wikipedia.org/wiki/Феникс_(созвездие)
                   6103:    3. http://www.symbolarium.ru/index.php/Феникс,_птица
                   6104:    4. http://ru.wikipedia.org/wiki/Воскресение_(роман)
                   6105:    5. http://ru.wikipedia.org/wiki/Герб_Воскресенска
                   6106: 
                   6107: Автор:
                   6108: Кирилл Третьяков (Коломна)
                   6109: 
                   6110: Вопрос 21:
                   6111: Коса, зулу, сесото, тсвана, свази, венда, тсонга - это официальные ОНИ.
                   6112: Назовите ИХ двумя словами, начинающимися на соседние буквы.
                   6113: 
                   6114: Ответ:
                   6115: Языки ЮАР.
                   6116: 
                   6117: Источник:
                   6118: http://ru.wikipedia.org/wiki/Языки_Южно-Африканской_Республики
                   6119: 
                   6120: Автор:
                   6121: Кирилл Третьяков (Коломна)
                   6122: 
                   6123: Вопрос 22:
                   6124: По правилам футбола гол не засчитывается, если мяч был направлен в
                   6125: ворота первым касанием при пробитии штрафного удара, свободного
                   6126: штрафного удара, удара от ворот, углового удара или непосредственно
                   6127: вбросом аута. Согласно футбольной шутке прошлого десятилетия, защитник
                   6128: сборной России по футболу Юрий Ковтун установил новый рекорд, забив уже
                   6129: в первом тайме два гола. Достижение прославленного защитника могло быть
                   6130: еще более выдающимся, если бы сразу после второго гола он не был
                   6131: травмирован Александром Филимоновым. Какие четыре буквы мы неоднократно
                   6132: пропустили в этом вопросе?
                   6133: 
                   6134: Ответ:
                   6135: авто.
                   6136: 
                   6137: Комментарий:
                   6138: Все слова "автогол" в этом вопросе были заменены просто на "гол".
                   6139: Согласно шутке, любитель забивать в свои ворота Юрий Ковтун после
                   6140: второго автогола был благоразумно травмирован своим же вратарем.
                   6141: 
                   6142: Источник:
                   6143: http://lurkmore.to/Российский_футбол
                   6144: 
                   6145: Автор:
                   6146: Александр Романкин (Коломна)
                   6147: 
                   6148: Вопрос 23:
                   6149: Статья в "Нью-Йорк Таймс" от 26 марта 2014 года носит название "Crime
                   6150: and Punishment". Какую букву мы пропустили в этом вопросе?
                   6151: 
                   6152: Ответ:
                   6153: A.
                   6154: 
                   6155: Комментарий:
                   6156: "Crimea and Punishment".
                   6157: 
                   6158: Источник:
                   6159: http://www.nytimes.com/2014/03/27/opinion/blow-crimea-and-punishment.html
                   6160: 
                   6161: Автор:
                   6162: Кирилл Третьяков (Коломна)
                   6163: 
                   6164: Вопрос 24:
                   6165: В эфиопской письменности названия многих букв сходны с другими
                   6166: семитскими системами письма, но есть и оригинальные. Например,
                   6167: двенадцатую букву, название которой в других семитских алфавитах
                   6168: означает "рыба", эфиопы назвали в честь другого животного, так, будто
                   6169: слишком буквально поняли известное евангельское изречение. А один
                   6170: афоризм гласит, что если человек очень легко меняет свои убеждения, то
                   6171: доверять ему стоит так же, как этому животному. Что это за животное?
                   6172: 
                   6173: Ответ:
                   6174: Змея.
                   6175: 
                   6176: Комментарий:
                   6177: Христос сказал: "И когда попросит рыбы, подал бы ему змею?"
                   6178: (продолжением фразы о том, что любящий отец не дал бы голодному сыну
                   6179: вместо просимого хлеба камень). Это единственное изречение в Евангелии,
                   6180: в котором упоминается замена рыбы на что-либо иное. Согласно афоризму,
                   6181: если человек меняет свои убеждения так же легко, как змея - кожу, то
                   6182: доверять ему стоит тоже как змее.
                   6183: 
                   6184: Источник:
                   6185: Евангелие от Матфея, 7:10.
                   6186: 
                   6187: Автор:
                   6188: Иван Морозов (Коломна)
                   6189: 
                   6190: Тур:
                   6191: 15 тур. "Немчиновка" (Москва)
                   6192: 
                   6193: Вопрос 1:
                   6194: Изобретатель матрешки завещал похоронить себя [пропуск]. Восстановите
                   6195: шесть пропущенных слов.
                   6196: 
                   6197: Ответ:
                   6198: "... в гробу в гробу в гробу".
                   6199: 
                   6200: Источник:
                   6201: http://ibigdan.livejournal.com/9225875.html
                   6202: 
                   6203: Автор:
                   6204: Игорь Доскоч
                   6205: 
                   6206: Вопрос 2:
                   6207: (pic: 20150161.jpg)
                   6208:    На картинке из сообщества политических карикатур приведены "сведения"
                   6209: по экстремизму - вероятно, склонности вести священные войны - среди
                   6210: последователей разных религий. Какой символ мы закрыли на последнем
                   6211: столбце?
                   6212: 
                   6213: Ответ:
                   6214: Надкушенное яблоко.
                   6215: 
                   6216: Зачет:
                   6217: Эмблема "Apple".
                   6218: 
                   6219: Комментарий:
                   6220: Одна из наиболее популярных тем холиваров (священных войн) в Интернете -
                   6221: это войны между макинтошниками и виндузятниками.
                   6222: 
                   6223: Источник:
                   6224: http://politicartoons.livejournal.com/3287606.html
                   6225: 
                   6226: Автор:
                   6227: Игорь Доскоч
                   6228: 
                   6229: Вопрос 3:
                   6230: Это слово встречается в Базе Вопросов более 1000 раз. Редактор Microsoft
                   6231: Word этого слова не знает и предлагает заменить его на повинность.
                   6232: Воспроизведите это слово.
                   6233: 
                   6234: Ответ:
                   6235: Борок.
                   6236: 
                   6237: Комментарий:
                   6238: Дмитрий Борок - автор более чем 1000 вопросов ЧГК, лежащих в Базе. Word
                   6239: предлагает заменить слово "Борок" на "оброк".
                   6240: 
                   6241: Источник:
                   6242:    1. http://db.chgk.info/
                   6243:    2. Microsoft Word.
                   6244: 
                   6245: Автор:
                   6246: Мишель Матвеев
                   6247: 
                   6248: Вопрос 4:
                   6249: В прошлом году автор вопроса занимался закупкой компьютерных мышей.
                   6250: После получения заказа выяснилось, что ушлые или невнимательные продавцы
                   6251: пропустили в описании шесть букв. Каких?
                   6252: 
                   6253: Ответ:
                   6254: ОМЕХАН.
                   6255: 
                   6256: Комментарий:
                   6257: Вместо ОПТИЧЕСКИХ продали ОПТомеханИЧЕСКИХ, т.е. шариковых.
                   6258: 
                   6259: Автор:
                   6260: Игорь Доскоч
                   6261: 
                   6262: Вопрос 5:
                   6263:    <раздатка>
                   6264:    Пiнгвiни Живуть Зимою
                   6265:    </раздатка>
                   6266:    В раздатке мы показали лишь чуть больше 40% от полного текста. Вас же
                   6267: мы попросим назвать часть тела, имя и источник из аналогичного текста на
                   6268: русском языке.
                   6269: 
                   6270: Ответ:
1.10      rubashki 6271: Голова, Жак, фонарь.
1.1       rubashki 6272: 
                   6273: Комментарий:
                   6274: Это украинская мнемоника для запоминания цветов спектра: "Чому Пiнгвiни
                   6275: Живуть Зимою Без Сво&#1111;х Фантазiй". Русский аналог - "Как Однажды
                   6276: Жак-Звонарь Головою Сбил Фонарь".
                   6277: 
                   6278: Источник:
                   6279: http://www.zapominalki.ru/interest-ukrainian-zapominalki/
                   6280: 
                   6281: Автор:
                   6282: Евгений Майсюк
                   6283: 
                   6284: Вопрос 6:
                   6285: Почтовый сервис Google, который располагается в домене gmail.com,
                   6286: изначально был так назван по первой букве имени весьма известного
                   6287: персонажа, но, вероятно, содержать этот адрес изначальным владельцам
                   6288: стало лень, и они продали адрес Google. Что это был за персонаж?
                   6289: 
                   6290: Ответ:
                   6291: [Кот] Гарфилд.
                   6292: 
                   6293: Комментарий:
                   6294: Кот Гарфилд известен своей ленью.
                   6295: 
                   6296: Источник:
                   6297: http://www.factroom.ru/facts/26623
                   6298: 
                   6299: Автор:
                   6300: Надежда Герасименко
                   6301: 
                   6302: Вопрос 7:
                   6303: Внимание, в вопросе есть замена.
                   6304:    Прослушайте цитату из Брокгауза и Ефрона, "Каштан": "Он образует
                   6305: обширные насаждения в средней и южной Франции, Испании и Италии и
                   6306: разводится у нас в Крыму и Туркестане, но здорово произрастает и в
                   6307: Закавказье - нижней и средней горных полосах, изредка поднимаясь до 5-6
                   6308: тысяч футов высоты". Какое слово мы заменили?
                   6309: 
                   6310: Ответ:
                   6311: Дико.
                   6312: 
                   6313: Комментарий:
                   6314: Вместо "здорово". Во времена Брокгауза и Ефрона такое употребление слова
                   6315: "здорово" выглядело несколько дико.
                   6316: 
                   6317: Источник:
                   6318: http://dic.academic.ru/dic.nsf/brokgauz_efron/50717/
                   6319: 
                   6320: Автор:
                   6321: Игорь Доскоч
                   6322: 
                   6323: Вопрос 8:
                   6324: На специальной наклейке в упаковке с тирольским пирогом Черри-Бренди
                   6325: напечатано предостережение об опасности, которой могут подвергнуться и
                   6326: взрослые, и дети. Какой?
                   6327: 
                   6328: Ответ:
                   6329: [Попадания] косточки.
                   6330: 
                   6331: Комментарий:
                   6332: Черри - вишня, в пироге случаются и косточки, чего нет в пирогах с
                   6333: другими составами.
                   6334: 
                   6335: Источник:
                   6336: (pic: 20150162.jpg)
                   6337: 
                   6338: Автор:
                   6339: Игорь Доскоч
                   6340: 
                   6341: Вопрос 9:
                   6342: (pic: 20150163.jpg)
                   6343:    На раздатке мы закрыли нечто, к чему пользователи Интернета
                   6344: пририсовали видимую надпись. Назовите двумя словами то, что мы закрыли.
                   6345: 
                   6346: Ответ:
                   6347: Пирамида Маслоу.
                   6348: 
                   6349: Источник:
                   6350: http://weknowmemes.com/2013/08/maslows-hierarchy-of-needs-updated/
                   6351: 
                   6352: Автор:
                   6353: Игорь Доскоч
                   6354: 
                   6355: Вопрос 10:
                   6356: Внезапный взрыв популярности этого предмета в Америке в 1970-х связан с
                   6357: тем, что много болевший к тому времени Роберт Хайнлайн уделил ему немало
                   6358: внимания в своих романах, включая культовый "Чужак в чужой стране".
                   6359: Назовите этот предмет.
                   6360: 
                   6361: Ответ:
                   6362: Водяной матрас.
                   6363: 
                   6364: Зачет:
                   6365: Водяная кровать.
                   6366: 
                   6367: Комментарий:
                   6368: Хайнлайну пришлось провести много времени в постели, и он занимался
                   6369: обдумыванием оптимальной конструкции такой кровати.
                   6370: 
                   6371: Источник:
                   6372: http://en.wikipedia.org/wiki/Waterbed
                   6373: 
                   6374: Автор:
                   6375: Надежда Герасименко
                   6376: 
                   6377: Вопрос 11:
                   6378: (pic: 20150164.jpg)
                   6379:    Восстановите второе слово, входящее в название изображенного жука.
                   6380: 
                   6381: Ответ:
                   6382: Schwarzeneggeri.
                   6383: 
                   6384: Зачет:
                   6385: С незначительными ошибками.
                   6386: 
                   6387: Комментарий:
                   6388: Передние лапки насекомого напоминают накачанные бицепсы.
                   6389: 
                   6390: Источник:
                   6391: http://ru.wikipedia.org/wiki/Agra_schwarzeneggeri
                   6392: 
                   6393: Автор:
                   6394: Виталий Федоров
                   6395: 
                   6396: Вопрос 12:
                   6397: Отрывок из мемуаров немецкого танкиста Отто Кариуса: "Повсюду активно
                   6398: работали саперы. Они даже СДЕЛАЛИ ЭТО! Такая уловка иногда удавалась
                   6399: позднее на Западном фронте в отношении американцев, но никак не
                   6400: проходила с русскими". Конец цитаты. Маленький гном Вася тоже СДЕЛАЛ
                   6401: ЭТО, и ему уловка удалась. Что мы заменили на "СДЕЛАТЬ ЭТО"?
                   6402: 
                   6403: Ответ:
                   6404: Повернуть в противоположную сторону предупредительные знаки.
                   6405: 
                   6406: Зачет:
                   6407: Повернуть указатели.
                   6408: 
                   6409: Источник:
                   6410:    1. http://militera.lib.ru/memo/german/carius_o/05.html
                   6411:    2. М. Липскеров. Маленький гном Вася.
                   6412: 
                   6413: Автор:
                   6414: Игорь Доскоч
                   6415: 
                   6416: Вопрос 13:
                   6417: Знакомый автора вопроса ошибочно полагал, что АЛЬФА - это отделение
                   6418: компании "БЕТА мотор корпорейшн". Назовите АЛЬФУ и БЕТУ.
                   6419: 
                   6420: Ответ:
                   6421: Акура, Мазда.
                   6422: 
                   6423: Автор:
                   6424: Анна Рогозина
                   6425: 
                   6426: Вопрос 14:
                   6427: Прослушайте шуточный стишок-клерихью:
                   6428:    Мы в России считаем, что Тютчев
                   6429:    Ихних Вордсвортов в тридцать раз круче.
                   6430:    Но по мненью (пропуск один),
                   6431:    Эдвард Лир всё же лучше (пропуск два).
                   6432:    Заполните любой из пропусков.
                   6433: 
                   6434: Ответ:
                   6435: "... британских ученых...", "... Кручёных".
                   6436: 
                   6437: Источник:
                   6438: http://dohlik-nemruchi.livejournal.com/69264.html
                   6439: 
                   6440: Автор:
                   6441: Игорь Биткин
                   6442: 
                   6443: Вопрос 15:
                   6444: В романе Лукьяненко маг применяет заклинание "абсолютного ИКСА", чтобы
                   6445: остановить людей, штурмующих помещение. Далее уточняется, что заклинаний
                   6446: "абсолютного ИКСА" два, но второе применяется только к одушевленным
                   6447: объектам. Какое слово мы заменили на ИКС?
                   6448: 
                   6449: Ответ:
                   6450: Тормоз.
                   6451: 
                   6452: Автор:
                   6453: Виталий Федоров
                   6454: 
                   6455: Вопрос 16:
                   6456: В 1931 году в Мюнхене начался суд над неким Францем Таузендом, сумевшим
                   6457: ввести в заблуждение многих влиятельных людей Германии - видимо,
                   6458: благодаря отчаянному положению Веймарской республики. Последний подобный
                   6459: процесс в Баварии закончился в начале XVII века, однако был назначен
                   6460: следственный эксперимент, в ходе которого Таузенд должен был... Сделать
                   6461: что?
                   6462: 
                   6463: Ответ:
                   6464: Получить золото.
                   6465: 
                   6466: Источник:
                   6467: http://www.webznayka.ru/interes-fakty/prestupleniya/77-afera-sintez-zolota-frants-tauzend
                   6468: 
                   6469: Автор:
                   6470: Виталий Федоров
                   6471: 
                   6472: Вопрос 17:
                   6473: Как ни странно, ОНА - часто не ОНА и может состоять из белков, жиров,
                   6474: свободных жирных кислот, минеральных солей, а может и содержать аж целых
                   6475: 4,2% ЕЕ. Назовите ЕЕ.
                   6476: 
                   6477: Ответ:
                   6478: Сера.
                   6479: 
                   6480: Комментарий:
                   6481: Приведен состав ушной серы.
                   6482: 
                   6483: Автор:
                   6484: Игорь Доскоч
                   6485: 
                   6486: Вопрос 18:
                   6487: В 1938 году сотрудники лаборатории Кюри работали над новым радиоактивным
                   6488: элементом. Его поспешно объявляли то изотопом актиния, то лантана, а то
                   6489: и вовсе трансураном. Коллеги в насмешку предлагали назвать элемент
                   6490: Curium. Какую систему мы пропустили?
                   6491: 
                   6492: Ответ:
                   6493: IOS.
                   6494: 
                   6495: Комментарий:
                   6496: CuriOSium, iOS - операционная система iPhone, iPad.
                   6497: 
                   6498: Автор:
                   6499: Виталий Федоров
                   6500: 
                   6501: Вопрос 19:
                   6502: Увидев сильно подержанный внедорожник Сузуки, автор вопроса подумал, что
                   6503: его название слишком мало, и ЭТО надо повторить. Что именно?
                   6504: 
                   6505: Ответ:
                   6506: Гранд.
                   6507: 
                   6508: Комментарий:
                   6509: Внедорожник назывался "Grand-Vitara". Grand-grandmother = прабабушка.
1.15      rubashki 6510:    z-checkdb: На самом деле прабабушка по-английски - great-grandmother
                   6511: (Андрей Кокуленко).
1.1       rubashki 6512: 
                   6513: Автор:
                   6514: Игорь Доскоч
                   6515: 
                   6516: Вопрос 20:
                   6517: Средневековый богослов Ефрем Сирин вычислил точную дату сотворения мира.
                   6518: На момент появления Адама на деревьях были плоды, следовательно, стоял
                   6519: сентябрь. Далее Сирин определяет и число, исходя из того, что Бог никак
                   6520: не мог сотворить ничего ТАКОГО. Какого?
                   6521: 
                   6522: Ответ:
                   6523: Ущербного.
                   6524: 
                   6525: Комментарий:
                   6526: Бог не мог создать ущербную луну, следовательно, на четвертый день
                   6527: творения она была полной. Остальные вычисления вполне по силам
                   6528: математике девятого века. В итоге вышло 1 сентября.
                   6529: 
                   6530: Автор:
                   6531: Виталий Федоров
                   6532: 
                   6533: Вопрос 21:
                   6534: Закончите стихотворение-порошок:
                   6535:    душа так жаждет обновленья
                   6536:    мозг тщится новое узнать
                   6537:    и палец тянется к заветной
                   6538:    ...
                   6539: 
                   6540: Ответ:
                   6541: эф пять
                   6542: 
                   6543: Зачет:
                   6544: F5
                   6545: 
                   6546: Источник:
                   6547: https://vk.com/sandalporoshki?w=wall-31481258_36173
                   6548: 
                   6549: Автор:
                   6550: Игорь Доскоч
                   6551: 
                   6552: Вопрос 22:
                   6553: На Байконуре есть традиция: за полтора километра до пусковой площадки
                   6554: космонавт выходит из автобуса и стоит несколько минут у заднего колеса.
                   6555: Назовите космонавта, который привел первоначальную традицию, которая
                   6556: идет со времен Гагарина, в существующий формат.
                   6557: 
                   6558: Ответ:
                   6559: Терешкова.
                   6560: 
                   6561: Комментарий:
                   6562: Ранее суеверные космонавты-мужчины повторяли поступок Гагарина и орошали
                   6563: колесо. А потом полетела Терешкова.
                   6564: 
                   6565: Автор:
                   6566: Александр Кузьмич
                   6567: 
                   6568: Вопрос 23:
                   6569: По мнению комментатора Александра Шмурнова, добивающиеся ЭТОГО подобны
                   6570: женщинам, мечтающим не иметь детей. Назовите ЭТО максимально точно.
                   6571: 
                   6572: Ответ:
                   6573: 0:0.
                   6574: 
                   6575: Зачет:
                   6576: Нулевая ничья и т.п.
                   6577: 
                   6578: Автор:
                   6579: Виталий Федоров
                   6580: 
                   6581: Вопрос 24:
                   6582: Из подписи одного интернет-пользователя: "У меня три АЛЬФЫ, поэтому я
                   6583: ничего не должен". Назовите любой из его долгов.
                   6584: 
                   6585: Ответ:
                   6586: Родить сына / посадить дерево / построить дом.
                   6587: 
                   6588: Комментарий:
                   6589: У пользователя три дочери.
                   6590: 
                   6591: Автор:
                   6592: Александр Фаюстов
                   6593: 
                   6594: Тур:
                   6595: 16 тур. "Ксеп" (Москва)
                   6596: 
                   6597: Редактор:
                   6598: Ярослав Котышов
                   6599: 
                   6600: Вопрос 1:
                   6601: Разбогатевший и ставший светским человеком рэпер говорит, что теперь у
                   6602: него в шкафу висят ОНИ. На одной картинке среди НИХ есть, например,
                   6603: 454545, E8E8E8 и 2F2F2F. Назовите ИХ тремя словами.
                   6604: 
                   6605: Ответ:
                   6606: Пятьдесят оттенков серого.
                   6607: 
                   6608: Комментарий:
                   6609: Шон Андерсон, который, по его словам, прежде по 50 раз в день слушал 50
                   6610: Cent, очевидно, намекает на появившиеся у него костюмы. На картинке
                   6611: представлены 50 оттенков серого, записанные шестнадцатеричным кодом.
                   6612: 
                   6613: Источник:
                   6614:    1. http://genius.com/4349269
                   6615:    2. https://twitter.com/tproger/status/560406953332330496/
                   6616: 
                   6617: Автор:
                   6618: Сергей Даровских, Ярослав Котышов
                   6619: 
                   6620: Вопрос 2:
                   6621: Дочь Василия и Доротеи Васильевых родилась на корабле, шедшем из
                   6622: Мурманска во Владивосток. Поэтому новорождённой дали имя Марина - в
                   6623: честь моря. В тексте вопроса мы немного изменили одно из слов. Напишите
                   6624: это слово в исходном виде.
                   6625: 
                   6626: Ответ:
                   6627: Карина.
                   6628: 
                   6629: Комментарий:
                   6630: Девочка родилась в Карском море, в честь которого и получила имя.
                   6631: 
                   6632: Источник:
                   6633: http://www.trud.ru/article/10-04-2013/1292062_poslednjaja_iz_cheljuskintsev_karina_vasiljeva_otmechaet_79-j_den_spasenija.html
                   6634: 
                   6635: Автор:
                   6636: Евгений Пашковский
                   6637: 
                   6638: Вопрос 3:
                   6639: Святой Иероним, наставляя будущую монахиню Евстохию, советует ей бежать
                   6640: от искушений внешнего мира и предостерегает ее от судьбы другой женщины.
                   6641: Кто эта женщина?
                   6642: 
                   6643: Ответ:
                   6644: Жена Лота.
                   6645: 
                   6646: Комментарий:
                   6647: Мир Иероним сравнивает с Содомом, из которого Евстохия должна бежать без
                   6648: оглядки.
                   6649: 
                   6650: Источник:
                   6651: http://www.ccel.org/ccel/schaff/npnf206.v.XXII.html
                   6652: 
                   6653: Автор:
                   6654: Юрий Разумов
                   6655: 
                   6656: Вопрос 4:
                   6657: Назовите самый древний из побратимов бразильского города Натал.
                   6658: 
                   6659: Ответ:
                   6660: Вифлеем.
                   6661: 
                   6662: Комментарий:
                   6663: Название бразильского города этимологически связано с Рождеством.
                   6664: 
                   6665: Источник:
                   6666: http://ru.wikipedia.org/wiki/Натал_(город)
                   6667: 
                   6668: Автор:
                   6669: Евгений Пашковский
                   6670: 
                   6671: Вопрос 5:
                   6672: После перехода футболиста Стивена Дефура в "Андерлехт" болельщики
                   6673: "пролетарского" клуба "Стандард" вывесили в его адрес угрожающий баннер
                   6674: с надписью на английском языке. Эта надпись совпадает с названием марки
                   6675: одежды, которую основал сын индейского вождя. Надпись содержит два
                   6676: прилагательных. Напишите их.
                   6677: 
                   6678: Ответ:
                   6679: Red, dead.
                   6680: 
                   6681: Зачет:
                   6682: Красный, мертвый.
                   6683: 
                   6684: Комментарий:
                   6685: На баннере было написано "Red or dead". Надпись отсылает к выражению
                   6686: "Better red than dead" времен Холодной войны.
                   6687: 
                   6688: Источник:
                   6689: http://www.bbc.com/sport/0/football/30976806
                   6690: 
                   6691: Автор:
                   6692: Сергей Даровских
                   6693: 
                   6694: Вопрос 6:
                   6695: Герой известной пьесы говорит, что из шести обезьян примерно половина
                   6696: должна СДЕЛАТЬ ЭТО головами, а половина - хвостами. Что именно сделать?
                   6697: 
                   6698: Ответ:
                   6699: Приземлиться.
                   6700: 
                   6701: Зачет:
                   6702: Упасть и другие синонимы.
                   6703: 
                   6704: Комментарий:
                   6705: Розенкранц и Гильденстерн в пьесе Тома Стоппарда играют в орлянку,
                   6706: которая по-английски называется "heads or tails". Гильденстерн
                   6707: рассуждает о теории вероятностей и говорит, что шесть обезьян, будучи
                   6708: подброшенными в воздух, поровну приземлятся на хвосты и на головы. Этим
                   6709: диалогом Стоппард, видимо, отсылает читателя к "Теореме о бесконечных
                   6710: обезьянах", согласно которой абстрактная обезьяна рано или поздно
                   6711: напечатает любой текст, например, "Гамлета" Шекспира.
                   6712: 
                   6713: Источник:
                   6714: http://lib.ru/PXESY/STOPPARD/r_g_engl.txt
                   6715: 
                   6716: Автор:
                   6717: Александр Марков
                   6718: 
                   6719: Вопрос 7:
                   6720: Внимание, в вопросе есть замены.
                   6721:    Пользователь сайта booking.com иронизирует, что в мотеле в Пущино
                   6722: сложно находиться из-за ПЕРВОГО, зато там есть ВТОРАЯ. Что мы заменили
                   6723: на ВТОРУЮ?
                   6724: 
                   6725: Ответ:
                   6726: Дискотека.
                   6727: 
                   6728: Комментарий:
                   6729: ПЕРВЫЙ - шум.
                   6730: 
                   6731: Источник:
                   6732: http://www.booking.com/hotel/ru/f1-motel.ru.html#tab-reviews
                   6733: 
                   6734: Автор:
                   6735: Ярослав Котышов
                   6736: 
                   6737: Вопрос 8:
                   6738: Какого поэта родители до пяти лет одевали как девочку и называли Софией?
                   6739: 
                   6740: Ответ:
                   6741: [Райнер Мария] Рильке.
                   6742: 
                   6743: Комментарий:
                   6744: Хотя среди его имен было другое женское имя. Мать Рильке очень
                   6745: переживала из-за смерти дочери в недельном возрасте.
                   6746: 
                   6747: Источник:
                   6748: http://en.wikipedia.org/wiki/Rainer_Maria_Rilke
                   6749: 
                   6750: Автор:
                   6751: Юрий Разумов
                   6752: 
                   6753: Вопрос 9:
                   6754: Внимание, в вопросе есть замена.
                   6755:    Писатель Иехиэль Динур после Второй мировой войны включил в свой
                   6756: псевдоним ИКС. Другой ИКС был в первоначальном варианте названия
                   6757: рассказа. Кто автор этого рассказа?
                   6758: 
                   6759: Ответ:
                   6760: [Александр] Солженицын.
                   6761: 
                   6762: Комментарий:
                   6763: ИКС - лагерный номер. Иехиэль Динур два года был узником Аушвица, а
                   6764: потом стал писать под псевдонимом Кацетник 135633, где 135633 - его
                   6765: лагерный номер. Солженицын переименовал рассказ "Щ-854" о заключенном
                   6766: Иване Шухове в "Один день Ивана Денисовича".
                   6767: 
                   6768: Источник:
                   6769:    1. http://en.wikipedia.org/wiki/Yehiel_De-Nur
                   6770:    2. http://festival.1september.ru/articles/531138/
                   6771: 
                   6772: Автор:
                   6773: Александр Марков
                   6774: 
                   6775: Вопрос 10:
                   6776: Герой современного произведения считает, что в Лос-Анджелесе сложно
                   6777: найти девушку, если у тебя всего одна строчка там. Ответьте, где именно,
                   6778: использовав аббревиатуру.
                   6779: 
                   6780: Ответ:
                   6781: [В профиле на] IMDB.
                   6782: 
                   6783: Комментарий:
                   6784: Девушки в Лос-Анджелесе, по мнению героя, избалованы голливудскими
                   6785: знаменитостями и не обращают внимания на начинающих актеров.
                   6786: 
                   6787: Источник:
                   6788: https://itunes.apple.com/us/album/the-women-of-la/id603620511?i=603620518
                   6789: 
                   6790: Автор:
                   6791: Александр Марков
                   6792: 
                   6793: Вопрос 11:
                   6794: Персонаж Ивлина Во, узнав о НЕЙ, упоминает украшение, сделавшее его
                   6795: одиноким царем лесов. Назовите героя, закричавшего о НЕЙ после взрыва.
                   6796: 
                   6797: Ответ:
                   6798: Мальчиш-Кибальчиш.
                   6799: 
                   6800: Комментарий:
                   6801: ОНА - измена. Персонаж Во говорит о появившихся у него рогах.
                   6802: 
                   6803: Источник:
                   6804:    1. И. Во. Возвращение в Брайдсхед.
                   6805: http://www.flibusta.net/b/337765/read
                   6806:    2. А.П. Гайдар. Сказка про военную тайну, Мальчиша-Кибальчиша и его
                   6807: твердое слово. http://www.flibusta.net/b/17629/read
                   6808: 
                   6809: Автор:
                   6810: Евгений Пашковский
                   6811: 
                   6812: Вопрос 12:
                   6813: Находящийся в Нью-Йорке магазин итальянских продуктов называется
                   6814: "Итали". Воспроизведите название этого магазина так, как оно написано на
                   6815: вывеске.
                   6816: 
                   6817: Ответ:
                   6818: Eataly.
                   6819: 
                   6820: Источник:
                   6821: http://www.eataly.com/nyc-locations-and-hours
                   6822: 
                   6823: Автор:
                   6824: Иделия Айзятулова
                   6825: 
                   6826: Вопрос 13:
                   6827: По одной из версий, смерть героини британского сериала была вызвана
                   6828: проблемами с Исламским государством Ирака и Леванта. В честь какой
                   6829: богини была названа эта героиня?
                   6830: 
                   6831: Ответ:
                   6832: Изида.
                   6833: 
                   6834: Комментарий:
                   6835: Речь идет о лабрадоре из сериала "Аббатство Даунтон". Левант по-арабски
                   6836: al-Sham, поэтому название террористической организации сокращают как
                   6837: ISIS. Кроме того, другое ее название - Исламское государство Ирака и
                   6838: Сирии.
                   6839: 
                   6840: Источник:
                   6841: http://www.dailymail.co.uk/news/article-2823184/From-mouth-Lord-Grantham-thinks-Downton-killing-Isis-labrador-Islamists-complete-berk.html
                   6842: 
                   6843: Автор:
                   6844: Дмитрий Карякин
                   6845: 
                   6846: Вопрос 14:
                   6847: Сторонник румынской антисемитской ультраправой православной организации
                   6848: Мирча Элиаде писал, что она "одерживает победу по воле Господней". Чье
                   6849: имя первоначально входило в название этой организации?
                   6850: 
                   6851: Ответ:
                   6852: Михаила [Архангела].
                   6853: 
                   6854: Комментарий:
                   6855: Румынская партия называлась "Легион Михаила Архангела", почти как
                   6856: русский "Союз имени Михаила Архангела".
                   6857: 
                   6858: Источник:
                   6859: http://traditio-ru.org/wiki/Легион_(движение)
                   6860: 
                   6861: Автор:
                   6862: Михаил Новосёлов
                   6863: 
                   6864: Вопрос 15:
                   6865: Говоря об исламском экстремизме, Фарит Салман, муфтий Ямало-Ненецкого
                   6866: округа, признаётся, что ваххабиты всё чаще приходят в мечети с НИМИ.
                   6867: Это, впрочем, неудивительно. Согласно одному утверждению, шесть ИХ часто
                   6868: соседствуют с двенадцатью коленами Израилевыми. Назовите ИХ.
                   6869: 
                   6870: Ответ:
                   6871: Обрезы.
                   6872: 
                   6873: Источник:
                   6874:    1. http://expert.ru/russian_reporter/2013/43/halifat-vseya-rusi/
                   6875:    2. Бородатый анекдот.
                   6876: 
                   6877: Автор:
                   6878: Дмитрий Карякин
                   6879: 
                   6880: Вопрос 16:
                   6881: Какой футбольный клуб, согласно шутке, назван в честь двух вещей,
                   6882: которые больше всего не любят исламские террористы?
                   6883: 
                   6884: Ответ:
                   6885: "Вест Хэм [Юнайтед]".
                   6886: 
                   6887: Комментарий:
                   6888: Ham - ветчина. Террористы, очевидно, не любят Запад, но и к свинине вряд
                   6889: ли относятся хорошо.
                   6890: 
                   6891: Источник:
                   6892: https://twitter.com/paulsinha/status/526005721809186816/
                   6893: 
                   6894: Автор:
                   6895: Юрий Разумов
                   6896: 
                   6897: Вопрос 17:
                   6898: В статье Нурсултана Назарбаева упоминается невидимая метла и огненный
                   6899: язык дракона Айдахара. Название какого города встречается в этой статье?
                   6900: 
                   6901: Ответ:
                   6902: Семипалатинск.
                   6903: 
                   6904: Зачет:
                   6905: Курчатов, Семипалатинск-21.
                   6906: 
                   6907: Комментарий:
                   6908: Назарбаев рассказывает о ядерных испытаниях в Казахстане.
                   6909: 
                   6910: Источник:
                   6911: Н. Назарбаев. Когда земля вскрикнула. // "Дилетант", 2015, N 1.
                   6912: 
                   6913: Автор:
                   6914: Дмитрий Карякин
                   6915: 
                   6916: Вопрос 18:
                   6917: Физик Чарльз Чой отмечает, что ИКСЫ так же сильны и опасны, как Халк.
                   6918: Героиня пьесы Пола Зиндела выясняет, что ИКСЫ могут сделать цветы
                   6919: красивее, а могут и погубить их. Какое слово, пишущееся через дефис, мы
                   6920: заменили на ИКСЫ?
                   6921: 
                   6922: Ответ:
                   6923: Гамма-лучи.
                   6924: 
                   6925: Комментарий:
                   6926: Физик считает, что предполагаемая концентрация гамма-лучей убила бы
                   6927: Брюса Бэннера, а не превратила в Халка. Героиня Пола Зиндела изучала
                   6928: влияние гамма-излучения на маргаритки.
                   6929: 
                   6930: Источник:
                   6931:    1. http://www.livescience.com/2590-gamma-rays-incredible-hulking-reality.html
                   6932:    2. http://en.wikipedia.org/wiki/The_Effect_of_Gamma_Rays_on_Man-in-the-Moon_Marigolds
                   6933: 
                   6934: Автор:
                   6935: Юрий Разумов
                   6936: 
                   6937: Вопрос 19:
                   6938: Знакомая Ирины Насановской, находясь с подругой во Львове, решила, что к
                   6939: русским там плохо относятся. Причиной этого стал ответ местного жителя
                   6940: на ее вопрос. Ответьте на этот вопрос.
                   6941: 
                   6942: Ответ:
                   6943: Текущее время словами или цифрами (22:40 на момент задания вопроса).
                   6944: 
                   6945: Комментарий:
                   6946: На вопрос "Который час?" мужчина ответил: "Две години", т.е. "Два часа".
                   6947: Девушки подумали, что он назвал их "гадинами".
                   6948: 
                   6949: Источник:
                   6950: http://www.anekdot.ru/id/733008/
                   6951: 
                   6952: Автор:
                   6953: Ярослав Котышов
                   6954: 
                   6955: Вопрос 20:
                   6956: В Бельгии автор вопроса купил себе чехол для телефона. Мы не спрашиваем,
                   6957: что на нем изображено. Что на нем написано?
                   6958: 
                   6959: Ответ:
                   6960: "Ceci n'est pas une pipe".
                   6961: 
                   6962: Зачет:
                   6963: "Это не трубка".
                   6964: 
                   6965: Комментарий:
                   6966: Изображение на чехле - знаменитая картина Магритта.
                   6967: 
                   6968: Источник:
                   6969: Чехол для телефона.
                   6970: 
                   6971: Автор:
                   6972: Сергей Даровских
                   6973: 
                   6974: Вопрос 21:
                   6975: Комментируя статистику выживаемости, Ада Суонсон сказала: "Если не
                   6976: можешь быть ИКСОМ, будь маленьким парнем, прячущимся в углу". Какое
                   6977: слово мы заменили на ИКС?
                   6978: 
                   6979: Ответ:
                   6980: Король.
                   6981: 
                   6982: Комментарий:
                   6983: Речь идет о выживаемости шахматных фигур к концу партии. Короли выживают
1.10      rubashki 6984: в 100% случаев, потому что их нельзя побить. На втором месте - белая
1.1       rubashki 6985: пешка с поля h2, которая выживает в 73,9% партий.
                   6986: 
                   6987: Источник:
                   6988: http://knowmore.washingtonpost.com/2014/10/24/what-are-the-chances-of-survival-for-each-chess-piece/
                   6989: 
                   6990: Автор:
                   6991: Ярослав Котышов
                   6992: 
                   6993: Вопрос 22:
                   6994: Героиня романа Кацетника "Кукольный дом" в концлагере попадает в отряд
                   6995: проституток, которые должны обслуживать немецких солдат. Назовите
                   6996: англичанина, который прочитал этот роман за несколько лет до
                   6997: самоубийства.
                   6998: 
                   6999: Ответ:
                   7000: Иен Кёртис.
                   7001: 
                   7002: Комментарий:
                   7003: Такой отряд в книге Кацетника называется "Joy Division" [джой дивИжн].
                   7004: Это название в 1978 году позаимствовал для своей группы Иен Кёртис. Два
                   7005: года спустя музыкант повесился.
                   7006: 
                   7007: Источник:
                   7008:    1. http://curiousjew.blogspot.ru/2009/04/house-of-dolls.html
                   7009:    2. http://www.theguardian.com/books/2008/may/10/popandrock.joydivision
                   7010: 
                   7011: Автор:
                   7012: Александр Марков
                   7013: 
                   7014: Вопрос 23:
                   7015: После появления на вулканическом острове Санторини ИКСА погонщики ослов
                   7016: стали получать компенсацию за каждый проданный билет. Какое слово мы
                   7017: заменили на ИКС?
                   7018: 
                   7019: Ответ:
                   7020: Фуникулер.
                   7021: 
                   7022: Комментарий:
                   7023: Крупнейшие поселения острова Санторини расположены на обрывистой части
1.19      rubashki 7024: острова. Подъем или спуск там был возможен только с помощью ослов. С
1.1       rubashki 7025: появлением фуникулера работа у погонщиков ослов практически пропала.
                   7026: 
                   7027: Источник:
                   7028: http://www.santonet.com/holidays/cablecar.html
                   7029: 
                   7030: Автор:
                   7031: Юрий Разумов
                   7032: 
                   7033: Вопрос 24:
                   7034: Книга о том, насколько различны вампиры и оборотни, называется "Вампиры
                   7035: [ПЕРВЫЙ ПРОПУСК], оборотни [ВТОРОЙ ПРОПУСК]". Заполните любой из
                   7036: пропусков.
                   7037: 
                   7038: Ответ:
                   7039: С Марса.
                   7040: 
                   7041: Зачет:
                   7042: С Венеры.
                   7043: 
                   7044: Комментарий:
                   7045: "Вампиры с Венеры, оборотни с Марса". Более известная книга "Мужчины с
                   7046: Марса, женщины с Венеры" рассказывает о непреодолимых различиях мужчин и
                   7047: женщин.
                   7048: 
                   7049: Источник:
                   7050: http://www.amazon.com/Vampires-Venus-Werewolves-Mars-Comprehensive-ebook/dp/B00AC6Q2MW/
                   7051: 
                   7052: Автор:
                   7053: Юрий Разумов
                   7054: 
                   7055: Тур:
                   7056: 17 тур. "Разведёнка с прицепом" (Москва - Серпухов)
                   7057: 
                   7058: Редактор:
                   7059: Артем Колесов
                   7060: 
                   7061: Вопрос 1:
                   7062: Обсуждение пакета вопросов команда вела в Google Docs. В начале
                   7063: обсуждения каждый участник должен был кое-что выбрать, и первую же
                   7064: реплику на эту тему можно было посчитать признанием. Что интересно,
                   7065: драки, аналогичной описанной в культовом фильме 90-х, не возникло, так
                   7066: как никто не захотел быть... Кем?
                   7067: 
                   7068: Ответ:
                   7069: Черным.
                   7070: 
                   7071: Зачет:
                   7072: Мистером Черным, Mr. Black.
                   7073: 
                   7074: Комментарий:
                   7075: Для удобства обсуждения каждый член команды должен был выбрать, какого
                   7076: цвета будут его комментарии. Первый же комментарий "Я голубой" повлек
                   7077: шуточки на тему каминг-аута. Что интересно, черным не захотел писать
                   7078: никто. К примеру, одному из редакторов было пофиг, и он стал фиолетовым.
                   7079: В культовом фильме Квентина Тарантино "Бешеные псы" есть сцена, где
                   7080: криминальный босс раздает "цветные" клички бандитам. А когда недовольный
                   7081: своим цветом Мистер Розовый спросил, почему они сами не могут выбрать
                   7082: себе цвета, босс ответил: "Я уже попробовал однажды, ничего не вышло.
                   7083: Четверо парней подрались из-за того, кто будет Мистером Черным".
                   7084: 
                   7085: Источник:
                   7086:    1. Обсуждение пакета вопросов.
                   7087:    2. Фильм "Бешеные псы", реж. Квентин Тарантино.
                   7088: 
                   7089: Автор:
                   7090: Артем Колесов
                   7091: 
                   7092: Вопрос 2:
                   7093: Печально известный с 60-х ИКС ИГРЕК был двойным и по составу, и по
                   7094: действию. Назовите фильм, в котором Мистер ИГРЕК оказывается двойным
                   7095: ИКСОМ.
                   7096: 
                   7097: Ответ:
                   7098: "Бешеные псы".
                   7099: 
                   7100: Комментарий:
                   7101: В фильме "Бешеные псы" Мистер Оранж (он же Мистер Оранжевый) оказывается
                   7102: двойным агентом. Агент Оранж - это печально известная со времен войны во
                   7103: Вьетнаме смесь дефолиантов и гербицидов в пропорции 1:1. Кстати, помимо
                   7104: Агента Оранжа, использовались также Агент Блю, Агент Уайт и другие
                   7105: "Агенты". Прямо как в "Бешеных псах".
                   7106: 
                   7107: Источник:
                   7108:    1. http://ru.wikipedia.org/wiki/Агент_Оранж
                   7109:    2. Фильм "Бешеные псы", реж. Квентин Тарантино.
                   7110: 
                   7111: Автор:
                   7112: Дмитрий Сахаров, Артем Колесов
                   7113: 
                   7114: Вопрос 3:
                   7115: В одном из ранних вариантов второстепенные героини пытаются обмануть
                   7116: героя, но их выдают пятна крови, в результате чего выдают заглавную
                   7117: героиню. Назовите эту героиню.
                   7118: 
                   7119: Ответ:
                   7120: Золушка.
                   7121: 
                   7122: Зачет:
                   7123: Синдерелла.
                   7124: 
                   7125: Комментарий:
                   7126: Ее сестер выдают пятна крови на ступнях. А саму Золушку в итоге выдают
                   7127: замуж за принца.
                   7128: 
                   7129: Источник:
                   7130: Valerie Steele "Shoes and the Erotic Imagination".
                   7131: 
                   7132: Автор:
                   7133: Антон Пинчук
                   7134: 
                   7135: Вопрос 4:
                   7136: Рассказывают, что фильм "Интерстеллар" настолько не понравился
                   7137: известному физику, что тот прямо во время сеанса СДЕЛАЛ ЭТО. Какое
                   7138: выражение из трех слов мы заменили словами "СДЕЛАЛ ЭТО"?
                   7139: 
                   7140: Ответ:
                   7141: Встал и вышел.
                   7142: 
                   7143: Зачет:
                   7144: Встал и ушел.
                   7145: 
                   7146: Комментарий:
                   7147: Известный физик - конечно, Стивен Хокинг. Надеемся, вы досидите тур до
                   7148: конца.
                   7149: 
                   7150: Источник:
                   7151: http://drugoe-kino.livejournal.com/3047266.html?thread=65087586#t65087586
                   7152: 
                   7153: Автор:
                   7154: Артем Колесов
                   7155: 
                   7156: Вопрос 5:
                   7157: Как пишет Евгений Морозов, в 90-х всемирная паутина казалась романтичной
                   7158: и неизведанной, что отразилось в названии ИКСА. Назовите ИКС двумя
                   7159: словами.
                   7160: 
                   7161: Ответ:
                   7162: Internet Explorer.
                   7163: 
                   7164: Источник:
                   7165: https://www.readability.com/articles/nxri0fo8
                   7166: 
                   7167: Автор:
                   7168: Антон Пинчук
                   7169: 
                   7170: Вопрос 6:
                   7171: Рассказывая о возможностях дракона, Огастес Браун упоминает скрип двери
                   7172: и мяуканье кошки. Кого мы заменили драконом?
                   7173: 
                   7174: Ответ:
                   7175: Пересмешника.
                   7176: 
                   7177: Зачет:
                   7178: Пересмешник.
                   7179: 
                   7180: Комментарий:
                   7181: Пересмешник может имитировать не только голоса птиц, но и скрип двери, а
                   7182: также мяуканье кошки. "Убить дракона" и "Убить пересмешника" -
                   7183: знаменитые экранизации.
                   7184: 
                   7185: Источник:
                   7186: Огастес Браун. Почему панда стоит на голове?
                   7187: http://www.flibusta.net/b/288520/read
                   7188: 
                   7189: Автор:
                   7190: Дмитрий Сахаров
                   7191: 
                   7192: Вопрос 7:
                   7193: В песне "Полька-байка" группы "Green Crow" повествуется о том, как
                   7194: лирический герой, перебравшийся из Дублина в Бирмингем, спит с шестью
                   7195: мужчинами. Напишите шесть букв, которые мы пропустили в предыдущем
                   7196: предложении.
                   7197: 
                   7198: Ответ:
                   7199: гастар.
                   7200: 
                   7201: Комментарий:
                   7202: Песня называется "Полька-гастарбайка" и повествует о нелегкой судьбе
                   7203: трудового народа, и спят герои песни всемером на одной кровати вовсе не
                   7204: потому, почему вы могли подумать, а просто денег нет.
                   7205: 
                   7206: Источник:
                   7207: http://webkind.ru/text/595958627_99362393p656475520_text_pesni_polka-gastarbajka.html
                   7208: 
                   7209: Автор:
                   7210: Руслан Хаиткулов
                   7211: 
                   7212: Вопрос 8:
                   7213: По словам Софии Губайдулиной, утверждение Брамса о том, что скорее
                   7214: мужчина сумеет родить ребенка, чем женщина сможет создать хорошую
                   7215: музыку, сейчас неактуально. В подтверждение своих слов Губайдулина
                   7216: приводит историю альфы. Какое слово мы заменили в этом вопросе?
                   7217: 
                   7218: Ответ:
                   7219: Арфы.
                   7220: 
                   7221: Зачет:
                   7222: Арфа.
                   7223: 
                   7224: Комментарий:
                   7225: София Губайдулина - композитор, что, впрочем, знать совсем не
                   7226: обязательно. По ее словам, если, например, в Древней Греции на арфе
                   7227: играли только мужчины, то в наши дни можно встретить только арфисток.
                   7228: Соответственно, Губайдулина приводит в пример историю арфы.
                   7229: 
                   7230: Источник:
                   7231: http://www.viperson.ru/wind.php?ID=277118
                   7232: 
                   7233: Автор:
                   7234: Антон Волосатов
                   7235: 
                   7236: Вопрос 9:
                   7237: Жена героя романа Сафрана Фоера сбегает с городским помещиком.
                   7238: Прощальная записка жены сильно огорчает героя. В ходе дальнейших событий
                   7239: записка начинает напоминать предмет из романа первой половины XX века.
                   7240: Кому этот предмет принадлежал?
                   7241: 
                   7242: Ответ:
                   7243: Фриде.
                   7244: 
                   7245: Комментарий:
                   7246: Герой хочет спрятать записку или потерять ее, но она каждый раз без
                   7247: всякой мистики снова попадается ему на глаза, заставляя раз за разом
                   7248: переживать огорчения. Фриде, задушившей платком ребенка, каждый день
                   7249: подкладывают этот самый платок на тумбочку.
                   7250: 
                   7251: Источник:
                   7252:    1. Джонатан Сафран Фоер. Полная иллюминация.
                   7253:    2. http://ru.wikipedia.org/wiki/Мастер_и_Маргарита#.D0.A4.D1.80.D0.B8.D0.B4.D0.B0
                   7254: 
                   7255: Автор:
                   7256: Дмитрий Сахаров
                   7257: 
                   7258: Вопрос 10:
                   7259: Отмечая назидательный характер мексиканской живописи, Петр Вайль писал,
                   7260: что эпики были этиками. Кого мы заменили эпиками и этиками?
                   7261: 
                   7262: Ответ:
                   7263: Муралисты, моралисты.
                   7264: 
                   7265: Комментарий:
                   7266: Мурализм - движение в монументальной живописи Мексики второй четверти XX
                   7267: века. Монументализм - в каком-то роде синоним эпичности, а мораль -
                   7268: этики. :-) Кстати, одним из самых известных представителей мексиканского
                   7269: мурализма был Диего Ривера, муж Фриды Кало.
                   7270: 
                   7271: Источник:
                   7272: Петр Вайль, цикл телепередач "Гений места".
                   7273: 
                   7274: Автор:
                   7275: Руслан Хаиткулов
                   7276: 
                   7277: Вопрос 11:
                   7278: Герой одного фильма обращается к своему малознакомому собеседнику:
                   7279: "Сударь, до [ПРОПУСК] нам еще полверсты". Восстанови пропущенное слово.
                   7280: 
                   7281: Ответ:
                   7282: Ты.
                   7283: 
                   7284: Комментарий:
                   7285: Герой фильма "Пена дней" говорит в рифму: "Сударь, до "ты" нам еще
                   7286: полверсты". Своеобразный аналог фразы "Мы с вами на брудершафт еще не
                   7287: пили".
                   7288: 
                   7289: Источник:
                   7290: Фильм "Пена дней", реж. Мишель Гондри.
                   7291: 
                   7292: Автор:
                   7293: Артем Колесов
                   7294: 
                   7295: Вопрос 12:
                   7296: Слово "ОНИ" в вопросе заменяет несколько слов.
                   7297:    В абсурдистском романе Бориса Виана фигурирует гардероб, в котором
                   7298: вместо вешалок используются ОНИ. На чьих концертах в 2012 году также
                   7299: использовались ОНИ?
                   7300: 
                   7301: Ответ:
                   7302: Леди Гага.
                   7303: 
                   7304: Комментарий:
                   7305: ОНИ - крюки для подвешивания мяса. Вместо вешалок у Виана использовались
                   7306: крюки для мяса. В мае 2012 года во время тура в поддержку своего
                   7307: студийного альбома "Born This Way" Леди Гага дала сольный концерт в
                   7308: Токио. Одной из декораций послужили подвешенные на крюках туши коров.
                   7309: Правда, туши были пластмассовые. :-)
                   7310: 
                   7311: Источник:
                   7312:    1. Борис Виан. Пена дней. http://www.flibusta.net/b/133416/read
                   7313:    2. http://ru.wikipedia.org/wiki/Мясное_платье_Леди_Гаги
                   7314: 
                   7315: Автор:
                   7316: Дмитрий Сахаров
                   7317: 
                   7318: Вопрос 13:
                   7319: В XIX веке французские повара изобрели способ отбивания мяса, не
                   7320: нарушающий его внешнего вида. Назовите профессию людей, которые почти
                   7321: сразу начали пользоваться этим способом в своих целях.
                   7322: 
                   7323: Ответ:
                   7324: Жандармы.
                   7325: 
                   7326: Зачет:
                   7327: Жандармерия, полиция, полицейские, дознаватели, следователи.
                   7328: 
                   7329: Комментарий:
                   7330: Инструмент стало удобно использовать при пытках.
                   7331: 
                   7332: Источник:
                   7333: Вильям Похлёбкин. Кухня века. http://www.flibusta.net/b/339182/read
                   7334: 
                   7335: Автор:
                   7336: Антон Пинчук
                   7337: 
                   7338: Вопрос 14:
                   7339: Дуплет.
                   7340:    1. "Оценка позиции алгоритмизируется плохо". "Эффективная реализация
                   7341: влияния - один из важнейших стратегических элементов, но столь же важным
                   7342: является понимание момента, когда от схватки за влияние следует
                   7343: переходить к окончательному оформлению территории". В каждой из этих
                   7344: цитат мы пропустили одни и те же два коротких слова. Напишите их.
                   7345:    2. "Самым большим барабаном является изготавливаемый из дзельквы
                   7346: тайко высотой 14 см и диаметром 28 см". "Сценическая речь стилизована и
                   7347: неразборчива". "Движения имеют медленный темпоритм". В каждой из этих
                   7348: цитат мы пропустили одни и те же два коротких слова. Напишите их.
                   7349: 
                   7350: Ответ:
                   7351:    1. В го.
                   7352:    2. В но.
                   7353: 
                   7354: Комментарий:
                   7355: Надеемся, вы еще сохраняете трезвость ума. Цитаты в первом вопросе
                   7356: дуплета взяты из статьи об игре го, цитаты во втором вопросе дуплета -
                   7357: из статьи о театре но.
                   7358: 
                   7359: Источник:
                   7360:    1. http://ru.wikipedia.org/wiki/Го
                   7361:    2. http://ru.wikipedia.org/wiki/Но
                   7362: 
                   7363: Автор:
                   7364: Артем Колесов
                   7365: 
                   7366: Вопрос 15:
                   7367: В одном из эпизодов романа "Прощай, оружие!" рядом с героем разрывается
                   7368: снаряд, после чего слышен стук падающих ИХ. Чуть позже герой признаёт,
                   7369: что действительно испугался. Назовите ИХ.
                   7370: 
                   7371: Ответ:
                   7372: Кирпичи.
                   7373: 
                   7374: Зачет:
                   7375: Кирпичики.
                   7376: 
                   7377: Комментарий:
                   7378: Без комментариев.
                   7379: 
                   7380: Источник:
                   7381: Эрнест Хемингуэй. Прощай, оружие! http://www.flibusta.net/b/153877/read
                   7382: 
                   7383: Автор:
                   7384: Анна Бахарева
                   7385: 
                   7386: Вопрос 16:
                   7387: Дуплет. Два вопроса по 30 секунд обсуждения на каждый. Ответ на первый
                   7388: вопрос дуплета должен быть написан в левой части бланка, ответа на
                   7389: второй вопрос дуплета - в правой.
                   7390:    1. ЕГО история началась так: вЫрезали белую часть, оставив две
                   7391: другие. Назовите ЕГО двумя словами.
                   7392:    2. Крест на НЕМ подчеркивает, что бОльшая часть населения страны -
                   7393: католики, в отличие от соседей. Назовите ЕГО тремя словами.
                   7394: 
                   7395: Ответ:
                   7396:    1. Флаг Гаити.
                   7397:    2. Флаг Доминиканской республики.
                   7398: 
                   7399: Зачет:
                   7400:    1. Гаитянский флаг.
                   7401: 
                   7402: Комментарий:
                   7403:    1. Красный и синий цвета полос флага восходят к цветам полос на флаге
                   7404: Франции. В данном случае они выступают в качестве символического
                   7405: воплощения союза мулатов и чернокожих. Восставшие на Гаити рабы вЫрезали
                   7406: не только белую полосу, но и часть белого населения.
                   7407:    2. Большинство населения Доминиканской республики - католики, а
                   7408: гаитяне - вудуисты.
                   7409:    Форма ответа - подсказка: государство Гаити находится на западе
                   7410: острова Гаити, а Доминиканская Республика - на востоке.
                   7411: 
                   7412: Источник:
                   7413: http://www.vokrugsveta.ru/vs/article/1305/
                   7414: 
                   7415: Автор:
                   7416: Павел Бордачев, Дмитрий Сахаров
                   7417: 
                   7418: Вопрос 17:
                   7419: (pic: 20150165.jpg)
                   7420:    Чтобы использовать эту фотографию в рекламных целях, одна фирма
                   7421: совершила над ней некое действие. Кто совершил аналогичное действие в
                   7422: первой четверти XX века?
                   7423: 
                   7424: Ответ:
                   7425: [Сергей] Эйзенштейн.
                   7426: 
                   7427: Комментарий:
                   7428: Вам была роздана фотография покорителя Южного полюса норвежца Оскара
                   7429: Вистинга. На черно-белой фотографии, если не знать, кто на ней
                   7430: изображен, трудно определить, какой флаг перед нами - исландский или
                   7431: норвежский. Норвежский производитель пищевых добавок для животных
                   7432: "Obersten" раскрасил флаг на упаковках своей продукции. Эйзенштейн в
                   7433: фильме "Броненосец "Потёмкин"" раскрасил флаг вручную.
                   7434: 
                   7435: Источник:
                   7436:    1. "Вокруг света", 2014, N 11. - С. 100.
                   7437:    2. http://ru.wikipedia.org/wiki/Броненосец_%C2%ABПотёмкин%C2%BB_(фильм)
                   7438: 
                   7439: Автор:
                   7440: Дмитрий Сахаров
                   7441: 
                   7442: Вопрос 18:
                   7443: (pic: 20150166.jpg)
                   7444:    Эта пилотка, популярная в годы Второй мировой войны, была названа в
                   7445: честь объекта, являющегося частью единственного национального парка....
                   7446: Какой страны?
                   7447: 
                   7448: Ответ:
                   7449: Словении.
                   7450: 
                   7451: Комментарий:
                   7452: Эта пилотка словенских партизан-антифашистов из-за характерной формы
                   7453: была прозвана триглавкой в честь национального символа Словении - горы
                   7454: Триглав. Эта гора изображена, кстати, и на флаге Словении. :-)
                   7455: 
                   7456: Источник:
                   7457: http://ru.wikipedia.org/wiki/Триглавка
                   7458: 
                   7459: Автор:
                   7460: Дмитрий Сахаров
                   7461: 
                   7462: Вопрос 19:
                   7463: На картинке, призывающей задуматься о судьбе вымирающего албанского
                   7464: языка, изображен ОН. Своим названием ОН обязан президенту Американского
                   7465: музея естественной истории. Назовите ЕГО.
                   7466: 
                   7467: Ответ:
                   7468: Тираннозавр.
                   7469: 
                   7470: Зачет:
                   7471: Т. Рекс, Тираннозавр Рекс.
                   7472: 
                   7473: Комментарий:
                   7474: Название переводится как "королевский ящер-тиран". Президент
                   7475: Американского музея естественной истории Осборн подчеркивал размеры
                   7476: животного и доминирование над другими животными того времени. На
                   7477: картинке-меме "задумчивый динозавр" тираннозавр задумался, не ждет ли
                   7478: тезаурус Тираны участь тираннозавров.
                   7479: 
                   7480: Источник:
                   7481:    1. В вопросе.
                   7482:    2. http://ru.wikipedia.org/wiki/Тираннозавр
                   7483: 
                   7484: Автор:
                   7485: Артем Колесов
                   7486: 
                   7487: Вопрос 20:
                   7488: Георгий Михайлович в 2011 году удостоился медали за заслуги в освоении
                   7489: космоса. При этом Гречко - глубоко религиозный человек: на февраль 2014
                   7490: года в его доме были сотни икон, евангелий и мощей святых. Какое слово
                   7491: мы заменили в этом вопросе?
                   7492: 
                   7493: Ответ:
                   7494: Пшонка.
                   7495: 
                   7496: Зачет:
                   7497: Через "е" и "ё" тоже зачтем. И с "о" на конце.
                   7498: 
                   7499: Комментарий:
                   7500: В первом предложении вопроса замен не было, космонавт Георгий Михайлович
                   7501: Гречко действительно удостоился медали за освоение космоса. А вот во
                   7502: втором предложении мы заменили фамилию Пшонка фамилией Гречко. Виктор
                   7503: Пшонка - украинский прокурор, тоже занимавшийся освоением. 24 февраля
                   7504: 2014 года активисты Евромайдана попали в особняк бывшего генерального
                   7505: прокурора Украины Виктора Пшонки. А еще Гречко и Пшонка - это записанные
                   7506: по-олбански гречка и пшёнка.
                   7507: 
                   7508: Источник:
                   7509:    1. http://ru.wikipedia.org/wiki/Гречко,_Георгий_Михайлович
                   7510:    2. http://ru.wikipedia.org/wiki/Пшонка,_Виктор_Павлович
                   7511: 
                   7512: Автор:
                   7513: Артем Колесов
                   7514: 
                   7515: Вопрос 21:
                   7516: Внимание, ПЕРВАЯ и ВТОРАЯ заменяют словосочетания из четырех слов.
                   7517:    По мнению Александра Шевченко, характеристика "ПЕРВАЯ" больше
                   7518: подходит к периоду до 1917 года, а "ВТОРАЯ" - после. "ПЕРВАЯ" - название
                   7519: фильма времен развала Советского Союза. Напишите два глагола, которые
                   7520: присутствуют, соответственно, в ПЕРВОЙ и ВТОРОЙ.
                   7521: 
                   7522: Ответ:
                   7523: Потеряли, сломали.
                   7524: 
                   7525: Зачет:
                   7526: В любой форме, в любом порядке.
                   7527: 
                   7528: Комментарий:
                   7529: Россией, которую мы потеряли, можно назвать дореволюционную Россию. А
                   7530: послереволюционный период - Россия, которую мы сломали. Сочетание
                   7531: глаголов отсылает к известному анекдоту про два титановых шарика.
                   7532: 
                   7533: Источник:
                   7534:    1. Разговор автора вопроса с Александром Шевченко, 24.05.2014 г.
                   7535:    2. http://lurkmore.to/Россия,_которую_мы_потеряли
                   7536:    3. http://www.anekdot.ru/id/-9937345/
                   7537: 
                   7538: Автор:
                   7539: Антон Волосатов
                   7540: 
                   7541: Вопрос 22:
                   7542: В одной статье рассказывается о том, что Яндексу приходится в украинской
                   7543: версии своих карт показывать Крым территорией Украины, а в русской
                   7544: версии - территорией России. Забавно, что в веб-адресе этой статьи
                   7545: фигурируют два предмета. Какие?
                   7546: 
                   7547: Ответ:
                   7548: Стулья.
                   7549: 
                   7550: Зачет:
                   7551: Стульчики.
                   7552: 
                   7553: Комментарий:
                   7554: Россия, которую мы обрели. :-) Статья то ли намекает на попытку Яндекса
                   7555: усидеть на двух стульях, то ли просто на нелегкий выбор в ситуации,
                   7556: когда есть два стула.
                   7557: 
                   7558: Источник:
1.21      rubashki 7559: https://tjournal.ru/50180-yandex-two-chairs
1.1       rubashki 7560: 
                   7561: Автор:
                   7562: Руслан Хаиткулов
                   7563: 
                   7564: Вопрос 23:
                   7565: В названии статьи kommersant.ru о признании прав России на весь Охотский
                   7566: шельф упоминается "ОНА" моря. "ОНА" - подпись к демотиватору, на котором
                   7567: изображена свисающая с потолка петля. Назовите ЕЕ двумя словами.
                   7568: 
                   7569: Ответ:
                   7570: Последняя капля.
                   7571: 
                   7572: Зачет:
                   7573: Последняя капелька.
                   7574: 
                   7575: Комментарий:
                   7576: Статья называлась "Последняя капля моря". А силуэт петли напоминает
                   7577: каплю.
                   7578: 
                   7579: Источник:
                   7580:    1. http://www.kommersant.ru/doc/2347321
                   7581:    2. http://demotivators.to/p/70289/poslednyaya-kaplya-.htm
                   7582: 
                   7583: Автор:
                   7584: Елена Сидорова, Павел Бордачев
                   7585: 
                   7586: Вопрос 24:
                   7587: [Ведущему: обозначить цитату, кавычки не озвучивать.]
                   7588:    "Вы слышите? На улице играют "ВсЕнощную!"" - в этой фразе пропущено
                   7589: слово из трех букв. Назовите того, кто ее произнес незадолго до смерти в
                   7590: одной из американских больниц.
                   7591: 
                   7592: Ответ:
                   7593: [Сергей Васильевич] Рахманинов.
                   7594: 
                   7595: Комментарий:
                   7596: Пропущенное слово - "моЮ". Россию в лице Рахманинова мы тоже потеряли.
                   7597: Почти всю вторую половину жизни композитор провел на чужбине. Обработки
                   7598: Рахманиновым церковной музыки обрели большую популярность как в
                   7599: богослужениях, так и в светских выступлениях. Незадолго до смерти
                   7600: композитор услышал, что играют его "Всенощное бдение". А наше с вами
                   7601: бдение продолжается.
                   7602: 
                   7603: Источник:
                   7604: http://www.kicl.ru/archives/1910
                   7605: 
                   7606: Автор:
                   7607: Антон Волосатов
                   7608: 
                   7609: Тур:
                   7610: 18 тур. "Абажур Ожегова" (Москва)
                   7611: 
                   7612: Вопрос 1:
                   7613: (pic: 20150167.jpg)
                   7614:    На представленных двух картинах английский художник Джон Кольер
                   7615: изобразил первую и вторую. Назовите обеих в правильном порядке.
                   7616: 
                   7617: Ответ:
                   7618: Лилит, Ева.
                   7619: 
                   7620: Источник:
                   7621:    1. http://www.wikiart.org/en/john-collier/lilith-with-a-snake-1886
                   7622:    2. http://www.wikiart.org/en/john-collier/eve-1911
                   7623: 
                   7624: Автор:
                   7625: Алина Нестеренко
                   7626: 
                   7627: Вопрос 2:
                   7628: Рик Ли был большим поклонником Элвиса Пресли и, в частности, его хита
                   7629: 1956 года "Heartbreak Hotel". В 1966 году группа Рика получила название,
                   7630: которое в переводе на соответствующий язык можно обнаружить в названии
                   7631: известного романа. Назовите автора этого романа.
                   7632: 
                   7633: Ответ:
                   7634: [Александр] Дюма.
                   7635: 
                   7636: Комментарий:
                   7637: Группа называлась "Ten Years After".
                   7638: 
                   7639: Источник:
                   7640: http://bandnaming.blogspot.ru/2013/04/ten-years-after-band-name-meaning.html
                   7641: 
                   7642: Автор:
                   7643: Анастасия Грибановская
                   7644: 
                   7645: Вопрос 3:
                   7646: Известной балерине Матильде Кшесинской в начале XX века был преподнесен
                   7647: Высочайший подарок - браслет в виде некоего животного. А кто, согласно
                   7648: распространенной версии, помогал Николаю II выбирать подарок?
                   7649: 
                   7650: Ответ:
                   7651: Императрица.
                   7652: 
                   7653: Зачет:
                   7654: Александра Федоровна.
                   7655: 
                   7656: Комментарий:
                   7657: Согласно воспоминаниям фрейлины Анны Вырубовой, когда Николай II,
                   7658: увлекшийся балериной, решил преподнести ей подарок, императрица сперва
                   7659: приревновала, а потом предложила помочь с выбором и указала на браслет в
                   7660: виде змеи с крупным сапфиром и бриллиантами.
                   7661: 
                   7662: Источник:
                   7663:    1. И.В. Зимин. Царские деньги. Доходы и расходы Дома Романовых.
                   7664: http://www.flibusta.net/b/242726/read
                   7665:    2. А.А. Вырубова. Страницы моей жизни.
                   7666: http://www.flibusta.net/b/242568/read
                   7667: 
                   7668: Автор:
                   7669: Анастасия Грибановская
                   7670: 
                   7671: Вопрос 4:
                   7672: (pic: 20150168.jpg)
                   7673:    На раздатке социальная реклама Всемирного фонда дикой природы в
                   7674: защиту животных. Подпись в правом верхнем углу: "Мода требует больше
                   7675: жертв, чем вы думаете". Что мы закрыли от вас?
                   7676: 
                   7677: Ответ:
                   7678: [Размеры] S, XL.
                   7679: 
                   7680: Зачет:
                   7681: По смыслу.
                   7682: 
                   7683: Источник:
                   7684: http://www.ontare.ru/shokiruyushhie-reklamnyie-kampanii-o-problemah-zhivotnyih/
                   7685: 
                   7686: Автор:
                   7687: Алексей Авраменко
                   7688: 
                   7689: Вопрос 5:
                   7690: Дуплет.
                   7691:    1. Согласно "Словарю Сатаны" Амброза Бирса, на одном языке это -
                   7692: красивая женщина, на другом - смертельный яд, что демонстрирует
                   7693: удивительное сходство языков. Что это?
                   7694:    2. Согласно "Словарю Сатаны" Амброза Бирса, ИКС - Бог ведущей религии
                   7695: мира, храм которой находится в священном городе Нью-Йорке. Назовите ИКС.
                   7696: 
                   7697: Ответ:
1.6       rubashki 7698:    1. Белладонна.
                   7699:    2. Маммона.
1.1       rubashki 7700: 
                   7701: Источник:
                   7702: Амброз Бирс. Словарь Сатаны. http://www.flibusta.net/b/293076/read
                   7703: 
                   7704: Автор:
                   7705: Алина Нестеренко
                   7706: 
                   7707: Вопрос 6:
                   7708: ИКС Вильгельма - часто используемый в кино и телевидении эффект. Помимо
                   7709: "Звездных войн", его можно встретить в таких фильмах, как "Бешеные псы",
                   7710: "Убить Билла", "Доказательство смерти", "Бесславные ублюдки". В одной из
                   7711: серий сериала "Волшебники из Вэйверли Плэйс" при обсуждении "ИКСА"
                   7712: появилась версия, что одному человеку просто очень сильно жмут ботинки.
                   7713: Назовите ИКС.
                   7714: 
                   7715: Ответ:
                   7716: Крик.
                   7717: 
                   7718: Источник:
                   7719:    1. http://ru.wikipedia.org/wiki/Крик_Вильгельма
                   7720:    2. http://lurkmore.to/Крик
                   7721: 
                   7722: Автор:
                   7723: Илья Любич
                   7724: 
                   7725: Вопрос 7:
                   7726: В тезисе из четырех слов отчетного доклада Л.И. Брежнева на XXVI съезде
                   7727: КПСС, ставшем политическим лозунгом, несмотря на первое впечатление,
                   7728: тавтологии нет, если правильно понимать использованные в нем термины. На
                   7729: какое слово заменено первое слово этого тезиса в заголовке статьи о
                   7730: специальном "зеленом" шрифте, на 20% сокращающем расход тонера?
                   7731: 
                   7732: Ответ:
                   7733: Экология.
                   7734: 
                   7735: Комментарий:
                   7736: Брежнев сказал: "Экономика должна быть экономной". Статья озаглавлена
                   7737: "Экология должна быть экономной".
                   7738: 
                   7739: Источник:
                   7740: http://www.brainbang.ru/note/489/
                   7741: 
                   7742: Автор:
                   7743: Анастасия Грибановская
                   7744: 
                   7745: Вопрос 8:
                   7746: (pic: 20150169.jpg)
                   7747:    Давид Бонацци известен своими философскими иллюстрациями на
                   7748: социальные темы. На представленной вам иллюстрации автор размышляет о
                   7749: свободе. Во что превращаются птицы с проводами на закрытой от вас части
                   7750: иллюстрации?
                   7751: 
                   7752: Ответ:
                   7753: Колючая проволока.
                   7754: 
                   7755: Комментарий:
                   7756: (pic: 20150170.jpg)
                   7757: 
                   7758: Источник:
                   7759: http://www.adme.ru/tvorchestvo-hudozhniki/30-ironichnyh-nablyudenij-o-nashem-mire-v-illyustraciyah-692305/
                   7760: 
                   7761: Автор:
                   7762: Алексей Авраменко
                   7763: 
                   7764: Вопрос 9:
                   7765: Фенеки обладают очень маленьким телом и большими ушами-локаторами. Уши
                   7766: фенеков занимают 20% поверхности тела и позволяют слышать перемещения
                   7767: насекомых даже под землей, а также избежать перегрева благодаря большой
                   7768: площади кровеносных сосудов. Как называется самое известное литературное
                   7769: произведение, в котором прототипом одного из главных героев послужил
                   7770: фенек?
                   7771: 
                   7772: Ответ:
                   7773: Маленький Принц.
                   7774: 
                   7775: Источник:
                   7776:    1. http://ru.wikipedia.org/wiki/Фенек
                   7777:    2. Журнал "Geo", февраль 2015 г.
                   7778: 
                   7779: Автор:
                   7780: Анастасия Кривенкова
                   7781: 
                   7782: Вопрос 10:
                   7783: Киты, как известно, общаются друг с другом с помощью пения. Обычно киты
                   7784: поют свои песни на частоте от 10 до 20 герц, но в мире существует
                   7785: уникальная особь, чье пение раздается на частоте 52 герца. Этого кита
                   7786: называют самым одиноким в мире, потому что его сородичи не могут его
                   7787: понять. Ученым, изучавшим кита, приходили письма от ТАКИХ людей, в
                   7788: которых они высказывали версию, что кит тоже является ТАКИМ. Каким?
                   7789: 
                   7790: Ответ:
                   7791: Глухим.
                   7792: 
                   7793: Источник:
                   7794: http://ru.wikipedia.org/wiki/Пятидесятидвухгерцевый_кит
                   7795: 
                   7796: Автор:
                   7797: Алина Нестеренко
                   7798: 
                   7799: Вопрос 11:
                   7800: В фильме Вуди Аллена "Разбирая Гарри" звучит такой диалог:
                   7801:    - У тебя нет моральных ценностей, вся твоя жизнь - сплошной нигилизм,
                   7802: цинизм, сарказм и оргазм.
                   7803:    - Кстати, во Франции с таким лозунгом легко СДЕЛАТЬ ЭТО.
                   7804:    Какие три слова мы заменили на "СДЕЛАТЬ ЭТО", если известный
                   7805: голливудский актер делал это дважды?
                   7806: 
                   7807: Ответ:
                   7808: Победить на выборах.
                   7809: 
                   7810: Комментарий:
                   7811: Речь идет о Шварценеггере.
                   7812: 
                   7813: Источник:
                   7814:    1. http://ru.wikiquote.org/wiki/Разбирая_Гарри_(фильм)
                   7815:    2. http://ru.wikipedia.org/wiki/Шварценеггер,_Арнольд
                   7816: 
                   7817: Автор:
                   7818: Анастасия Грибановская
                   7819: 
                   7820: Вопрос 12:
                   7821: (pic: 20150171.jpg)
                   7822:    Этот рекламный плакат был сделан по случаю одной популярной ежегодной
                   7823: акции. Назовите ее.
                   7824: 
                   7825: Ответ:
                   7826: Час Земли.
                   7827: 
                   7828: Комментарий:
                   7829: Скрыта надпись "For once, the Dark Side saves the planet".
                   7830: 
                   7831: Источник:
                   7832: http://www.brainbang.ru/note/490/
                   7833: 
                   7834: Автор:
                   7835: Анастасия Грибановская
                   7836: 
                   7837: Вопрос 13:
                   7838: По поводу происхождения названия этой американской рок-группы существует
                   7839: множество версий. По одной из них, оно появилось, когда музыканты
                   7840: подъехали поздно вечером к придорожной пиццерии с неисправно работающей
                   7841: вывеской. Назовите группу.
                   7842: 
                   7843: Ответ:
                   7844: "ZZ Top".
                   7845: 
                   7846: Комментарий:
                   7847: Согласно этой версии, автобус с голодными музыкантами остановился у
                   7848: неоновой вывески "PIZZA STOP" с выключенными "PI", "A" и "S".
                   7849: 
                   7850: Источник:
                   7851: http://bandnaming.blogspot.ru/2013/04/zz-top-band-name-meaning.html
                   7852: 
                   7853: Автор:
                   7854: Анастасия Грибановская
                   7855: 
                   7856: Вопрос 14:
                   7857: В конце фильма "Залечь на дно в Брюгге" один из героев умирает в городе,
                   7858: который успел возненавидеть за время пребывания в нем. Последние дни
                   7859: своей жизни он сравнивает с ИКСОМ. ИКСОВАЯ версия тетриса имеет
                   7860: закругленное дно. Что мы заменили словом "ИКС"?
                   7861: 
                   7862: Ответ:
                   7863: Ад.
                   7864: 
                   7865: Источник:
                   7866:    1. http://ru.wikiquote.org/wiki/Залечь_на_дно_в_Брюгге
                   7867:    2. http://www.kongregate.com/games/banthar/hell-tetris
                   7868: 
                   7869: Автор:
                   7870: Илья Любич
                   7871: 
                   7872: Вопрос 15:
                   7873: В одном учебнике английского языка есть фраза, дословно переводящаяся
                   7874: как "Если вы сидите на заборе, то вы ДЕЛАЕТЕ ЭТО. Однако рано или поздно
                   7875: вам придется слезть справа или слева". Ответьте, что такое "ДЕЛАТЬ ЭТО",
                   7876: тремя словами, начинающимися на три последовательные буквы.
                   7877: 
                   7878: Ответ:
                   7879: Откладывать принятие решения.
                   7880: 
                   7881: Комментарий:
                   7882: Sit on the fence = откладывать принятие решения.
                   7883: 
                   7884: Автор:
                   7885: Роман Игла
                   7886: 
                   7887: Вопрос 16:
                   7888: В фильме Вуди Аллена "Разбирая Гарри" один из персонажей появляется в
                   7889: кадре на несколько минут, чтобы подосадовать на то, что он ТАКОЙ, и что
                   7890: это очень мешает ему в его карьере актера. "Когда мир ТАКОЙ" - заголовок
                   7891: статьи о тренировке зрения. Какие три слова мы заменили на "ТАКОЙ"?
                   7892: 
                   7893: Ответ:
                   7894: Не в фокусе.
                   7895: 
                   7896: Источник:
                   7897:    1. Фильм "Разбирая Гарри" (1997).
                   7898:    2. Газета "Помоги себе сам", апрель 2005 г.
                   7899: 
                   7900: Автор:
                   7901: Анастасия Грибановская
                   7902: 
                   7903: Вопрос 17:
                   7904: В одном из выпусков юмористического проекта "Epic Rap Battles of
                   7905: History" этот известный человек, ссылаясь на свою работу, заявляет, что
                   7906: он крут как два обычных рэпера. Назовите этого человека.
                   7907: 
                   7908: Ответ:
                   7909: Эйнштейн.
                   7910: 
                   7911: Комментарий:
                   7912: E (instein) = MC^2. MC [эм-си] (сокр. англ. Master of Ceremonies), в
                   7913: регги-культуре и хип-хопе - артист, в сопровождении электронной
                   7914: танцевальной музыки произносящий со сцены слова - заранее сочиненные или
                   7915: импровизированные.
                   7916: 
                   7917: Источник:
                   7918: http://genius.com/408252
                   7919: 
                   7920: Автор:
                   7921: Илья Любич
                   7922: 
                   7923: Вопрос 18:
1.10      rubashki 7924: По окончании перелетов на "Боингах" стюардессы российских авиалиний
                   7925: иногда обнаруживают пропажу ИХ и находятся в недоумении. Назовите ИХ.
1.1       rubashki 7926: 
                   7927: Ответ:
                   7928: Спасательные жилеты.
                   7929: 
                   7930: Комментарий:
                   7931: В журнале сообщается, что большинство людей при поимке заявляет, что
                   7932: любят в них рыбачить.
                   7933: 
                   7934: Источник:
                   7935: Журнал "Geo", февраль 2015 г.
                   7936: 
                   7937: Автор:
                   7938: Анастасия Кривенкова
                   7939: 
                   7940: Вопрос 19:
                   7941: (pic: 20150172.jpg)
                   7942:    Доля резюме с ЭТИМ от общего количества представлена на диаграмме. С
                   7943: чем?
                   7944: 
                   7945: Ответ:
                   7946: С фотографией.
                   7947: 
                   7948: Источник:
                   7949: http://www.career.ru/article/15908
                   7950: 
                   7951: Автор:
                   7952: Дмитрий Лобанов
                   7953: 
                   7954: Вопрос 20:
                   7955: Герой Орсона Скотта Карда пишет несколько политических статей под
                   7956: псевдонимом, который он взял в честь мыслителя XVII века. Но переводчик
                   7957: не понял хитрого авторского приема и решил, что псевдоним герой выбрал в
                   7958: честь бога. Назовите и мыслителя, и бога.
                   7959: 
                   7960: Ответ:
                   7961: [Джон] Локк, Локи.
                   7962: 
                   7963: Источник:
                   7964: http://ru.wikipedia.org/wiki/Локк,_Джон
                   7965: 
                   7966: Автор:
                   7967: Илья Любич
                   7968: 
                   7969: Вопрос 21:
                   7970: Кельвин Кляйн считается одним из самых скандально известных новаторов в
                   7971: моде. Он известен провокационной рекламой с участием практически
                   7972: обнаженных несовершеннолетних и тем, что ввел в моду стиль "унисекс".
                   7973: Одна из его идей была настолько возмутительной, что имела небывалый
                   7974: успех. Она не давала модникам всего мира забыть его имя ни днем, ни
                   7975: ночью. Какая?
                   7976: 
                   7977: Ответ:
                   7978: Его имя на резинке трусов.
                   7979: 
                   7980: Зачет:
                   7981: По смыслу.
                   7982: 
                   7983: Автор:
                   7984: Анастасия Кривенкова
                   7985: 
                   7986: Вопрос 22:
                   7987: В сериале "Сверхъестественное" в серии "Девушка с татуировкой драконов"
                   7988: главные герои встречают хакершу Чарли, которая помогает им сражаться с
                   7989: их врагами Левиафанами. Она постоянно думает, как бы поступили на ее
                   7990: месте герои различных фентезийных миров, как бы примеряя их образы на
1.20      rubashki 7991: себя, чтобы не струсить и не убежать в самый опасный и ответственный
1.1       rubashki 7992: момент. Какие два слова мы пропустили в первом предложении?
                   7993: 
                   7994: Ответ:
                   7995: Подземелий и.
                   7996: 
                   7997: Комментарий:
                   7998: Серия называется "Девушка с татуировкой Подземелий и Драконов". В D&D
                   7999: люди обычно, наоборот, представляют себя на месте других героев. Чарли
                   8000: является сильным гиком.
                   8001: 
                   8002: Источник:
                   8003: http://www.imdb.com/title/tt1995114/
                   8004: 
                   8005: Автор:
                   8006: Илья Любич
                   8007: 
                   8008: Вопрос 23:
                   8009: (pic: 20150173.jpg)
                   8010:    Перед вами иллюстрация к одной карте из коллекционной карточной игры
                   8011: "Magic: The Gathering". Действие карты ход за ходом снимает жизни у всех
                   8012: игроков, однако игроки могут свести эти повреждения к минимуму. Название
                   8013: карты является отсылкой к проекту, начатому в 1947 году. Назовите этот
                   8014: проект.
                   8015: 
                   8016: Ответ:
                   8017: "Часы Судного дня".
                   8018: 
                   8019: Источник:
                   8020: http://ru.wikipedia.org/wiki/Часы_Судного_дня
                   8021: 
                   8022: Автор:
                   8023: Илья Любич
                   8024: 
                   8025: Вопрос 24:
                   8026: Один из героев фильма "Кадиллак Рекордз" говорит: "Когда первая девчонка
                   8027: сняла трусы и бросила их на сцену, там пели блюз. Когда дело дошло до
                   8028: НИХ, это назвали рок-н-ролл". Какие два слова мы заменили на "НИХ"?
                   8029: 
                   8030: Ответ:
                   8031: Белых девчонок.
                   8032: 
                   8033: Зачет:
                   8034: По смыслу.
                   8035: 
                   8036: Источник:
                   8037: Фильм "Кадиллак Рекордз" (2008).
                   8038: 
                   8039: Автор:
                   8040: Анастасия Грибановская
                   8041: 
                   8042: Тур:
                   8043: 19 тур. "Hangover" (Архангельск - Северодвинск - Санкт-Петербург)
                   8044: 
                   8045: Вопрос 1:
                   8046: Некий дворянин был вынужден уволиться с государственной службы, когда
                   8047: перепутал ИКС с АЛЬФОЙ. Впрочем, такие случаи происходят до сих пор.
                   8048: Так, Артемий Лебедев, собирая свою фотоколлекцию под названием "6666
                   8049: рублей", тоже сначала перепутал ИКС и АЛЬФУ. Что мы заменили на ИКС и
                   8050: АЛЬФУ?
                   8051: 
                   8052: Ответ:
                   8053: Архангельск, Астрахань.
                   8054: 
                   8055: Комментарий:
                   8056: Коллекция Лебедева посвящена денежным знакам; на купюре в 500 российских
                   8057: рублей присутствует город Архангельск, однако Лебедев не первый, кто
                   8058: перепутал этот город с Астраханью. Дворянин, сделавший это до него, -
                   8059: Обломов.
                   8060: 
                   8061: Источник:
                   8062:    1. Иван Гончаров. Обломов. http://www.flibusta.net/b/217953/read
                   8063:    2. http://tema.livejournal.com/1788181.html
                   8064: 
                   8065: Автор:
                   8066: Екатерина Макоед (Архангельск)
                   8067: 
                   8068: Вопрос 2:
                   8069: Роберт Персиг в своей книге упоминает реакционные указы губернатора
                   8070: Монтаны Дональда Наттера и замечает, впрочем, что в его случае настоящий
                   8071: политик-радикал может позволить себе делать что угодно, потому что
                   8072: оппозиция уже выставила себя... Закончите фразу одним словом.
                   8073: 
                   8074: Ответ:
                   8075: "... ослами".
                   8076: 
                   8077: Комментарий:
                   8078: Оппозиция была представлена демократами, символом которых является осел.
                   8079: 
                   8080: Источник:
                   8081: Роберт Пирсиг. Дзен и искусство ухода за мотоциклом.
                   8082: http://www.flibusta.net/b/42253/read
                   8083: 
                   8084: Автор:
                   8085: Денис Никитенко (Архангельск - Санкт-Петербург)
                   8086: 
                   8087: Вопрос 3:
                   8088: Хотя это млекопитающее не является хищником и питается преимущественно
                   8089: насекомыми и плодами, знакомый автора вопроса утверждал, что оно
                   8090: неожиданно напало на его команду, прямо посреди турнира по "Что? Где?
                   8091: Когда?". Назовите это животное.
                   8092: 
                   8093: Ответ:
                   8094: Тупайя.
                   8095: 
                   8096: Источник:
                   8097:    1. Шутка знакомого автора вопроса.
                   8098:    2. http://ru.wikipedia.org/wiki/Тупайи
                   8099: 
                   8100: Автор:
                   8101: Сергей Коновалов (Санкт-Петербург)
                   8102: 
                   8103: Вопрос 4:
                   8104: В осовремененной постановке трагедии Еврипида "Медея" в первом акте на
                   8105: сцене появляются два спальника, которые впоследствии будут использованы
                   8106: как ИКСЫ. На сайте vkostume.ru [в костюме точка ру] можно заказать
                   8107: хэллоуинский вариант ИКСА с прорезями для ног и головы, сшитый из
                   8108: 100%-ного полиэстера. Назовите ИКС тремя или двумя словами.
                   8109: 
                   8110: Ответ:
                   8111: Мешок для трупа / патанатомичский мешок.
                   8112: 
                   8113: Зачет:
                   8114: По смыслу.
                   8115: 
                   8116: Комментарий:
                   8117: Медея упаковывает тела убитых сыновей в спальники, как в
                   8118: патанатомические мешки.
                   8119: 
                   8120: Источник:
                   8121:    1. Постановка Королевского Национального театра, 2014.
                   8122:    2. http://www.vkostume.ru/item/Meshok_dlya_tela/
                   8123: 
                   8124: Автор:
                   8125: Екатерина Макоед (Архангельск)
                   8126: 
                   8127: Вопрос 5:
                   8128: На пьедестале памятника этому человеку выбиты слова, исходя из которых
                   8129: можно предположить, что это памятник либо руководителю еврейского народа
                   8130: времен завоевания Ханаана, либо французскому сенатору XIX века. Назовите
                   8131: человека, которому в действительности поставлен этот памятник.
                   8132: 
                   8133: Ответ:
                   8134: Николай Коперник.
                   8135: 
                   8136: Комментарий:
                   8137: Надпись гласит: "Остановившему Солнце, сдвинувшему Землю". Иисус Навин
                   8138: остановил Солнце, а Жорж Дантес убил Пушкина - "Солнце русской поэзии".
                   8139: 
                   8140: Источник:
                   8141: http://ru.wikipedia.org/wiki/Коперник,_Николай
                   8142: 
                   8143: Автор:
                   8144: Виталий Егоров (Мурманск)
                   8145: 
                   8146: Вопрос 6:
                   8147: Дуплет.
                   8148:    1. Леонид Мацих утверждает, что умирающий икс в эмблеме одной из
                   8149: масонских лож - символ разгадки всех тайн природы. В одном из слов
                   8150: предыдущего предложения мы пропустили три буквы. Напишите их.
                   8151:    2. Петер Идлинг пишет, что нагрудный карман красного кхмера с разным
                   8152: количеством шариковых ручек выполнял ту же функцию, что и он в других
                   8153: странах. Какие три буквы мы пропустили?
                   8154: 
                   8155: Ответ:
                   8156:    1. с, ф, н.
                   8157:    2. п, о, г.
                   8158: 
                   8159: Комментарий:
                   8160: Соответственно, СФиНкс и ПОГон.
                   8161: 
                   8162: Источник:
                   8163:    1. Л.М. Мацих. Серия программ "Братья" на радио "Эхо Москвы".
                   8164:    2. Петер Фреберг Идлинг. Улыбка Пол Пота. - М.: Астрель, 2014.
                   8165: 
                   8166: Автор:
                   8167: Денис Никитенко (Архангельск - Санкт-Петербург)
                   8168: 
                   8169: Вопрос 7:
                   8170: Внимание, в вопросе есть замены.
                   8171:    "ДВОРОВЫЕ ПСЫ" стали устойчивым выражением в январе 1948 года.
                   8172: Кропоткина и Бакунина вполне можно назвать ПОРОДИСТЫМИ ПСАМИ. Назовите
                   8173: человека, который считается первым ПСОМ.
                   8174: 
                   8175: Ответ:
                   8176: Диоген.
                   8177: 
                   8178: Комментарий:
                   8179: Дворовые псы - безродные космополиты, породистые псы - родовитые
                   8180: космополиты, а первым космополитом называли Диогена. Псом его, впрочем,
                   8181: тоже называли.
                   8182: 
                   8183: Источник:
                   8184:    1. http://www.bibliotekar.ru/encSlov/2/33.htm
                   8185:    2. http://ru.wikipedia.org/wiki/Кропоткин,_Пётр_Алексеевич
                   8186:    3. http://ru.wikipedia.org/wiki/Бакунин,_Михаил_Александрович
                   8187:    4. Диоген Лаэртский. О жизни, учениях и изречениях знаменитых
                   8188: философов. http://www.psylib.org.ua/books/diogenl/txt06.htm
                   8189: 
                   8190: Автор:
                   8191: Александр Цыганов (Архангельск)
                   8192: 
                   8193: Вопрос 8:
                   8194: В вопросе ИКСОМ заменены несколько слов.
                   8195:    Американцам, скорее всего, не был известен сериал "Coronation
                   8196: Street", поэтому ИКС, который в Британии воспринимался как пародия на
                   8197: узнаваемые женские образы из этого сериала, в США был запрещен. Назовите
                   8198: любого из снявшихся в ИКСЕ в главных ролях.
                   8199: 
                   8200: Ответ:
                   8201: Фредди Меркьюри / Брайан Мэй / Роджер Тейлор / Джон Дикон.
                   8202: 
                   8203: Комментарий:
                   8204: Речь идет о клипе на песню "I want to break free", содержащем
                   8205: провокационные образы.
                   8206: 
                   8207: Источник:
                   8208: http://ru.wikipedia.org/wiki/I_Want_to_Break_Free
                   8209: 
                   8210: Автор:
                   8211: Екатерина Макоед (Архангельск)
                   8212: 
                   8213: Вопрос 9:
                   8214: Один пользователь Рунета, говоря о защите компьютерных программ,
                   8215: пошутил, что на полях борьбы правообладателей и пиратов было пролито
                   8216: много крови. Какие два слова мы заменили в тексте вопроса?
                   8217: 
                   8218: Ответ:
                   8219: Сломано копий.
                   8220: 
                   8221: Комментарий:
                   8222: Замена связана со смыслом идиоматических выражений "пролито крови" и
                   8223: "сломано копий". Игра слов с отсылкой к пиратским копиям.
                   8224: 
                   8225: Источник:
                   8226: http://bash.im/quote/420736/
                   8227: 
                   8228: Автор:
                   8229: Иван Беляев (Архангельск)
                   8230: 
                   8231: Вопрос 10:
                   8232: Как известно, для бразильцев нет ничего важнее футбола. Перед финальным
1.10      rubashki 8233: матчем Кубка страны газета "Estado de Minas" вышла с большим постером на
                   8234: первой полосе, посвященным предстоящему матчу. Назовите короткое слово,
                   8235: встречающееся во всех остальных статьях в этом выпуске.
1.1       rubashki 8236: 
                   8237: Ответ:
                   8238: Bla.
                   8239: 
                   8240: Зачет:
                   8241: Бла.
                   8242: 
                   8243: Комментарий:
                   8244: Редакция газеты, видимо, посчитала, что ничего важнее финала кубка нет,
                   8245: поэтому все остальные статьи газеты состояли из слов "bla bla bla".
                   8246: 
                   8247: Источник:
                   8248: http://portalmakingof.com.br/estado_de_minas_coloca_quotbla_bla_blaquot_na_capa_para_focar_no_classico_mineiro.html
                   8249: 
                   8250: Автор:
                   8251: Сергей Коновалов (Санкт-Петербург)
                   8252: 
                   8253: Вопрос 11:
                   8254: В романе Джоан Роулинг "Шелкопряд" герои посещают старинное заведение с
                   8255: традиционной британской кухней. Место настолько кичится своей
                   8256: консервативностью, что "в пику французам" это слово там не
                   8257: употребляется. Назовите это слово.
                   8258: 
                   8259: Ответ:
                   8260: Меню.
                   8261: 
                   8262: Комментарий:
                   8263: Даже привычное и почти обязательное слово для любого места общественного
                   8264: питания, имеющее французские корни, британцы стараются избегать.
                   8265: 
                   8266: Источник:
                   8267: Джоан Роулинг. Шелкопряд.
                   8268: 
                   8269: Автор:
                   8270: Екатерина Макоед (Архангельск)
                   8271: 
                   8272: Вопрос 12:
                   8273: Сидя на собрании анонимных алкоголиков, Гомер Симпсон спрашивает, где
                   8274: пончики. Когда ему указывают, он говорит: "Слишком далеко. Целых...".
                   8275: Закончите фразу двумя словами.
                   8276: 
                   8277: Ответ:
                   8278: "... двенадцать шагов".
                   8279: 
                   8280: Комментарий:
                   8281: Широко известна программа "12 шагов", которую также часто называют одной
                   8282: из самых эффективных для лечения и реабилитации страдающих зависимостью
                   8283: людей.
                   8284: 
                   8285: Источник:
                   8286: "Симпсоны", s11e18 "Days of Wine and D'oh'ses".
                   8287: 
                   8288: Автор:
                   8289: Екатерина Макоед (Архангельск)
                   8290: 
                   8291: Вопрос 13:
                   8292: В книге "Огнем и мечом" герой, рассуждая о военных успехах Речи
                   8293: Посполитой, говорит: "Разгромила их как АЛЬФА, но простит их как БЕТА".
                   8294: АЛЬФА-БЕТА - единственный в своем роде носитель звания Почетного
                   8295: гражданина города Волгограда за особые заслуги в период Второй мировой
                   8296: войны. Что мы заменили словами "АЛЬФА" и "БЕТА"?
                   8297: 
                   8298: Ответ:
                   8299: Королева, мать.
                   8300: 
                   8301: Комментарий:
                   8302: Звание "Почетный гражданин города-героя Волгограда" присвоено
                   8303: Королеве-матери Великобритании Елизавете Виндзорской постановлением
                   8304: Волгоградского городского Совета народных депутатов от 6 января 2000 г.
                   8305: за особые заслуги в организации помощи Сталинграду жителями
                   8306: Великобритании в период Второй мировой войны и развитии дружественных
                   8307: связей с Россией. На данный момент это единственный случай присвоения
                   8308: звания почетного гражданина города Волгограда иностранцу.
                   8309: 
                   8310: Источник:
                   8311:    1. Г. Сенкевич. Огнем и мечом. http://www.flibusta.net/b/243372/read
                   8312:    2. http://ru.wikipedia.org/wiki/Почётные_граждане_города-героя_Волгограда#2000.C2.A0.D0.B3.D0.BE.D0.B4
                   8313: 
                   8314: Автор:
                   8315: Александра Ермолина (Архангельск)
                   8316: 
                   8317: Вопрос 14:
                   8318: Фильм, посвященный убийству Кеннеди, называется "ИКС". Первый лондонский
                   8319: ИКС вышел на улицы в 1829 году. Назовите ИКСА.
                   8320: 
                   8321: Ответ:
                   8322: Бобби.
                   8323: 
                   8324: Комментарий:
                   8325: Фильм посвящен убийству Роберта Кеннеди; Бобби - также прозвище
                   8326: английских полицейских.
                   8327: 
                   8328: Источник:
                   8329:    1. http://www.kinopoisk.ru/film/31918/
                   8330:    2. http://www.vokrugsveta.ru/quiz/104/
                   8331: 
                   8332: Автор:
                   8333: Екатерина Макоед (Архангельск)
                   8334: 
                   8335: Вопрос 15:
                   8336: Внимание, в вопросе есть замена.
                   8337:    В театральных постановках АЛЬФУ сыграли Мехрангиз Гасанова и Алсу
                   8338: Густова, а в фильме - Татьяна Федотова, которая попалась на глаза
                   8339: режиссера случайно. "Если бы Танечка не сбежала с уроков, - пишет
                   8340: Людмила Андреева, - у АЛЬФЫ было бы совсем другое...". Закончите
                   8341: предложение одним словом абсолютно точно.
                   8342: 
                   8343: Ответ:
                   8344: "... личико".
                   8345: 
                   8346: Комментарий:
                   8347: АЛЬФА - Гюльчатай из "Белого солнца пустыни". Широкую известность
                   8348: получила фраза из фильма "Гюльчатай, открой личико!".
                   8349: 
                   8350: Источник:
                   8351:    1. http://ru.wikipedia.org/wiki/Гюльчатай
                   8352:    2. http://www.smena.ru/media/numbers/24803/24803.pdf
                   8353: 
                   8354: Автор:
                   8355: Денис Никитенко (Архангельск - Санкт-Петербург)
                   8356: 
                   8357: Вопрос 16:
                   8358: Биатлонист Бьорн Ферри летом 2014 года презентовал автобиографическую
                   8359: книгу. Название книги состоит из двух слов и на слух практически
                   8360: неотличимо от "имени" рекордсменки, установившей свой рекорд в России в
                   8361: мае 2009 года. Воспроизведите название книги.
                   8362: 
                   8363: Ответ:
                   8364: "Ferry Tales".
                   8365: 
                   8366: Зачет:
                   8367: "Ferry Tale".
                   8368: 
                   8369: Комментарий:
                   8370: Рекордсменка - песня Александра Рыбака "Fairytale", набравшая на
                   8371: конкурсе Евровидение-2009 рекордное число очков.
                   8372: 
                   8373: Источник:
                   8374:    1. http://www.sports.ru/biathlon/1021218863.html
                   8375:    2. http://ru.wikipedia.org/wiki/Fairytale
                   8376: 
                   8377: Автор:
                   8378: Сергей Коновалов (Санкт-Петербург)
                   8379: 
                   8380: Вопрос 17:
                   8381: Несмотря на вполне устоявшуюся традицию изображать ЕЕ определенным
                   8382: образом, у НЕЕ, находящейся в Центральном административном округе
                   8383: Москвы, нет одного из важнейших атрибутов. Этот факт стал причиной для
                   8384: множества шуток и обсуждений до такой степени, что несколько лет назад
                   8385: депутаты предложили президенту начать одну из проводимых реформ именно с
                   8386: этого. Ответьте максимально точно: что предложили сделать депутаты?
                   8387: 
                   8388: Ответ:
                   8389: Завязать Фемиде глаза.
                   8390: 
                   8391: Комментарий:
                   8392: ОНА - это Фемида. Статуя Фемиды на фасаде Верховного суда РФ изображена
                   8393: вопреки устоявшейся традиции без повязки на глазах. Депутаты на встрече
                   8394: с тогдашним президентом Дмитрием Медведевым предложили начать судебную
                   8395: реформу с изменения статуи.
                   8396: 
                   8397: Источник:
                   8398: http://lenta.ru/news/2011/05/13/femida/
                   8399: 
                   8400: Автор:
                   8401: Александра Ермолина (Архангельск)
                   8402: 
                   8403: Вопрос 18:
                   8404: В своей статье Дмитрий Быков рассуждает о новой волне целенаправленных
                   8405: нападок на нобелевского лауреата и упоминает французский вариант его
                   8406: имени и АЛЬФУ, отличающиеся друг от друга только одной буквой. Назовите
                   8407: АЛЬФУ.
                   8408: 
                   8409: Ответ:
                   8410: Мишень.
                   8411: 
                   8412: Комментарий:
                   8413: Нобелевский лауреат - Михаил Горбачёв, обыгрывается созвучие "Мишель -
                   8414: мишень".
                   8415: 
                   8416: Источник:
                   8417: http://www.sobesednik.ru/dmitriy-bykov/20140416-dmitriy-bykov-kak-razvlech-rossiyu-yarche-suda-nad-gorbachev
                   8418: 
                   8419: Автор:
                   8420: Денис Никитенко (Архангельск - Санкт-Петербург)
                   8421: 
                   8422: Вопрос 19:
                   8423: В своей статье о Владимире Тарасенко Артем Зырянов пишет, что этот
                   8424: хоккеист то забьет кистевым броском, то щелкнет, то подставит клюшку на
                   8425: пятачке, то выдаст хитрый финт. Далее голы Тарасенко сравниваются с
                   8426: АЛЬФАМИ. В известной песне АЛЬФА сравнивается с транспортным средством.
                   8427: Назовите это транспортное средство тремя словами.
                   8428: 
                   8429: Ответ:
                   8430: Огромный седой вертолет.
                   8431: 
                   8432: Комментарий:
                   8433: Голы Тарасенко, по мнению автора статьи, не похожи друг на друга, как
                   8434: снежинки. В песне из спектакля "День радио" есть строки "Над костром
                   8435: пролетает снежинка, как огромный седой вертолет".
                   8436: 
                   8437: Источник:
                   8438:    1. http://www.sports.ru/tribuna/blogs/centre/709069.html
                   8439:    2. http://www.muzland.ru/songs.html?auth=244&song=5
                   8440: 
                   8441: Автор:
                   8442: Сергей Коновалов (Санкт-Петербург)
                   8443: 
                   8444: Вопрос 20:
                   8445: В вопросе словами "СДЕЛАЛ ЭТО" заменены два других слова.
                   8446:    Автор вопроса получил небольшую травму и впервые за 6,5 лет СДЕЛАЛ
                   8447: ЭТО. Назовите литературного персонажа, который не впервые СДЕЛАЛ ЭТО и
                   8448: понял, что страшно устал, но его воля окрепла и на сердце стало легче.
                   8449: 
                   8450: Ответ:
                   8451: Фродо Бэггинс.
                   8452: 
                   8453: Комментарий:
                   8454: Автор вопроса повредил безымянный палец и вынужден был снять обручальное
                   8455: кольцо, которое до этого носил не снимая.
                   8456: 
                   8457: Источник:
                   8458:    1. ЛОАВ.
                   8459:    2. http://www.flibusta.net/b/220980/read
                   8460: 
                   8461: Автор:
                   8462: Сергей Коновалов (Санкт-Петербург)
                   8463: 
                   8464: Вопрос 21:
                   8465: Англоязычное идиоматическое выражение "СДЕЛАТЬ ЭТО" с середины XX века
                   8466: использовалось в среде любителей бейсбола для того, чтобы отметить
                   8467: хорошую игру игрока обороняющейся команды. В настоящее время фраза
                   8468: "СДЕЛАТЬ ЭТО" используется в смысле "достичь грандиозного и неожиданного
                   8469: успеха" или "решить очень сложную задачу". Назовите человека, которому
                   8470: удалось первым СДЕЛАТЬ ЭТО и не погибнуть.
                   8471: 
                   8472: Ответ:
                   8473: [Бенджамин] Франклин.
                   8474: 
                   8475: Комментарий:
                   8476: Выражение - to catch lightning in a bottle - поймать молнию в бутылку,
                   8477: которое, собственно, изначально и пошло от его опыта с лейденской банкой
                   8478: и атмосферным электричеством. Подобный опыт пытался провести Рихман, но
                   8479: погиб от удара шаровой молнией.
                   8480: 
                   8481: Источник:
                   8482:    1. http://en.wiktionary.org/wiki/lightning_in_a_bottle
                   8483:    2. http://en.wikipedia.org/wiki/Leo_Durocher
                   8484:    3. http://ru.wikipedia.org/wiki/Франклин,_Бенджамин
                   8485:    4. http://ru.wikipedia.org/wiki/Рихман,_Георг_Вильгельм
                   8486: 
                   8487: Автор:
1.2       rubashki 8488: Иван Беляев (Архангельск)
1.1       rubashki 8489: 
                   8490: Вопрос 22:
                   8491: Мастерская в городе Петрозаводске, занимающаяся производством
                   8492: средневековых предметов быта аутентичным способом из аутентичных
                   8493: материалов, называется "Как спичка". Какие три слова мы заменили в
                   8494: вопросе двумя?
                   8495: 
                   8496: Ответ:
                   8497: Кожа да кости.
                   8498: 
                   8499: Комментарий:
                   8500: Логика замены: выражения, обозначающие худобу: "Худой, как спичка" и
                   8501: "Худой - кожа да кости".
                   8502: 
                   8503: Источник:
                   8504:    1. https://vk.com/kozhadakosti
                   8505:    2. http://dic.academic.ru/dic.nsf/dic_synonims/179561/
                   8506: 
                   8507: Автор:
                   8508: Сергей Коновалов (Санкт-Петербург)
                   8509: 
                   8510: Вопрос 23:
                   8511: Сергей Шнуров, комментируя происходящую, по его мнению, девальвацию
                   8512: патриотизма, сказал, что патриотизм стал "предметом" массовой
                   8513: культуры... оболочка стала внутренним содержанием, и сравнил российский
                   8514: патриотизм с ИКСОМ, который также очень часто используют не по
                   8515: назначению. Мы не спрашиваем, какие слова были заменены на ИКС. Назовите
                   8516: основанную в 1948 году компанию, производящую популярные в России ИКСЫ.
                   8517: 
                   8518: Ответ:
                   8519: "Adidas".
                   8520: 
                   8521: Комментарий:
                   8522: Патриотизм, по мнению Шнурова, стал подобен спортивному костюму, который
                   8523: предназначен для тренировок и занятий спортом, а используется многими в
                   8524: нашей стране в качестве повседневной одежды.
                   8525: 
                   8526: Источник:
                   8527:    1. http://lenta.ru/news/2014/10/06/leningrad/
                   8528:    2. http://ru.wikipedia.org/wiki/Adidas
                   8529: 
                   8530: Автор:
                   8531: Сергей Коновалов (Санкт-Петербург)
                   8532: 
                   8533: Вопрос 24:
                   8534: Внимание, в вопросе есть замена.
                   8535:    Несмелый герой Джоан Роулинг сравнивает первую скамью в церкви с
1.10      rubashki 8536: первыми местами на ОЛИВЬЕ, где нужно принимать на себя все обрывы и
1.1       rubashki 8537: виражи. Что мы заменили на "ОЛИВЬЕ"?
                   8538: 
                   8539: Ответ:
                   8540: Американские горки.
                   8541: 
                   8542: Зачет:
                   8543: Русские горки.
                   8544: 
                   8545: Комментарий:
                   8546: "Оливье" - салат, называемый в Европе русским, как и американские горки,
                   8547: однако герой сравнил скамью с более привычным понятием.
                   8548: 
                   8549: Источник:
                   8550: Дж. Роулинг. Случайная вакансия.
1.10      rubashki 8551: http://www.e-reading.link/bookreader.php/1015597/Rouling_Dzhoan_-_Sluchaynaya_vakansiya.html
1.1       rubashki 8552: 
                   8553: Автор:
                   8554: Екатерина Макоед (Архангельск)
                   8555: 
                   8556: Тур:
                   8557: 20 тур. "Ночь реабилитации" (Москва - Санкт-Петербург)
                   8558: 
1.5       rubashki 8559: Редактор:
                   8560: 1-12 - Владислав Король; 13-24 - Серафим Шибанов
                   8561: 
1.1       rubashki 8562: Вопрос 1:
                   8563: В стихотворении Арсения Тарковского ОНИ ходят "по слепым глазам
                   8564: старухи". Назовите ИХ.
                   8565: 
                   8566: Ответ:
                   8567: Мухи, мухи, мухи.
                   8568: 
                   8569: Зачет:
                   8570: Мухи.
                   8571: 
                   8572: Комментарий:
                   8573: Стихотворение Тарковского называется "Портрет".
                   8574: 
                   8575: Источник:
                   8576: Арсений Тарковский. Портрет. http://www.stihi-rus.ru/1/Tarkovskiy/76.htm
                   8577: 
                   8578: Автор:
                   8579: Владислав Король
                   8580: 
                   8581: Вопрос 2:
                   8582: Внимание, в вопросе есть замена.
                   8583:    Представители МЧС отметили, что памятник комсомольцам Сталинграда
                   8584: обрушился из-за сильного ветра и красной плесени. Какие два слова
                   8585: заменены в тексте вопроса?
                   8586: 
                   8587: Ответ:
                   8588: Коррозия металла.
                   8589: 
                   8590: Комментарий:
                   8591: Сам памятник не стальной, а бронзовый - проржавели поддерживающие его
                   8592: металлические конструкции. "Красная плесень" и "Коррозия металла" -
                   8593: популярные группы.
                   8594: 
                   8595: Источник:
                   8596: http://ria.ru/incidents/20090419/168625659.html
                   8597: 
                   8598: Автор:
                   8599: Владислав Король
                   8600: 
                   8601: Вопрос 3:
                   8602: Дуплет.
                   8603:    1. Состоящий из десяти треков альбом "Некрофилия" группы "Сексуальные
                   8604: меньшинства" имеет подзаголовок "ТАКАЯ десятка". Какая - ТАКАЯ?
                   8605:    2. В каком белорусском городе был записан этот альбом?
                   8606: 
                   8607: Ответ:
                   8608:    1. Холодная.
                   8609:    2. Могилев.
                   8610: 
                   8611: Комментарий:
                   8612:    1. В противовес многочисленным "горячим десяткам".
                   8613:    2. Информация с обложки компакт-диска.
                   8614: 
                   8615: Источник:
                   8616: http://www.lebedev.com/MusicPhone/Review/golubitsky/sm-disk.htm
                   8617: 
                   8618: Автор:
                   8619: Владислав Король
                   8620: 
                   8621: Вопрос 4:
                   8622: Прожив чуть более 17 тысяч дней и ночей, в 2007 году известный человек
                   8623: СДЕЛАЛ ЭТО. Жан-Бедель Бокасса СДЕЛАЛ ЭТО, пообщавшись с Муаммаром
                   8624: Каддафи. Какие два слова заменены словами "СДЕЛАЛ ЭТО"?
                   8625: 
                   8626: Ответ:
                   8627: Принял ислам.
                   8628: 
                   8629: Комментарий:
                   8630: "Двадцать тысяч дней и ночей пройдет - человек родился, человек умрет".
                   8631: После смертей Александра Литвиненко и Ильи Кормильцева (автора текста
                   8632: песни "20.000") словосочетание "принять ислам" в значении "умереть"
                   8633: стало мемом. После встречи с ливийским лидером в сентябре 1976 года
                   8634: Бокасса принял ислам, однако спустя три месяца снова стал добрым
                   8635: католиком.
                   8636: 
                   8637: Источник:
                   8638:    1. http://ru.wikipedia.org/wiki/Кормильцев,_Илья_Валерьевич
                   8639:    2. http://www.nautilus.ru/SONGS/S820.shtml
                   8640:    3. http://lurkmore.to/Принял_ислам
                   8641:    4. http://ru.wikipedia.org/wiki/Бокасса,_Жан-Бедель
                   8642: 
                   8643: Автор:
                   8644: Владислав Король, Серафим Шибанов
                   8645: 
                   8646: Вопрос 5:
                   8647: В одной из песен с альбома "Некрофилия" ПРОПУСК поясняется словами
                   8648: "живая ты не в кайф". У этого вопроса тоже ПРОПУСК. Заполните пропуск
                   8649: тремя словами.
                   8650: 
                   8651: Ответ:
                   8652: Позишн намбер файф.
                   8653: 
                   8654: Зачет:
                   8655: Position Number Five.
                   8656: 
                   8657: Комментарий:
                   8658: Сама песня является творческим переосмыслением нетленной работы Кая
                   8659: Метова "Position Number Two".
                   8660: 
                   8661: Источник:
                   8662:    1. http://www.lebedev.com/MusicPhone/Review/golubitsky/gol03.htm
                   8663:    2. http://www.lebedev.com/MusicPhone/Review/golubitsky/smd/02-position-number-five-zhivaja-ty-ne-v-kaif.mp3
                   8664: 
                   8665: Автор:
                   8666: Серафим Шибанов
                   8667: 
                   8668: Вопрос 6:
                   8669: [Ведущему: немного выделить слово "помешала".]
                   8670:    Внимание, словами "АЛЬФА" и "БЕТА" заменены два других слова.
                   8671:    Героям одного романа, путешествовавшим из столицы в провинцию,
                   8672: устроиться на ночлег помешала АЛЬФА, увившая фасад здания гостиницы.
                   8673: Один из них потребовал, чтобы фасад был непременно увит БЕТОЙ. Назовите
                   8674: АЛЬФУ и БЕТУ в любом порядке.
                   8675: 
                   8676: Ответ:
                   8677: Повилика, жимолость.
                   8678: 
                   8679: Комментарий:
                   8680: Во время другого путешествия из столицы в провинцию звучали слова:
                   8681: "Приготовленную таким образом смесь надо двадцать минут помешивать
                   8682: веткой жимолости. Иные, правда, утверждают, что в случае необходимости
                   8683: можно жимолость заменить повиликой. Это неверно и преступно". Некоторые
                   8684: литературоведы считают этот фрагмент поэмы Венедикта Ерофеева отсылкой к
                   8685: эпизоду романа Джерома К. Джерома (перевод 1958 года).
                   8686: 
                   8687: Источник:
                   8688: Джером К. Джером. Трое в лодке (не считая собаки), перевод Э. Линецкой и
                   8689: М. Донского. http://www.flibusta.net/b/345347/read
                   8690: 
                   8691: Автор:
                   8692: Владислав Король
                   8693: 
                   8694: Вопрос 7:
                   8695: Узнав о планируемом телемосте между мэром Киева и одним из депутатов
                   8696: российской Государственной Думы, автор вопроса заметил, что ОН вышел на
                   8697: новый технологический уровень. Назовите ЕГО тремя словами.
                   8698: 
                   8699: Ответ:
                   8700: Бокс по переписке.
                   8701: 
                   8702: Комментарий:
                   8703: Депутат Николай Валуев и мэр Виталий Кличко должны были поговорить о
                   8704: политике. Однако телемост не состоялся.
                   8705: 
                   8706: Источник:
                   8707:    1. ЛОАВ.
                   8708:    2. http://lenta.ru/news/2014/09/19/valuev/
                   8709: 
                   8710: Автор:
                   8711: Владислав Король
                   8712: 
                   8713: Вопрос 8:
                   8714: В романе Йозефа Рота "Марш Радецкого" посетитель трактира заметил, что
                   8715: ЕГО "белое одеяние... казалось пробитым бесчисленными дробинками". Чье
                   8716: одеяние?
                   8717: 
                   8718: Ответ:
                   8719: [Государь император] Франц-Иосиф [Первый].
                   8720: 
                   8721: Комментарий:
                   8722: В романе "Марш Радецкого" описано начало Первой мировой войны в
                   8723: Австро-Венгрии. Как и в более известном романе, висевший в трактире
                   8724: портрет государя императора был засижен мухами. Но Ярослав Гашек написал
                   8725: об этом раньше.
                   8726: 
                   8727: Источник:
                   8728: Йозеф Рот. Марш Радецкого. http://www.flibusta.net/b/175490/read
                   8729: 
                   8730: Автор:
                   8731: Владислав Король
                   8732: 
                   8733: Вопрос 9:
                   8734: Дмитрий Галковский пишет, что русские издавна предпочитали квас, репу,
                   8735: капусту и ржаной хлеб, стремясь опираться не на внешний мир, а на
                   8736: внутренний. Следствием этого стало первенство в некой области. Назовите
                   8737: уроженца Волынской губернии, с которым связано это первенство.
                   8738: 
                   8739: Ответ:
                   8740: Сергей Королёв.
                   8741: 
                   8742: Комментарий:
                   8743: В философском романе "Бесконечный тупик" известный социогуманитарный
                   8744: мыслитель отметил, что "природная склонность к реактивной форме
                   8745: движения" привела к тому, что русские первыми вышли в космос.
                   8746: 
                   8747: Источник:
                   8748: Дмитрий Галковский. Бесконечный тупик (примечание к N 902).
                   8749: http://www.samisdat.com/3/311-902.htm
                   8750: 
                   8751: Автор:
                   8752: Владислав Король, Серафим Шибанов
                   8753: 
                   8754: Вопрос 10:
                   8755: В романтической комедии "Частная школа" урок по теме "Механика полового
                   8756: акта" ведет преподавательница с подходящей фамилией. Назовите
                   8757: произведение английской литературы, несколько героев которого носят
                   8758: фамилию, созвучную с фамилией преподавательницы.
                   8759: 
                   8760: Ответ:
                   8761: "Ромео и Джульетта".
                   8762: 
                   8763: Комментарий:
                   8764: Половой акт - копуляция. Фамилия героини Сильвии Кристель - Копулетта
                   8765: (Copuletta).
                   8766: 
                   8767: Источник:
                   8768: Фильм "Private School".
                   8769: 
                   8770: Автор:
                   8771: Владислав Король
                   8772: 
                   8773: Вопрос 11:
                   8774: Жак часто развлекался, кидая из окна золотые монеты, а его младший брат
                   8775: Ахмед как-то получил на день рождения автомобиль "Порше". Назовите их
                   8776: фамилию.
                   8777: 
                   8778: Ответ:
                   8779: Бокасса.
                   8780: 
                   8781: Комментарий:
                   8782: Большинство из более чем сорока детей эксцентричного диктатора носит
                   8783: французские имена, а вот Ахмед родился в тот короткий период, когда
                   8784: Бокасса был мусульманином.
                   8785: 
                   8786: Источник:
                   8787: http://gazeta.aif.ru/_/online/aif/1085/06_01
                   8788: 
                   8789: Автор:
                   8790: Владислав Король
                   8791: 
                   8792: Вопрос 12:
                   8793: Герой песни Валерия Зуйкова "Нюхаю трусы" уверен, что его супруга на
                   8794: курорте чувствует себя отлично и на НИХ не смотрит. Назовите ИХ.
                   8795: 
                   8796: Ответ:
                   8797: Часы.
                   8798: 
                   8799: Комментарий:
                   8800: Счастливые часов не наблюдают.
                   8801: 
                   8802: Источник:
                   8803: Песня Валерия Зуйкова "Нюхаю трусы".
                   8804: 
                   8805: Автор:
                   8806: Владислав Король
                   8807: 
                   8808: Вопрос 13:
                   8809: Согласно одной шутке, основанной на недавних наблюдениях, озвучку
                   8810: главного злодея мультфильма "Пингвины Мадагаскара" стоило поручить
                   8811: британскому певцу. Какому?
                   8812: 
                   8813: Ответ:
                   8814: Seal.
                   8815: 
                   8816: Зачет:
                   8817: Сил.
                   8818: 
                   8819: Комментарий:
                   8820: Seal - это морской котик на английском. Последние исследования доказали,
                   8821: что морские котики проявляют к пингвинам своеобычный интерес.
                   8822: 
                   8823: Источник:
                   8824:    1. http://samir-befish.livejournal.com/937.html
                   8825:    2. http://ru.wikipedia.org/wiki/Пингвины_Мадагаскара
                   8826:    3. http://ru.wikipedia.org/wiki/Seal
                   8827:    4. http://www.bbc.com/earth/story/20141117-why-seals-have-sex-with-penguins
                   8828: 
                   8829: Автор:
                   8830: Серафим Шибанов
                   8831: 
                   8832: Вопрос 14:
                   8833: Город Бобруйск был основан в 1387 году. Помня об этом факте, некоторые
                   8834: местные жители используют эту дату вместо... Чего?
                   8835: 
                   8836: Ответ:
                   8837: 14/88.
                   8838: 
                   8839: Зачет:
                   8840: С чертой и без.
                   8841: 
                   8842: Комментарий:
                   8843: На стикере фанатов бобруйского футбольного клуба вместо традиционного
                   8844: для правой сцены 14/88 крупно написано 13/87.
                   8845: 
                   8846: Источник:
                   8847: Стикер ФК "Белшина" на плафоне эскалатора станции метро "Площадь
                   8848: Восстания" (Санкт-Петербург).
                   8849: 
                   8850: Автор:
                   8851: Владислав Король
                   8852: 
                   8853: Вопрос 15:
                   8854: Один из альбомов группы "Коловрат", известной своими радикальными
                   8855: политическими взглядами, называется "Эра ЕЕ". Вероятно, в течение
                   8856: следующей минуты многие из здесь присутствующих задействуют ЕЕ. Назовите
                   8857: ЕЕ двумя словами, начинающимися на соседние буквы алфавита.
                   8858: 
                   8859: Ответ:
                   8860: Правая рука.
                   8861: 
                   8862: Комментарий:
                   8863: Вскинутая вверх правая рука считается нацистским приветствием. При
                   8864: записи вопроса многие из вас задействуют правую руку, так как правшей по
                   8865: статистике больше.
                   8866: 
                   8867: Источник:
                   8868:    1. http://ru.wikipedia.org/wiki/Эра_правой_руки
                   8869:    2. http://ru.wikipedia.org/wiki/Коловрат_(группа)
                   8870:    3. http://ru.wikipedia.org/wiki/Правша
                   8871: 
                   8872: Автор:
                   8873: Серафим Шибанов
                   8874: 
                   8875: Вопрос 16:
                   8876: Председатель Московской областной организации общества "Динамо" Виктор
                   8877: Пауков говорит, что под его руководством существенно повысился средний
                   8878: уровень результатов подмосковных спортсменов. Например, лыжников, часами
                   8879: плутающих по НЕМУ, в командах, тащемта, уже нет. Назовите ЕГО двумя
                   8880: словами.
                   8881: 
                   8882: Ответ:
                   8883: Химкинский лес.
                   8884: 
                   8885: Комментарий:
                   8886: Рок-музыкант Паук, баллотировавшийся на пост мэра Химок, в свое время
                   8887: поддержал вырубку местного леса, обещая на его месте сделать русский
                   8888: Лас-Вегас.
                   8889: 
                   8890: Источник:
                   8891: Интервью с председателем Московской областной организации ОГО ВФСО
                   8892: "Динамо", начальником ГУ МВД России по Московской области
                   8893: генерал-майором полиции Виктором Кузьмичом Пауковым, журнал "Динамо",
                   8894: январь 2015 г.
                   8895: 
                   8896: Автор:
                   8897: Антон Снятковский
                   8898: 
                   8899: Вопрос 17:
                   8900: (pic: 20150174.jpg)
                   8901:    Перед вами инструкция, следуя которой вы сможете в буквальном смысле
                   8902: СДЕЛАТЬ ЭТО. Ответьте двумя словами, начинающимися на соседние буквы
                   8903: алфавита, что такое "СДЕЛАТЬ ЭТО".
                   8904: 
                   8905: Ответ:
                   8906: Склеить телку.
                   8907: 
                   8908: Комментарий:
                   8909: Бумажную телку.
                   8910: 
                   8911: Источник:
                   8912: (pic: 20150175.jpg)
                   8913: 
                   8914: Автор:
                   8915: Серафим Шибанов
                   8916: 
                   8917: Вопрос 18:
                   8918: Дуплет.
                   8919:    1. На гербе Гвинеи, действовавшем до 1993 года, можно было найти ЕГО.
                   8920: Против другого ЕГО в свое время выступили участники "Союза офицеров
                   8921: России". Назовите ЕГО двумя словами.
                   8922:    2. Судя по тексту известной песни, ОНИ были у юных распутных девиц. В
                   8923: фильме 2001 года каждая из НИХ стоила 50 долларов США. Назовите ИХ тремя
                   8924: словами.
                   8925: 
                   8926: Ответ:
                   8927:    1. Зеленый слоник.
                   8928:    2. Пять бутылок водки.
                   8929: 
                   8930: Комментарий:
                   8931: В фильме Светланы Басковой офицеры ведут себя несколько сомнительно. В
                   8932: первой части вопроса речь идет о песне Андрея Алексина, в которых есть
                   8933: строчки "Было к вечеру у них / две дешевых шоколадки, / пять бутылок на
                   8934: троих". Автор предполагает, что речь идет именно о бутылках водки,
                   8935: поскольку дальше есть строчки "Где-то в загородной бане / телки водку
                   8936: будут жрать". Во второй части имеется в виду другой фильм Светланы
                   8937: Басковой.
                   8938: 
                   8939: Источник:
                   8940:    1. http://en.wikipedia.org/wiki/Coat_of_arms_of_Guinea
                   8941:    2. http://www.ridus.ru/news/150661 со ссылкой на
                   8942: https://vk.com/wall-7660419_503
                   8943:    3. http://www.karaoke.ru/song/4644.htm
                   8944:    4. http://www.imdb.com/title/tt2653232/
                   8945: 
                   8946: Автор:
                   8947: Серафим Шибанов, идея дуплета - Владислав Король
                   8948: 
                   8949: Вопрос 19:
                   8950:    <раздатка>
                   8951:    По рублених стiнах бiгали червонi вiдблиски вогню, що палав у камiнi,
                   8952: в кованих канделябрах горiли свiчки. Бiля широкого лiжка стояли два
                   8953: дерев'яних стiльцi та маленький стiл пiд темним сукном. На стiльцi перед
                   8954: вогнем сидiв гном, який вже скинув дощовика. У кiмнатi пахло його мiцним
                   8955: тютюном.
                   8956:    &nbsp;
                   8957:    Перу какого писателя принадлежит отрывок, который мы перевели на
                   8958:    мову?
                   8959:    </раздатка>
                   8960: 
                   8961: Ответ:
                   8962: Ника Перумова.
                   8963: 
                   8964: Комментарий:
                   8965: Это была загадка в духе "на "Ля" начинается...". Начинается на "Перу",
                   8966: кончается на "мову". Отрывок из романа "Алмазный меч, деревянный меч".
                   8967: 
                   8968: Источник:
                   8969:    1. Ник Перумов. Эльфийский клинок.
                   8970: http://books.rusf.ru/xussr_mr/perumn04/perumn04.3.html
                   8971:    2. Перевод Владислава Короля.
                   8972: 
                   8973: Автор:
                   8974: Владислав Король, Серафим Шибанов
                   8975: 
                   8976: Вопрос 20:
                   8977: На днях наркополицейские привлекли к ответственности москвича,
                   8978: торговавшего запрещенными препаратами. В сейфе задержанного нашли
                   8979: марихуану, гашиш и амфетамины. Напишите трехзначный пароль, который тот
                   8980: установил на свой сейф.
                   8981: 
                   8982: Ответ:
                   8983: 228.
                   8984: 
                   8985: Комментарий:
                   8986: Именно такой номер имеет статья, предусматривающая ответственность за
                   8987: оборот наркотиков.
                   8988: 
                   8989: Источник:
                   8990: http://www.fsknmsk.ru/news/release/parol-228.html
                   8991: 
                   8992: Автор:
                   8993: Серафим Шибанов
                   8994: 
                   8995: Вопрос 21:
                   8996: Прослушайте четверостишие:
                   8997:    ... Чтоб расширялася ОНА
                   8998:    В два полушария округлых,
                   8999:    Где дверь запретная видна
                   9000:    Пленительно в долинах смуглых...
                   9001:    Назовите ЕЕ.
                   9002: 
                   9003: Ответ:
                   9004: Спина.
                   9005: 
                   9006: Комментарий:
                   9007: Данное стихотворение Михаила Кузмина, сами понимаете о чем.
                   9008: 
                   9009: Источник:
                   9010: http://az.gay.ru/authors/russian/kuzmin.html
                   9011: 
                   9012: Автор:
                   9013: Серафим Шибанов
                   9014: 
                   9015: Вопрос 22:
                   9016: Блиц.
                   9017:    1. Сайт edim-doma.ru [едим дома точка ру] говорит, что для
                   9018: бостонского ЕГО понадобятся: 50 г сахара, 3 яичных желтка, 20 г муки, 20
                   9019: г кукурузного крахмала, 300 мл молока, 1 чайная ложка ванильной эссенции
                   9020: и бисквит. Назовите ЕГО.
                   9021:    2. Самое длинное ОНО, изготовленное в префектуре Миэ, имеет размер
                   9022: 222 метра и вес 20 килограммов. На шее вряд ли удержишь. Назовите ЕГО
                   9023: двумя словами.
                   9024:    3. В стихотворении Татьяны Матвеевой упоминается ОНА заката.
                   9025: Станислав Малярчук пишет, что некоторые из групп, выступавшие на
                   9026: Вудстоке, исчезли в НЕЙ кислотного тумана. Назовите ЕЕ двумя словами.
                   9027: 
                   9028: Ответ:
                   9029:    1. Кремовый пирог.
                   9030:    2. Жемчужное ожерелье.
                   9031:    3. Пурпурная дымка.
                   9032: 
                   9033: Источник:
                   9034:    1. http://www.edimdoma.ru/retsepty/49792-bostonskiy-kremovyy-pirog-boston-cream-pie
                   9035:    2. http://lenta.ru/news/2013/04/12/necklace/
                   9036:    3. https://vk.com/wall-59758826_355
                   9037:    4. http://www.jazzquad.ru/index.pl?act=PRODUCT&id=3275
                   9038: 
                   9039: Автор:
                   9040: Серафим Шибанов
                   9041: 
                   9042: Вопрос 23:
                   9043: Фамилия одного из сценаристов фильма, в котором кавалерийский отряд
                   9044: сопровождает пассажиров через земли, кишащие бандитами, совпадает с
                   9045: названием города. Назовите этот город.
                   9046: 
                   9047: Ответ:
                   9048: Лион.
                   9049: 
                   9050: Комментарий:
                   9051: В фильме действие крутится вокруг дилижанса. Сам фильм называется
                   9052: "Дилижанс ярости" (впрочем, знать это не нужно). Сценаристом был Эрл
                   9053: Лион, такой вот вышел "лионский дилижанс". :-)
                   9054: 
                   9055: Источник:
                   9056: http://www.kinonews.ru/movie_73207/stagecoach-to-fury
                   9057: 
                   9058: Автор:
                   9059: Серафим Шибанов
                   9060: 
                   9061: Вопрос 24:
                   9062: [Ведущему: слово "гоккун" обязательно повторить по буквам как указано.]
                   9063:    Японское слово "гоккун" [Георгий, Олег, Константин, Кузьма, Устин,
                   9064: Николай] является звукоподражанием глотку. Ответьте словом, также
                   9065: содержащим удвоенную "К", разновидностью чего является гоккун.
                   9066: 
                   9067: Ответ:
                   9068: Буккакэ.
                   9069: 
                   9070: Зачет:
                   9071: Буккакэру, буккейк; также с "е" вместо "э".
                   9072: 
                   9073: Комментарий:
                   9074: Неспроста все шесть имен мужские.
                   9075: 
                   9076: Источник:
                   9077:    1. http://ru.wikipedia.org/wiki/Гоккун
                   9078:    2. http://ru.wikipedia.org/wiki/Буккакэ
                   9079: 
                   9080: Автор:
                   9081: Серафим Шибанов
                   9082: 
                   9083: Тур:
                   9084: 21 тур. "ЗаПущинная наука" (Пущино)
                   9085: 
                   9086: Вопрос 1:
                   9087: Хотя на дворе ночь, расслабляться не время.
                   9088:    Цитата с сайта bash.im: "Я пролила ИКС на ноутбук, теперь он не
                   9089: уходит в ЗЕТ". Какое слово мы заменили на ИКС, а какие два слова - на
                   9090: ЗЕТ?
                   9091: 
                   9092: Ответ:
                   9093: Кофе, спящий режим.
                   9094: 
                   9095: Источник:
                   9096: http://bash.im/quote/422320/
                   9097: 
                   9098: Автор:
                   9099: Данила Аладин (Пущино)
                   9100: 
                   9101: Вопрос 2:
                   9102: Во Вьетнаме водители грузовиков часто прикрепляют светоотражающие знаки
                   9103: на передний бампер. С большой популярностью какого транспорта это
                   9104: связано?
                   9105: 
                   9106: Ответ:
                   9107: Мотоциклетного.
                   9108: 
                   9109: Зачет:
                   9110: По упоминанию мотоцикла, мотороллера, велосипеда.
                   9111: 
                   9112: Комментарий:
                   9113: Чтобы встречные не приняли грузовик за два мотоцикла и не попытались
                   9114: "проскочить" между ними.
                   9115: 
                   9116: Источник:
                   9117: ЛНА.
                   9118: 
                   9119: Автор:
                   9120: Данила Аладин (Пущино)
                   9121: 
                   9122: Вопрос 3:
                   9123: Закончите стихотворение-порошок двумя словами, начинающимися на одну и
                   9124: ту же букву:
                   9125:    в своей дилемме о футболе
                   9126:    и сексе с ольгой на столе
                   9127:    влад предпочтенье отдал ...
                   9128:    ...
                   9129: 
                   9130: Ответ:
                   9131: оле оле
                   9132: 
                   9133: Источник:
                   9134: https://vk.com/sandalporoshki?w=wall-31481258_91608
                   9135: 
                   9136: Автор:
                   9137: Данила Аладин (Пущино)
                   9138: 
                   9139: Вопрос 4:
                   9140: (pic: 20150176.jpg)
                   9141:    Два слова, которые мы от вас скрыли, обозначают всемирную проблему.
                   9142: Напишите эти слова.
                   9143: 
                   9144: Ответ:
                   9145: Глобальное потепление.
                   9146: 
                   9147: Источник:
                   9148: http://demotivatorium.ru/demotivators/d/43473/
                   9149: 
                   9150: Автор:
                   9151: Алексей Дубровский (Пущино)
                   9152: 
                   9153: Вопрос 5:
                   9154: Закончите двумя словами цитату с сайта bash.im: "Накупили продуктов.
                   9155: Российская капуста, израильская морковка, голландский лук, английская
                   9156: свекла, египетская картошка, белорусское сало. Так вот ты какой - ...".
                   9157: 
                   9158: Ответ:
                   9159: "... украинский борщ".
                   9160: 
                   9161: Источник:
                   9162: http://bash.im/quote/427173/
                   9163: 
                   9164: Автор:
                   9165: Данила Аладин (Пущино)
                   9166: 
                   9167: Вопрос 6:
                   9168: На одной картинке человек раскрывает печенья с предсказаниями. Лежащие
                   9169: перед ним на столе бумажки из печений складываются в слова
                   9170: стихотворения-порошка, первые три строчки которого такие:
                   9171:    сегодня ждите неудачу
                   9172:    забудьте слово хорошо
                   9173:    несчастье будет бесконечным
                   9174:    Ответьте, что написано на четвертой, не до конца извлеченной из
                   9175: печенья, бумажке.
                   9176: 
                   9177: Ответ:
                   9178: вам жо
                   9179: 
                   9180: Источник:
                   9181: (pic: 20150177.jpg)
                   9182: 
                   9183: Автор:
                   9184: Алексей Дубровский (Пущино)
                   9185: 
                   9186: Вопрос 7:
                   9187: В 2006 году в городе Аракатака прошел референдум. Предлагалось изменить
                   9188: название города, но, к сожалению, явка не достигла необходимого минимума
                   9189: в 7400 человек, хотя 90% проголосовавших были не против добавить в свою
                   9190: жизнь магического реализма. Какое название должен был получить город?
                   9191: 
                   9192: Ответ:
                   9193: Макондо.
                   9194: 
                   9195: Комментарий:
                   9196: Аракатака - родной город Гарсиа Маркеса. Магический реализм -
                   9197: литературное направление, в котором работал нобелевский лауреат.
                   9198: 
                   9199: Источник:
                   9200: http://www.mk.ru/social/article/2014/04/18/1016713-7-faktov-iz-zhizni-gabrielya-garsia-markesa-korrespondent-v-moskve-zapret-na-vezd-v-ssha-i-obet-molchaniya.html
                   9201: 
                   9202: Автор:
                   9203: Евгения Крутинина (Пущино)
                   9204: 
                   9205: Вопрос 8:
                   9206: Внимание, в вопросе заменена одна буква.
                   9207:    Один из товарищей по команде автора вопроса на фестивале "ишАчил", а
                   9208: другой - "ажИл". Мы не спрашиваем вас, кто из них чем занимался.
                   9209: Ответьте, какое слово сокращают до замененной буквы.
                   9210: 
                   9211: Ответ:
                   9212: Жюри.
                   9213: 
                   9214: Комментарий:
                   9215: Один из участников команды входил в ИЖ фестиваля, другой - в АЖ,
                   9216: соответственно родились неологизмы "ИЖачил" и "АЖил".
                   9217: 
                   9218: Источник:
                   9219: ЛНА.
                   9220: 
                   9221: Автор:
                   9222: Никита Поздняков (Пущино)
                   9223: 
                   9224: Вопрос 9:
                   9225: (pic: 20150178.jpg)
                   9226:    Перед вами самый расистский из антирасистских плакатов. Скрытая от
                   9227: вас надпись гласит: "Внутри мы все ПЕРПЕНДИКУЛЯРНЫЕ". Какое слово мы
                   9228: заменили?
                   9229: 
                   9230: Ответ:
                   9231: Нормальные.
                   9232: 
                   9233: Комментарий:
                   9234: Под "нормальным" автор плаката понимает белый цвет кожи. :-)
                   9235: 
                   9236: Источник:
                   9237: http://joyreactor.cc/post/1753904
                   9238: 
                   9239: Автор:
                   9240: Алексей Дубровский (Пущино)
                   9241: 
                   9242: Вопрос 10:
                   9243: Согласно шутке с сайта bash.im, "глава российского государства, который
                   9244: завел себе блог, войдет в историю как Дмитрий". Какие две буквы латиницы
                   9245: мы пропустили?
                   9246: 
                   9247: Ответ:
                   9248: LJ.
                   9249: 
                   9250: Комментарий:
                   9251: LJ Дмитрий - как обладатель Живого Журнала.
                   9252: 
                   9253: Источник:
                   9254: http://bash.im/quote/407684/
                   9255: 
                   9256: Автор:
                   9257: Данила Аладин (Пущино)
                   9258: 
                   9259: Вопрос 11:
                   9260: (pic: 20150179.jpg)
                   9261:    Автор вопроса считает, что в дудле от 9 сентября 2014 года большая
                   9262: буква "G", как ни странно, очень напоминает нимб. Кого мы от вас скрыли?
                   9263: 
                   9264: Ответ:
                   9265: Льва Толстого.
                   9266: 
                   9267: Комментарий:
                   9268: Странно, что художник нарисовал букву "G", напоминающую нимб, несмотря
                   9269: на то что Толстой был отлучен от церкви.
                   9270: 
                   9271: Источник:
                   9272: http://www.ruscur.ru/themes/0/00/57/5751.shtml?news/0/05/21/52180
                   9273: 
                   9274: Автор:
                   9275: Мария Томашевская (Пущино)
                   9276: 
                   9277: Вопрос 12:
                   9278: Компания "Valtec" торгует инженерной сантехникой: трубами, счетчиками
                   9279: для воды... К своему большому разочарованию, автор вопроса не нашел в их
                   9280: каталоге ЭТО. Ответьте абсолютно точно, какие два слова мы заменили на
                   9281: "ЭТО".
                   9282: 
                   9283: Ответ:
                   9284: Водяной чип.
                   9285: 
                   9286: Комментарий:
                   9287: Главый квест в культовой игре "Fallout" - найти водяной чип,
                   9288: произведенный компанией "Vault-Tec".
                   9289: 
                   9290: Источник:
                   9291:    1. http://www.valtec.ru
                   9292:    2. http://ru.fallout.wikia.com/wiki/Волт-Тек
                   9293: 
                   9294: Автор:
                   9295: Данила Аладин (Пущино)
                   9296: 
                   9297: Вопрос 13:
                   9298: Вот и прозвучала половина вопросов тура.
                   9299:    Прослушайте перевод отрывка одной из песен британской группы "Arena":
                   9300: ""Возьми ребенка. Он твой!" - единственное, что она могла сказать...".
                   9301: Какое имя собственное носит эта песня?
                   9302: 
                   9303: Ответ:
                   9304: "Соломон".
                   9305: 
                   9306: Комментарий:
                   9307: В тексте песни обыгрывается известная притча о младенце и двух женщинах,
                   9308: в которой Соломон предложил рассечь ребенка пополам.
                   9309: 
                   9310: Источник:
                   9311: http://www.mp3lemon.net/lyric/50955/
                   9312: 
                   9313: Автор:
                   9314: Данила Аладин (Пущино)
                   9315: 
                   9316: Вопрос 14:
                   9317: (pic: 20150180.jpg)
                   9318:    Назовите музыкальный инструмент, который мы от вас скрыли.
                   9319: 
                   9320: Ответ:
                   9321: Балалайка.
                   9322: 
                   9323: Комментарий:
                   9324: Вот такой вот "Битлз" по-русски.
                   9325: 
                   9326: Источник:
                   9327: http://kunst-camera.livejournal.com/24238.html
                   9328: 
                   9329: Автор:
                   9330: Данила Аладин (Пущино)
                   9331: 
                   9332: Вопрос 15:
                   9333: Блиц. В рассказе Акутагавы Рюноскэ "О-Гин" упоминаются заимствованные
                   9334: имена и понятия.
                   9335:    1. Как по-русски мы называем того, кто в рассказе упоминается как
                   9336: Сан-Дзеан Батиста?
                   9337:    2. Как по-русски мы называем того, кто в рассказе упоминается как
                   9338: Габуриэру?
                   9339:    3. Как по-русски мы называем то, что в рассказе упоминается как
                   9340: сагурамэнто?
                   9341: 
                   9342: Ответ:
                   9343:    1. Иоанн Креститель.
                   9344:    2. Гавриил.
                   9345:    3. Крещение.
                   9346: 
                   9347: Зачет:
                   9348:    3. Причастие.
                   9349: 
                   9350: Комментарий:
                   9351: Рассказ повествует о первых христианах в Японии. И японские термины суть
                   9352: искажения португальских, т.к. именно они первыми из европейцев
                   9353: взаимодействовали с японцами.
                   9354: 
                   9355: Источник:
                   9356: Акутагава Рюноскэ. О-Гин. http://www.flibusta.net/b/121845/read
                   9357: 
                   9358: Автор:
                   9359: Данила Аладин (Пущино)
                   9360: 
                   9361: Вопрос 16:
                   9362: (pic: 20150181.jpg)
                   9363:    Воспроизведите надпись под демотиватором, несильно отличающуюся от
                   9364: устойчивого выражения.
                   9365: 
                   9366: Ответ:
                   9367: Пирамида непотребств [Маслоу].
                   9368: 
                   9369: Зачет:
                   9370: Пирамида непотребностей [Маслоу].
                   9371: 
                   9372: Источник:
                   9373: http://demotivators.to/p/968817/piramida-nepotrebstv-maslou.htm
                   9374: 
                   9375: Автор:
                   9376: Алексей Дубровский (Пущино)
                   9377: 
                   9378: Вопрос 17:
                   9379: Внимание, в вопросе есть замена.
                   9380:    На обложке книжки "Жесткий тайм-менеджмент" циферблаты часов
                   9381: заключены в ножницы. Какое слово мы заменили?
                   9382: 
                   9383: Ответ:
                   9384: Наручники.
                   9385: 
                   9386: Комментарий:
                   9387: (pic: 20150182.jpg)
                   9388: 
                   9389: Источник:
                   9390: http://www.livelib.ru/book/1000596852
                   9391: 
                   9392: Автор:
                   9393: Ирина Алисова (Пущино)
                   9394: 
                   9395: Вопрос 18:
                   9396: Автор вопроса - не сторонник садомазохизма, но статья в журнале "Maxim"
                   9397: его заинтересовала. В начале статьи журналист сокрушается: "Пусть ...
                   9398: люди стали в целом добрее друг к другу, зато из их жизни ушла
                   9399: изобретательность", а затем предлагает вспомнить лучшие из хитроумных и
                   9400: жестоких пыток древности. Названием статьи стала русская пословица из
                   9401: четырех слов, в которой заменили первую букву. Как называется эта
                   9402: статья?
                   9403: 
                   9404: Ответ:
                   9405: "Боль на выдумки хитра".
                   9406: 
                   9407: Источник:
                   9408: "Maxim", май 2007 г. - С. 120.
                   9409: 
                   9410: Автор:
                   9411: Олег Холодов (Серпухов)
                   9412: 
                   9413: Вопрос 19:
                   9414: Приведенная на одном из рисунков пирамида потребностей содержит, как ни
                   9415: странно, четыре уровня, верхний из которых называется "Уничтожение
                   9416: человечества". Всё дело в том, что вместо правильного содержания второго
                   9417: сверху уровня на рисунке показаны белые буквы на синем фоне. Третий же и
                   9418: четвертый уровни этой пирамиды пункта приведены в соответствие ИМ.
                   9419: Назовите ИХ тремя словами.
                   9420: 
                   9421: Ответ:
                   9422: Три законами робототехники.
                   9423: 
                   9424: Зачет:
                   9425: Три закона Азимова.
                   9426: 
                   9427: Комментарий:
                   9428: (pic: 20150183.jpg)
                   9429:    Белые буквы на синем фоне обозначают своего рода сбой мозга робота
                   9430: при попытке выполнить первый закон робототехники.
                   9431: 
                   9432: Источник:
                   9433: http://lol24.ee/post/101059/
                   9434: 
                   9435: Автор:
                   9436: Никита Поздняков (Пущино)
                   9437: 
                   9438: Вопрос 20:
                   9439: (pic: 20150184.jpg)
                   9440:    Назовите двумя словами животное, изображение которого мы от вас
                   9441: скрыли.
                   9442: 
                   9443: Ответ:
                   9444: Большая панда.
                   9445: 
                   9446: Источник:
                   9447: http://pikabu.ru/story/tatu_1752435
                   9448: 
                   9449: Автор:
                   9450: Мария Томашевская (Пущино)
                   9451: 
                   9452: Вопрос 21:
                   9453: В фильме "Внутри Льюина Дэвиса" с главным героем путешествует кот. После
                   9454: множества приключений кот возвращается к хозяевам. Напишите состоящую из
                   9455: одного слова кличку кота.
                   9456: 
                   9457: Ответ:
                   9458: Одиссей.
                   9459: 
                   9460: Зачет:
                   9461: Улисс.
                   9462: 
                   9463: Источник:
                   9464: Х/ф "Внутри Льюина Дэвиса".
                   9465: 
                   9466: Автор:
                   9467: Мария Томашевская (Пущино)
                   9468: 
                   9469: Вопрос 22:
                   9470: (pic: 20150185.jpg)
                   9471:    Восстановите скрытое от вас указание из пяти слов.
                   9472: 
                   9473: Ответ:
                   9474: После свадьбы читать снизу вверх.
                   9475: 
                   9476: Источник:
                   9477: http://demotivators.to/p/345533/posle-svadbyi-chitat-snizu-vverh.htm
                   9478: 
                   9479: Автор:
                   9480: Алексей Дубровский (Пущино)
                   9481: 
                   9482: Вопрос 23:
                   9483: Любимым увлечением его детства стали воздушные змеи, которых он сам
                   9484: мастерил. Запустив в небо очередное творение, мальчик по нитке отправлял
                   9485: "почту" - спичечный коробок с живым тараканом. Когда ему исполнилось 22
                   9486: года, он построил первую в мире центробежную машину и записал в
                   9487: дневнике: "Вес рыжего таракана был увеличен в 300 раз, а вес цыпленка -
                   9488: в 10, без малейшего для них вреда". Назовите этого человека.
                   9489: 
                   9490: Ответ:
                   9491: [Константин Эдуардович] Циолковский.
                   9492: 
                   9493: Источник:
                   9494: "Maxim", май 2007 г. - С. 170.
                   9495: 
                   9496: Автор:
                   9497: Олег Холодов (Серпухов)
                   9498: 
                   9499: Вопрос 24:
                   9500: "ИКСЫ и ИГРЕКИ" - круглосуточный канал телевизионных игр и викторин.
                   9501: Осенью 2014 года от многих из вас также требовали ИКСЫ, а зимой 2015
                   9502: года от вас ждут уже ИГРЕКОВ. Назовите ИКС и ИГРЕК в правильном порядке.
                   9503: 
                   9504: Ответ:
                   9505: Вопрос, ответ.
                   9506: 
                   9507: Источник:
                   9508: http://tv.yandex.ru/5/channels/333
                   9509: 
                   9510: Автор:
                   9511: Олег Холодов (Серпухов)
                   9512: 
                   9513: Тур:
                   9514: 22 тур. "Несомненный скрамасакс" (Москва)
                   9515: 
1.5       rubashki 9516: Редактор:
                   9517: Андрей Алдашев - автор всех вопросов тура; Серафим Шибанов - редактор
                   9518: 
1.1       rubashki 9519: Вопрос 1:
                   9520: Музыканты московской алко-трэш-метал-группы называют себя
                   9521: "собутыльниками Дьявола". Если не считать пробел, название группы
                   9522: состоит из 11 символов. Напишите это название.
                   9523: 
                   9524: Ответ:
                   9525: "Портвейн 666".
                   9526: 
                   9527: Комментарий:
                   9528: Более известен, конечно, портвейн "777".
                   9529: 
                   9530: Источник:
                   9531:    1. http://www.portwine666.com/band/
                   9532:    2. http://vk.com/portwine666
                   9533: 
                   9534: Автор:
                   9535: Андрей Алдашев (Москва)
                   9536: 
                   9537: Вопрос 2:
                   9538: В заголовке о продолжении рекламной акции "Зимний марафон" есть слова
                   9539: "[ПРОПУСК 1]". В Москве 16 декабря 2013 года после ремонта [ПРОПУСК 2].
                   9540: Заполните любой из пропусков.
                   9541: 
                   9542: Ответ:
                   9543: Открылось второе дыхание.
                   9544: 
                   9545: Зачет:
                   9546: Открылось "Второе дыхание".
                   9547: 
                   9548: Комментарий:
                   9549: Культовая московская рюмочная была закрыта, но, как оказалось,
                   9550: ненадолго.
                   9551: 
                   9552: Источник:
                   9553:    1. http://www.tourprom.ru/profi/content/informer/15357/
                   9554:    2. http://msk-zabegalovka.livejournal.com/492224.html
                   9555: 
                   9556: Автор:
                   9557: Андрей Алдашев (Москва)
                   9558: 
                   9559: Вопрос 3:
                   9560: (pic: 20150186.jpg)
                   9561:    Внимание, АССА - это замена.
                   9562:    Телебашню на розданной вам фотографии, как и множество ей подобных,
                   9563: можно сравнить с АССОЙ. Назовите город, где находится эта телебашня.
                   9564: 
                   9565: Ответ:
                   9566: Вена.
                   9567: 
                   9568: Комментарий:
                   9569: АССА - игла. Название одного фильма с Виктором Цоем мы заменили другим.
                   9570: Такая вот игла в Вене.
                   9571: 
                   9572: Источник:
                   9573:    1. http://gjabu.livejournal.com/34048.html
                   9574:    2. http://de.wikipedia.org/wiki/Donauturm
                   9575: 
                   9576: Автор:
                   9577: Андрей Алдашев (Москва)
                   9578: 
                   9579: Вопрос 4:
                   9580: Внимание, в вопросе есть замена.
                   9581:    В Базе Вопросов ЧГК буквосочетание "ХЛЕБ" встречается более 900 раз.
                   9582: Назовите марку ХЛЕБА, которая упоминается при этом чаще всего.
                   9583: 
                   9584: Ответ:
                   9585: Гиннесс.
                   9586: 
                   9587: Комментарий:
                   9588: Согласно анекдоту, если в слове "ХЛЕБ" сделать четыре ошибки, получится
                   9589: "ПИВО". Чаще всего - это рекорд, так что имеет место очередной рекорд
                   9590: Гиннесса.
                   9591: 
                   9592: Источник:
                   9593: http://db.chgk.info/search/questions/пиво
                   9594: 
                   9595: Автор:
                   9596: Андрей Алдашев (Москва)
                   9597: 
                   9598: Вопрос 5:
                   9599: Почти все уже забыли, что в названии предприятия с турецким капиталом
                   9600: когда-то фигурировал князь Рюрик. Какой город, помимо Москвы,
                   9601: упоминается в нем сейчас?
                   9602: 
                   9603: Ответ:
                   9604: Эфес.
                   9605: 
                   9606: Комментарий:
                   9607: В названии ЗАО "Пивоварня Москва-Эфес" недолгое время после ее появления
                   9608: вместо Москвы был князь Рюрик. Оборот "все забыли" намекает на
                   9609: Герострата, который сжег храм Артемиды Эфесской.
                   9610: 
                   9611: Источник:
                   9612: Этикетка "Старого мельника" из коллекции автора вопроса.
                   9613: 
                   9614: Автор:
                   9615: Андрей Алдашев (Москва)
                   9616: 
                   9617: Вопрос 6:
                   9618: На этикетке одного сорта эля изображены 15 одинаково одетых людей, и
                   9619: специально подчеркнуто, что этот эль - светлый. Для его приготовления
                   9620: использовался хмель, выведенный в стране из Южного полушария. В какой
                   9621: стране?
                   9622: 
                   9623: Ответ:
                   9624: Новая Зеландия.
                   9625: 
                   9626: Комментарий:
                   9627: Пиво носит название "All blacks" в честь знаменитой новозеландской
                   9628: регбийной сборной. В одной из версий этой игры на площадке находится 15
                   9629: игроков. Черной является их форма, и, чтобы не сбить с толку покупателя
                   9630: данного светлого эля, производители указали, что это не "черное" пиво.
                   9631: 
                   9632: Источник:
                   9633:    1. http://www.1tonna.ru/beer/
                   9634:    2. http://www.nubo.ru/pavel_egorov/moscow/m1t1801.jpg
                   9635: 
                   9636: Автор:
                   9637: Андрей Алдашев (Москва)
                   9638: 
                   9639: Вопрос 7:
                   9640: Домашний пивовар Михаил Венёвцев, вероятно, недолго мучился, придумывая
                   9641: название для своего крепкого светлого сорта. Кто изображен на этикетке
                   9642: указанного пива?
                   9643: 
                   9644: Ответ:
                   9645: Фрекен Бок.
                   9646: 
                   9647: Зачет:
                   9648: Кошка Матильда.
                   9649: 
                   9650: Комментарий:
                   9651: Сваренное крепкое пиво относится к типу "бок". Название "Freken Bock"
                   9652: для подобного пива придумать, как нам кажется, несложно. Слова
                   9653: "домашний" и "мучился" - подсказка.
                   9654: 
                   9655: Источник:
                   9656: http://www.nubo.ru/pavel_egorov/moscow/mzv0201.jpg
                   9657: 
                   9658: Автор:
                   9659: Андрей Алдашев (Москва)
                   9660: 
                   9661: Вопрос 8:
                   9662: Назовите марку российского пива, на этикетке которого была телеведущая
                   9663: Маша Малиновская.
                   9664: 
                   9665: Ответ:
                   9666: "Три медведя".
                   9667: 
                   9668: Комментарий:
                   9669: Три медведя и Машенька - прямо как в сказке.
                   9670: 
                   9671: Источник:
                   9672: http://www.nubo.ru/pavel_egorov/russian/rpi0519.jpg
                   9673: 
                   9674: Автор:
                   9675: Андрей Алдашев (Москва)
                   9676: 
                   9677: Вопрос 9:
                   9678: Дуплет.
                   9679:    1. Согласно шутке автора вопроса, сидя в жюри телешоу "Голос", Дима
                   9680: Билан услышал свою песню и прокомментировал ее исполнение цитатой. Чьей?
                   9681:    2. Согласно другой версии шутки автора вопроса, один из членов жюри
                   9682: телешоу "Голос" посчитал исполнение участником своей песни настолько
                   9683: дурным, что раскритиковал того цитатой. Какую пьесу она завершает?
                   9684: 
                   9685: Ответ:
                   9686:    1. [Константина] Станиславского.
                   9687:    2. "На дне".
                   9688: 
                   9689: Комментарий:
                   9690: Билан сказал "Не верю!", а неизвестный член жюри - "Эх... испортил
                   9691: песню... дур-рак!".
                   9692: 
                   9693: Источник:
                   9694:    1. ЛОАВ.
                   9695:    2. http://ru.wikipedia.org/wiki/Не_верю
                   9696:    3. Максим Горький. На дне.
                   9697: http://az.lib.ru/g/gorxkij_m/text_0180.shtml
                   9698: 
                   9699: Автор:
                   9700: Андрей Алдашев (Москва)
                   9701: 
                   9702: Вопрос 10:
                   9703: Ожидая зимой общественного транспорта на остановках, например,
                   9704: Савёловский вокзал или Хамовнический вал, автор вопроса характеризовал
                   9705: состояние окружающей среды известной цитатой. Воспроизведите ее двумя
                   9706: словами.
                   9707: 
                   9708: Ответ:
                   9709: Ветер северный.
                   9710: 
                   9711: Комментарий:
                   9712: Названия остановок звучат похоже на "Владимирский централ", а зимой
                   9713: весьма прохладно. "Окружающей" - небольшая подсказка.
                   9714: 
                   9715: Источник:
                   9716:    1. ЛОАВ.
                   9717:    2. Михаил Круг - Владимирский централ.
                   9718: 
                   9719: Автор:
                   9720: Андрей Алдашев (Москва)
                   9721: 
                   9722: Вопрос 11:
                   9723: Певец Валентин Стрыкало стал известен благодаря Интернету. Закончите
                   9724: одним словом строку из его песни: "Я ухожу к другому, его зовут...".
                   9725: 
                   9726: Ответ:
                   9727: "... Рустем".
                   9728: 
                   9729: Комментарий:
                   9730: В этой строчке можно усмотреть намек на блогера Рустема Адагамова,
                   9731: известного под ником drugoi.
                   9732: 
                   9733: Источник:
                   9734:    1. Валентин Стрыкало - Рустем.
                   9735:    2. http://drugoi.livejournal.com/profile
                   9736: 
                   9737: Автор:
                   9738: Андрей Алдашев (Москва)
                   9739: 
                   9740: Вопрос 12:
1.11      rubashki 9741: [Каждый из бланков раздатки должен быть порван пополам на месте пробела
                   9742: между словами "Saxifraga" и "adscendens".]
1.1       rubashki 9743:    <раздатка>
                   9744:    Saxifraga adscendens
                   9745:    </раздатка>
                   9746:    Лирический герой группы "Иван-царевич", собираясь в путешествие,
                   9747: сулящее богатство, намерен запастись АЛЬФОЙ. Назовите АЛЬФУ.
                   9748: 
                   9749: Ответ:
                   9750: Разрыв-трава.
                   9751: 
                   9752: Комментарий:
                   9753: Разорванная раздатка должна служить намеком. Растение Saxifraga
                   9754: adscendens, по одной из версий, является прототипом разрыв-травы из
                   9755: сказок.
                   9756: 
                   9757: Источник:
                   9758:    1. Иван-царевич - На острове Буян.
                   9759:    2. http://ru.wikipedia.org/wiki/Камнеломка
                   9760: 
                   9761: Автор:
                   9762: Андрей Алдашев (Москва)
                   9763: 
                   9764: Вопрос 13:
                   9765: Цитата из журнала "Вокруг света": "Вначале НОЖКА была острой палкой,
                   9766: потом шилом, потом в ее истории случилась ПОДНОЖКА". Что мы заменили
                   9767: словами "НОЖКА" и "ПОДНОЖКА"?
                   9768: 
                   9769: Ответ:
                   9770: Вилка, развилка.
                   9771: 
                   9772: Источник:
                   9773: "Вокруг света", 2013, N 6. - С. 28.
                   9774: 
                   9775: Автор:
                   9776: Андрей Алдашев (Москва)
                   9777: 
                   9778: Вопрос 14:
                   9779: Известный в Советском Союзе "ИКС" скалил зубы, в том числе, и на
                   9780: Гитлера. По некоторым источникам, ИКС Гитлера в 2012 году жил в Москве.
                   9781: Назовите ИКСА.
                   9782: 
                   9783: Ответ:
                   9784: Крокодил.
                   9785: 
                   9786: Комментарий:
                   9787: Крокодилы живут по сотне лет, так что "личный крокодил Гитлера" по
                   9788: кличке Сатурн, привезенный в Московский зоопарк, вероятно, жив до сих
                   9789: пор. Сатирический журнал "Крокодил" неоднократно изображал Гитлера на
                   9790: карикатурах.
                   9791: 
                   9792: Источник:
                   9793:    1. http://www.journal-club.ru/?q=image/tid/88
                   9794:    2. http://www.aif.ru/society/article/51397
                   9795: 
                   9796: Автор:
                   9797: Андрей Алдашев (Москва)
                   9798: 
                   9799: Вопрос 15:
                   9800: Однажды автор вопроса обратил внимание на дату упаковки и СДЕЛАЛ ЭТО.
                   9801: Назовите супруга героини, которая СДЕЛАЛА ЭТО на дынных грядках.
                   9802: 
                   9803: Ответ:
                   9804: Наг.
                   9805: 
                   9806: Комментарий:
                   9807: СДЕЛАТЬ ЭТО - отложить яйца. Автор вопроса не постеснялся сказать об
                   9808: этом. Имя кобры из рассказа Киплинга в разных переводах пишется
                   9809: по-разному, поэтому мы решили спросить ее мужа.
                   9810: 
                   9811: Источник:
                   9812:    1. ЛОАВ.
                   9813:    2. Р. Киплинг. Рикки-Тикки-Тави.
                   9814: http://www.flibusta.net/b/190177/read
                   9815: 
                   9816: Автор:
                   9817: Андрей Алдашев (Москва)
                   9818: 
                   9819: Вопрос 16:
                   9820: "Восемь гусениц" - это пример ЗАМЕНЫ. Управляемые силой мысли ЗАМЕНЫ в
                   9821: XXI веке перестают быть фантастикой. Ответьте одним словом, что мы в
                   9822: обоих случаях обозначили как ЗАМЕНЫ.
                   9823: 
                   9824: Ответ:
                   9825: Протезы.
                   9826: 
                   9827: Комментарий:
                   9828: Протезы используются как замены конечностей. А еще мы проиллюстрировали
                   9829: лингвистическое явление протезы, когда перед одним звуком появляется
                   9830: другой для удобства произношения.
                   9831: 
                   9832: Источник:
                   9833:    1. http://ru.wikipedia.org/wiki/Протеза
                   9834:    2. http://ru.wikipedia.org/wiki/Протезирование
                   9835: 
                   9836: Автор:
                   9837: Андрей Алдашев (Москва)
                   9838: 
                   9839: Вопрос 17:
                   9840: В СССР было [ПРОПУСК 1] для поставок их продукции в необходимых для
                   9841: [ПРОПУСК 2] объемах. ПРОПУСК 1 и ПРОПУСК 2 малоразличимы на слух.
                   9842: Заполните оба пропуска.
                   9843: 
                   9844: Ответ:
                   9845: "... мало козоводов...", "... молокозаводов...".
                   9846: 
                   9847: Источник:
                   9848: http://www.fermer.ru/book/export/html/15170
                   9849: 
                   9850: Автор:
                   9851: Андрей Алдашев (Москва)
                   9852: 
                   9853: Вопрос 18:
                   9854: Изначально словом "кобан" обозначались будки, в которых располагался
                   9855: ЁКАРНЫЙ БАБАЙ. Напишите два слова, которые мы заменили словами "ЁКАРНЫЙ
                   9856: БАБАЙ".
                   9857: 
                   9858: Ответ:
                   9859: Японский городовой.
                   9860: 
                   9861: Комментарий:
                   9862: Одно экспрессивное восклицание мы заменили другим.
                   9863: 
                   9864: Источник:
                   9865: http://ru.wikipedia.org/wiki/Кобан_(полицейский_участок)
                   9866: 
                   9867: Автор:
                   9868: Андрей Алдашев (Москва)
                   9869: 
                   9870: Вопрос 19:
                   9871: В песне украинской группы "Лихолесье" медведи по утрам делают то же, что
                   9872: и один из героев Антуана де Сент-Экзюпери. Намеком на попытки
                   9873: российского руководства прекратить подачу газа это, однако, не является.
                   9874: Ответьте двумя словами, что они делают.
                   9875: 
                   9876: Ответ:
                   9877: Гасят фонари.
                   9878: 
                   9879: Зачет:
                   9880: Синонимичные ответы с указанием фонарей.
                   9881: 
                   9882: Комментарий:
                   9883: Упомянутый герой - не сам Маленький принц, а Фонарщик.
                   9884: 
                   9885: Источник:
                   9886:    1. http://www.liholesie.ru/publ/statia_marchen
                   9887:    2. А. Сент-Экзюпери. Маленький Принц.
                   9888: http://www.flibusta.net/b/79276/read
                   9889: 
                   9890: Автор:
                   9891: Андрей Алдашев (Москва)
                   9892: 
                   9893: Вопрос 20:
                   9894: Дуплет.
                   9895:    1. На гербе небольшого российского города изображены представители
                   9896: бобовых, а название, по одной версии, означает "укрытие на
                   9897: возвышенности". Назовите этот город.
                   9898:    2. Назовите российский город, в котором действует пивоварня Sheep
                   9899: Scull Brewery [шип скалл брЮэри].
                   9900: 
                   9901: Ответ:
                   9902:    1. Гороховец.
                   9903:    2. Череповец.
                   9904: 
                   9905: Комментарий:
                   9906: В слове Гороховец некоторые исследователи выделяют корень "-хов-",
                   9907: означающий "прятать" (сравните "ховать").
                   9908: 
                   9909: Источник:
                   9910:    1. http://www.biografia.ru/about/moskva02.html
                   9911:    2. https://vk.com/sheep_skull_brewery_club
                   9912: 
                   9913: Автор:
                   9914: Андрей Алдашев (Москва)
                   9915: 
                   9916: Вопрос 21:
                   9917: Улица Космонавтов ТАМ, вероятно, названа в честь МирОслава
                   9918: ГермашЕвского. Московская улица Космонавтов находится не ТАМ. Ответьте
                   9919: двумя словами, где ТАМ.
                   9920: 
                   9921: Ответ:
                   9922: В Люблине.
                   9923: 
                   9924: Комментарий:
                   9925: Мирослав Гермашевский - пока единственный польский космонавт. А топонимы
                   9926: типа Люблино склоняются.
                   9927: 
                   9928: Источник:
                   9929:    1. Карты Google Москвы и Люблина.
                   9930:    2. http://ru.wikipedia.org/wiki/Гермашевский,_Мирослав
                   9931: 
                   9932: Автор:
                   9933: Андрей Алдашев (Москва)
                   9934: 
                   9935: Вопрос 22:
                   9936: Знакомый автора вопроса назвал День работников СИЗО и тюрем России
                   9937: иноязычным словом, в котором выделил три последние буквы. Напишите
                   9938: получившееся название.
                   9939: 
                   9940: Ответ:
                   9941: ХэллоУИН.
                   9942: 
                   9943: Зачет:
                   9944: ХеллоУИН.
                   9945: 
                   9946: Комментарий:
                   9947: День работников СИЗО и тюрем приходится на 31 октября, как и Хэллоуин.
                   9948: УИН - Управление исполнения наказаний, к которому относятся СИЗО и
                   9949: тюрьмы в регионе России.
                   9950: 
                   9951: Источник:
                   9952: Шутка знакомого автора вопроса.
                   9953: 
                   9954: Автор:
                   9955: Андрей Алдашев (Москва)
                   9956: 
                   9957: Вопрос 23:
                   9958: Дуплет.
                   9959:    1. Гаубица - это и оружие супергероя, и название украинской компании,
                   9960: предоставляющей тариф "Киноман". Какие три буквы мы заменили в
                   9961: предыдущем предложении?
                   9962:    2. Украинский психолог предостерегает, что "Град" или "Смерч" могут
                   9963: привести к появлению магического стража. Какие три буквы мы заменили в
                   9964: предыдущем предложении?
                   9965: 
                   9966: Ответ:
                   9967:    1. П, т, н.
                   9968:    2. П, н, х.
                   9969: 
                   9970: Зачет:
                   9971: В любом порядке, в любом регистре.
                   9972: 
                   9973: Комментарий:
                   9974:    1. Паутина, а не гаубица. Супергерой - Человек-паук, да и для
                   9975: интернет-провайдера подходящее название.
                   9976:    2. Не "магический страж", а "панический страх". Панический страх
                   9977: нередко охватывает новобранцев при длительном обстреле.
                   9978: 
                   9979: Источник:
                   9980:    1. http://www.pautina.ch.ua/tarifs.html
                   9981:    2. http://www.ukrinform.ua/rus/news/v_boy___bez_paniki_psihologi_uchat_preodolevat_boevoy_stress_1686443
                   9982: 
                   9983: Автор:
                   9984: Андрей Алдашев (Москва)
                   9985: 
                   9986: Вопрос 24:
                   9987: Вероятно, что самоубийство стало для НЕГО поводом расслабиться. Назовите
                   9988: ЕГО двумя словами, начинающимися на одни и те же пять букв.
                   9989: 
                   9990: Ответ:
                   9991: Сфинктер Сфинкса.
                   9992: 
                   9993: Комментарий:
                   9994: Теперь все могут расслабиться.
                   9995: 
                   9996: Источник:
                   9997: Общие соображения.
                   9998: 
                   9999: Автор:
                   10000: Андрей Алдашев (Москва)
                   10001: 
                   10002: Тур:
                   10003: 23 тур. "Яблоко в штанах" (Воронеж - Омск - Смоленск)
                   10004: 
                   10005: Инфо:
                   10006: Команда благодарит за помощь в подготовке пакета Константина Науменко
                   10007: (Киев), Юрия Мотькина (Самара), Юлию Лободу (Томск), Сергея Терентьева
                   10008: (Санкт-Петербург), Максима Мерзлякова и команду "Сфинкс" (Воронеж).
                   10009: 
                   10010: Вопрос 1:
                   10011: Заголовок заметки о рейтинге самых употребляемых слов русского
                   10012: разговорного языка короче известной пословицы на одно слово. Напишите
                   10013: эту пословицу.
                   10014: 
                   10015: Ответ:
                   10016: Первый блин комом.
                   10017: 
                   10018: Комментарий:
                   10019: Слово "блин" является самым употребительным в русской разговорной речи.
                   10020: Заголовок "Первый - блин".
                   10021: 
                   10022: Источник:
                   10023: "Русский репортер", 2014, N 23.
                   10024: 
                   10025: Автор:
                   10026: Александр Нечаев (Воронеж)
                   10027: 
                   10028: Вопрос 2:
                   10029: В этом вопросе мы использовали четыре источника.
                   10030:    В музее Кьярамонти ОНА держит в руке рог изобилия. Другая ОНА
                   10031: установлена в американском университете Пердью. Назовите ЕЕ двумя
                   10032: словами.
                   10033: 
                   10034: Ответ:
                   10035: Статуя Нила.
                   10036: 
                   10037: Комментарий:
                   10038: Разливы Нила были рогом изобилия для Египта. А в университете Пердью
                   10039: находится статуя Нила Армстронга. Фонтан четырех рек символизирует
                   10040: четыре источника, одним из которых является Нил.
                   10041: 
                   10042: Источник:
                   10043:    1. http://ru.wikipedia.org/wiki/Музей_Кьярамонти
                   10044:    2. http://www.russianla.com/common/humor/story.php/474033
                   10045:    3. http://ru.wikipedia.org/wiki/Армстронг,_Нил
                   10046:    4. http://ru.wikipedia.org/wiki/Фонтан_Четырёх_рек
                   10047: 
                   10048: Автор:
                   10049: Александр Сидоренков (Смоленск)
                   10050: 
                   10051: Вопрос 3:
                   10052: Лозунг одной социальной рекламы гласит: "Не торопись за решетку, ДЕЛАЙ
                   10053: ЭТО", а сама решетка отбрасывает тень в виде ИКСА. Мы не спрашиваем, что
                   10054: такое "ДЕЛАТЬ ЭТО". Назовите ИКС двумя словами.
                   10055: 
                   10056: Ответ:
                   10057: Пешеходный переход.
                   10058: 
                   10059: Комментарий:
                   10060: Свет, прошедший сквозь решетку, проецируется на дорогу как "зебра"
                   10061: пешеходного перехода. ДЕЛАТЬ ЭТО - пропускать пешехода.
                   10062: 
                   10063: Источник:
                   10064: http://www.sostav.ru/publication/prizrak-rebenka-predupredil-toroplivykh-voditelej-5605.html
                   10065: 
                   10066: Автор:
                   10067: Александр Нечаев (Воронеж)
                   10068: 
                   10069: Вопрос 4:
                   10070: К вам обращается автор вопроса.
                   10071:    В 2014 году я (а возможно, и кто-то из вас) отыграл несколько
                   10072: вопросов с ответом "АЛЬФА". Ограниченность тем в вопросах сегодняшнего
                   10073: ЧГК ужасно расстраивает. Вас ведь тоже? Из-за этого построить какие-то
                   10074: новые ходы в этом вопросе не получилось. Простите.
                   10075:    Назовите АЛЬФУ.
                   10076: 
                   10077: Ответ:
                   10078: Четвертая стена.
                   10079: 
                   10080: Комментарий:
                   10081: "Четвертая стена" - изначально театральный термин, обозначающий
                   10082: воображаемую "стену" между актерами и зрителями, позднее
                   10083: распространившийся на другие виды искусств. В тексте вопроса автор
                   10084: всячески (насколько это возможно в отношении автора и игроков) пытается
                   10085: эту стену сломать, прямо обращаясь к игрокам от лица игрока.
                   10086: "Ограниченность", "построить" - подсказки. Как и номер вопроса.
                   10087: 
                   10088: Источник:
                   10089: ЛОАВ.
                   10090: 
                   10091: Автор:
                   10092: Дмитрий Тарарыков (Москва - Воронеж)
                   10093: 
                   10094: Вопрос 5:
                   10095: (pic: 20150187.jpg)
                   10096:    Перед вами английское название популярного советского фильма, в
                   10097: котором полосками закрыты одни и те же четыре буквы. Назовите эти буквы.
                   10098: 
                   10099: Ответ:
                   10100: trip.
                   10101: 
                   10102: Комментарий:
                   10103: Фильм - "Полосатый рейс", а "stripe" в переводе с английского - полоска.
                   10104: 
                   10105: Источник:
                   10106: http://en.wikipedia.org/wiki/Striped_Trip
                   10107: 
                   10108: Автор:
                   10109: Андрей Яблонских (Воронеж)
                   10110: 
                   10111: Вопрос 6:
                   10112: ОН был придуман в швейцарской компании. На постере документального
                   10113: фильма о нем изображены 26 актеров. Назовите ЕГО точно.
                   10114: 
                   10115: Ответ:
                   10116: Шрифт "Гельветика".
                   10117: 
                   10118: Зачет:
                   10119: Гельветика, Гельвеция, Neue Haas Grotesk, Нойе Гельветика.
                   10120: 
                   10121: Комментарий:
                   10122: На постере изображены 26 букв английского алфавита, а сам шрифт назван в
                   10123: честь древнего латинского названия Швейцарии. Образец шрифта
                   10124: "Гельветика" вы могли видеть на раздатке к прошлому вопросу.
                   10125: 
                   10126: Источник:
                   10127:    1. http://ru.wikipedia.org/wiki/Гельветика
                   10128:    2. http://en.wikipedia.org/wiki/Helvetica_(film)
                   10129: 
                   10130: Автор:
                   10131: Андрей Кокуленко (Омск)
                   10132: 
                   10133: Вопрос 7:
                   10134: На карикатуре, посвященной ЭТОМУ, белый медведь использует скотч.
                   10135: Назовите ЭТО двумя словами.
                   10136: 
                   10137: Ответ:
                   10138: Глобальное потепление.
                   10139: 
                   10140: Зачет:
                   10141: Таяние льдин, таяние льдов, таяние ледников.
                   10142: 
                   10143: Комментарий:
                   10144: Скотчем медведь пытается "склеить" таящие льдины.
                   10145: 
                   10146: Источник:
                   10147: http://9gag.com/gag/aeNB4m5
                   10148: 
                   10149: Автор:
                   10150: Аркадий Илларионов (Воронеж)
                   10151: 
                   10152: Вопрос 8:
                   10153: Женщинам в Объединенных Арабских Эмиратах запретят делать макияж, а
                   10154: мужчинам - надевать куфию. Есть, пить и писАть тоже скоро будет нельзя.
                   10155: В тексте вопроса мы пропустили название, получившее известность с 1928
                   10156: года. Напишите это название.
                   10157: 
                   10158: Ответ:
                   10159: "За рулем".
                   10160: 
                   10161: Источник:
                   10162:    1. http://news.bigmir.net/life/808171-Jenshinam-v-OAE-zapretyat-krasitsya-za-rylem-a-myjchinam-odevat-obrych
                   10163:    2. http://ru.wikipedia.org/wiki/За_рулём
                   10164: 
                   10165: Автор:
                   10166: Александр Нечаев (Воронеж)
                   10167: 
                   10168: Вопрос 9:
                   10169: [Ведущему: "Лу Ю" прочитать отчетливо раздельно.]
                   10170:    В этом вопросе слово "АЛЬФА" заменяет другое слово.
                   10171:    Согласно "Чайному канону" Лу Ю АЛЬФА должна быть столь же блестящей,
                   10172: как сугроб, и столь же роскошной, как весенний лотос. АЛЬФА входит в
                   10173: название французского романа. Какого?
                   10174: 
                   10175: Ответ:
                   10176: "Пена дней".
                   10177: 
                   10178: Комментарий:
                   10179: В "Чайном каноне" упоминается поэма, которая говорит о пене как о
                   10180: пламенеющем блеске. "Пена дней" - наиболее известный из романов
                   10181: французского писателя Бориса Виана.
                   10182: 
                   10183: Источник:
                   10184:    1. Лу Ю. Чайный канон.
                   10185: http://www.e-reading.link/bookreader.php/1015344/Lu_-_Chaynyy_Kanon.html
                   10186:    2. http://ru.wikipedia.org/wiki/Пена_дней
                   10187: 
                   10188: Автор:
                   10189: Михаил Эктов (Воронеж)
                   10190: 
                   10191: Вопрос 10:
                   10192: (pic: 20150188.jpg)
                   10193:    [Ведущему: голосом слегка выделить "кораллово".]
                   10194:    Звезды на Аллее славы сделаны из кораллово-розового композитного
                   10195: материала. Мы не спрашиваем фамилию актера, звезда которого была открыта
                   10196: 18 августа 1997 года. Назовите его имя.
                   10197: 
                   10198: Ответ:
                   10199: Патрик.
                   10200: 
                   10201: Комментарий:
                   10202: Патрик Стар (персонаж мультсериала "Губка Боб Квадратные Штаны") -
                   10203: розовая морская звезда. 18 августа 1997 года была открыта звезда Патрика
                   10204: Суэйзи.
                   10205: 
                   10206: Источник:
                   10207:    1. http://ru.wikipedia.org/wiki/Голливудская_%C2%ABАллея_славы%C2%BB
                   10208:    2. http://www.imdb.com/event/ev0000693/1997
                   10209:    3. http://ru.wikipedia.org/wiki/Патрик_Стар
                   10210: 
                   10211: Автор:
                   10212: Аркадий Илларионов (Воронеж)
                   10213: 
                   10214: Вопрос 11:
                   10215: В романе Стивена Кинга герой проверяет сочинения своих учеников.
                   10216: Отмечая, что один из учеников очень сильно нажимает на ручку, он
                   10217: упоминает фамилию. Какую?
                   10218: 
                   10219: Ответ:
                   10220: Брайль.
                   10221: 
                   10222: Комментарий:
                   10223: Выдавленные строки сочинения можно было читать тактильным способом, как
                   10224: если бы оно было написано шрифтом Брайля.
                   10225: 
                   10226: Источник:
                   10227: Стивен Кинг. 11/22/63.
                   10228: 
                   10229: Автор:
                   10230: Андрей Яблонских (Воронеж)
                   10231: 
                   10232: Вопрос 12:
                   10233: [Ведущему: немного выделить слова "Мюнхена" и "ребенок".]
                   10234:    Брайан Аттбери считает, что название этих персонажей связано с гербом
                   10235: Мюнхена, на котором изображен ребенок. Назовите их аналог в
                   10236: произведениях советского писателя.
                   10237: 
                   10238: Ответ:
                   10239: Жевуны.
                   10240: 
                   10241: Комментарий:
                   10242: Они - манчкины. Возможно, "манчкин" произошло из немецкого слова,
                   10243: означающего "карлик", "маленькое существо". Семья Баума имела немецкие
                   10244: корни. Аттбери предполагает, что писатель мог видеть репродукцию герба в
                   10245: детстве. Munch по-английски - жевать.
                   10246: 
                   10247: Источник:
                   10248:    1. http://en.wikipedia.org/wiki/Munchkin#Origin_of_the_term
                   10249:    2. http://ru.wikipedia.org/wiki/Манчкин
                   10250: 
                   10251: Автор:
                   10252: Аркадий Илларионов (Воронеж)
                   10253: 
                   10254: Вопрос 13:
                   10255: В уголовном деле по ограблению закусочной полицейские смогли доказать
                   10256: вину подозреваемого с помощью АЛЬФЫ. В статье "Когда в СССР появилась
                   10257: АЛЬФА" упоминаются гирлянды на шее. Назовите АЛЬФУ.
                   10258: 
                   10259: Ответ:
                   10260: Туалетная бумага.
                   10261: 
                   10262: Комментарий:
                   10263: На туалетной бумаге остались следы записки, которой преступник
                   10264: шантажировал администрацию закусочной. В СССР был дефицит туалетной
                   10265: бумаги. Обычно ей торговали по десять штук в одни руки, и покупатели шли
                   10266: по городу с гирляндами на шее.
                   10267: 
                   10268: Источник:
                   10269:    1. http://www.newsru.com/crime/30dec2014/robtoiletpaperus.html
                   10270:    2. http://silver-fancy.livejournal.com/206741.html
                   10271: 
                   10272: Автор:
                   10273: Александр Нечаев (Воронеж)
                   10274: 
                   10275: Вопрос 14:
                   10276: Хирург Пирогов, пребывая в Риге, выкроил одному цирюльнику новый ИКС.
                   10277: Хотя АЛЬФА вдохновила многих, сам классик хронологически никак не мог
                   10278: выкроить ИКС из АЛЬФЫ. Мы не просим назвать классика. Назовите ИКС и
                   10279: АЛЬФУ.
                   10280: 
                   10281: Ответ:
                   10282: Нос, шинель.
                   10283: 
                   10284: Комментарий:
                   10285: Пирогов выкроил цирюльнику новый нос, хотя читал ли Пирогов Гоголя -
                   10286: автору неизвестно. "Шинель" появилась позже "Носа", поэтому Гоголь не
                   10287: мог сам из нее сделать "Нос". Известно выражение "Все мы вышли из
                   10288: гоголевской шинели".
                   10289: 
                   10290: Источник:
                   10291:    1. http://www.booksite.ru/localtxt/sci/ent/ist/sto/45.htm
                   10292:    2. http://ru.wikipedia.org/wiki/Гоголь,_Николай_Васильевич
                   10293:    3. http://www.bibliotekar.ru/encSlov/3/182.htm
                   10294: 
                   10295: Автор:
                   10296: Александр Сидоренков (Смоленск)
                   10297: 
                   10298: Вопрос 15:
                   10299: Отмечая способности славян к торговле и предпринимательству, Борис
                   10300: Акунин говорит, что даже ЭТО они делали не авансом, а только в случае
                   10301: успеха. "ЭТО" - название фильма 1986 года. Назовите ЭТО сложным словом.
                   10302: 
                   10303: Ответ:
                   10304: Жертвоприношение.
                   10305: 
                   10306: Комментарий:
                   10307: Жертвы богам славяне приносили только когда уже получили их милость.
                   10308: "Жертвоприношение" - фильм Андрея Тарковского.
                   10309: 
                   10310: Источник:
                   10311:    1. Борис Акунин. История государства Российского. Том 1.
                   10312:    2. http://ru.wikipedia.org/wiki/Жертвоприношение_(фильм,_1986)
                   10313: 
                   10314: Автор:
                   10315: Александр Нечаев (Воронеж)
                   10316: 
                   10317: Вопрос 16:
                   10318: Герой Виктора Пелевина, испытывая противоречивые чувства к школьной
                   10319: программе по литературе, сравнивает ее с ИКСОМ. Британские ученые
                   10320: недавно заявили о планах получения ИКСА из генномодифицированных
                   10321: растений. Назовите ИКС двумя словами.
                   10322: 
                   10323: Ответ:
                   10324: Рыбий жир.
                   10325: 
                   10326: Комментарий:
                   10327: Пелевин замечает, что школьная программа по литературе это полезно, но
                   10328: совершенно невкусно.
                   10329: 
                   10330: Источник:
                   10331:    1. Виктор Пелевин. Любовь к трем цукербринам.
                   10332:    2. http://medportal.ru/mednovosti/news/2014/04/17/082fishoil/
                   10333: 
                   10334: Автор:
                   10335: Александр Нечаев (Воронеж)
                   10336: 
                   10337: Вопрос 17:
                   10338: [Ведущему: "ИГРЕКОМ рассказывать о природе" прочитать как единую
                   10339: конструкцию. Не <ИКС с ИГРЕКОМ> <рассказывать о природе>, а <ИКС> с его
                   10340: <ИГРЕКОМ рассказывать о природе>.]
                   10341:    На американской вилле, где собираются русские, Тимофей Пнин сожалеет,
                   10342: что не пришел ИКС с его ИГРЕКОМ рассказывать о природе. Назовите ИКСА и
                   10343: ИГРЕК.
                   10344: 
                   10345: Ответ:
                   10346: Набоков, дар.
                   10347: 
                   10348: Комментарий:
                   10349: Набоков увлекался энтомологией и написал произведение "Дар". "Пнин" -
                   10350: также произведение Набокова. Писатель, как известно, жил и в Америке.
                   10351: 
                   10352: Источник:
                   10353:    1. В. Набоков. Пнин. http://www.flibusta.net/b/158647/read
                   10354:    2. http://ru.wikipedia.org/wiki/Дар_(роман)
                   10355: 
                   10356: Автор:
                   10357: Александр Сидоренков (Смоленск)
                   10358: 
                   10359: Вопрос 18:
                   10360: В стихотворении Натальи Ромашовой про барана с необычной кличкой ПРОПУСК
                   10361: есть такие строчки:
                   10362:    Стоит лишь в разговоре
                   10363:    Ответить: "Да ну!",
                   10364:    И вам сразу припомнят
                   10365:    Знакомство с ПРОПУСК!
                   10366:    Заполните пропуск.
                   10367: 
                   10368: Ответ:
                   10369: Кигну.
                   10370: 
                   10371: Комментарий:
                   10372: Баран Кигну - персонифицированная присказка баранки гну.
                   10373: 
                   10374: Источник:
                   10375: http://www.artlebedev.ru/everything/izdal/baran-kignu/
                   10376: 
                   10377: Автор:
                   10378: Аркадий Илларионов (Воронеж)
                   10379: 
                   10380: Вопрос 19:
                   10381: Уважаемые знатоки, не печальтесь, если не возьмете этот вопрос.
                   10382:    Путешественник Владимир Севриновский так пишет о своем походе к
                   10383: курящемуся вулкану: "Пришлось ползти вниз по едва различимой тропинке. Я
                   10384: то и дело оскальзывался и цеплялся за ветки, щедро ДЕЛАЯ ЭТО". Какое
                   10385: устойчивое словосочетание было заменено словами "ДЕЛАТЬ ЭТО"?
                   10386: 
                   10387: Ответ:
                   10388: Посыпать голову пеплом.
                   10389: 
                   10390: Источник:
                   10391: http://www.vokrugsveta.ru/article/190618/
                   10392: 
                   10393: Автор:
                   10394: Андрей Яблонских (Воронеж)
                   10395: 
                   10396: Вопрос 20:
                   10397: В связи с особенностями ремонта дорог в России дорожное полотно иногда
                   10398: сравнивают с НИМ. ОНО обычно упоминается при описании одного из периодов
                   10399: отечественной истории. Назовите ЕГО двумя словами.
                   10400: 
                   10401: Ответ:
                   10402: Лоскутное одеяло.
                   10403: 
                   10404: Комментарий:
                   10405: Землю в иллюминаторе, дорожное полотно и карты различных государств
                   10406: периода феодальной раздробленности довольно часто сравнивают с лоскутным
                   10407: одеялом.
                   10408: 
                   10409: Источник:
                   10410: http://ru.wikipedia.org/wiki/Лоскутное_одеяло
                   10411: 
                   10412: Автор:
                   10413: Юлия Мещерякова (Воронеж)
                   10414: 
                   10415: Вопрос 21:
                   10416: (pic: 20150189.jpg)
                   10417:    [Ведущему: после слова "аллюзией" сделать смысловую паузу.]
                   10418:    Дуплет.
                   10419:    1. Розданный кадр из эпизода мультсериала "Симпсоны" сопровождает
                   10420: хорошо известная всем вам музыка. В том же эпизоде за открывшейся дверью
                   10421: туалета мы видим человека, держащего в руках... Что?
                   10422:    2. Название упомянутого эпизода является аллюзией с поправкой на
                   10423: характерную особенность персонажей сериала. Напишите это название.
                   10424: 
                   10425: Ответ:
                   10426:    1. Винтовка.
                   10427:    2. "A Clockwork Yellow".
                   10428: 
                   10429: Зачет:
                   10430:    1. М14, автомат, карабин, ружье.
                   10431:    2. "Заводной желтый", "Заводной лимон".
                   10432: 
                   10433: Комментарий:
                   10434: Эпизод целиком посвящен творчеству Стэнли Кубрика. На розданной картинке
                   10435: Гомер одет как персонаж "Заводного апельсина", а поднятая над головой
                   10436: кость и музыка Рихарда Штрауса из симфонической поэмы "Так говорил
                   10437: Заратустра" отсылают к фильму "Космическая одиссея 2001 года". Рядовой
                   10438: Леонард "Куча" Лоуренс, сидящий в туалете с винтовкой, - одна из
                   10439: наиболее известных сцен в фильме Кубрика "Цельнометаллическая оболочка".
                   10440: Название эпизода обыгрывает то, что все персонажи "Симпсонов" имеют кожу
                   10441: желтого цвета, что хорошо видно на раздаточном материале.
                   10442: 
                   10443: Источник:
                   10444: "The Simpsons", s26e04.
                   10445: 
                   10446: Автор:
                   10447: Дмитрий Тарарыков (Москва - Воронеж)
                   10448: 
                   10449: Вопрос 22:
                   10450: Одна из цитат с сайта bash.im предполагает, что полное название
                   10451: известного фильма 2010 года заканчивается словами "... только,
                   10452: пожалуйста, не выкладывай всё это в Инстаграм". Назовите этот фильм.
                   10453: 
                   10454: Ответ:
                   10455: "Ешь, молись, люби".
                   10456: 
                   10457: Комментарий:
                   10458: Фильм снят по мотивам романа Элизабет Гилберт "Есть, молиться, любить"
                   10459: (Eat, Pray, Love; 2006).
                   10460: 
                   10461: Источник:
                   10462: http://bash.im/abyssbest/20150107
                   10463: 
                   10464: Автор:
                   10465: Аркадий Илларионов (Воронеж)
                   10466: 
                   10467: Вопрос 23:
                   10468: Пушпака, колесница Рамачандры, была, наверное, самая быстрая, да и
                   10469: управлялась ЛОЖЬЮ. Что мы заменили ЛОЖЬЮ?
                   10470: 
                   10471: Ответ:
                   10472: Мысль.
                   10473: 
                   10474: Комментарий:
                   10475: Мысль изреченная есть ложь. Чтобы не солгать, слово "мысль" мы в вопросе
                   10476: заменили. Согласно другому известному изречению, самое быстрое - это
                   10477: мысль.
                   10478: 
                   10479: Источник:
                   10480:    1. http://ru.wikipedia.org/wiki/Пушпака
                   10481:    2. http://ru.wikiquote.org/wiki/Мысль
                   10482:    3. http://www.gumer.info/bogoslov_Buks/Philos/Spirk/03.php
                   10483: 
                   10484: Автор:
                   10485: Александр Сидоренков (Смоленск)
                   10486: 
                   10487: Вопрос 24:
                   10488: Внимание, в вопросе есть замены.
                   10489:    На рекламном плакате компании "Jeep" со слоганом "У каждого свой ритм
                   10490: сердца!" изображен ИКС определенной формы. Назовите ИКС двумя словами,
                   10491: одно из которых - английского происхождения.
                   10492: 
                   10493: Ответ:
                   10494: След протектора.
                   10495: 
                   10496: Зачет:
                   10497: Узор протектора, отпечаток протектора и другие синонимичные варианты со
                   10498: словом "протектор".
                   10499: 
                   10500: Комментарий:
                   10501: На плакате изображен след протектора шины в виде кардиограммы.
                   10502: 
                   10503: Источник:
                   10504:    1. http://www.autogrodno.by/stati-partnerov/6826-luchshaya-avtomobilnaya-reklama.html
                   10505:    2. http://ru.wikipedia.org/wiki/Протектор_(элемент_шины)
                   10506: 
                   10507: Автор:
                   10508: Александр Нечаев (Воронеж)
                   10509: 
                   10510: Тур:
                   10511: 24 тур. "Сборная Царицына" (Волгоград)
                   10512: 
                   10513: Вопрос 1:
                   10514: В немецкой мифологии есть злой дух по имени ИКС, который подсовывает
                   10515: рудокопам металл, внешне похожий на медь. Как рассказал Виктор Бочаров,
                   10516: один житель Воронежской области предложил вместо традиционного
                   10517: обозначения ИКСА использовать череп с костями. Назовите ИКС.
                   10518: 
                   10519: Ответ:
                   10520: Никель.
                   10521: 
                   10522: Комментарий:
                   10523: В Воронежской области обнаружены крупнейшие месторождения никеля. Среди
                   10524: жителей Прихоперья распространилось много слухов об опасности этого
                   10525: металла.
                   10526: 
                   10527: Источник:
                   10528: http://www.vlg.aif.ru/society/ecology/1071777
                   10529: 
                   10530: Автор:
                   10531: Роман Мерзляков (Волгоград)
                   10532: 
                   10533: Вопрос 2:
                   10534: Литературовед Вадим Борисов утверждает, что в мандельштамовской строке
                   10535: "Успенье нежное - Флоренция в Москве" дважды упоминается ОНА. Назовите
                   10536: ЕЕ.
                   10537: 
                   10538: Ответ:
                   10539: Марина Цветаева.
                   10540: 
                   10541: Комментарий:
                   10542: "Цветущее" название итальянского города содержит явную отсылку к фамилии
                   10543: Марины Цветаевой. Скрытая аллюзия содержится в упоминании Успенского
                   10544: собора, который построил известный итальянский зодчий Аристотель
                   10545: Фиораванти. По-итальянски fiore - "цветок".
                   10546: 
                   10547: Источник:
                   10548: Л. Видгоф. Французские мотивы. Нестрогие рассуждения о стихах и прозе О.
                   10549: Мандельштама. // "Вопросы литературы", 2013, N 6. - С. 116.
                   10550: 
                   10551: Автор:
                   10552: Роман Мерзляков (Волгоград)
                   10553: 
                   10554: Вопрос 3:
                   10555: У представителей ЭТОГО подошва на левом ботинке из тефлона, а на правом
                   10556: - из пористой резины. Какого деятеля российской истории первой половины
                   10557: XX века упомянул наблюдавший за ЭТИМ журналист?
                   10558: 
                   10559: Ответ:
                   10560: Чапаев.
                   10561: 
                   10562: Комментарий:
                   10563: В керлинге у спортсменов один ботинок служит для хорошего сцепления со
                   10564: льдом, а на тефлоновой подошве, наоборот, можно скользить при запуске
                   10565: камня. Керлинг напоминает не только бильярд на льду, но и такую игру,
                   10566: как "Чапаев".
                   10567: 
                   10568: Источник:
                   10569:    1. http://gazeta.aif.ru/_/online/ss/245/vkl01_01
                   10570:    2. Трансляция кёрлингового матча Россия - Дания, телеканал "Россия
                   10571: 2", 10.02.2014 г.
                   10572: 
                   10573: Автор:
                   10574: Роман Мерзляков (Волгоград)
                   10575: 
                   10576: Вопрос 4:
                   10577: Комментатор хоккейного матча с участием женской олимпийской сборной
                   10578: России указывал, что не всегда наши хоккеистки находят верные решения.
                   10579: При этом, оценивая важную роль в команде игрока под N 8, комментатор
                   10580: вспомнил Константина Бескова. Назовите фамилию игрока N 8.
                   10581: 
                   10582: Ответ:
                   10583: Гаврилова.
                   10584: 
                   10585: Комментарий:
                   10586: Бесков говорил "спартаковцам": "Не знаешь, что делать с мячом, - отдай
                   10587: его Гаврилову". Так же оценивал комментатор роль Ии Гавриловой.
                   10588: 
                   10589: Источник:
                   10590:    1. Трансляция матча Россия - Германия, телеканал "Россия 2",
                   10591: 09.02.2014 г.
                   10592:    2. http://ru.wikipedia.org/wiki/Гаврилова,_Ия_Викторовна
                   10593:    3. http://ru.wikipedia.org/wiki/Гаврилов,_Юрий_Васильевич
                   10594: 
                   10595: Автор:
                   10596: Роман Мерзляков (Волгоград)
                   10597: 
                   10598: Вопрос 5:
                   10599: Говоря о движении прогресса вспять, Умберто Эко предрекает переход на
                   10600: кабельное телевидение. Использование проводов, считает философ,
                   10601: отодвинет нас к эпохе Антонио Меуччи - ученого, изобретшего в 1857 году
                   10602: телефон. Однако до этого, как считает непоследовательный Эко,
                   10603: человечество откатится к другой стадии. С чьим именем он связывает эту
                   10604: стадию?
                   10605: 
                   10606: Ответ:
                   10607: [Гульельмо] Маркони.
                   10608: 
                   10609: Комментарий:
                   10610: Меуччи изобрел телефон на 19 лет раньше Александра Белла, однако из-за
                   10611: неправильного оформления патента лишился права официального
                   10612: первооткрывателя. Однако патриотичный Эко эпоху радиовещания связывает с
                   10613: Маркони, который изобрел радио позже Александра Попова, но зато
                   10614: позаботился оформить документы. Впрочем, термин "галактика Маркони" ввел
                   10615: канадец Маршалл Маклюэн, а Умберто Эко лишь воспользовался им.
                   10616: 
                   10617: Источник:
                   10618: У. Эко. Полный назад! - М., 2012. - С. 11-12.
                   10619: 
                   10620: Автор:
                   10621: Роман Мерзляков (Волгоград)
                   10622: 
                   10623: Вопрос 6:
                   10624: Вергилий из Бигорра пишет, как два ритора четырнадцать суток напролет
                   10625: спорили о звательном падеже одного слова. Назовите это слово.
                   10626: 
                   10627: Ответ:
                   10628: Я.
                   10629: 
                   10630: Комментарий:
                   10631: Бессмысленность спора в том, что сам себя человек ни в каких ситуациях
                   10632: не зовет.
                   10633: 
                   10634: Источник:
                   10635: У. Эко. Полный назад! - М., 2012. - С. 562.
                   10636: 
                   10637: Автор:
                   10638: Роман Мерзляков (Волгоград)
                   10639: 
                   10640: Вопрос 7:
                   10641: (pic: 20150190.jpg)
                   10642:    Этот морской рисунок разместил в своей книге Яков Перельман. Назовите
                   10643: любую страну, у берегов которой мог быть сделан этот рисунок.
                   10644: 
                   10645: Ответ:
                   10646: Любая из стран: Сан-Томе и Принсипи, Экваториальная Гвинея, Габон,
                   10647: Конго, Кения, Сомали, Мальдивы, Индонезия, Кирибати, Эквадор, Колумбия,
                   10648: Бразилия. Незачет: Уганда и Демократическая Республика Конго (у первой
                   10649: нет выхода к морю, у второй он довольно далеко от экватора).
                   10650: 
                   10651: Комментарий:
                   10652: Яков Перельман, как известно, занимался популяризацией точных наук. На
                   10653: правдоподобном с астрономической точки зрения рисунке видно, что у
                   10654: человека почти нет тени. Такое можно видеть на экваторе.
                   10655: 
                   10656: Источник:
1.4       rubashki 10657:    1. Я.И. Перельман. Занимательная астрономия. - М., 2011. - С. 30-31.
1.1       rubashki 10658:    2. http://www.vokrugsveta.ru/encyclopedia/index.php?title=Экватор
                   10659: 
                   10660: Автор:
                   10661: Роман Мерзляков (Волгоград)
                   10662: 
                   10663: Вопрос 8:
                   10664: (pic: 20150191.jpg)
                   10665:    Надеемся, что этот вопрос доставит вам удовольствие.
                   10666:    Яков Перельман, описывая лунную орбиту, говорит, что Луна в АЛЬФЕ
                   10667: занимает такое же положение по отношению к Земле, как и в перигее. Что
                   10668: мы заменили на АЛЬФУ?
                   10669: 
                   10670: Ответ:
                   10671: Точка G.
                   10672: 
                   10673: Комментарий:
                   10674: Перигей приходится на чертеже на точку A. А противоположная ей точка G
                   10675: соответствует апогею.
                   10676: 
                   10677: Источник:
                   10678: Я.И. Перельман. Занимательная астрономия. - М., 2011. - С. 90-91.
                   10679: 
                   10680: Автор:
                   10681: Роман Мерзляков (Волгоград)
                   10682: 
                   10683: Вопрос 9:
                   10684:    <раздатка>
                   10685:    Возобновляемые источники (солнце, ветер, геотермальные) - чистые,
                   10686: фотогеничные, но крайне непредсказуемы в поведении. Быстро наращивают
                   10687: долю, но начинают с очень низкой базы. Атом - профессионал в летах,
                   10688: всегда заканчивает оплаченную работу, за карьеру правда у него случалось
                   10689: разное. Развивается 40 лет, но пока так и остался в аутсайдерах. Более
                   10690: того, сейчас его доля даже сокращается. Гидроэнергетика - простая,
                   10691: "грубая", но очень эффективная технология, вышедшая из деревенских
                   10692: водяных мельниц.
                   10693:    </раздатка>
                   10694:    В блогерском сообществе "Мир вокруг нас" было опубликовано
                   10695: экономическое исследование перспектив альтернативной энергетики.
                   10696: Аналогию с каким фильмом проводят авторы, если его бюджет был на 600
                   10697: тысяч долларов больше, чем у предыдущей картины того же режиссера?
                   10698: 
                   10699: Ответ:
                   10700: "Хороший, плохой, злой".
                   10701: 
                   10702: Комментарий:
                   10703: Возобновляемые источники - это хороший персонаж. Атом - плохой. А
                   10704: гидроэнергетика - злой. Предыдущий фильм Серджо Леоне назывался "На
                   10705: несколько долларов больше".
                   10706: 
                   10707: Источник:
                   10708:    1. http://mirvn.livejournal.com/8149.html
                   10709:    2. http://ru.wikipedia.org/wiki/Хороший,_плохой,_злой
                   10710:    3. http://ru.wikipedia.org/wiki/На_несколько_долларов_больше
                   10711: 
                   10712: Автор:
                   10713: Роман Мерзляков (Волгоград)
                   10714: 
                   10715: Вопрос 10:
                   10716: У персонажа Виктора Пелевина сопоставление шансов ПЕРВОГО и ВТОРОГО
                   10717: вызвало римско-пугачёвские ассоциации. Назовите ПЕРВОГО и ВТОРОГО.
                   10718: 
                   10719: Ответ:
                   10720: "Спартак" и "Салават Юлаев".
                   10721: 
                   10722: Комментарий:
                   10723: Речь о московском и уфимском хоккейных клубах, названных в честь
                   10724: древнеримского гладиатора и башкирского бандита из армии Емельяна
                   10725: Пугачёва.
                   10726: 
                   10727: Источник:
                   10728: В. Пелевин. Желтая стрела.
                   10729: 
                   10730: Автор:
                   10731: Роман Мерзляков (Волгоград)
                   10732: 
                   10733: Вопрос 11:
                   10734: В финале рассказа Виктора Пелевина, изображающего социалистический быт,
                   10735: главный герой видит ковш Большой Медведицы. Какое слово мы заменили в
                   10736: тексте вопроса?
                   10737: 
                   10738: Ответ:
                   10739: Совок.
                   10740: 
                   10741: Комментарий:
                   10742: Заменили слово "совок" на "ковш". Советскую действительность часто
                   10743: уничижительно называли совком.
                   10744: 
                   10745: Источник:
                   10746: В. Пелевин. Желтая стрела.
                   10747: 
                   10748: Автор:
                   10749: Роман Мерзляков (Волгоград)
                   10750: 
                   10751: Вопрос 12:
                   10752: Когда пожилые прихожане пожаловались волгоградскому священнику Дмитрию
                   10753: Нестерову, что не могут запомнить псалмы, он пошел по пути своего
                   10754: западноевропейского коллеги и обучил молитвам десять ИХ. Назовите ИХ
                   10755: словом арабского происхождения.
                   10756: 
                   10757: Ответ:
                   10758: Попугаи.
                   10759: 
                   10760: Комментарий:
                   10761: Монах Франциск Ассизский, как известно, читал Библию, в том числе, и
                   10762: птицам. А Дмитрий Нестеров показал прихожанам попугаев, затвердивших
                   10763: тексты молитв, как пример усердия. Слово "попугай" восходит к арабскому
                   10764: "babagh&#257;", от которого попало во французский и испанский, а оттуда
                   10765: уже в русский.
                   10766: 
                   10767: Источник:
                   10768:    1. Пресс-релиз областного правительства о смерти иеромонаха Дидима
                   10769: (Дмитрия Нестерова).
                   10770:    2. http://put.ucoz.ru/index/0-76
                   10771:    3. http://ru.wiktionary.org/wiki/попугай
                   10772: 
                   10773: Автор:
                   10774: Роман Мерзляков (Волгоград)
                   10775: 
                   10776: Вопрос 13:
                   10777: Согласно шуточной версии Ирины Антоновой, в 1937 году художник Аминадав
                   10778: Каневский начал рисовать плюшевого медвежонка, но успел очертить только
                   10779: его контуры. Набросок сбежал с мольберта и нырнул в заросли молодых
                   10780: одуванчиков. А как оживший рисунок представился, когда пришел
                   10781: устраиваться на работу?
                   10782: 
                   10783: Ответ:
                   10784: Мурзилка.
                   10785: 
                   10786: Комментарий:
                   10787: Сначала рисунок был похож на белого медвежонка, но в одуванчиковом море
                   10788: он сам окрасился в ярко-желтый цвет. Персонаж пришел устраиваться
                   10789: корреспондентом в журнал, где работал его автор.
                   10790: 
                   10791: Источник:
                   10792: И. Антонова. Желтый и пушистый. // "Мурзилка", 2014, N 5. - С. 8-11.
                   10793: 
                   10794: Автор:
                   10795: Роман Мерзляков (Волгоград)
                   10796: 
                   10797: Вопрос 14:
                   10798: 28 января 1925 года Иван Бунин написал стихотворение, в котором были
                   10799: такие строки:
                   10800:    И всё ж придет, придет пора
                   10801:    И воскресенья и деянья,
                   10802:    Прозрения и покаянья.
                   10803:    Чьей памяти посвящены эти стихи?
                   10804: 
                   10805: Ответ:
                   10806: Петр I.
                   10807: 
                   10808: Комментарий:
                   10809: Дата написания стихотворения - это 200-я годовщина со дня смерти первого
                   10810: российского императора. Последняя строка четверостишия содержит рифму к
                   10811: слову "пора": "Россия! Помни же Петра".
                   10812: 
                   10813: Источник:
                   10814: http://pesni.voskres.ru/poems/bunin17.htm
                   10815: 
                   10816: Автор:
                   10817: Роман Мерзляков (Волгоград)
                   10818: 
                   10819: Вопрос 15:
                   10820: Критик Тимофеев пишет: чтобы современные российские читатели этого
                   10821: английского романа испытали те же ощущения, что и в свое время западная
                   10822: публика, переводчику нужно было б некоторые русские слова заменить на
                   10823: чеченские. Назовите этот роман.
                   10824: 
                   10825: Ответ:
                   10826: "Заводной апельсин".
                   10827: 
                   10828: Комментарий:
                   10829: По словам Тимофеева, "русский язык, особенно среди западных читателей
                   10830: старшего поколения, прочно ассоциировался с угрозой советского
                   10831: тоталитаризма". В "Заводном апельсине" Энтони Бёрджесс воспроизводит
                   10832: молодежный жаргон, в котором очень много русских слов. Эти русские слова
                   10833: в англоязычном тексте и предлагается при переводе заменить на чеченские.
                   10834: 
                   10835: Источник:
                   10836: В. Тимофеев. Послесловие. // Э. Бёрджесс. Заводной апельсин. - СПб.,
                   10837: 2004. - С. 226.
                   10838: 
                   10839: Автор:
                   10840: Роман Мерзляков (Волгоград)
                   10841: 
                   10842: Вопрос 16:
                   10843: Персонаж Милорада Павича говорил, что ПЕРВОЕ идет в начале, ВТОРОЕ - в
                   10844: конце. ПЕРВОЕ ассоциируется с жизнью, а ВТОРОЕ - с кладбищами и
                   10845: могильными цветами. Назовите ВТОРОЕ двумя словами на одну и ту же букву.
                   10846: 
                   10847: Ответ:
                   10848: Четное число.
                   10849: 
                   10850: Комментарий:
                   10851: В рядах целых чисел нечетные идут раньше четных: 1, 2; затем 3, 4 и т.д.
                   10852: Могильные букеты состоят из четного количества цветов, тогда как живым
                   10853: дарят нечетное число бутонов.
                   10854: 
                   10855: Источник:
                   10856: М. Павич. Русская борзая. - СПб., 2001. - С. 32.
                   10857: 
                   10858: Автор:
                   10859: Роман Мерзляков (Волгоград)
                   10860: 
                   10861: Вопрос 17:
                   10862:    <раздатка>
                   10863:    f,sh
                   10864:    </раздатка>
                   10865:    Это набранное в английской раскладке первое слово, произнесенное ИМ.
                   10866: Напишите ЕГО имя или фамилию.
                   10867: 
                   10868: Ответ:
                   10869: Полиграф.
                   10870: 
                   10871: Зачет:
                   10872: Шариков.
                   10873: 
                   10874: Комментарий:
                   10875: Так в английской раскладке выглядит слово "абыр", т.е. "рыба" наоборот.
                   10876: 
                   10877: Источник:
                   10878: http://bash.im/quote/419694/
                   10879: 
                   10880: Автор:
                   10881: Денис Леонтьев (Волгоград)
                   10882: 
                   10883: Вопрос 18:
                   10884: В одном документальном фильме о русском ученом использованы кадры
                   10885: поздней режиссерской работы Василия Шукшина. Зрители могут видеть героя
                   10886: "Калины красной" по прозвищу Горе. О каком ученом снят этот докфильм?
                   10887: 
                   10888: Ответ:
                   10889: [Сергей] Прокудин-Горский.
                   10890: 
                   10891: Комментарий:
                   10892: "Калина красная" - единственный фильм Василия Шукшина, снятый им на
                   10893: цветную пленку. Главного героя картины зовут Егор Прокудин. В фильме
                   10894: Леонида Парфёнова о пионере цветной фотографии Прокудине-Горском
                   10895: происходит своеобразная встреча изобретателя и его кинематографического
                   10896: однофамильца.
                   10897: 
                   10898: Источник:
                   10899:    1. Документальный фильм "Цвет нации"
                   10900: (http://www.youtube.com/watch?v=2yIWsXe8s2Y), 1:15:45.
                   10901:    2. http://ru.wikipedia.org/wiki/Калина_красная_(фильм)
                   10902: 
                   10903: Автор:
                   10904: Роман Мерзляков (Волгоград)
                   10905: 
                   10906: Вопрос 19:
                   10907: На чемпионате мира по футболу корреспондент спортивного СМИ заметил на
                   10908: матче сборной Бразилии Леонардо Ди Каприо. Журналист съязвил, что
                   10909: никогда еще актер не был так близок к Халку. Какое слово мы заменили в
                   10910: тексте вопроса?
                   10911: 
                   10912: Ответ:
                   10913: Оскару.
                   10914: 
                   10915: Комментарий:
                   10916: Халк и Оскар - игроки сборной Бразилии. Журналист имел в виду, что Ди
                   10917: Каприо, несмотря на довольно успешную карьеру, до сих пор ни разу не
                   10918: получил премию "Оскар".
                   10919: 
                   10920: Источник:
                   10921: http://www.sports.ru/football/1021291329.html
                   10922: 
                   10923: Автор:
                   10924: Роман Мерзляков (Волгоград)
                   10925: 
                   10926: Вопрос 20:
                   10927: (pic: 20150192.jpg)
                   10928:    Деревенские герои рассказа Юрия Коваля, гадая, как проводить
                   10929: реанимационные действия в полевых условиях, решают, что нужно СДЕЛАТЬ
                   10930: ЭТО. Назовите слово, которое мы заменили на "СДЕЛАТЬ ЭТО".
                   10931: 
                   10932: Ответ:
                   10933: Вдуть.
                   10934: 
                   10935: Комментарий:
                   10936: "Вдувай! Вдувай! - сразу обрадовался Генка. - В нас воздух живой. Он
                   10937: войдет в него и оживит", - так персонажи рассуждают об искусственном
                   10938: дыхании. Фотографии сексуальных девушек в Интернете часто сопровождаются
                   10939: пользовательскими комментариями с однообразным содержанием: "Я бы вдул".
                   10940: 
                   10941: Источник:
                   10942:    1. http://www.e-reading.link/chapter.php/105418/11/Koval'_-_Samaya_legkaya_lodka_v_mire_(sbornik).html
                   10943:    2. http://www.gotowall.com/gallery/people/models_female/13376.html
                   10944: 
                   10945: Автор:
                   10946: Роман Мерзляков (Волгоград)
                   10947: 
                   10948: Вопрос 21:
                   10949: В июле 2014 года в российском Интернете появилась такая шутка. Некий
                   10950: человек оказывается в глухом лесу и с облегчением говорит: "Наконец я
                   10951: совсем один". Мы не спрашиваем, что он услышал в ответ. Назовите этого
                   10952: человека.
                   10953: 
                   10954: Ответ:
                   10955: Луис Фелипе Сколари.
                   10956: 
                   10957: Комментарий:
                   10958: На домашнем чемпионате мира по футболу бразильцы проиграли в полуфинале
                   10959: немцам. И эхо напомнило главному тренеру Бразилии счет матча: "семь -
                   10960: один, семь - один, семь - один".
                   10961: 
                   10962: Источник:
                   10963: http://wc2014.rsport.ru/wc2014/20140709/757078009.html
                   10964: 
                   10965: Автор:
                   10966: Роман Мерзляков (Волгоград)
                   10967: 
                   10968: Вопрос 22:
                   10969: Говоря о том, что жизнь Сократа еще до суда над ним оказалась под
                   10970: угрозой, Михаил Гаспаров пишет, что ОНИ сгущаются. Назовите ИХ,
                   10971: используя восемь символов.
                   10972: 
                   10973: Ответ:
                   10974: "Облака".
                   10975: 
                   10976: Комментарий:
                   10977: Первым предупреждением своевольному философу была комедия Аристофана
                   10978: "Облака", в которой показывалось вредное влияние Сократа на молодежь. В
                   10979: финале пьесы земледелец Стрепсиад поджигает школу мудреца.
                   10980: 
                   10981: Источник:
                   10982:    1. М.Л. Гаспаров. Занимательная Греция: Рассказы о древнегреческой
                   10983: культуре; Капитолийская волчица: Рим до цезарей. - М.: Эксмо, 2012. - С.
                   10984: 306-308.
                   10985:    2. http://ru.wikipedia.org/wiki/Облака_(комедия)
                   10986: 
                   10987: Автор:
                   10988: Роман Мерзляков (Волгоград)
                   10989: 
                   10990: Вопрос 23:
                   10991: Нападающему команды "Ротор-Волгоград" Дмитрию Шишину после победы в
                   10992: чемпионате России по пляжному футболу вручили премию "ИКС". На
                   10993: германском телевидении ИКС присутствует в немецких аналогах передачи
                   10994: "Спокойной ночи, малыши!". Назовите ИКСА двумя словами.
                   10995: 
                   10996: Ответ:
                   10997: Песочный человек.
                   10998: 
                   10999: Комментарий:
                   11000: В пляжный футбол играют на песке. А фольклорный персонаж Песочный
                   11001: человек помогает детям заснуть.
                   11002: 
                   11003: Источник:
                   11004:    1. http://www.volganet.ru/news/3060/
                   11005:    2. http://ru.wikipedia.org/wiki/Песочный_человек
                   11006: 
                   11007: Автор:
                   11008: Роман Мерзляков (Волгоград)
                   11009: 
                   11010: Вопрос 24:
                   11011: Дуплет.
                   11012:    У Хулио Кортасара есть серия ироничных инструкций, как совершать
                   11013: какие-либо банальные действия, например, как правильно петь или как
                   11014: правильно подниматься по лестнице.
                   11015:    1. В инструкции, как правильно ДЕЛАТЬ ЭТО, рекомендуется подумать "о
                   11016: селезне, подвергшемся нападению полчища муравьев". Также следует при
                   11017: этом избегать сходства с улыбкой. Что учит делать эта инструкция?
                   11018:    2. В инструкции, как правильно ДЕЛАТЬ ЭТО, рекомендуется сначала
                   11019: приподнять головку. Далее следует фраза: "И вот начинаются новые сроки".
                   11020: Что учит делать эта инструкция?
                   11021: 
                   11022: Ответ:
                   11023:    1. Плакать.
                   11024:    2. Заводить часы.
                   11025: 
                   11026: Зачет:
                   11027:    2. Переводить часы, переводить время, переводить стрелки.
                   11028: 
                   11029: Комментарий:
                   11030:    2. Кортасар советует поднять головку завода.
                   11031: 
                   11032: Источник:
                   11033: Х. Кортасар. Собрание сочинений в четырех томах. - Т. 2. - СПб., 1992. -
                   11034: С. 531, 532, 534.
                   11035: 
                   11036: Автор:
                   11037: Роман Мерзляков (Волгоград)
                   11038: 
                   11039: Тур:
                   11040: 25 тур. "Сборная с. Тупино" (Ступино - Коломна)
                   11041: 
                   11042: Вопрос 1:
                   11043:    <раздатка>
                   11044:    Если вы читаете это, тогда это предупреждение для вас. Каждое слово,
                   11045: прочитанное вами на этой бесполезной картинке, крадет у вас секунду
                   11046: жизни. Вам больше нечем заняться? Ваша жизнь настолько пуста, что вы
                   11047: действительно не можете придумать лучшего времяпрепровождения? Или вы
                   11048: настолько впечатлены властью, что готовы уважать каждого и доверять
                   11049: всем, кто этого требует? Вы читаете всё, что вам предлагают прочесть?
                   11050: Думаете обо всём, о чем предлагают подумать? Покупаете то, что вам
                   11051: навязывают? Выйдите из вашей квартиры. Встретьтесь с кем-нибудь
                   11052: противоположного пола. Прекратите чрезмерно покупать и мастурбировать.
                   11053: Уйдите с работы. Затейте драку. Докажите, что вы живы. Если вы не
                   11054: докажете свое право быть человеком, вы превратитесь в статистику. Я вас
                   11055: предупредил...
                   11056:    </раздатка>
                   11057:    Какое имя стоит в подписи к этому предупреждению?
                   11058: 
                   11059: Ответ:
                   11060: Тайлер.
                   11061: 
                   11062: Зачет:
                   11063: Дёрден.
                   11064: 
                   11065: Комментарий:
                   11066: Еще одно предупреждение перед началом "Бойцовского клуба".
                   11067: 
                   11068: Источник:
                   11069: http://www.adme.ru/tvorchestvo-kino/25-tajn-bojcovskogo-kluba-612655/
                   11070: 
                   11071: Автор:
                   11072: Юлия Кощеева (Коломна), Виктор Плотников (Великий Новгород)
                   11073: 
                   11074: Вопрос 2:
                   11075: Внимание, в вопросе есть одна замена.
                   11076:    На логотипе интернет-магазина, продающего автомобильные шины и диски,
                   11077: изображены две покрышки, лежащие друг на друге. Этот интернет-магазин
                   11078: называется "Поршенон". В предыдущем предложении мы заменили одно слово.
                   11079: Какое?
                   11080: 
                   11081: Ответ:
                   11082: "Колесей".
                   11083: 
                   11084: Зачет:
                   11085: "Колисей".
                   11086: 
                   11087: Комментарий:
                   11088: Поршенон - Парфенон, Колесей - Колизей.
                   11089: 
                   11090: Источник:
                   11091: http://www.kolesey.ru/
                   11092: 
                   11093: Автор:
                   11094: Андрей Волков, Игорь Коршунов (Коломна)
                   11095: 
                   11096: Вопрос 3:
                   11097: Внимание, в вопросе есть одна замена.
                   11098:    Герой сериала "Интерны" венеролог Купитман говорит: "Ко мне пришел
                   11099: депутат. Пойду, полюбуюсь, так сказать, на ИКС". Назовите ИКС двумя
                   11100: словами, и не ошибитесь веткой.
                   11101: 
                   11102: Ответ:
                   11103: Законодательный орган.
                   11104: 
                   11105: Источник:
                   11106: Телесериал "Интерны", s4e33, 15'17".
                   11107: 
                   11108: Автор:
                   11109: Юлия Кощеева (Коломна), Виктор Плотников (Великий Новгород)
                   11110: 
                   11111: Вопрос 4:
                   11112: Во время Зимних Олимпийских игр 2014 года в Сочи многие интернет-сайты
                   11113: вели текстовые трансляции происходящих событий. Онлайн-трансляция
                   11114: мужской эстафеты в биатлоне на сайте Lenta.ru носила подзаголовок:
                   11115: "Лыжи, палки, ОНИ". Напишите два слова, которые мы заменили словом
                   11116: "ОНИ".
                   11117: 
                   11118: Ответ:
                   11119: Четыре ствола.
                   11120: 
                   11121: Комментарий:
                   11122: Мужская эстафета - 30 км (4 участника по 7,5 км). Ну и вопрос тоже
                   11123: четвертый.
                   11124: 
                   11125: Источник:
                   11126: http://sochi.lenta.ru/onlines/2014/02/22/thlon/
                   11127: 
                   11128: Автор:
                   11129: Юлия Кощеева (Коломна), Виктор Плотников (Великий Новгород)
                   11130: 
                   11131: Вопрос 5:
                   11132: [Ведущему: выделить голосом слово "ФОРМУЛУ".]
                   11133:    Напоминаем ФОРМУЛУ стишка-порошка: 9+8+9+2, вторая и четвертая строки
                   11134: рифмуются. Прослушайте первые две строки порошка:
                   11135:    вот обнажившись кофеманка
                   11136:    ныряет в море кофиЯ
                   11137:    Напишите восьмибуквенное существительное, присутствующее в третьей
                   11138: строке этого грустного порошка.
                   11139: 
                   11140: Ответ:
                   11141: Капучина.
                   11142: 
                   11143: Зачет:
                   11144: Капучино.
                   11145: 
                   11146: Комментарий:
                   11147:    вот обнажившись кофеманка
                   11148:    ныряет в море кофиЯ
                   11149:    и капучина поглощает
                   11150:    ея
                   11151: 
                   11152: Источник:
                   11153: https://vk.com/perawki?w=wall-28122932_48096
                   11154: 
                   11155: Автор:
                   11156: Олег Евстафьев (Ступино)
                   11157: 
                   11158: Вопрос 6:
                   11159: (pic: 20150193.jpg)
                   11160:    Воспроизведите подпись под этим изображением, совпадающую со строкой
                   11161: из детской песни.
                   11162: 
                   11163: Ответ:
                   11164: "Голубой вагон бежит, качается".
                   11165: 
                   11166: Источник:
                   11167: (pic: 20150194.jpg)
                   11168: 
                   11169: Автор:
                   11170: Борис Скоморохов (Ступино)
                   11171: 
                   11172: Вопрос 7:
                   11173: В популярной сетевой игре "Годвилль" герой автора вопроса записал у себя
                   11174: в дневнике: "Прошелся по овсяному полю, потом по пшеничному, по
                   11175: ячменному. В общем, посетил кучу ИХ, а денег не потратил". Назовите ИХ
                   11176: двумя словами.
                   11177: 
                   11178: Ответ:
                   11179: Злачные места.
                   11180: 
                   11181: Источник:
                   11182: http://www.godville.net
                   11183: 
                   11184: Автор:
                   11185: Юлия Кощеева (Коломна), Виктор Плотников (Великий Новгород)
                   11186: 
                   11187: Вопрос 8:
                   11188: Дуплет.
                   11189:    1. В одном из эпизодов телевизионного шоу "Утка-детектив" из
                   11190: мультсериала "Гравити Фолз" констебль в ответ на реплику УТКИ говорит,
                   11191: что она фактически ПРОПУСК дело. Какое слово мы заменили на слово
                   11192: "ПРОПУСК"?
                   11193:    2. Когда в финале мультфильма "Шрек-2" фея превратила короля
                   11194: Гарольда, один из второстепенных персонажей сказал: "Он ПРОПУСК". Какое
                   11195: слово мы заменили на "ПРОПУСК"?
                   11196: 
                   11197: Ответ:
                   11198:    1. Крякнула.
                   11199:    2. Квакнулся.
                   11200: 
                   11201: Комментарий:
                   11202: Вот такой вот птичий вопрос.
                   11203: 
                   11204: Источник:
                   11205:    1. Телесериал "Гравити Фолз".
                   11206:    2. М/ф "Шрек-2", 1h15'.
                   11207: 
                   11208: Автор:
                   11209: Игорь Коршунов (Коломна), Олег Евстафьев (Ступино)
                   11210: 
                   11211: Вопрос 9:
                   11212: Прослушайте четверостишие:
                   11213:    Постигая дамские причуды,
                   11214:    сделайте заметку, господа:
                   11215:    иногда слова "иди отсюда"
                   11216:    означают...
                   11217:    Закончите последнюю строку двумя словами.
                   11218: 
                   11219: Ответ:
                   11220: "Идиот, сюда".
                   11221: 
                   11222: Комментарий:
                   11223: Важны не только запятые, но и пробелы.
                   11224: 
                   11225: Источник:
                   11226: http://tamsyam.livejournal.com/406028.html
                   11227: 
                   11228: Автор:
                   11229: Олег Евстафьев (Ступино)
                   11230: 
                   11231: Вопрос 10:
                   11232: (pic: 20150195.jpg)
                   11233:    Подпись к этому изображению всего на одну букву длиннее названия
                   11234: популярной детской сказки. Воспроизведите подпись к рисунку.
                   11235: 
                   11236: Ответ:
                   11237: "Храпунцель".
                   11238: 
                   11239: Источник:
                   11240: https://vk.com/namalevich?w=wall-26217154_100517
                   11241: 
                   11242: Автор:
                   11243: Юлия Кощеева (Коломна), Виктор Плотников (Великий Новгород)
                   11244: 
                   11245: Вопрос 11:
                   11246: Внимание, в вопросе есть одна замена.
                   11247:    Согласно твиттеру сайта Kinopoisk.ru [кинопоиск точка ру], одно из
                   11248: правил создания постеров к фильмам с очень крутым героем таково: на
                   11249: постере герой ПОКАЗЫВАЕТ СВОЕ НАСТОЯЩЕЕ ЛИЦО. До XIX века дирижеры не
                   11250: ПОКАЗЫВАЛИ СВОЕ НАСТОЯЩЕЕ ЛИЦО. Напишите четыре слова, которые мы
                   11251: заменили словами "ПОКАЗЫВАТЬ СВОЕ НАСТОЯЩЕЕ ЛИЦО".
                   11252: 
                   11253: Ответ:
                   11254: Стоять спиной к зрителю.
                   11255: 
                   11256: Комментарий:
                   11257: Герои на многих постерах стоят спиной к зрителю. До XIX века дирижеры
                   11258: стояли лицом к зрителям.
                   11259: 
                   11260: Источник:
                   11261: https://twitter.com/kinopoiskru/status/388609450711724032/
                   11262: 
                   11263: Автор:
                   11264: Ксения Калинина (Великий Новгород - Санкт-Петербург)
                   11265: 
                   11266: Вопрос 12:
                   11267: Одна из немногих FM-радиостанций, работающих в АнАдыре, носит название
                   11268: "Радио Фигня". Какое слово мы заменили словом "Фигня"?
                   11269: 
                   11270: Ответ:
                   11271: Пурга.
                   11272: 
                   11273: Комментарий:
                   11274: Как еще назвать радиостанцию в снежном Анадыре?
                   11275: 
                   11276: Источник:
                   11277: http://www.laradiofm.ru/station-950/
                   11278: 
                   11279: Автор:
                   11280: Юлия Кощеева (Коломна), Виктор Плотников (Великий Новгород)
                   11281: 
                   11282: Вопрос 13:
                   11283: Прослушайте цитату из песни:
                   11284:    И сердце огнем пылает,
                   11285:    И разум злой туман затмил,
                   11286:    И снова дым извергает ...
                   11287:    В последней строке четверостишия упоминается вымышленный топоним.
                   11288: Назовите этот топоним.
                   11289: 
                   11290: Ответ:
                   11291: Сайлент Хилл.
                   11292: 
                   11293: Источник:
                   11294: КняZz - Пепел города грез.
                   11295: http://www.knyazz.ru/releases/tajna-krivyx-zerkal
                   11296: 
                   11297: Автор:
1.2       rubashki 11298: Андрей Волков (Коломна)
1.1       rubashki 11299: 
                   11300: Вопрос 14:
                   11301: (pic: 20150196.jpg)
                   11302:    Перед вами двое рабочих - один с шуруповертом, другой с молотком. В
                   11303: какую игру они играют?
                   11304: 
                   11305: Ответ:
                   11306: "Крестики-нолики".
                   11307: 
                   11308: Источник:
                   11309: http://tango2010weibo.tumblr.com/post/86223935582
                   11310: 
                   11311: Автор:
                   11312: Юлия Кощеева (Коломна), Виктор Плотников (Великий Новгород)
                   11313: 
                   11314: Вопрос 15:
                   11315: В этом библейском сюжете старообрядцы усматривали современный им смысл:
                   11316: Петр I упразднил патриаршество. Написанная в XVIII веке в Невьянске
                   11317: икона на этот сюжет изображает Петра в виде воина с мечом. Назовите этот
                   11318: сюжет.
                   11319: 
                   11320: Ответ:
                   11321: Усекновение главы Иоанна Предтечи.
                   11322: 
                   11323: Зачет:
                   11324: По смыслу.
                   11325: 
                   11326: Комментарий:
                   11327: Петр I обезглавил церковь.
                   11328: 
                   11329: Источник:
                   11330: http://roizman.livejournal.com/1690491.html
                   11331: 
                   11332: Автор:
                   11333: Юлия Кощеева (Коломна), Виктор Плотников (Великий Новгород)
                   11334: 
                   11335: Вопрос 16:
                   11336: В одном сериале полицейский собирается на пенсию. Он приобретает себе
                   11337: ресторан. Зайдя в это заведение, другой герой сериала видит большое
                   11338: количество посетителей-копов и говорит: "Отличная встроенная ОНА".
                   11339: Назовите ЕЕ двумя словами.
                   11340: 
                   11341: Ответ:
                   11342: Система безопасности.
                   11343: 
                   11344: Источник:
                   11345: "Декстер", s07e08, 24'.
                   11346: 
                   11347: Автор:
                   11348: Юлия Кощеева (Коломна), Виктор Плотников (Великий Новгород)
                   11349: 
                   11350: Вопрос 17:
                   11351: Еще раз напомним формулу стишка-порошка: 9+8+9+2, вторая и четвертая
                   11352: строки рифмуются. Закончите порошок:
                   11353:    вошел король и всё затихло
                   11354:    придворные упали ниц
                   11355:    и только шарканье по полу
                   11356:    ...
                   11357: 
                   11358: Ответ:
                   11359: ресниц
                   11360: 
                   11361: Источник:
                   11362: https://vk.com/sandalporoshki?w=wall-31481258_93111
                   11363: 
                   11364: Автор:
                   11365: Олег Евстафьев (Ступино)
                   11366: 
                   11367: Вопрос 18:
                   11368: Управляя внутренними двигателями старых дисководов, можно получать звуки
                   11369: разной частоты. Пользователь YouTube собрал в одном корпусе несколько
                   11370: дисководов и заставил их играть разные произведения мировой музыкальной
                   11371: культуры. Видеоролик об этом он назвал "Призрак оперы". Какие три буквы
                   11372: мы пропустили в этом вопросе?
                   11373: 
                   11374: Ответ:
                   11375: ф, л, п.
                   11376: 
                   11377: Комментарий:
                   11378: "Призрак флопперы" ("Phantom of the Floppera").
                   11379: 
                   11380: Источник:
                   11381: http://www.youtube.com/watch?v=dmoDLyiQYKw
                   11382: 
                   11383: Автор:
                   11384: Юлия Кощеева (Коломна), Виктор Плотников (Великий Новгород)
                   11385: 
                   11386: Вопрос 19:
                   11387: В фильме 1972 года его персонаж на самом деле не болен, а только
                   11388: притворяется больным. В фильме 1981 года его персонажа действительно
                   11389: потрепало. В обоих случаях за ним ухаживают женщины, правда одна
                   11390: чересчур молчаливая, а другая чересчур болтливая. Назовите исполнителя
                   11391: этих ролей.
                   11392: 
                   11393: Ответ:
                   11394: Никита Михалков.
                   11395: 
                   11396: Комментарий:
                   11397: Ротмистр Минский из "Станционного смотрителя" и Генри Баскервиль из
                   11398: "Собаки Баскервилей". Дуня Вырина за весь фильм ни слова не говорит, а
                   11399: Эльза Бэрримор "трындычит" так, что у похмельного Баскервиля голова
                   11400: раскалывается.
                   11401: 
                   11402: Источник:
                   11403: Указанные фильмы.
                   11404: 
                   11405: Автор:
                   11406: Юлия Кощеева (Коломна), Виктор Плотников (Великий Новгород)
                   11407: 
                   11408: Вопрос 20:
                   11409: В современном фильме Карлосон в исполнении Михаила Галустяна задает
                   11410: Малышу вопрос: "А что такое "не кипишуй"?". В ответ он слышит три слова.
                   11411: Напишите эти три слова.
                   11412: 
                   11413: Ответ:
                   11414: "Спокойствие, только спокойствие".
                   11415: 
                   11416: Источник:
                   11417: "Тот еще Карлосон!", 11'30".
                   11418: 
                   11419: Автор:
                   11420: Юлия Кощеева (Коломна), Виктор Плотников (Великий Новгород)
                   11421: 
                   11422: Вопрос 21:
                   11423: Надеемся, вы еще не забыли, что такое стишок-порошок. Персонаж одного
                   11424: стишка-порошка утонул в северной русской реке, так и не закончив свою
                   11425: знаменитую фразу. Назовите фамилию этого персонажа.
                   11426: 
                   11427: Ответ:
                   11428: Станиславский.
                   11429: 
                   11430: Зачет:
                   11431: Алексеев.
                   11432: 
                   11433: Комментарий:
                   11434:    неугомонный станиславский
                   11435:    тонув в стремительной неве
                   11436:    закончил жизнь на полуфразе
                   11437:    не ве
                   11438: 
                   11439: Источник:
                   11440: https://vk.com/perawki?w=wall-28122932_47968
                   11441: 
                   11442: Автор:
                   11443: Олег Евстафьев (Ступино)
                   11444: 
                   11445: Вопрос 22:
                   11446: В язык этого народа были введены несколько оригинальных слов. Например,
                   11447: слово "беш-ло" означает "железная рыба". Назовите этот народ.
                   11448: 
                   11449: Ответ:
                   11450: Навахо.
                   11451: 
                   11452: Комментарий:
                   11453: Для обозначения военных терминов, которых не было в языке навахо, были
                   11454: введены новые слова. "Беш-ло" - подводная лодка.
                   11455: 
                   11456: Источник:
                   11457: http://ru.wikipedia.org/wiki/Шифровальщики_навахо
                   11458: 
                   11459: Автор:
                   11460: Юлия Кощеева (Коломна), Виктор Плотников (Великий Новгород)
                   11461: 
                   11462: Вопрос 23:
                   11463: Статья "Московского комсомольца" об успехах наших паралимпийцев в Сочи
                   11464: призывает встать с дивана и начать заниматься спортом. В заголовок
                   11465: статьи был вынесен вопрос, который с 1992 года некоторые телезрители
                   11466: еженедельно задают с экрана телевизора другим телезрителям.
                   11467: Воспроизведите этот вопрос из трех слов.
                   11468: 
                   11469: Ответ:
                   11470: "А вам слабо?".
                   11471: 
                   11472: Комментарий:
                   11473: Речь идет о рубрике в передаче "Сам себе режиссер".
                   11474: 
                   11475: Источник:
                   11476:    1. http://www.mk.ru/daily/newspaper/2014/3/18/
                   11477:    2. http://ru.wikipedia.org/wiki/Сам_себе_режиссёр
                   11478: 
                   11479: Автор:
                   11480: Борис Скоморохов (Ступино)
                   11481: 
                   11482: Вопрос 24:
                   11483: [Ведущему: выделить голосом слово "ВЕРНОЕ".]
                   11484:    Внимание, в вопросе есть одна замена.
                   11485:    Согласно популярной интернет-шутке, алкаша, который каждый день по
                   11486: несколько часов ждет открытия винно-водочного магазина, продавцы ласково
                   11487: прозвали "Бушарик". На самом деле его прозвали по-другому. Поняв, что мы
                   11488: заменили в этом вопросе, напишите ВЕРНОЕ прозвище.
                   11489: 
                   11490: Ответ:
                   11491: Бухатико.
                   11492: 
                   11493: Комментарий:
                   11494: Одну кличку собаки - Шарик - заменили на другую - Хатико.
                   11495: 
                   11496: Источник:
                   11497: http://www.anekdot.ru/id/706891/
                   11498: 
                   11499: Автор:
                   11500: Олег Евстафьев (Ступино)
                   11501: 
                   11502: Тур:
                   11503: 26 тур. "Hookah Project" (Астрахань - Истра - Москва)
                   11504: 
                   11505: Инфо:
                   11506: Команда благодарит за ценные замечания Алексея Хмелькова, Александра
                   11507: Грачёва, Анатолия Полетаева, Веронику Лапину, Анастасию Вилкову и Сергея
                   11508: Бакшаева.
                   11509: 
                   11510: Вопрос 1:
                   11511: ИКСОМ в карьере Михаила Ботвинника стало школьное соревнование 1923
                   11512: года, на котором, возможно, был объявлен его ИГРЕК. ИКС и ИГРЕК
                   11513: различаются одной буквой. Назовите ИГРЕК двумя словами.
                   11514: 
                   11515: Ответ:
                   11516: Первый шах.
                   11517: 
                   11518: Комментарий:
                   11519: Первым шагом Ботвинника к шахматной короне был обычный школьный матч.
                   11520: 
                   11521: Источник:
                   11522: http://ru.wikipedia.org/wiki/Ботвинник,_Михаил_Моисеевич
                   11523: 
                   11524: Автор:
                   11525: Михаил Сенкевич (Астрахань)
                   11526: 
                   11527: Вопрос 2:
                   11528: Во вселенной Вархаммера есть навигаторы - существа, использующие
                   11529: нечеловеческую энергию, чтобы видеть и указывать путь кораблям, и
                   11530: поэтому имеющие ЕГО. Героя Кинга излечивают от НЕГО, делая похожим на
                   11531: Гарри Поттера. Назовите ЕГО.
                   11532: 
                   11533: Ответ:
                   11534: Третий глаз.
                   11535: 
                   11536: Комментарий:
                   11537: Наука на грани эзотерики. Мальчику в книге "Темная половина" удаляют
                   11538: третий глаз, в результате чего у него появляется шрам на лбу.
                   11539: 
                   11540: Источник:
                   11541:    1. http://ru.ffg.wikia.com/wiki/Навигатор
                   11542:    2. С. Кинг. Темная половина.
                   11543: 
                   11544: Автор:
                   11545: Михаил Сенкевич (Астрахань)
                   11546: 
                   11547: Вопрос 3:
                   11548: В полусказке Феликса Кривина, когда зашел спор о прекрасном среди
                   11549: растений, Колючка одобрила ЕЕ остротУ и проникновение до самых глубин,
                   11550: но осудила за то, что ОНА представляет всё ТАК. Какие три слова мы
                   11551: заменили на "ТАК"?
                   11552: 
                   11553: Ответ:
                   11554: В розовом свете.
                   11555: 
                   11556: Зачет:
                   11557: Сквозь розовые очки.
                   11558: 
                   11559: Комментарий:
                   11560: Колючка критиковала розу.
                   11561: 
                   11562: Источник:
                   11563: Ф. Кривин. Полусказки. http://www.flibusta.net/b/170551/read
                   11564: 
                   11565: Автор:
                   11566: Вероника Лапина (Астрахань)
                   11567: 
                   11568: Вопрос 4:
                   11569: В фильме "Сквозь горизонт" есть сцена: персонаж рисует на листе линию,
                   11570: складывает лист пополам и протыкает его ручкой. Назовите фильм, в
                   11571: котором эта сцена была полностью повторена.
                   11572: 
                   11573: Ответ:
                   11574: "Интерстеллар".
                   11575: 
                   11576: Комментарий:
                   11577: Персонаж показывает сущность кротовой норы и мгновенных перемещений в
                   11578: пространстве.
                   11579: 
                   11580: Источник:
                   11581: http://www.kinopoisk.ru/film/258687/
                   11582: 
                   11583: Автор:
                   11584: Михаил Сенкевич (Астрахань)
                   11585: 
                   11586: Вопрос 5:
                   11587: Герой Маяковского, бывший рабочий, бывший партиец, а ныне жених,
                   11588: требовал, чтобы была ОНА и никаких богов. Другая ОНА состоялась в 299
                   11589: году. Назовите ЕЕ двумя словами.
                   11590: 
                   11591: Ответ:
                   11592: Красная свадьба.
                   11593: 
                   11594: Комментарий:
                   11595: В 299 году по летоисчислению Семи Королевств.
                   11596: 
                   11597: Источник:
                   11598:    1. В. Маяковский. Клоп.
                   11599: http://www.feb-web.ru/feb/mayakovsky/texts/ms0/msb/msb-215-.htm
                   11600:    2. Джордж Мартин. Буря Мечей.
                   11601: 
                   11602: Автор:
                   11603: Вероника Лапина (Астрахань)
                   11604: 
                   11605: Вопрос 6:
                   11606: Персонажи романа Бориса Виана "Пена дней" во время венчания едят
                   11607: божественное мороженое и пьют газированную воду с сиропом. Какое слово в
                   11608: предыдущем предложении мы пропустили?
                   11609: 
                   11610: Ответ:
                   11611: Святую.
                   11612: 
                   11613: Комментарий:
                   11614: Венчание происходит в церкви, где даже газированная вода с сиропом -
                   11615: святая, а мороженое - божественное.
                   11616: 
                   11617: Источник:
                   11618: Б. Виан. Пена дней. http://www.flibusta.net/b/133416/read
                   11619: 
                   11620: Автор:
                   11621: Вероника Лапина (Астрахань)
                   11622: 
                   11623: Вопрос 7:
                   11624: Евгения Ивановна, заглавная героиня повести Леонида Леонова, -
                   11625: эмигрантка, которая бежала из царской России и вышла замуж за
                   11626: англичанина. Запишите название повести, как это сделал автор.
                   11627: 
                   11628: Ответ:
                   11629: "Evgenia Ivanovna".
                   11630: 
                   11631: Зачет:
                   11632: Транслитом с незначительными ошибками.
                   11633: 
                   11634: Комментарий:
                   11635: Такой заголовок подчеркивает, что героиня во многом и сама стала
                   11636: англичанкой.
                   11637: 
                   11638: Источник:
                   11639: Л. Леонов. Evgenia Ivanovna. http://www.flibusta.net/b/198326/read
                   11640: 
                   11641: Автор:
                   11642: Вероника Лапина (Астрахань)
                   11643: 
                   11644: Вопрос 8:
                   11645: Сразу после знакомства со своим спасителем заглавный герой детского
                   11646: фильма попросил его назвать самое нежное из имен его отца. Назовите имя
                   11647: отца заглавного героя.
                   11648: 
                   11649: Ответ:
                   11650: Хоттаб.
                   11651: 
                   11652: Комментарий:
                   11653: Потому он и называл Вольку ибн Алеша, а не ибн Алексей.
                   11654: 
                   11655: Источник:
                   11656: Л. Лагин. Старик Хоттабыч. http://www.flibusta.net/b/266897/read
                   11657: 
                   11658: Автор:
                   11659: Вероника Лапина (Астрахань)
                   11660: 
                   11661: Вопрос 9:
                   11662: Саре Бернар приписывают следующее утверждение: "Жизнь похожа на лежание
                   11663: на полке в бане.... Тебе мылят голову и ДЕЛАЮТ ЭТО". ДЕЛАЮТ ЭТО
                   11664: работники некоторых музеев. Какие два слова мы заменили на "ДЕЛАТЬ ЭТО"?
                   11665: 
                   11666: Ответ:
                   11667: Перебирать косточки.
                   11668: 
                   11669: Зачет:
                   11670: Перебирать кости; перемывать косточки; перемывать кости.
                   11671: 
                   11672: Источник:
                   11673: А.П. Чехов. Философские определения жизни.
                   11674: http://www.feb-web.ru/feb/chekhov/texts/sp0/sp1/sp1-470-.htm
                   11675: 
                   11676: Автор:
                   11677: Вероника Лапина (Астрахань)
                   11678: 
                   11679: Вопрос 10:
                   11680: Барселонский клуб комедии установил на спинке каждого кресла планшет.
                   11681: Эта технология позволила установить необычный, но справедливый способ
                   11682: оплаты: теперь зритель платит по 30 центов за каждую АЛЬФУ. Какое слово
                   11683: мы заменили АЛЬФОЙ?
                   11684: 
                   11685: Ответ:
                   11686: Улыбка.
                   11687: 
                   11688: Комментарий:
                   11689: Планшет считывает улыбку на лице зрителя, а после выступления улыбки
                   11690: подсчитываются, и их количество определяет, сколько тот должен
                   11691: заплатить. Максимальная сумма, которую можно потратить, - 24 евро, после
                   11692: этого все улыбки бесплатны.
                   11693: 
                   11694: Источник:
                   11695: http://www.theguardian.com/stage/2014/oct/14/standup-comedy-pay-per-laugh-charge-barcelona
                   11696: 
                   11697: Автор:
                   11698: Илья Иванов (Пермь)
                   11699: 
                   11700: Вопрос 11:
                   11701: В повести Теда Чана описывается строительство Вавилонской башни. При
                   11702: этом не было недостатка в кирпичах, зато каждый строитель боялся
                   11703: потерять то, что в советские времена было названием детского журнала о
                   11704: творчестве и рукоделии. Что именно?
                   11705: 
                   11706: Ответ:
                   11707: Мастерок.
                   11708: 
                   11709: Зачет:
                   11710: Мастерки.
                   11711: 
                   11712: Комментарий:
                   11713: Вавилонская башня была очень высокой. Утеря кирпича для каменщика была
                   11714: пустяком, а вот без мастерка он на три-четыре месяца лишался работы,
                   11715: пока ему наверх не доставляли новый.
                   11716: 
                   11717: Источник:
                   11718:    1. Тед Чан. Вавилонская башня. http://www.flibusta.net/b/96583/read
                   11719:    2. http://ru.wikipedia.org/wiki/Кельма
                   11720: 
                   11721: Автор:
                   11722: Виктор Мялов (Днепропетровск)
                   11723: 
                   11724: Вопрос 12:
                   11725: Некоторые муравьи рода Adetomyrma [адетомИрма] питаются гемолИмфой своих
                   11726: личинок. За это их прозвали муравьями-акУлами. Одно из слов в предыдущем
                   11727: предложении мы немного изменили. Напишите его в первоначальном виде.
                   11728: 
                   11729: Ответ:
                   11730: Муравьями-дрАкулами.
                   11731: 
                   11732: Зачет:
                   11733: ДрАкулами.
                   11734: 
                   11735: Комментарий:
                   11736: Гемолимфа у насекомых является аналогом крови у животных.
                   11737: 
                   11738: Источник:
                   11739: http://en.wikipedia.org/wiki/Adetomyrma
                   11740: 
                   11741: Автор:
                   11742: Илья Иванов (Пермь)
                   11743: 

FreeBSD-CVSweb <freebsd-cvsweb@FreeBSD.org>